Sie sind auf Seite 1von 279

General Knowledge Today

Current Affairs: August, 2017 [Date-


wise Compendium]

Published Date: September 8, 2017


Published by: GKTODAY.IN
GKToday 2017 | All Rights Reserved

Suitable for Bank Probationary, Bank Clerical, IBPS Recruitment, NDA, CDS,
LIC/GIC, Railways, SSC, UGC and state / UPSC preliminary Examination.
Current Aairs: August, 2017 [Date-wise Compendium]

Contents
Month Highlights: July, 2017 ............................................................................................................................ 3
Facts in News [FIN] August, 2017 for Banking, SSC, MBA, CLAT etc. .............................................. 18
August 4, 2017 .......................................................................................................................................... 18
August 5, 2017 ........................................................................................................................................... 19
August 7, 2017 .......................................................................................................................................... 20
August 8, 2017 ........................................................................................................................................... 21
August 9, 2017 ........................................................................................................................................... 21
August 10, 2017 ........................................................................................................................................ 22
August 12, 2017 ......................................................................................................................................... 22
August 13, 2017 ......................................................................................................................................... 23
August 14, 2017 ........................................................................................................................................ 24
August 17, 2017 ........................................................................................................................................ 24
18 August 2017 ......................................................................................................................................... 24
August 20-21, 2017 .................................................................................................................................. 25
22 August 2017 ......................................................................................................................................... 26
25 August 2017 ......................................................................................................................................... 26
26 August 2017 ........................................................................................................................................ 27
27-28 August 2017 .................................................................................................................................. 28
29 August 2017 ........................................................................................................................................ 29
30-31 August 2017 ................................................................................................................................... 30
Date Wise Current Affairs ................................................................................................................................ 31
August 1, 2017 .................................................................................................................................................... 31
BSF and NSDC sign MoU on Skill Development ............................................................................... 31
Key Facts ...................................................................................................................................................... 31
About National Skill Development Corporation (NSDC) ................................................................ 31
India-Bhutan New Trade and Transit agreement come into force ................................................ 31
Background ................................................................................................................................................ 32
19th RCEP Trade Negotiating Committee Meeting held in Hyderabad ....................................... 32
Key takeaways of 19th round ................................................................................................................. 32
About Regional Comprehensive Economic Partnership (RCEP) .................................................... 32
SC bans use of ve heavy metals in recrackers ................................................................................ 32
SC order ...................................................................................................................................................... 32
Heavy metals in recrackers ................................................................................................................... 33
About Petroleum and Explosives Safety Organisation (PESO) ........................................................ 33
Dhrupad maestro Ustad Sayeeduddin Dagar passes away .............................................................. 33
About Sayeeduddin Dagar ...................................................................................................................... 33
About Dhrupad .......................................................................................................................................... 33
Arvind Panagariya resigns as Vice-Chairman of NITI Aayog ...................................................... 34
About Arvind Panagariya ........................................................................................................................ 34

2017 GKToday | All Rights Reserved | www.gktoday.in

2
Current Aairs: August, 2017 [Date-wise Compendium]

About NITI Aayog .................................................................................................................................... 34


Shahid Khaqan Abbasi elected as Prime Minister of Pakistan ...................................................... 34
About Shahid Khaqan Abbasi ............................................................................................................... 34
Chemical from turmeric treats cancer in children: study ................................................................. 35
Key Facts ..................................................................................................................................................... 35
Neuroblastoma (NB) ................................................................................................................................ 35
August 2, 2017 .................................................................................................................................................. 35
Eminent Scientist Pushpa Bhargava passes away ............................................................................... 35
About Pushpa Mittra Bhargava .............................................................................................................. 35
India, Somali sign agreement on transfer of sentenced persons .................................................... 36
Background ................................................................................................................................................ 36
Parliament passes Right of Children to Free and Compulsory Education (Amendment) Bill,
2017 ..................................................................................................................................................... 36
Key Facts ..................................................................................................................................................... 36
Comment .................................................................................................................................................... 36
Government launches e-RaKAM portal ............................................................................................... 37
About e-RaKAM ....................................................................................................................................... 37
Government constitutes Committee to study issues related to data protection .......................... 37
Terms of Reference of Committee ....................................................................................................... 37
Background ................................................................................................................................................ 37
India, China home to 39% of young Internet users: UN report .................................................... 37
Key Highlights of Report ......................................................................................................................... 37
About International Telecommunication Union (ITU) ..................................................................... 38
Scientists successfully use CRISPR to x a genetic mutation causing Hypertrophic
Cardiomyopathy disease .................................................................................................................. 38
Key Facts ..................................................................................................................................................... 39
How it was achieved? .............................................................................................................................. 39
........................................................................................................................................................................ 39
CRISPR-Cas9 gene-editing technology ................................................................................................ 39
Rising temperatures drive up farmer suicides in India: US study ................................................... 40
Key Highlights of Study .......................................................................................................................... 40
Limitations of Study ................................................................................................................................. 40
India can construct Kishanganga, Ratle Hydro Power Plants: World Bank ................................. 40
Key Facts .................................................................................................................................................... 40
Background ................................................................................................................................................. 41
About Indus Waters Treaty (IWT) ........................................................................................................ 41
RBI cuts repo rate by 25 bps to 6% ..................................................................................................... 42
Policy Rates ................................................................................................................................................. 42
About Monetary Policy Committee (MPC) ........................................................................................ 42
August 3, 2017 .................................................................................................................................................. 42
Indian Army develops Humraaz app for soldiers to track promotions, postings ....................... 43

2017 GKToday | All Rights Reserved | www.gktoday.in

3
Current Aairs: August, 2017 [Date-wise Compendium]

Key Facts ..................................................................................................................................................... 43


Union Cabinet approves MoU between India and Spain on cooperation in renewable energy
................................................................................................................................................................ 43
Key Facts ..................................................................................................................................................... 43
Lok Sabha passes two key bills for implementation of GST in J&K .............................................. 43
Background ................................................................................................................................................ 43
Cabinet approves MoU to set up BRICS Agriculture Research Platform ..................................... 43
BRICS Agriculture Research Platform (BRICS-ARP) ....................................................................... 44
Background ................................................................................................................................................ 44
SEBI constitutes TK Viswanathan committee on fair market conduct ......................................... 44
Need ............................................................................................................................................................ 44
Terms and Reference of Committee .................................................................................................... 44
About Securities and Exchange Board of India (SEBI) .................................................................... 44
Union Cabinet clears proxy voting for NRIs ....................................................................................... 45
Key Facts ..................................................................................................................................................... 45
Need for Proxy Voting ............................................................................................................................. 45
Expert committee Recommendation ..................................................................................................... 45
Comment .................................................................................................................................................... 45
BRICS nations vow to ght protectionism .......................................................................................... 45
Highlights of Joint statement .................................................................................................................. 45
About BRICS ............................................................................................................................................. 46
Israel launches rst environmental research satellite Venus ............................................................ 46
Key facts ..................................................................................................................................................... 46
Lok Sabha passes Banking Regulation (Amendment) Bill, 2017 ................................................... 47
Key Features of the Bill ........................................................................................................................... 47
Need for Amendment .............................................................................................................................. 47
Lok Sabha passes NABARD (Amendment) Bill, 2017 ..................................................................... 47
Key Features of the Bill ........................................................................................................................... 47
August 4, 2017 ................................................................................................................................................. 48
Qatar to become rst Arab state to offer permanent residency to some non-citizens ............. 48
Key Facts .................................................................................................................................................... 48
Background ................................................................................................................................................ 48
Maharashtra Government launches two online portals MahaDBT, MahaVASTU .................... 48
Key Facts .................................................................................................................................................... 48
Suns core rotates 4 times faster than its surface: Study ................................................................. 49
Key Facts .................................................................................................................................................... 49
Parliament passes IIIT (Amendment) Bill, 2017 ................................................................................ 49
Key Features of the Bill .......................................................................................................................... 49
Government proposes to set up 14 new AIIMS hospitals under PMSSY .................................... 50
About Pradhan Mantri Swasthya Suraksha Yojana ........................................................................... 50
ISRO, NASA working towards realisation of NISAR Mission by 2021 ........................................ 50

2017 GKToday | All Rights Reserved | www.gktoday.in

4
Current Aairs: August, 2017 [Date-wise Compendium]

Salient Features .......................................................................................................................................... 50


Indias rst private missile production facility unveiled in Hyderabad ............................................ 51
Key Facts ...................................................................................................................................................... 51
Recommendations: Justice Srikrishna committee on Institutionalization of Arbitration
Mechanism .......................................................................................................................................... 51
Key Recommendations ............................................................................................................................. 52
August 5, 2017 .................................................................................................................................................. 52
Government to launch AGRI UDAAN-Food and Agribusiness Accelerator 2.0 programme
................................................................................................................................................................ 52
Key facts ...................................................................................................................................................... 52
India signs loan pact with AIIB for Rural Roads Project .................................................................. 53
About Asian Infrastructure Investment Bank (AIIB) ......................................................................... 53
ISRO and CSIR-NPL sign MoU for time and frequency traceability services .............................. 53
Signicance of MoU ................................................................................................................................. 53
NavIC ........................................................................................................................................................... 53
National Physical Laboratory (NPL) ..................................................................................................... 54
Indian Railways signs rst EPC contract with L&T to speed up electrication ........................... 54
Background ................................................................................................................................................ 54
Comment .................................................................................................................................................... 54
Union Government launches Bharat 22 ETF to sell stakes in 22 rms ........................................ 54
Exchange-traded funds (ETFs) .............................................................................................................. 54
Key Facts ..................................................................................................................................................... 55
Background ................................................................................................................................................ 56
Venkaiah Naidu elected 13th Vice President of India ........................................................................ 57
Background ................................................................................................................................................ 57
About M. Venkaiah Naidu ...................................................................................................................... 57
Vice-President Elections ........................................................................................................................... 57
Economist Rajiv Kumar appointed as Vice Chairman of NITI Aayog .......................................... 57
About Rajiv Kumar ................................................................................................................................... 57
NITI Aayog ................................................................................................................................................ 58
August 6, 7, 2017 ............................................................................................................................................. 58
Aparna appointed as Executive Director of World Bank ................................................................. 58
About S. Aparna ........................................................................................................................................ 58
About World Bank .................................................................................................................................... 58
August 7, 2017 .................................................................................................................................................. 58
7 August: National Handloom Day ...................................................................................................... 58
Background ................................................................................................................................................ 59
Comment .................................................................................................................................................... 59
Astronomers found WASP-121b exoplanet with glowing water atmosphere .............................. 59
Key Facts ..................................................................................................................................................... 59
About WASP-121b exoplanet .................................................................................................................. 59
2017 GKToday | All Rights Reserved | www.gktoday.in

5
Current Aairs: August, 2017 [Date-wise Compendium]

IIT Delhi researchers develops new nanotechnology-based drug delivery system .................... 59
Need ............................................................................................................................................................. 59
Key Facts .................................................................................................................................................... 60
IISc researchers develop low-cost, sensitive CO sensor ................................................................... 60
Carbon Monoxide (CO) .......................................................................................................................... 60
Key Facts .................................................................................................................................................... 60
Signicance ................................................................................................................................................ 60
Two new species of Cycas discovered ................................................................................................. 60
Cycas ........................................................................................................................................................... 60
Key Facts ..................................................................................................................................................... 61
Government launches Tele-Law scheme in Bihar .............................................................................. 61
Key Facts ..................................................................................................................................................... 61
Government launches second phase of Measles-rubella vaccination campaign .......................... 61
Key Facts ..................................................................................................................................................... 61
Measles-rubella (MR) ............................................................................................................................... 62
August 8, 2017 .................................................................................................................................................. 62
India to MGC nations: Expand areas of cooperation ........................................................................ 62
Key Facts ..................................................................................................................................................... 62
Mekong Ganga Cooperation (MGC) bloc .......................................................................................... 62
Government merges NCRB with BPRD .............................................................................................. 63
Key Facts ..................................................................................................................................................... 63
Reasons for merger .................................................................................................................................. 63
About NCRB and BPRD .......................................................................................................................... 63
Entire Assam declared as disturbed area under AFSPA for another month .............................. 63
Background ................................................................................................................................................ 63
About Armed Forces (Special Powers) Act ........................................................................................ 63
India, Iran call for early operationalisation of Chabahar Port ......................................................... 64
Key Facts .................................................................................................................................................... 64
Background ................................................................................................................................................ 64
Chabahar Port ........................................................................................................................................... 64
Signicance of Chabahar Port ............................................................................................................... 64
Maharashtra Government to raise $270 m from Green Climate Fund ....................................... 65
Maharashtra Water Resources Multi-Stakeholder Platform (MWRMP) ...................................... 65
2030WRG .................................................................................................................................................. 66
Green Climate Fund (GCF) ................................................................................................................... 66
Government constitute committee to make drugs more affordable ............................................. 66
Terms of reference of the committee .................................................................................................. 66
Justice Dipak Misra appointed as 45th Chief Justice of India ......................................................... 66
About Justice Dipak Misra ...................................................................................................................... 66
August 9, 2017 ................................................................................................................................................. 67
75th anniversary of Quit India Movement .......................................................................................... 67

2017 GKToday | All Rights Reserved | www.gktoday.in

6
Current Aairs: August, 2017 [Date-wise Compendium]

Quit India Movement ............................................................................................................................... 67


Commerce Ministry to shut down DGS&D ....................................................................................... 67
Directorate General of Supplies and Disposals (DGS&D) .............................................................. 68
Need for shutting down DGS&D .......................................................................................................... 68
NITI Aayog partners with 6 states to transform Health & Education sectors ............................ 68
Key Facts .................................................................................................................................................... 68
Selection of States ..................................................................................................................................... 69
Sustainable Action for Transforming Human Capital (SATH) initiative ..................................... 69
NITI Aayog ............................................................................................................................................... 69
NSDC, Google India launch mobile app development programme ............................................. 69
Key Facts .................................................................................................................................................... 69
National Skill Development Corporation (NSDC) ............................................................................ 69
Government to expand Logistics Data Bank project to South India .......................................... 70
About Logistics Data Bank (LDB) project .......................................................................................... 70
Signicance of project ............................................................................................................................. 70
Government launches Swachh Survekshan Gramin 2017 .............................................................. 70
Key Highlights of Swachh Survekshan Gramin ................................................................................. 70
About Quality Council of India (QCI) .................................................................................................. 71
India ranks 43rd in 2017 Global Retirement Index ........................................................................... 71
Key Highlights of 2017 GRI ................................................................................................................... 72
India raties 2nd commitment period of Kyoto Protocol ................................................................ 73
About Kyoto Protocol .............................................................................................................................. 73
Comment .................................................................................................................................................... 73
August 10, 2017 ................................................................................................................................................ 73
Anti-dumping duty on 93 products from China: Government ...................................................... 73
Anti-dumping duty ................................................................................................................................... 74
Why it is imposed? ................................................................................................................................... 74
Mechanism for imposition ...................................................................................................................... 74
NCERT launches web portal to supply Textbooks online ............................................................... 74
Key Facts .................................................................................................................................................... 74
About National Council of Educational Research and Training (NCERT) .................................. 74
August 10: International Biodiesel Day ................................................................................................ 75
About Biodiesel .......................................................................................................................................... 75
Benets of Biodiesel .................................................................................................................................. 75
UP Government launches Namami Gange Jagriti Yatra .................................................................. 75
Key Facts ..................................................................................................................................................... 75
ISRO to develop full-edged Hyperspectral Imaging Earth observation satellite ....................... 75
Hyperspectral Imaging ............................................................................................................................ 76
Key Facts .................................................................................................................................................... 76
Background ................................................................................................................................................ 76
National Cyber Coordination Centre made operational: Government ...................................... 77

2017 GKToday | All Rights Reserved | www.gktoday.in

7
Current Aairs: August, 2017 [Date-wise Compendium]

Key Facts .................................................................................................................................................... 78


Government to launch Aajeevika Grameen Express Yojana under DAY-NRLM ....................... 78
Key Facts .................................................................................................................................................... 78
About Deendayal Antyodaya Yojana National Rural Livelihoods Mission (DAY-NRLM)
............................................................................................................................................................... 78
NGT bans use of non-biodegradable plastic bags in Delhi ............................................................. 79
NGT order ................................................................................................................................................. 79
Plastic waste ............................................................................................................................................... 79
Lok Sabha passes State Banks (Repeal and Amendment) Bill, 2017 ............................................ 79
Key Facts .................................................................................................................................................... 79
Background ................................................................................................................................................ 80
August 11, 2017 ................................................................................................................................................. 80
India and US to co-host Global Entrepreneurship Summit ............................................................. 80
Key Facts .................................................................................................................................................... 80
Parliament passes Banking Regulation (Amendment) Bill, 2017 ................................................... 80
Key Features of the Bill ............................................................................................................................ 81
Need for Amendment ............................................................................................................................... 81
National Mission for Clean Ganga approves 10 projects .................................................................. 81
Approved Projects ..................................................................................................................................... 81
About National Mission for Clean Ganga (NMCG) .......................................................................... 81
Venkaiah Naidu sworn in as 13th Vice President of India ................................................................ 82
About M Venkaiah Naidu ....................................................................................................................... 82
About ofce of Vice President ................................................................................................................ 82
15th BIMSTEC Ministerial meeting begins in Kathmandu .............................................................. 82
Key Facts ..................................................................................................................................................... 82
About BIMSTEC ....................................................................................................................................... 83
US Scientists develop micropropulsion system based on liquid water .......................................... 83
CubeSats ..................................................................................................................................................... 83
Film-Evaporation MEMS Tunable Array (FEMTA) .......................................................................... 83
August 12, 2017 ................................................................................................................................................ 84
12 August: International Youth Day ..................................................................................................... 84
Background ................................................................................................................................................ 84
MCX gets SEBI approval to launch Indias rst gold options contract ........................................ 84
Key Facts .................................................................................................................................................... 84
About Multi Commodity Exchange Ltd (MCX) ................................................................................ 84
Odisha government and Facebook launches SheMeansBusiness programme .......................... 85
Key Facts ..................................................................................................................................................... 85
SheMeansBusiness programme ............................................................................................................. 85
Prasoon Joshi appointed as new chairman of CBFC ......................................................................... 85
Prasoon Joshi .............................................................................................................................................. 85
About Central Board of Film Certication (CBFC) .......................................................................... 85

2017 GKToday | All Rights Reserved | www.gktoday.in

8
Current Aairs: August, 2017 [Date-wise Compendium]

IIP output contracts 0.1% in June 2017 ............................................................................................... 86


Key Facts .................................................................................................................................................... 86
About Index of Industrial Production (IIP) ......................................................................................... 86
Economic Survey 2016-17 Volume II: Agriculture and Food Management Reforms Measures
............................................................................................................................................................... 87
Challenges .................................................................................................................................................. 87
Reforms suggested ................................................................................................................................... 87
Economic Survey proposes Transparency of Rules Act ................................................................... 87
Need ............................................................................................................................................................ 88
Key Features of TORA ............................................................................................................................ 88
Way Forward ............................................................................................................................................. 88
Government launches nationwide campaign Gaj Yatra to protect elephants ............................. 88
Gaj Yatra Campaign ................................................................................................................................. 88
World Elephant Day ................................................................................................................................. 88
Elephants ..................................................................................................................................................... 89
Indian Coast Guard Ship Shaurya commissioned in Goa ................................................................ 89
About ICGS Shaurya ............................................................................................................................... 89
CSIR-IGIB and NCDC Researchers develop DNA sensor for quick pathogen detection ........ 89
Key Facts .................................................................................................................................................... 89
S. pyogenes infections .............................................................................................................................. 90
August 13, 14, 2017 ......................................................................................................................................... 90
India-ASEAN Youth Summit begins in Bhopal .................................................................................. 90
Key Facts .................................................................................................................................................... 90
About Association of South East Asian Nations (ASEAN) .............................................................. 90
Indias rst online course on Infusion Therapy launched by INS .................................................. 90
About Infusion therapy ............................................................................................................................. 91
Indra: India, Russia for rst time conduct tri-services exercise ....................................................... 91
Key Facts ..................................................................................................................................................... 91
Comment ..................................................................................................................................................... 91
Background ................................................................................................................................................. 91
Scientists develop worlds rst plant-based Zika vaccine .................................................................. 91
Key Facts ................................................................................................................................................... 92
About Zika virus ........................................................................................................................................ 92
IMMSAREX: IONSs maiden maritime search and rescue exercise will be held in November
2017 ..................................................................................................................................................... 92
Comment .................................................................................................................................................... 92
About Indian Ocean Naval Symposium (IONS) ................................................................................ 93
Members to IONS .................................................................................................................................... 93
El Nino of 2014-16 aided in massive carbon dioxide release: Study ............................................. 93
El Nino ......................................................................................................................................................... 93
Key Highlights of study ........................................................................................................................... 93

2017 GKToday | All Rights Reserved | www.gktoday.in

9
Current Aairs: August, 2017 [Date-wise Compendium]

Earths Largest volcanic region found in Antarctica .......................................................................... 94


Key Facts .................................................................................................................................................... 94
August 16, 2017 ................................................................................................................................................ 95
GeM and CII sign MoU to establish GeMSamvad ............................................................................ 95
Key Facts ..................................................................................................................................................... 95
About Government eMarketplace (GeM) ........................................................................................... 95
Scientists develop injectable tissue bandage to repair hearts ........................................................... 95
Key Facts .................................................................................................................................................... 96
Black Carbon released by aeroplanes may be affecting ozone, monsoon: Study ....................... 96
Key Facts .................................................................................................................................................... 96
India to host steering committee meeting of TAPI gas pipeline .................................................... 96
About Turkmenistan-Afghanistan-Pakistan-India (TAPI) gas pipeline ........................................ 96
....................................................................................................................................................................... 97
Comment .................................................................................................................................................... 97
Nepal, China sign three pacts to boost energy, economic ties ....................................................... 97
Key Facts .................................................................................................................................................... 98
CCEA approves procedure and mechanism for Strategic Disinvestment .................................... 98
Revised mechanism for strategic disinvestment ................................................................................. 98
Strategic Disinvestment ........................................................................................................................... 98
Department of Investment and public Asset Management (DIPAM) .......................................... 98
Union Cabinet approves new Metro Policy ........................................................................................ 98
Salient Features of new policy ............................................................................................................... 99
Proposed ways for private sector participation in O & M of metro services ............................. 100
NPPA caps prices of orthopaedic knee implants ............................................................................. 100
Background .............................................................................................................................................. 100
Capped Prices .......................................................................................................................................... 100
Comment .................................................................................................................................................. 100
National Pharmaceutical Pricing Authority (NPPA) ........................................................................ 100
Cabinet approves creation non-lapsable Madhyamik and Uchchtar Shiksha Kosh .................. 101
Madhyamik and Uchchtar Shiksha Kosh (MUSK) ........................................................................... 101
Background ............................................................................................................................................... 101
August 17, 2017 ............................................................................................................................................... 101
Malpelo plate: New tectonic microplate discovered off the coast of Ecuador ........................... 101
Key Facts .................................................................................................................................................... 101
Tectonic Plate ........................................................................................................................................... 102
India has 27,312 elephants: 2017 Census Estimation ...................................................................... 102
Key Highlights of the survey ................................................................................................................. 102
Comment .................................................................................................................................................. 103
Melbourne tops in 2017 EIU Liveability index .................................................................................. 103
Key Facts ................................................................................................................................................... 103
US designates Hizbul Mujahideen as Foreign Terrorist Organisation ........................................ 104

2017 GKToday | All Rights Reserved | www.gktoday.in

10
Current Aairs: August, 2017 [Date-wise Compendium]

What does it means? .............................................................................................................................. 104


Background .............................................................................................................................................. 105
Comment .................................................................................................................................................. 105
About Hizbul Mujahideen ..................................................................................................................... 105
Cabinet approves MoU between India and Sweden on IPRs ........................................................ 105
Features of MoU ...................................................................................................................................... 105
Signicance of the MoU ........................................................................................................................ 106
Union Government inks Global Environment Facility Grant Agreement with World Bank
.............................................................................................................................................................. 106
Ecosystem Service Improvement Project ........................................................................................... 106
Global Environment Facility (GEF) ..................................................................................................... 106
National Mission for Green India (GIM) ........................................................................................... 106
Cabinet clears stalled North Koel Reservoir Project in Bihar ........................................................ 106
Background .............................................................................................................................................. 107
North Koel River ..................................................................................................................................... 107
Cabinet approves raising Rs. 9020 crore for Long Term Irrigation Fund ................................ 107
Utilization of Funds ................................................................................................................................. 107
Background .............................................................................................................................................. 107
Pradhan Mantri Krishi Sinchayee Yojana (PMKSY) ........................................................................ 107
August 18, 2017 .............................................................................................................................................. 108
Veteran photographer S Paul passes away ........................................................................................ 108
About S Paul ............................................................................................................................................ 108
Veteran photographer S Paul passes away ........................................................................................ 108
About S Paul ............................................................................................................................................ 108
NHAI launches MyFASTag and FASTag Partner mobile apps .................................................... 108
Key Facts ................................................................................................................................................... 109
Background .............................................................................................................................................. 109
Electronic Toll Collection (ETC) ......................................................................................................... 109
FASTags .................................................................................................................................................... 109
Government launches Harit Diwali, Swasth Diwali Campaign .................................................... 109
Harit Diwali, Swasth Diwali Campaign .............................................................................................. 109
Background ............................................................................................................................................... 110
Defence Ministry clears proposal to procure 6 Apache attack helicopters for Army .............. 110
Decision taken by DAC .......................................................................................................................... 110
Navika Sagar Parikrama: Indian Navys all-woman team to circumnavigate globe .................. 110
Key Facts .................................................................................................................................................... 110
INSV Tarini ................................................................................................................................................ 111
Earlier Expeditions ..................................................................................................................................... 111
Scientists develops articial womb to save premature babies ......................................................... 111
Key Facts ..................................................................................................................................................... 111
Signicance ................................................................................................................................................. 111

2017 GKToday | All Rights Reserved | www.gktoday.in

11
Current Aairs: August, 2017 [Date-wise Compendium]

August 19, 2017 ................................................................................................................................................ 111


5 Indian-origin persons in Fortunes 2017 40 Under 40 list ....................................................... 111
Key Facts ..................................................................................................................................................... 111
RBI to issue new Rs 50 currency note ................................................................................................ 112
Key Features new Rs.50 note ................................................................................................................ 112
19 August: World Humanitarian Day .................................................................................................. 113
Background ............................................................................................................................................... 113
Climate change costs India $10 billion every year: Government ................................................... 113
Impact of Climate change on agriculture ........................................................................................... 113
NASA successfully launches Tracking and Data Relay Satellite-M .............................................. 114
TDRS-M .................................................................................................................................................... 114
Tracking and Data Relay Satellite System (TDRSS) ........................................................................ 114
India, US establish Two-By-Two Ministerial Dialogue ................................................................... 114
Key Facts .................................................................................................................................................... 115
Comment ................................................................................................................................................... 115
Two new species of Earthworm discovered in Kerala ..................................................................... 115
Key Facts .................................................................................................................................................... 115
August 21, 2017 ................................................................................................................................................ 115
China launches rst cyber court in e-commerce hub of Hangzhou ............................................. 115
Key Facts .................................................................................................................................................... 116
Defence Ministry approves delegation of powers to BRO .............................................................. 116
Need ............................................................................................................................................................ 116
Key Facts .................................................................................................................................................... 116
About Border Roads Organisation (BRO) .......................................................................................... 117
Indias rst calf born to surrogate cow in Pune ................................................................................ 117
Key Facts .................................................................................................................................................... 117
In Vitro Fertilisation (IVF) technology ................................................................................................. 117
Scientists discover new state of matter ............................................................................................... 117
Key Facts .................................................................................................................................................... 117
Superconductivity ..................................................................................................................................... 118
Versius: worlds smallest surgical robot developed by UK scientists ............................................ 118
Key Facts .................................................................................................................................................... 118
Government imposes anti-dumping duty on tempered glass from China .................................. 118
Textured toughened (tempered) glass ................................................................................................ 118
Background ............................................................................................................................................... 118
Anti-dumping duty .................................................................................................................................. 118
Anti-dumping duty on Chinese Products ........................................................................................... 119
Indian Navys second LCU Mark IV L52 ship commissioned at Port Blair ................................. 119
Key Facts .................................................................................................................................................... 119
Government launches digital police portal under CCTNS ............................................................. 119
Digital police portal ................................................................................................................................. 119

2017 GKToday | All Rights Reserved | www.gktoday.in

12
Current Aairs: August, 2017 [Date-wise Compendium]

About Crime and Criminal Tracking Network and Systems Project (CCTNS) ....................... 120
August 22, 2017 .............................................................................................................................................. 120
Karnataka Government launches clouding seeding Project Varshadhari .................................... 120
Project Varshadhari ................................................................................................................................. 120
Cloud seeding ........................................................................................................................................... 120
Scientists discover new therapy to prevent sepsis in new born babies ........................................ 120
Need ........................................................................................................................................................... 120
New prevention technique ..................................................................................................................... 121
Sepsis ........................................................................................................................................................... 121
Swasth Bachche, Swasth Bharat Programme launched ................................................................... 121
About Swasth Bachche, Swasth Bharat programme ........................................................................ 121
ICMR, Health Ministry ink MoU with IVI for vaccine R&D ........................................................... 121
Background ............................................................................................................................................... 122
International Vaccine Institute (IVI) ..................................................................................................... 122
Indian Council of Medical Research (ICMR) .................................................................................... 122
ISA extends Indias exclusive rights to explore Polymetallic Nodules .......................................... 122
Background ............................................................................................................................................... 122
Polymetallic nodules ................................................................................................................................ 122
International Seabed Authority (ISA) .................................................................................................. 123
17th meeting of FSDC held in New Delhi ......................................................................................... 123
Key Highlights of Meeting ..................................................................................................................... 123
About Financial Stability and Development Council (FSDC) ........................................................ 123
Government announces National Sports Awards 2017 ................................................................. 124
2017 Rajiv Gandhi Khel Ratna Award ................................................................................................ 124
2017 Dronacharya Award ..................................................................................................................... 124
2017 Arjuna Award ................................................................................................................................ 124
2017 Dhyan Chand Award ................................................................................................................... 124
TRAI ink LoI with Malaysian Communications and Multimedia Commission .......................... 125
Key Facts ................................................................................................................................................... 125
Telecom Regulatory Authority of India (TRAI) ............................................................................... 125
August 23, 2017 .............................................................................................................................................. 125
Former Manipur CM Rishang Keishing passes away ...................................................................... 125
About Rishang Keishing ......................................................................................................................... 125
SC strikes down instant Triple Talaq .................................................................................................. 125
SC Judgement .......................................................................................................................................... 126
Background .............................................................................................................................................. 126
Nasikabatrachus bhupathi: New frog species with pig face discovered ...................................... 127
Nasikabatrachus bhupathi ..................................................................................................................... 127
Signicance of Discovery ....................................................................................................................... 127
NITI Aayog launches Mentor India Campaign ................................................................................ 127
Atal Tinkering Labs ................................................................................................................................ 127

2017 GKToday | All Rights Reserved | www.gktoday.in

13
Current Aairs: August, 2017 [Date-wise Compendium]

Atal Innovation Mission ......................................................................................................................... 127


NITI Aayog ............................................................................................................................................... 128
Cabinet approves commission on sub-categorisation of OBCs .................................................... 128
Terms of references of Commission ................................................................................................... 128
Background .............................................................................................................................................. 128
Cabinet approves India-Nepal MoU on Prevention of Illicit Trafcking in Narcotic Drugs
.............................................................................................................................................................. 128
Features of MoU ...................................................................................................................................... 129
Background .............................................................................................................................................. 129
CCEA approves closure of Bharat Wagon and Engineering Company ....................................... 129
Background .............................................................................................................................................. 129
Bharat Wagon and Engineering Company Limited (BWEL) ......................................................... 129
Cabinet approves alternative mechanism Framework for consolidation of Public Sector Banks
.............................................................................................................................................................. 129
Features of approved Framework for Consolidation of PSBs ....................................................... 130
Benets ...................................................................................................................................................... 130
Background .............................................................................................................................................. 130
August 24, 2017 .............................................................................................................................................. 130
Union Cabinet approves new Bridge over Mechi River at India-Nepal border ......................... 130
Key facts .................................................................................................................................................... 130
Government to establish rst National Sports Museum in New Delhi ........................................ 131
Key Facts .................................................................................................................................................... 131
Government noties new Rs.200 notes .............................................................................................. 131
Key Facts .................................................................................................................................................... 131
Comment ................................................................................................................................................... 132
Printing of currency notes ..................................................................................................................... 132
Odisha Government launches rooftop solar project ........................................................................ 132
Key Facts ................................................................................................................................................... 132
Karnataka ties up with Microsoft to develop Indias rst Farm Price Forecasting Model ...... 132
Key Facts ................................................................................................................................................... 132
Signicance ............................................................................................................................................... 133
CCEA approves renaming SAMPADA Scheme as PM Kisan Sampada Yojana ........................ 133
Pradhan Mantri Kisan Sampada Yojana (PMKSY) .......................................................................... 133
Financial Allocation ................................................................................................................................. 133
Signicance ............................................................................................................................................... 133
Supreme Court declares right to privacy as Fundamental right under constitution ................ 134
Supreme Court Judgment ..................................................................................................................... 134
Comment .................................................................................................................................................. 134
India, Nepal sign 8 MoUs ...................................................................................................................... 134
Signed MoUs are ..................................................................................................................................... 134
August 25, 2017 .............................................................................................................................................. 135

2017 GKToday | All Rights Reserved | www.gktoday.in

14
Current Aairs: August, 2017 [Date-wise Compendium]

Government appoints Ashwani Lohani as New Chairman of Railway Board ............................ 135
About Ashwani Lohani ........................................................................................................................... 135
Railway Board ........................................................................................................................................... 135
Microsoft launches Project Brainwave for real-time Articial Intelligence ................................. 135
Key facts .................................................................................................................................................... 135
Articial Intelligence (AI) ...................................................................................................................... 136
Moss serves as a cheap pollution monitor: Scientists ...................................................................... 136
Key Facts ................................................................................................................................................... 136
Government eases norms under UDAN Scheme ............................................................................ 136
Key Facts ................................................................................................................................................... 136
About Ude Desh Ka Aam Nagrik (UDAN) Scheme ....................................................................... 136
August 26, 2017 .............................................................................................................................................. 137
GM crops only after biosafety, socio-economic evaluation: Parliamentary Committee .......... 137
Report Highlights .................................................................................................................................... 137
Background .............................................................................................................................................. 137
Government constitutes task force on articial intelligence .......................................................... 137
Key Facts ................................................................................................................................................... 137
Background .............................................................................................................................................. 138
Government bans import of gold, silver items from South Korea ............................................... 138
Sudden surge ............................................................................................................................................ 138
Background .............................................................................................................................................. 138
Directorate General of Foreign Trade (DGFT) ............................................................................... 138
NPCI approves Spice Digitals operation under Bharat Bill Payment System ............................. 138
Need ........................................................................................................................................................... 138
Bharat Bill Payment System (BBPS) .................................................................................................... 139
National Payments Corporation of India (NPCI) ............................................................................. 139
CSIR-NIIST develop new technique to produce bioethanol from cotton-stalks ...................... 139
New Technique ....................................................................................................................................... 139
Signicance ............................................................................................................................................... 139
IIT Researchers develop biosensor to detect kidney disorders ..................................................... 140
Drawbacks of conventional tests ......................................................................................................... 140
Biosensor Tests ........................................................................................................................................ 140
Signicance ............................................................................................................................................... 140
August 27, 28, 2017 ...................................................................................................................................... 140
PV Sindhu wins silver medal at 2017 World Badminton Championship .................................... 140
PV Sindhu ................................................................................................................................................. 140
Indias achievements at World Badminton Championship .............................................................. 141
Government imposes anti-dumping duty on sodium nitrate imports from China .................... 141
Sodium Nitrite .......................................................................................................................................... 141
Anti-dumping Duty ................................................................................................................................. 141
Government noties Banking Regulation (Amendment) Act, 2017 ............................................ 141

2017 GKToday | All Rights Reserved | www.gktoday.in

15
Current Aairs: August, 2017 [Date-wise Compendium]

Key Facts .................................................................................................................................................... 141


Background .............................................................................................................................................. 142
DRDO, IAI to produce MRSAM missile system by 2020 for Indian Army .............................. 142
Key Facts ................................................................................................................................................... 142
Background .............................................................................................................................................. 142
Government inaugurates Indias rst Videsh Bhavan in Mumbai ................................................. 142
Key Facts ................................................................................................................................................... 142
Justice Dipak Misra sworn in as 45th Chief Justice of India ......................................................... 143
About Justice Dipak Misra .................................................................................................................... 143
ICOSIS unveils Ocean forecasting system for Comoros, Madagascar and Mozambique ....... 143
Key Facts ................................................................................................................................................... 143
Background .............................................................................................................................................. 144
Indian National Centre for Ocean Information Services (INCOIS) ............................................. 144
August 28, 2017 ............................................................................................................................................. 144
ICEX launches worlds rst diamond futures contracts ................................................................. 144
Key Facts ................................................................................................................................................... 145
Indian Commodity Exchange (ICEX) ................................................................................................. 145
August 29: International Day Against Nuclear Tests ...................................................................... 145
International Day Against Nuclear Tests ........................................................................................... 145
Semipalatinsk Nuclear Test site ........................................................................................................... 145
August 29: National Sports Day .......................................................................................................... 145
About Dhyan Chand .............................................................................................................................. 146
Government launches National Sports Talent Search Portal ........................................................ 146
Key Facts ................................................................................................................................................... 146
India, Germany ink agreement to improve grid integration of renewable energy ................... 146
Indo-German Energy ProgrammeGreen Energy Corridors (IGEN-GEC) .............................. 146
Green Energy Corridor Project ............................................................................................................ 147
Need for synchronisation ...................................................................................................................... 147
Government to launch Deep Ocean Mission .................................................................................... 147
Need ........................................................................................................................................................... 147
Deep Ocean Mission .............................................................................................................................. 147
West Bengals Gobindobhog rice gets geographical indication status ........................................ 147
Gobindobhog rice ................................................................................................................................... 147
Signicance ............................................................................................................................................... 148
About Geographical Indication (GI) ................................................................................................... 148
IAEA opens worlds rst low Enriched Uranium bank in Kazakhstan ........................................ 148
Key Facts ................................................................................................................................................... 148
Purpose of Bank ...................................................................................................................................... 149
International Atomic Energy Agency (IAEA) ................................................................................... 149
August 30, 2017 ............................................................................................................................................. 149
Government clears 100% strategic sale of CEL ............................................................................... 149

2017 GKToday | All Rights Reserved | www.gktoday.in

16
Current Aairs: August, 2017 [Date-wise Compendium]

Alternative mechanism ........................................................................................................................... 149


Strategic disinvestment .......................................................................................................................... 149
Delhi Police launches YUVA a skill development programme .................................................. 150
Key Facts ................................................................................................................................................... 150
Pradhan Mantri Kaushal Vikas Yojana (PMKVY) ............................................................................ 150
Government to release new industrial policy .................................................................................... 150
Features of New policy .......................................................................................................................... 150
NITI Aayog proposes Judicial Performance Index for lower judiciary ......................................... 151
Judicial performance index (JPI) ........................................................................................................... 151
Other recommendations ........................................................................................................................ 152
Cabinet approves MoU on India-Israel Industrial R&D and Technological Innovation Fund
.............................................................................................................................................................. 152
Cabinet approves promulgation of GST (Compensation to States) Ordinance, 2017 ............ 153
Key Facts ................................................................................................................................................... 153
Background ............................................................................................................................................... 153
Cabinet approves MoU between India-Brazil for cooperation in Zebu Cattle Genomics and
ARTs ................................................................................................................................................... 153
Key Facts ................................................................................................................................................... 153
Zebu Cattle ............................................................................................................................................... 154
IIT Researchers develops bioarticial implantable pancreas .......................................................... 154
Key Facts ................................................................................................................................................... 154
August 31, 2017 ............................................................................................................................................... 154
India-Myanmar to sign MoU for conservation of quake-damaged Pagodas at Bagan ............ 154
Signicance of Project ............................................................................................................................ 155
Konkani writer Mahabaleshwar Sail receives 2016 Saraswati Samman ..................................... 155
Mahabaleshwar Sail ................................................................................................................................. 155
Saraswati Samman .................................................................................................................................. 155
CCS approves rst phase of Army reforms ....................................................................................... 155
The rst phase of reforms involves ..................................................................................................... 156
Background .............................................................................................................................................. 156
US approves rst gene therapy for cancer ........................................................................................ 156
Key Facts ................................................................................................................................................... 156
CAR-T treatment .................................................................................................................................... 157
EU, India agrees to enhance counter-terrorism cooperation ......................................................... 157
Key Highlights 10th Dialogue ............................................................................................................... 157
Financial Action Task Force (FATF) .................................................................................................. 157
India, Switzerland ink two MoUs in the eld of Railways .............................................................. 157
Signed Pacts .............................................................................................................................................. 158
Key Takeaways of Joint press statement ............................................................................................ 158
India-Switzerland ..................................................................................................................................... 158
Launch of navigation satellite IRNSS-1H unsuccessful: ISRO ........................................................ 158

2017 GKToday | All Rights Reserved | www.gktoday.in

17
Current Aairs: August, 2017 [Date-wise Compendium]

IRNSS-1H .................................................................................................................................................. 158


IRNSS ......................................................................................................................................................... 158
Compendium of Multiple Choice Questions ............................................................................................. 159

2017 GKToday | All Rights Reserved | www.gktoday.in

18
Current Aairs: August, 2017 [Date-wise Compendium]

Month Highlights: July, 2017


Government Policy and Schemes
B N Srikrishna Committee
The Union Ministry of Electronics & Information Technology (MEITY) has constituted an
expert Committee to study and identify key data protection issues and recommend methods
for addressing them. The ten-member committee will be headed by Supreme Court Judge
(retired) Justice B N Srikrishna and comprise of members from government, academia and
Industry.
e-Rashtriya Kisan Agri Mandi
The Union Government has launched e-Rashtriya Kisan Agri Mandi (e-RaKAM) portal to
provide a platform for farmers to sell agricultural produce. e-RaKAM is a first of its kind
initiative that leverages technology to connect farmers from the smallest villages to the
biggest markets of the world through internet.
Genetic mutation repaired for rst time
Scientists for the first time have successfully repaired a genetic mutation in human embryos
by using a gene-editing tool called CRISPR-Cas9. It has freed embryos of faulty DNA that
causes deadly hereditary heart disease. It potentially opens the door to preventing 10,000
disorders that are passed down the generations.
BRICS Agriculture Research Platform
The Union Cabinet has approved for a Memorandum of Understanding (MoU) signed
among India and various BRICs countries for establishment of the BRICS Agriculture
Research Platform (BRICS-ARP).
Proxy voting to NRIs
The Union Cabinet has cleared a proposal to extend proxy voting to Non-Resident Indians
(NRIs) and overseas Indians. Currently, only service personnel are permitted to vote through
proxy.
MahaDBT and MahaVASTU Portal
Maharashtra Government has launched two online portals MahaDBT and MahaVASTU to
streamline the process of direct benefit transfer (DBT) and to bring in more transparency in
construction sector.
14 New AIIMS
The Union Government has proposed to establish an additional 14 new AIIMS (All India
Institute of Medical Science) under various phases
of Pradhan Mantri Swasthya Suraksha Yojana (PMSSY) in different parts of the country.
AGRI-UDAAN Food and Agribusiness Accelerator 2.0 programme
The Union Government is going to launch AGRI-UDAAN Food and Agribusiness

2017 GKToday | All Rights Reserved | www.gktoday.in

3
Current Aairs: August, 2017 [Date-wise Compendium]

Accelerator 2.0 programme in an attempt to promote innovation and entrepreneurship in


agriculture. It will mentor startups and help them connect with potential investors.
BN Srikrishna Committee
A High Level Committee set up to review the Institutionalization of Arbitration Mechanism
in India has submitted its report to the Union Law Ministry. It was headed by Justice (retired)
BN Srikrishna.
Gujarat Rural Roads Project (GRRP)
India has signed a loan agreement with China led-Asian Infrastructure Investment Bank
(AIIB) for financing of $329 million for Gujarat Rural Roads Project (GRRP).
2nd Phase of measles-rubella (MR) vaccination
Union Ministry of Health and Family Welfare launched the second phase of measles-rubella
(MR) vaccination campaign to reduce measles morbidity and mortality. The second phase of
campaign will cover 8 states.
Tele-Law scheme
The Union Ministry of Law & Justice in association with the National Legal Service
Authority (NALSA) launched Tele-Law scheme in Bihar. The scheme aims at providing
legal aid services to marginalised communities and citizens living in rural areas through
digital technology.
NCRB merged with BPRD
The Union Home Ministry has merged three-decade old National Crime Records Bureau
(NCRB) with Bureau of Police Research and Development (BPRD). The merger aims at
improving administrative efficiency and optimal utilisation of resources for development
works related to policing. It will also boost crime data collection and research efforts.
Committee to make drugs more affordable
The Union Ministry of Chemicals and Fertilisers has constituted a committee of joint
secretaries for ensuring enhanced affordability, availability and accessibility of drugs for the
citizens. The committee will suggest ways to make pricing policy in favour of poor patients.
Green Climate Fund (GCF)
The 2030 Water Resources Group (2030WRG) has decided to help the Maharashtra
Government to raise $270 million from the Green Climate Fund (GCF). The amount raised
will be invested in state governments integrated watershed programmes such
as Jalyukt Shivar Yojana which has successfully augmented the ground water in drought
affected areas of the state.
Sustainable Action for Transforming Human Capital (SATH)
The NITI (National Institution for Transforming India) Aayog has announced partnership
with three states each to radically transform their Health and Education sectors under its

2017 GKToday | All Rights Reserved | www.gktoday.in

4
Current Aairs: August, 2017 [Date-wise Compendium]

Sustainable Action for Transforming Human Capital (SATH) initiative.


Directorate General of Supplies and Disposals (DGS&D)
The Union Ministry of Commerce and Industry has started the process of shutting down
Directorate General of Supplies and Disposals (DGS&D), over 100-year-old government
procurement arm, in this fiscal. Now the public procurement of goods and services will be
managed by the Governments e-market platform (GeM).
Logistics Data Bank (LDB) project
Union Ministry of Shipping is planning to expand Logistics Data Bank (LDB) project to the
countrys southern region. So far, it had covered only the western logistics corridor. The LDB
project was launched at the Jawaharlal Nehru Port (JNPT), Mumbai in July 2016, making it
first port in India to provide this service.
Swachh Survekshan Gramin 2017
The Union Ministry of Drinking Water and Sanitation has
released Swachh Survekshan Gramin 2017, third party survey report to assess the
progress Swachh Bharat Mission in rural India. The survey was undertaken by the Quality
Council of India (QCI) between May and June 2017 to take into account status of rural
sanitation in all States and UTs.
Namami Gange Jagriti Yatra
The Uttar Pradesh Government has launched Namami Gange Jagriti Yatra, an awareness
drive along banks of Ganga River to maintain cleanliness. The campaign was flagged off by
Chief Minister Yogi Adityanath from the state capital in Lucknow.
Aajeevika Grameen Express Yojana
The Union Ministry of Rural Development is going to
launch Aajeevika Grameen Express Yojana (AGEY), a sub-scheme
under Deendayal Antyodaya Yojana National Rural Livelihoods Mission (DAY-NRLM).
National Cyber Coordination Centre (NCCC)
The Union Ministry of Electronics and Information Technology (MEITY) has announced
that the first phase of National Cyber Coordination Centre (NCCC) has become operational.
NCCC has been set up to scan the countrys web traffic to detect cyber security threats.
SheMeansBusiness programme
Odisha government and social networking giant Facebook has launched SheMeansBusiness
programme to train women entrepreneurs in the state. It was launched in partnership
between Odishas MSME (micro, small and medium enterprises) department and Project
Mission Shakti.
First online course on Infusion Therapy
The Infusion Nurses Society (INS)-India in its 6th National conference launched countrys

2017 GKToday | All Rights Reserved | www.gktoday.in

5
Current Aairs: August, 2017 [Date-wise Compendium]

first online course on Infusion Therapy for nurses. INS is global authority in infusion
therapy. Under the online course, the INS aims to train over 3000 nurses in the vital medical
procedure.
GeMSamvad
The Public procurement portal Government eMarketplace (GeM) and industry body
Confederation of Indian Industry (CII) have signed a MoU to establish GeMSamvad, a GeM -
Industry forum. GeMSamvad aims to create awareness about online purchase of goods and
services by government agencies and departments.
New Metro Policy
The Union Cabinet has approved a new policy for expanding and regulating metro rail
services in cities across India. This is the first such policy document prepared by the Union
Urban Development Ministry since metro rail operations began in Delhi in 2002.The policy
gives a big boost to private players by making private participation mandatory for all the
three funding options.
Alternative mechanism for strategic disinvestment
The Cabinet Committee on Economic Affairs (CCEA) chaired by the Prime
Minister Narendra Modi has approved revision in the procedure and mechanism for strategic
disinvestment.
Madhyamik and Uchchtar Shiksha Kosh (MUSK)
The Union Cabinet has approved creation of a single corpus fund non-lapsable known
as Madhyamik and Uchchtar Shiksha Kosh (MUSK). Under it, all proceeds of secondary and
higher education cess will be credited and utilised for the governments schemes in education
sector.
Orthopaedic knee implants
The drug price regulator National Pharmaceuticals Pricing Authority (NPPA) has capped the
prices of orthopaedic knee implants upto 70% lower than current market rates. It will be valid
for all orthopaedic knee implants sold within India whether domestic or imported.
Long Term Irrigation Fund (LTIF)
The Union Cabinet gave its approval for raising Extra Budgetary Resources of up to Rs, 9,020
crore for Long Term Irrigation Fund (LTIF) during the financial year 2017-18.The funds will
be raised by the National Bank for Agriculture and Rural Development (NABARD) through
the issuance of Bonds at 6% per annum as per requirement.
North Koel Reservoir Project
The Union Cabinet approved proposal to complete the balance works of
North Koel Reservoir Project in Bihar, stalled since 1993. It also revived the project to address
environmental concerns.
2017 GKToday | All Rights Reserved | www.gktoday.in

6
Current Aairs: August, 2017 [Date-wise Compendium]

MyFASTag and FASTag Partner mobile Apps


The National Highways Authority of India (NHAI) launched two mobile Apps
MyFASTag and FASTag Partner in to facilitate the availability of FASTags for Electronic
Toll Collection (ETC) project. The mobile apps will ease the process, making it possible to
buy or recharge FASTags easier using mobile button.
New Rs.50 Note
The Reserve Bank of India (RBI) will shortly issue new banknotes of Rs.50 denomination in
the Mahatma Gandhi (New) Series. The base colour of the new notes will be fluorescent
blue. RBI also has clarified that all banknotes in Rs.50 denomination issued in the earlier
series will continue to be legal tender.
Two-by-Two Ministerial Dialogue
India and US have established new two-by-two (2 by 2) ministerial dialogue to enhance
strategic coordination between them and maintaining peace and stability in the Indo-Pacific
region. The new dialogue format will replace the earlier India-US Strategic and Commercial
Dialogue. It will be similar to the India-Japan 2+2 dialogue format between foreign and
defence secretaries of the two countries.
Digital Police Portal
The Union Ministry of Home Affairs launched digital police portal under Crime and
Criminal Tracking Network and Systems Project (CCTNS).The portal is a smart policing
initiative of the Government to provide services to citizens and aid efficient police
investigation.
Project Varshadhari
Karnataka Government has launched clouding seeding Project Varshadhari in Bengaluru
under which an aircraft will spray chemicals to induce rainfall. The cloud seeding project is an
experimental project that will cover Bengaluru, Gadag and Yadgir districts of the state
Swasth Bachche, Swasth Bharat Programme
The Union Ministry of Human Resource Development (HRD) launched
Swasth Bachche, Swasth Bharat Programme. The programme is an initiative
of Kendriya Vidyalaya Sangathan (KVS) to prepare a physical Health and Fitness Profile Card
for more than 12 lakhs of Kendriya Vidyalaya students.
Mentor India Campaign
The National Institution for Transforming India (NITI) Aayog has launched Mentor India
Campaign, a strategic nation building initiative to engage leaders for guiding and mentoring
students at Atal Tinkering Labs.
Pradhan Mantri Kisan Sampada Yojana
The Cabinet Committee on Economic Affairs (CCEA) has approved renaming of SAMPADA

2017 GKToday | All Rights Reserved | www.gktoday.in

7
Current Aairs: August, 2017 [Date-wise Compendium]

(Scheme for Agro-Marine Processing and Development of Agro-Processing Clusters)


as Pradhan Mantri Kisan Sampada Yojana (PMKSY).
Indias rst Farm Price Forecasting Model
Karnataka Government has signed a Memorandum of Understanding (MoU) with software
giant Microsoft India to develop a unique farm price forecasting model. The proposed
initiative is claimed to be the first-of-its-kind in the country.
First Videsh Bhavan
The Union Ministry of External Affairs (MEA) launched countrys first Videsh Bhavan by
assimilating all Regional Passport Offices and allied departments under a single roof at
the Bandra Kurla Complex (BKC) in Mumbai, Maharashtra.
National Sports Talent Search Portal
The Union Ministry of Youth Affairs and Sports has launched National Sports Talent Search
Portal to unearth sporting talent from every nook and corner of the country. The portal will
be also available as smartphone application.
YUVA
The Delhi Police has launched YUVA a skill development programme
under Pradhan Mantri Kaushal Vikas Yojana (PMKVY). It aims to connect with youth by
upgrading their skill as per their competencies.
Deep Ocean Mission
The Union Ministry of Earth Sciences (MoES) is going to launch Deep Ocean Mission by
January 2018 to spur research activities in ocean science and develop technology to harness
ocean resources. The proposed inter-ministerial and interdisciplinary project aims to explore
and harness mineral resources beneath the ocean floor.
Persons in news Appointments/Elections
Shahid Khaqan Abbasi
Pakistans Petroleum Minister Shahid Khaqan Abbasi was elected the 18th Prime Minister of
Pakistan by National Assembly. Abbasi, nominee of ruling Pakistan Muslim League (N)
secured 221 votes in House of 342. Abbasi succeeds Nawaz Sharif who was disqualified by the
Supreme Court over Panama Papers scandal.
Arvind Panagariya
Arvind Panagariya (64) has resigned as Vice Chairman of NITI (National Institution for
Transforming India) Aayog and announced to return to academics in US.
Rajiv Kumar
The Union Government has appointed economist Rajiv Kumar as the new vice-chairman of
National Institution for Transforming India (NITI) Aayog. Rajiv Kumar will
succeed Arvind Panagariya. As Vice Chairman of NITI Aayog he will hold cabinet minister
rank.
2017 GKToday | All Rights Reserved | www.gktoday.in

8
Current Aairs: August, 2017 [Date-wise Compendium]

Muppavarapu Venkaiah Naidu


Former union minister Muppavarapu Venkaiah Naidu (68) was elected as 13th Vice President
of India. He will succeed Hamid Ansari, who held the position for two consecutive terms of 5
years.
S. Aparna
The Appointments Committee of the Cabinet (ACC) has appointed senior IAS officer from
Gujarat S. Aparna as Executive Director of the World Bank. At the World Bank, she will
represent the constituency of India, Bangladesh, Bhutan and Sri Lanka for three years.
Justice Dipak Misra
President Ram Nath Kovind has appointed Justice Dipak Misra (63), senior-most judge of the
Supreme Court as the next Chief Justice of India. He will be 45th CJI and succeeds Justice
JS Khehar.
Prasoon Joshi
The Union Information and Broadcasting Ministry has appointed Prasoon Joshi as the new
Chairman of Central Board of Film Certification (CBFC). He will replace Pahlaj Nihalani and
shall have tenure of three years.
Ashwani Lohani
The Union Government has appointed Ashwani Lohani as the new Chairman of the Railway
Board (Ministry of Railways). Prior to this appointment, he was serving as Chairman and
Managing Director (CMD) of Air India.
Science, Technology & Environment
SC bans use of heavy metals in recrackers
The Supreme Court has imposed ban on use of five harmful heavy metals like lithium,
antimony, mercury, arsenic and lead in the manufacturing of firecrackers as they cause air
pollution.
Curcumin: bioactive component of turmeric
Scientists from United States have found that curcumin, the bioactive component of turmeric
that is widely used in Indian cuisine can treat cancer in children. They have found that
nanoparticles loaded with curcumin can target and destroy neuroblastoma tumour cells, which
commonly affects children aged five or less.
Venus Satellite
Israel has launched its first environmental research satellite named as Venus (Vegetation and
Environment Monitoring New Micro-Satellite). It is a joint venture between the Israel Space
Agency (ISA) and its French counterpart CNES.
Suns core rotates 4 times faster than its surface: Study
A team of researchers have discovered solar seismic waves which revealed that Suns core is
rotating four times faster than its surface. Earlier it was assumed that suns core rotate at
2017 GKToday | All Rights Reserved | www.gktoday.in

9
Current Aairs: August, 2017 [Date-wise Compendium]

same speed as the surface.


NASA-ISRO Synthetic Aperture Radar (NISAR) Satellite
ISRO and NASA are jointly working on the NASA-ISRO Synthetic Aperture Radar (NISAR)
mission to co-develop and launch a dual frequency synthetic aperture radar (SAR) satellite by
2021. The satellite will be the worlds most expensive earth-imaging satellite till date, costing
around $1.5 billion. It aims to study global environmental change and natural disasters.
Time and frequency traceability services
The ISRO Telemetry Tracking and Command Network (ISTRAC) and CSIR-National
Physical Laboratory (NPL) have signed MoU to make indigenous regional positioning
system-NavIC independent from the US clock system.
Nanotechnology-based drug delivery system
An all-women team of researchers from Indian Institute of Technology (IIT) Delhi has
developed a new drug delivery platform using nanoparticles. The nanotechnology-based drug
delivery system has ability to boost the efficacy of antibiotics at the cellular level and improve
chances of recovery from cancer-related bacterial infections.
WASP-121b exoplanet
Astronomers using NASAs Hubble Space Telescope have found that exoplanet WASP-121b
which provides strongest evidence about existence of a stratosphere on it along with glowing
water molecules. The stratosphere is a layer of atmosphere in which temperature increases
with higher altitudes.
Two new species of Cycas discovered
Research conducted on Cycas pschannae, a lone tree found in the Acharya Jagadish Chandra
Bose Indian Botanic Garden, West Bengal has revealed two new species of Cycas to the
world. This discovery takes the total number of Cycas species found in India to 14.
Low Cost nanometre-scale carbon monoxide (CO) sensor
Indian Institute of Science (IISc) researchers from Bengaluru have developed a highly
sensitive, low cost nanometre-scale carbon monoxide (CO) sensor, with potential
applications in environmental pollution monitoring. The sensor was developed using novel
fabrication technique that does not involve costly and time consuming lithography
technology.
2nd commitment period of Kyoto Protocol
India has ratified the second commitment period of the Kyoto Protocol (or Doha
Amendment) that commits countries to contain the emission of greenhouse gases (GHGs). In
this regard, India deposited its Instrument of Acceptance of the Doha Amendment to the
Kyoto Protocol. With this, India became the 80th country to accept the amendment relating
to the second commitment period (2013- 2020) of the Kyoto Protocol.
2017 GKToday | All Rights Reserved | www.gktoday.in

10
Current Aairs: August, 2017 [Date-wise Compendium]

Hyperspectral Imaging Satellite


The Indian Space Research Organisation (ISRO) is planning to launch a full-fledged niche
Earth observation (EO) satellite called the Hyperspectral Imaging Satellite (HySIS).
The HySIS satellite has critical chip called an optical imaging detector array indigenously
developed by ISRO.
NGT bans non-biodegradable plastic bags in NCR Delhi
The National Green Tribunal (NGT) imposed an interim ban on use of non-biodegradable
plastic bags which are less than 50 microns in the entire national capital Delhi.
Film-Evaporation MEMS Tunable Array (FEMTA) thruster
Engineers from Purdue University in US have designed and tested a micropropulsion system
called a Film-Evaporation MEMS Tunable Array (FEMTA) thruster that uses liquid water as
the propellant for orbital maneuvering of tiny satellites called CubeSats.
Gaj Yatra
The Union Ministry of Environment and Forest has launched a nationwide
campaign Gaj Yatra to protect elephants. It was launched on the occasion of World Elephant
Day observed on August 12.
DNA chip based sensor
Researchers form CSIR-Institute of Genomics and Integrative Biology (CSIR-IGIB) and
National Centre for Disease Control (NCDC) have developed an ultrasensitive DNA chip
based sensor for quick pathogen detection. The senor can detect S. pyogenes, a bacterium
which causes a wide range of diseases in about 30 minutes.
Worlds rst plant-based Zika vaccine
Scientists from US have developed the worlds first plant-based Zika vaccine that may be more
effective, safer and cheaper than other vaccines against the mosquito-borne Zika virus. The
vaccine was developed using proteins derived from Tobacco plant.
Worlds largest volcanic region on Earth
Researchers from the University of Edinburgh in Britain have discovered the largest volcanic
region on Earth, two km below the surface of the vast ice sheet in west Antarctica. They have
found 91 previously unknown active volcanoes in the region known as the West Antarctic
rift system, adding to the exiting 47 volcanoes that were discovered earlier. This makes it
largest volcanic region on the Earth.
Malpelo plate
Researchers from US-based Rice University have found new tectonic mircoplate off Ecuadors
coast in the eastern Pacific Ocean. It has been named Malpelo plate, after a Colombian island
and an oceanic ridge it contains. It is overall 57th tectonic mircoplate to be discovered so far
and the first in nearly a decade. Microplates are tectonic plates with an area less than 1 million

2017 GKToday | All Rights Reserved | www.gktoday.in

11
Current Aairs: August, 2017 [Date-wise Compendium]

km2.
Ecosystem Service Improvement Project
The Union Government has inked US $24.64 million Grant Agreement from the Global
Environment Facility (GEF) of the World Bank for Ecosystem Service Improvement Project.
The Project will be entirely financed by the World Bank out of its GEF Trust Fund. The
projects duration is of five years.
Harit Diwali, Swasth Diwali campaign
The Union Ministry of Environment, Forest and Climate Change (MoEFCC) has launched
Harit Diwali, Swasth Diwali campaign. The campaign aims to reduce adverse environmental
conditions especially pollution in country after post Diwali celebrations due to excessive
bursting of crackers which contributes significantly to air and noise pollution.
Articial womb developed
Scientists have developed an artificial womb that can be used in future to save the lives of
extremely premature human babies. It was successfully used to incubate healthy baby lambs
for a week. The research showed that preterm lambs were successfully maintained in a
healthy, infection-free condition with significant growth, for a period of one week using ex-
vivo uterine environment (EVE) therapy.
Tracking and Data Relay Satellite-M (TDRS-M)
The National Aeronautics and Space Administration (NSAS) has successfully launched
Tracking and Data Relay Satellite-M (TDRS-M) into orbit. It was launched aboard a United
Launch Alliance (ULA) Atlas V rocket. TDRS-M is third and final satellite in a series of the
Tracking and Data Relay Satellite System (TDRSS), next generation communications
satellites.
New State of Matter
Scientists Los Alamos National Laboratory, US have discovered a potential new state of
matter that may help explain phenomena like superconductivity. It was discovered in the
high-magnetic-field state of the heavy fermion superconductor CeRhIn5.
Versius: worlds smallest surgical robot
Scientists in the United Kingdom (UK) have developed the worlds smallest surgical robot
called Versius. It was developed using low-cost technology used in mobile phones and space
industries. The robot can mimic human arm and can be used to carry out a wide range of
surgical procedures. It can be controlled by a surgeon using a console guided by a 3D screen
in the operating theatre.
Project Brainwave
Software giant Microsoft has launched Project Brainwave, a deep learning acceleration
platform for real-time artificial intelligence (AI).
2017 GKToday | All Rights Reserved | www.gktoday.in

12
Current Aairs: August, 2017 [Date-wise Compendium]

Worlds rst low Enriched Uranium bank


The UN global nuclear watchdog, International Atomic Energy Agency (IAEA) has opened
worlds first low Enriched Uranium (LEU) Bank in Oskemen in Kazakhstan. The bank is
owned and managed by IAEA. It will be the first of its kind LEU bank not to be under control
of any individual country.
Indo-German Energy Programme Green Energy Corridors (IGEN-GEC)
India and Germany have signed an agreement on technical cooperation under the Indo-
German Energy Programme Green Energy Corridors (IGEN-GEC). The agreement was
signed between Indias Ministry of New and Renewable Energy (MNRE) and
Deutsche Gesellschaft fr Internationale Zusammenarbeit (GIZ) GmbH India on behalf of
Germany.
Defence Related
Humraaz mobile application
Indian Army has developed Humraaz mobile application through which serving soldiers can
track details like postings and promotions. The app will also enable soldiers to view their
monthly salary slips, Form 16 and also download them.
Indias rst private missile production facility
Indias first private sector missile sub-systems manufacturing facility Kalyani Rafael Advanced
Systems (KRAS) plant was inaugurated near Hyderabad, capital of Telangana. It is 51:49
joint venture between Indias Kalyani Group and Israels Rafael Advanced Defence Systems
Ltd.
ICGS Shaurya
Indian Coast Guard Ship (ICGS) Shaurya was commission in Vasco, Goa. ICGS Shaurya is an
offshore patrol vessel designed and built indigenously by Goa Shipyard Ltd (GSL). It will be
based at Chennai under administrative and operational control of Commander Coast Guard
Region (East).
Indra tri-services exercise
India and Russia for first time are going to conduct Indra tri-services exercise, involving their
armies, navies and the air forces in October 2017 to further ramp up military ties. It will be
for the first time India will participate in a tri-services exercise with a foreign country with
such large scale participation by Army, Navy and Air Force.
IMMSAREX 2017
Indian Ocean Naval Symposiums (IONS) maiden International Maritime Search and Rescue
Exercise (IMMSAREX) will be held in November 2017 in the Bay of Bengal under the
chairmanship of Bangladesh. The ships and aircraft of the members and observers of the
IONS including China, India and Pakistan will participate in the exercise.

2017 GKToday | All Rights Reserved | www.gktoday.in

13
Current Aairs: August, 2017 [Date-wise Compendium]

Navika Sagar Parikrama


A six-member all-woman team of the Indian Navy will set sail to circumnavigate the globe on
the sailing vessel INSV Tarini. This is the first-ever Indian circumnavigation of the globe by
an all-woman crew. The expedition has been aptly titled Navika Sagar Parikrama with an aim
to promote women empowerment in the country and ocean sailing by the Indian Navy.
AH-64-E Apache attack helicopters
The Union Defence Ministry has cleared a proposal to procure six AH-64-E Apache attack
helicopters along with weapons systems from American aerospace giant Boeing for the
Indian Army.
LCU Mark IV L52
Indian Navys second LCU (landing craft utility) Mark IV L52 ship was commissioned at Port
Blair, capital of Andaman and Nicobar Islands. LCU L52 is the second LCU Mk-IV class ship
to be inducted into the Indian Navy. It has been indigenously designed and built by Garden
Reach Shipbuilders and Engineers (GRSE), Kolkata.
MRSAM defence system
The DRDO has signed Rs.17,000 crore deal with Israel Aerospace Industries (IAI) for
producing advanced mediumrange surface to air missile (MRSAM) system for Indian Army
by 2020. The deal envisages develping 40 firing units and around 200 missiles.
Finance, Business and Economy
RBI bimonthly policy review
The Monetary Policy Committee (MPC) of the Reserve Bank of India (RBI) has reduced
short-term lending rate, or repo rate, by 25 basis points to 6%. It was RBIs third bimonthly
policy review for the financial year 2017-18.
T K Viswanathan Committee
Market regulator Securities and Exchange Board of India (SEBI) has set up a committee on
fair market conduct. It will be headed former law secretary T K Viswanathan. The
committee will suggest measures for improving surveillance of the markets and strengthen
rules for algorithm trades, among other norms.
Bharat 22
The Union Finance Ministry has launched second exchange-traded fund (ETF), named
Bharat 22. It will help to speed up Governments disinvestment programme budgeted to raise
a record Rs 72,500 crore in the FY 2018. Bharat 22 comprise of 22 stocks including those of
central public sector enterprises (CPSEs), public sector banks (PSBs) and its holdings under
the Specified Undertaking of Unit Trust of India (SUUTI).
Global Retirement Index (GRI)
India was ranked last at 43rd position in 2017 Global Retirement Index (GRI) published by
French asset management company Natixis Global.
2017 GKToday | All Rights Reserved | www.gktoday.in

14
Current Aairs: August, 2017 [Date-wise Compendium]

Indias rst gold options contract


Commodity derivatives bourse Multi Commodity Exchange Ltd (MCX) has received markets
regulator Securities and Exchange Board of Indias (SEBI) approval to launch Indias first gold
options contract.The gold futures contract will have bi-monthly duration. The option will
also have the existing gold kilo futures contract as its underlier.
Framework for Consolidation of PSBs
The Union Cabinet has approved Framework for Consolidation of Public Sector Banks
(PSBs) through an Alternative Mechanism (AM). The decision would facilitate consolidation
among the nationalised banks to create strong and competitive banks and also improve their
competitiveness and efficiency.
New Rs. 200 note notied
The Union Finance Ministry on the recommendations of the Central Board of Directors of
the Reserve Bank of India (RBI) has notified currency notes of Rs.200.
Worlds rst diamond futures contracts
The Indian Commodity Exchange (ICEX) as launched worlds first diamond futures contracts
to provide exporters with a hedging tool.
Events & Observances
7 August: National Handloom Day
The National Handloom Day is being observed every year on 7 August to honour the
handloom weavers in the country and also carter an impetus to Indias handloom industry.
This year it was third edition of the National Handloom Day after it was instituted in 2015.
75th anniversary of Quit India Movement
The 75th anniversary of the Quit India Movement was observed across the country. This
years theme was Sankalp se Siddhi- the attainment through resolve.The campaign pledges
to build new India by 2022.
August 10: International Biodiesel Day
The International Biodiesel Day (IBD) is celebrated every year on August 10 in a bid to create
awareness about non fossil-fuels (Green Fuels). The day also honours the research
experiments by Sir Rudolf Diesel who ran an engine with peanut oil in the year of 1893.
India-US Global Entrepreneurship Summit
India and US are going to co-host the Global Entrepreneurship Summit at
Hyderabad, Telangana from 28th to 30th November 2017. American delegation will be led by
US President Donald Trumps daughter Ivanka Trump.
15th BIMSTEC ministerial meeting
The 15th edition of Bay of Bengal Initiative for Multi-Sectoral Technical and Economic
Cooperation (BIMSTEC) ministerial meeting was held in Kathmandu, Nepal.

2017 GKToday | All Rights Reserved | www.gktoday.in

15
Current Aairs: August, 2017 [Date-wise Compendium]

12 August: International Youth Day


The International Youth Day is observed across the world on 12 August to draw attention to
a given set of cultural and legal issues surrounding youth. The theme for this year is Youth
Building Peace. It aims at celebrating young peoples contributions in preventing conflicts
and bringing about transformations along with social justice, inclusion and sustainable peace.
India-ASEAN Youth Summit
The first India-ASEAN Youth Summit was held at Bhopal, Madhya Pradesh to commemorate
25th anniversary of Association of Southeast Asian Nations (ASEAN)India dialogue
partnership in the ongoing year.
19 August: World Humanitarian Day
The World Humanitarian Day (WHD) is observed every year on 19 August to recognize
work of humanitarian personnel and those who have lost their lives. The campaign theme for
this year is #NotATarget.
August 29: National Sports Day
The National Sports Day is observed every year on 29th august every year to birth
anniversary of the legendary hockey player Dhyan Chand who was born on 29 August 1905.
August 29: International Day Against Nuclear Tests
The International Day Against Nuclear Tests was observed across the world on 29 August
with an aim to raise awareness about the effects of nuclear weapon test explosions or any
other nuclear explosions.
Sports
First National Sports Museum
The Union Ministry of Sports and Youth Affairs is going to establish National Sports
Museum at Jawaharlal Nehru Stadium, New Delhi. It will be first of its kind sports museum
in India. The aim of the sports museum is to popularise the sports as a way of life within the
country. It will showcase the Indias achievements in sports and also focus on the traditional
sports within the country.
PV Sindhu
Indian ace shuttler and 2017 Rio Olympic silver medallist PV Sindhu won the silver medal in
2017 World Badminton Championship held at Glasgow, Scotland (United Kingdom). In the
final match, Sindhu lost to Japans Nozomi Okuhara by 19-21, 22-20, 20-22 score.
Persons in news: Died
Ustad Hussain Sayeeduddin Dagar
Ustad Hussain Sayeeduddin Dagar, one of the foremost exponents and custodian of the
venerable Dhrupad tradition of Hindustani classical music passed away in a Pune.
Pushpa Mittra Bhargava
Veteran molecular biologist Pushpa Mittra Bhargava passed away in Hyderabad. He was
2017 GKToday | All Rights Reserved | www.gktoday.in

16
Current Aairs: August, 2017 [Date-wise Compendium]

89. Bhargava was the founding director of Hyderabad based Centre for Cellular and Molecular
Biology (CCMB).
S Paul
Veteran photographer S Paul passed away in New Delhi. Paul was a self-taught photographer
who discovered his passion for the art as a teenager. He joined The Indian Express in the
1960s and retired in 1989.
Rishang Keishing
Former Manipur Chief Minister and Veteran Congress leader Rishang Keishing passed away
in Imphal following a brief illness. He was freedom fighter and member of Indias first
parliament (1952-57). He had served as the Chief Minister of Manipur for four terms from
1980 to 1988 and 1994 to 1997.
Miscellaneous Terms and Facts
19th RCEP Trade Negotiating Committee meetings
The 19th round of the Regional Comprehensive Economic Partnership (RCEP) Trade
Negotiating Committee (TNC) meetings and other related meetings were held in
Hyderabad, Telangana.
India-Bhutan New Trade and Transit agreement
The new bilateral Agreement on Trade, Commerce and Transit between India and Bhutan
came into force from July 2017. The new agreement aims to further strengthen the bilateral
trade relations between both countries.
Right of Children to Free and Compulsory Education (Amendment) Bill, 2017
Parliament has passed the Right of Children to Free and Compulsory Education
(Amendment) Bill, 2017 with Rajya Sabha approving it. Lok Sabha had passed it earlier. The
Bill amends the Right of Children to Free and Compulsory Education Act (RTE), 2009 by
extending the deadline for teachers to acquire the prescribed minimum qualifications for
appointment.
Banking Regulation (Amendment) Bill, 2017
Parliament passed the Banking Regulation (Amendment) Bill, 2017 by voice vote. It will
replace the Banking Regulation (Amendment) Ordinance, 2017. The bill seeks to amend the
Banking Regulation Act, 1949 to insert provisions for handling cases related to stressed assets
or non-performing assets (NPAs) of banks.
NABARD (Amendment) Bill, 2017
The Lok Sabha has passed the National Bank for Agriculture and Rural Development
(Amendment) Bill, 2017 by voice vote. The Bill seeks to amend the NABARD Act,
1981. NABARD is responsible for providing and regulating facilities like credit for
agricultural and industrial development in the rural areas.

2017 GKToday | All Rights Reserved | www.gktoday.in

17
Current Aairs: August, 2017 [Date-wise Compendium]

Indian Institutes of Information Technology (Amendment) Bill, 2017


Parliament has passed the Indian Institutes of Information Technology (Amendment) Bill,
2017 after the Rajya Sabha approved it. The Lok Sabha has already passed it. The Bill amends
the IIIT Act, 2014. The IIIT Act, 2014 declares certain Institutes of Technology as institutions
of national importance.
State Banks (Repeal and Amendment) Bill, 2017
The Lok Sabha has passed the State Banks (Repeal and Amendment) Bill, 2017. The bill seeks
to repeal the two Acts namely State Bank of India (Subsidiary Banks) Act, 1959, and State
Bank of Hyderabad Act, 1956. It also seeks to amend the State Bank of India (SBI) Act, 1955 to
remove references to subsidiary banks and powers of SBI to act as an agent of the RBI for
subsidiary banks.
Hizbul Mujahideen
The United States has named Pakistan based Kashmiri terrorist outfit Hizbul Mujahideen as a
Foreign Terrorist Organisation (FTO). The designation slaps a series of US sanctions on the
outfit.
Melbourne
According to the 2017 Global Liveability Report compiled by the Economist Intelligence Unit
(EIU), Australian city Melbourne is the most liveable city in the world. The EIU Liveability
index scores 140 major cities of the world on the scale ranging from 0 (least liveable city) to
100 (most liveable city) based on healthcare, education, stability, culture, environment and
infrastructure parameters. No Indian city was ranked in the top ten or bottom ten.
Facts in News [FIN] August, 2017 for Banking, SSC, MBA, CLAT etc.
August 4, 2017
Mohammad Mustafa appointed as CMD of SIDBI
The Appointments Committee of the Cabinet (ACC) has appointed senior bureaucrat Mohammad Mustafa
as Chairman and Managing Director (CMD) of the Small Industries Development Bank of India (SIDBI) for
a period of three years. Mustafa is 1995 batch Indian Administrative Service (IAS) officer of Uttar Pradesh
cadre. Prior to this appointment, he was working as a Joint Secretary in Department of Financial Services.
Earlier he had served as the Chairman of the Board of National Housing Bank (2014-2015).
US Senate conrms three Indian-Americans to key government positions
The US Senate has unanimously confirmed three Indian-Americans to key government positions in the
Trump administration. Neil Chaterjee was confirmed as a member of the Federal Energy Regulatory
Commission (FERC). Vishal Amin was confirmed as Intellectual Property (IP) Enforcement Coordinator.
Krishna Urs was confirmed as the US ambassador to Peru. He is second Indian American after Indian
American who have been selected for an ambassadorial position by the administration.
Government disinvests 6.83% of equity in HCL
The Union Government has disinvested 6.83% of paid up equity in Hindustan Copper Limited (HCL)
2017 GKToday | All Rights Reserved | www.gktoday.in

18
Current Aairs: August, 2017 [Date-wise Compendium]

through Offer for Sale. Government is likely to get approximately Rs. 400 crore from this disinvestment.
HCL is a government-owned corporation in CPSU under the Union Ministry of Mines. It is the only
vertically integrated copper producer in India engaged in a wide spectrum of activities ranging from Mining,
Beneficiation, Smelting, Refining and Continuous Cast Rod manufacturer.
Aadhaar necessary for registration of death
The Registrar General of India (RGI) has made Aadhaar number necessary for registration of death from 1
October 2017. However it is not mandatory. This move aims at ensuring accuracy of the details provided by
the relatives and dependents. It will also serve as an effective method to prevent identity fraud and also help
recording the identity of the deceased person. Besides, it will also obviate the need for producing multiple
documents to prove the identity of the deceased person. The provisions will come into effect immediately
for residents of all States except Jammu and Kashmir, Assam and Meghalaya for which a date will be notified
separately.
Government renames Mughalsarai railway station after Deen Dayal Upadhyaya
The Union Home Ministry has approved a proposal to rename the Mughalsarai railway station in Uttar
Pradesh after BJP ideologue Deen Dayal Upadhyaya. Proposal in this regard was earlier was forwarded by
Uttar Pradesh government. Mughalsarai junction is located on the main Howrah-Delhi rail line. Deen Dayal
Upadhyaya was found dead under mysterious circumstances on this platform in 1968. Mughalsarai is also the
birthplace of former Prime Minister Lal Bahadur Shastri.
NFAI acquires 2500 lm posters of Indian Cinema
Pune based National Film Archive of India (NFAI) has acquired about 2500 film posters of Indian cinema. It
is one of the major acquisition in recent times. The collection includes about 1500 Hindi film posters, in
addition to a number of Tamil, Telugu, Kannada and Malayalam ones. These posters range from 1942
onwards till recent times. The collection showcases the evolution of film publicity covering various
techniques of poster-making.
August 5, 2017
Joshna-Dipika settle for bronze at World Doubles Squash Championship
The Indian pair of Joshna Chinappa and Dipika Pallikal Karthik have bronze medal at the WSF World
doubles squash championship held in Manchester, England. In the semifinals, they lost to English duo of
Jenny Duncalf and Alison Waters by 11-6, 6-11, 8-11 score.
President Paul Kagame wins by 2017 Rwandan Presidential Elections
President Paul Kagame has won 2017 Rwandan Presidential Elections by winning 98% of the votes. It will be
Kagames third term in office who has been in power for 17 years.
FOREX reserves surge to record $392.867 billion
Indias foreign exchange reserves (FOREX) have surged by $1.536 billion to touch a fresh lifetime high of
$392.867 billion in the week ended of July 2017. The components of Indias Foreign Exchange Reserves
include Foreign currency assets (FCAs), Special Drawing Rights (SDRs), Gold and RBIs Reserve position
with International Monetary Fund (IMF). In July 2017, the foreign currency assets, a major part of the
overall reserves rose by $1.609 billion to 368.759 billion dollars. Gold reserves remained unchanged at

2017 GKToday | All Rights Reserved | www.gktoday.in

19
Current Aairs: August, 2017 [Date-wise Compendium]

$20.35 billion. Indias SDR with IMF went up by $3.9 million to $1.495 billion, while countrys reserve
position declined by $77.2 million to $2.263 billion.
Hassan Rouhani sworn in as Irans president for second term
Irans re-elected president Hassan Rouhani has been sworn in for his second term in an official ceremony
held at the Iranian parliament in capital city Tehran.
Indo-Canadian named Canadas consul general in San Francisco
Indo-Canadian entrepreneur Rana Sarkar was appointed as Canadas Consul General in San Francisco. He
was also appointed member of the Canadian official team tasked with renegotiating the North American
Free Trade Agreement (NAFTA) with the US and Mexico. Till recently, Sarkar was national director for
high growth markets at the Toronto-based multi-national KPMG.
Indian Navy band joins historic Edinburgh Military Tattoo
A 66-member Indian Navy band has joined the historic Royal Edinburgh Military Tattoo in United
Kingdom for 2017 musical event where they will mark the 70th year of Indias Independence and the India-
UK Year of Culture. The theme for this years event is titled Splash of Tartan, celebrating Scotlands Year of
History, Heritage and Archaeology. The Royal Edinburgh Military Tattoo is an annual series of military
tattoos performance of music by British Armed Forces, Commonwealth and international military
bands and artistic performance teams on the esplanade of Edinburgh Castle in Scotland. It was first staged in
1950 and since then British and international musicians and acts from over 50 countries have entertained
audiences.
Ankur Mittal wins gold in Asian Shotgun Championship
Indian shooter Ankur Mittal has won ndividual as well as the team gold medal in mens double trap at the
7th Asian Shotgun Championship held at Astana, Kazakhstan. He shot 71 in the six-man final to finish at the
top of the podium. With scores of Sangram Dahiya (43) and Mohammad Asab (34) added to Mittals final
total, India won gold medal in the team event also. In the individual event, Dahiya finished fourth, while
Asab had to be contend with the fifth position.
August 7, 2017
Sajan bags bronze in Junior World Wrestling
Indias Sajan has won bronze medal at Junior World Wrestling in Greco-Roman category after defeating Ali
Osman Erbay of Turkey held at Tampere, Finland. Sajan defeated the Turkish grappler by 6-1 score 74kg
class category. He was the lone Indian Greco-Roman wrestler to bag a medal at this championship.
Justin Gatlin wins Gold medal in 100 meters Final at World Athletics Championship
Veteran American sprinter Justin Gatlin won gold medal in the 100 meters final at the World Athletics
Championship held in London. He clocked 9.92 seconds ahead of Christian Coleman (9.94 seconds) and
fastest man on earth Usain Bolt (9.95). Since 2008, when Bolt had set his first world record, he had not lost a
championship race and collected 19 gold medals in 100, 200 and 4100 meters.
Tori Bowie wins womens 100m title at Worlds
Olympic silver medallist Tori Bowie (26) won the womens 100 metres title at the World Athletics
Championships by clocking season best of 10.85 seconds. Marie-Josee Ta Lou (Cote DIvoire) won silver

2017 GKToday | All Rights Reserved | www.gktoday.in

20
Current Aairs: August, 2017 [Date-wise Compendium]

medal by clocking 10.86 seconds and Dutch woman Dafne Schippers won bronze in 10.96 seconds.
Indias rst heli-taxi service to start in Bengaluru
Indias first helicopter-taxi (heli-taxi) service will be started in Bengaluru, Karnataka for those who cannot
afford to spend time battling traffic to travel across the city. The Kempegowda International Airport will
become the first airport in India to have this facility. The service will be operated by Thumby Aviation
Private Limited and is expected to take off within three months. Initially, the helicopter service will be
operated between Bengalurus Kempegowda International Airport and the Electronics City area. The travel
time will be cut down to 15 minutes instead of the two hours taken by road.
August 8, 2017
Goa Assembly passes bill to reclassify Coconut as tree
The Goa Legislative Assembly has passed a bill to amend the Goa, Daman and Diu (Preservation of) Trees
Act, 1984 to reclassify coconut as a tree. The bill aim at ensuring legal protection for coconut tree and
regulate its felling. The previous state government led by Laxmikant Parseka had changed classification of
coconut tree to a palm. However, the decision kicked up a controversy as status of palm would had made
easy to cut down coconut trees indiscriminately in the coastal state as it would require no formal permission.
Dawn of cruise tourism in India event held in Mumbai
In order to promote cruise tourism in India, a special event, The dawn of cruise tourism in India was held in
Mumbai. It was organised by Ministry of shipping and Mumbai Port to showcase their preparedness to host
cruise vessels in the country. The event also saw the launch of three reports, Road map for Sea cruise
tourism, Mumbai Port SoPs for cruise operations and Cruise terminals in India.
Kynan Chenai wins bronze at 7th Asian Shotgun
Indian shooter Kynan Chenai won a bronze medal in mens trap event at the 7th Asian Championship
Shotgun held at Astana, Kazakhstan. It is his first international medal at the senior level. Kynan first shot
116 out of 125 to qualify for the six men final round in third sport and then held on to the position till the
end of the finals. He shot a score of 30 to finish behind gold medallist Abdulrahman Al Faihan (Kuwait) and
Talal Al Rashidi (Kuwait) who bagged the silver with an effort of 38.
Six-Party Talks
The six-party talks aim to find a peaceful resolution to the security concerns as a result of the North Korean
nuclear weapons program. These talks were a result of North Korea withdrawing from the Nuclear Non-
Proliferation Treaty (NPT) in 2003. Countries involved in it are South Korea, North Korea, United States,
China, Japan and Russia. North Korea had pulled out of Six Party Talks in April 2009 and resumed its
nuclear enrichment program in order to boost its nuclear deterrent.
August 9, 2017
BJP MP and former Union Minister Sanwar Lal Jat passes away
BJPs Member of Parliament from Ajmer and former Union minister Prof Sanwar Lal Jat passed away in
New Delhi. He was 62. Prof Sanwar Lal Jat was member of the Rajasthan Legislative Assembly for five
terms. He also served as Minister in the Rajasthan Government thrice. In 2014, he was elected to the Lok
Sabha from Ajmer. He served as Union Minister of State for Water Resources, River Development and

2017 GKToday | All Rights Reserved | www.gktoday.in

21
Current Aairs: August, 2017 [Date-wise Compendium]

Ganga Rejuvenation till July last year.


Kenyas Kipruto wins 3,000m steeplechase world title
Olympic gold medallist Conseslus Kipruto won mens 3,000 metres steeplechase at the IAAF World
Athletics Championships in London. He clocked eight minutes and 14.12 seconds to continue his in-form
performance in the event after the Rio Olympic Games.
Wayde Van Niekerk retains mens 400m world title
Olympic Champion Wayde Van Niekerk from South Africa won the mens 400 meters event at the IAAF
World Athletics Championships and retained his title. He clocked 43.98 seconds to finish at the top of
podium. Steve Gardiner of Bahamas finished second while Abdalelah Haroun of Qatar claimed the Bronze.
Indias rst micro forest being built in Chhattisgarh
Indias first micro forest is being built in Chhattisgarhs Raipur in order to create an oxy-zone. The micro
forest will provide fresh air to Raipur which is the seventh most polluted city in the world. It will reportedly
be open to the public in the next 8 months.
Asian Shotgun Championship: Kynan, Shreyasi win bronze in mixed team trap event
Indias Kynan Chenai and Shreyasi Singh have won bronze in mixed team trap event in Asian Shotgun
Shooting Championship in Astana, Kazakhstan. The Indian pair defeated Lebanan shooters by 40-38 score.
It was Indias 4th medal in the Championship. Kynan Chenai earlier had also won bronze in mens trap
event.
August 10, 2017
Davinder Singh Kang: 1st Indian to qualify for javelin throw nal at World Athletics Championships
Davinder Singh Kang (26) has become the first Indian to qualify for the final round of the javelin throw at
the IAAF World Athletics Championships in London. Competing in the qualification round Group B, Kang
had cleared the automatic qualification mark of 83 meter in his third and final throw with distance of 84.22
meter. He had 82.22m in his opening throw and then came up with 82.14m in his second.
August 12, 2017
Uhuru Kenyatta wins 2017 Kenya Presidential election
Uhuru Kenyatta, the current President of Kenya was officially declared the winner of the 2017 Kenya
Presidential election. Kenyatta secured 54.27% of the ballots cast, while his rival, Raila Odinga, won 44.74%.
Milkha Singh appointed as WHO Goodwill Ambassador for physical activity in South-East Asia
Region
Legendary athlete Milkha Singh was appointed as the World Health Organisations (WHO) Goodwill
Ambassador for physical activity in the South-East Asia Region (SEAR). As goodwill ambassador, he will
promote WHO SEARs Non-Communicable Diseases (NCDs) prevention and control action plan which
seeks to reduce the level of insufficient physical activity by 10% and NCDs by 25% by 2025. Milkha Singh,
also known as the Flying Sikh has won numerous medals in athletics in Commonwealth and Asian games.
He has been awarded Padma Shri award for his contributions to sports.
Gerish Khemani and Akshat Nigam win The Hindu Playwright Award 2017
Gerish Khemani and Akshat Nigam have won The Hindu Playwright Award 2017 for their play In Search of
Dariya Sagar. They were presented the award at the 13th edition of The Hindu Theatre Fest in Mumbai.
2017 GKToday | All Rights Reserved | www.gktoday.in

22
Current Aairs: August, 2017 [Date-wise Compendium]

Their play In Search of Dariya Sagar brings together public events and private lives, political history and
personal memory, in a manner that resonates both the real and the magical. The Hindu Playwright Award
2017 was instituted in 2008 to reward the best new unpublished and unperformed play-script in English.
Bihar Government launches mobile therapy vans for elderly people
Bihar Government has launched mobile therapy vans to provide basic medical assistance to elderly people,
widows and other sections of society which requires social security. The therapy vans were launched as part
of a joint project of the World Bank and the Bihar government. These vans will be equipped with basic
medical apparatus and accompanied by a technical team and will be run by Social Welfare department of the
state.
Dr Ruth Pfau: Pakistans Mother Teresa passes away
Dr Ruth Pfau, a German doctor and nun who dedicated her life to eradicating leprosy in Pakistan passed in
Karachi, Pakistan. She was 87. She was hailed as Pakistans Mother Teresa for her contributions towards
helping people displaced by the 2010 Pakistan floods. Dr. Pfau had founded Pakistans National Leprosy
Control Programme and the Marie Adelaide Leprosy Centre, which has a presence in every Pakistani
province. She had trained Pakistani doctors and attracted foreign donations. She received numerous awards
in Pakistan and Germany including the Hilal-e-Imtiaz (Pakistans second highest civilian award) in 1979, the
Hilal-e-Pakistan (1989) and the German Staufer Medal (2015). She had written four books in German about
her work in Pakistan, including To Light A Candle, which has been translated into English.
August 13, 2017
Jharkhand Assembly passes Religious Freedom Bill
Jharkhand Assembly has passed Religious Freedom Bill 2017. It aims to stop the forced conversions in the
state. The Bill has provision for imprisonment of three years and fine of 50,000 rupees or both, and four year
imprisonment and 1 lakh rupee fine, or both, if the person converted is a minor, woman or a member of
Scheduled Caste (SC)/Scheduled Tribe (ST). It also has provision saying that a person converting willingly
will have to inform the Deputy Commissioner about details such as time, place and the person who
administers the conversion proceedings.
Mairaj-Rashmmi win gold medal at Asian Shotgun Championship
Indian duo Mairaj Ahmad Khan and Rashmmi Rathore won gold medal in the skeet mixed team event at 7th
Asian Shotgun Championship held in Astana. In this edition of championship, India won total of eight
medals. Ankur Mittal won a gold medal in the mens double trap and also team gold in the event. Kynan
Chenai won a bronze in the mens trap and also partnered with Shreyasi Singh to win bronze medal in trap
mixed team bronze. Maheshwari Chauhan won bronze mdeal in the womens skeet, along with team silver.
Mens skeet team also won bronze medal.
Indian Oil Corp buys Indias rst shale oil from US
State-owned Indian Oil (IOC) has bought the Indias first shale oil from the United States. It is also looking
to step up imports from US as part of its crude diversification strategy. With this, India, the worlds third-
largest oil importer, joins Asian countries like Japan, South Korea and China to buy US crude.

2017 GKToday | All Rights Reserved | www.gktoday.in

23
Current Aairs: August, 2017 [Date-wise Compendium]

August 14, 2017


Poet Vijay Nambisan passes away
The well-known poet, writer and journalist Vijay Nambisan passed away. He was 54. He was the first-ever
All India Poetry Champion in 1988 for his poem Madras Central. He was well known for writing in English
and had co-authored a book of poems Gemini with Dom Moraes and Jeet Thayil. He had passionately
championed the ethical use of language and made an appeal to readers, writers and enthusiasts of literature
in his monumental book Language as an Ethic. His poetry served keen observations about politics both
internal and external.
Yisrael Kristal: Worlds oldest man passes away
The worlds oldest man Yisrael Kristal passed away in Haifa, Israel. He was 113. He was officially recognised
as the worlds oldest man by the Guinness Book of Records in March 2016. He was a Polish-Israeli
supercentenaria. He had lived through both World Wars. He became the worlds oldest recognized
Holocaust survivor in 2014 and the worlds oldest man in 2016.
Maharashtra Government launches Namo Yuva Rojgar Kendra
Maharashtra Government has launched Namo Yuva Rojgar Kendra to provide job opportunities to
unemployed youth in the state capital Mumbai. Under this initiative, there are various schemes like farmers
weekly market, farmers mobile market and mothers tiffin. These products will be made available with the
help of mobile vans all over Mumbai and the information about the location of the vans is made available
through.
August 17, 2017
Navroz: Parsi New Year
Parsi New Year, Navroz was celebrated across the world. The festival is celebrated by followers of
Zoroastrianism. On this day, Parsis wear traditional clothes and visit fire temple also known as Agiary and
offer milk, flowers, fruits, sandalwood to the sacred fire. The vibrant festival began 3,000 years back by the
mighty emperor Jamshed, who introduced the new calendar in Persia. Hence, it is also known as Jamshedi
Navroz or Navroz. The day is celebrated to gain health, wealth, prosperity, and productivity. Four Fs Fire,
Fragrance, Food, and Friendship play an important role in the celebration. Besides celebration, Navroz is
also the day of repentance. The day is dedicated to clean and purify the mind and start a new year with love
and peace.
RBI makes Aadhar linkage mandatory for farmers to avail short-term crop loans
Reserve Bank of India (RBI) has made Aadhar linkage mandatory for farmers who wish to avail short-term
crop loans in 2017-18. The Aadhaar linkage has been made compulsory to ensure hassle-free benefits to
farmers under the Interest Subvention Scheme (ISS). Farmers can avail of short-term crop loans of up to Rs.
3 lakh at subsidised interest rate of 7% that could go down to 4% on prompt repayment.
18 August 2017
US, Japan to advance security, defence cooperation with India, South Korea and Australia
US and Japan have agreed to advance their multilateral security and defence cooperation with countries like
India, South Korea and Australia in view of North Koreas dangerous provocative behaviour. In cooperation

2017 GKToday | All Rights Reserved | www.gktoday.in

24
Current Aairs: August, 2017 [Date-wise Compendium]

with other nations, US and Japan will continue to employ diplomatic and economic pressure to convince
North Korea to end its illegal nuclear and ballistic missile program.
Lakshya Sen wins Bulgaria Open International Series
Young Indian shuttler Lakshya Sen (16) won the Bulgaria Open International Series title of badminton in
the mens single category. In the final match held in Sofia, Bulgaria he defeated Croatias Zvonimir Durkinjak
by 18-21 21-12 21-17 score. In the semifinals Lakshya had defeated Sri Lankas Dinuka Karunaratna 21-19
21-14.
Vishal Sikka resigns as Infosys CEO & MD
CEO and managing director of Indias second largest software export firm Infosys, Vishal Sikka, has
resigned. He has retained as the executive vice-chairman of the company. UB Pravin Rao, chief operating
officer of the Infosys, has been appointed as interim CEO and managing director.
August 20-21, 2017
Soumyajit Ghosh, G Sathiyan win silver medal at Bulgaria Open
The Indian pair of Soumyajit Ghosh and G Sathiyan won silver medal in the mens doubles category held at
the Bulgaria Open in Panagyurishte. In the final match Indian duo lost to the top-seeded Japanese pair of Jin
Uda and Maharu Yoshimura by 13-11, 7-11, 4-11, 11-6, 5-11 score. This was Japanesse duos second title in
three World Tour final appearances.
Shapath Bharadwaj wins bronze at Junior Shotgun World Cup
Indian teenager Shapath Bharadwaj (15) won a bronze medal in the double trap event at the ISSF Junior
Shotgun World Cup held in Porpetto, Italy. He shot 48 points in the final round to finish behind gold-medal
winning Briton James Dedman and silver medalist Finlands Miki Ylonen. It was Indias first individual
medal in this edition of competition. Earlier Lakshay Sheoran and Manisha Keer had won bronze medal in
the mixed team trap competition.
Telangana CM chosen for Agriculture Leadership Award
Telangana Chief Minister K Chandrashekhar Rao was selected for the prestigious Agriculture Leadership
Award-2017. He has been selected for the award for his innovative services rendered to the welfare of
farmers and the farm sector. The award was instituted by Indian Council of Food and Agriculture (ICFA) in
2008, is presented every year since then.
20 August: Sadbhavana Diwas
The Sadbhavana Diwas (Harmony Day) is celebrated every year on 20 August to commemorate the birth
anniversary of former Prime Minister Rajiv Gandhi. Observance of the day seeks to encourage, national
integration, peace, love, affection and communal harmony among all Indians. Rajiv Gandhi was born on
August 20, 1944, in Mumbai, Maharashtra. At 40, he was the youngest Prime Minister of India. He was the
6th Prime Minister of India, serving from 1984 to 1989. He was assassinated on 21 May 1991.
Sri Lanka appoints rst Tamil Navy Chief
Rear Admiral Travis Sinniah was appointed as the chief of Sri Lankas naval forces by President Maithripala
Sirisena. It makes him the first from the minority Tamil community to head the island nations Navy since
the civil war erupted in the country 45 years ago. He succeeds Vice Admiral Ravi Wijegunaratne who has

2017 GKToday | All Rights Reserved | www.gktoday.in

25
Current Aairs: August, 2017 [Date-wise Compendium]

retired. It should be noted that he is the second Tamil to head the Lankan Navy after Rajan Kadirgamar, who
was the commander in the late 1960s.
22 August 2017
Ulchi Freedom Guardian drill: US and South Korea annual Military Exercise
The Ulchi Freedom Guardian drill, 10-day-long annual military exercise between US and South Korea was
held recently. It was a computer-simulated defensive exercise designed to enhance readiness and maintain
stability on the Korean peninsula. It was the first joint military exercise between the allies since North Korea
successfully flight-tested two intercontinental ballistic missiles (ICBMs) in July 2017 and threatened to attack
Guam Island of US with intermediate range ballistic missile.
Indian-Origin Rahul Doshi crowned Child Genius in UK
A 12-year-old Indian-origin boy named Rahul Doshi was crowned as the UKs Child Genius in a popular
television quiz competition, days after he became an overnight sensation after answering all questions
correctly. Rahul won the Channel 4 show Child Genius by beating his nine-year-old opponent Ronan by
10-4 score.
Rajasthan Government ink JV Agreement with HPCL for Renery Project
Rajasthan government and state-owned oil firm HPCL have signed an agreement to form a joint venture
company for Barmer petroleum refinery and petrochemical complex. In the JV, HPCL Rajasthan Refinery
Limited (HRRL), state government will have 26% and HPCL will have 74% of partnership. Th refinery
project will be established in 4,813 acres of land area. Morevover, a 270 MW power plant will be also
established to generate power from the waste petcoke generated from the refinery.
First Regional Centre of NDB opens in South Africa
The first regional centre of the New Development Bank (NDB), set up by BRICS nations, was officially
opened by South African President Jacob Zuma and the banks Indian head K V Kamath. NDB has
anauthorized capital of the bank is US $100 billion. In 2018, it is expected to present about 15 to 20 projects
involving a total lending of about US $4 billion for the boards consideration.
Government to replace physical Police verication of Passport applicants with online verication
The Union Home Ministry is planning replace physical Police verification of Passport applicants with online
verification with the help of Crime and Criminal Tracking Network and Systems (CCTNS). Police will be
given hand-held devices to go to an applicants address and upload his or her details on to the network which
will minimise contact with police and reduce time.
25 August 2017
Indian Navy Band participates in International Military Music Festival in Russia
Indian Navy Band has participated in the International Military Music Festival Spasskaya Tower in Russia.
The Spasskaya Tower is the parade of the best military music bands of Russia and other countries. It takes
place every year at Red Square in Moscow. Every year about 1500 musicians, military men and other artists
from around 40 countries perform in this mega event. The Festival is one of the largest international events
in Russia and attracts tremendous response of the general public.
Indian Coast Guard ship Varuna decommissioned
Indian Coast Guard ship (ICGS) Varuna was decommissioned with full armed forces honours after an
2017 GKToday | All Rights Reserved | www.gktoday.in

26
Current Aairs: August, 2017 [Date-wise Compendium]

illustrious service spanning about 30 years. ICGS Varuna was fourth of its series, known as Offshore Patrol
Vessel (OPV) and built by Mazagaon Dock, Mumbai. It had numerous operational achievements to its credit
including record number of apprehension of smugglers with gold worth crores during late 1980s.
Nepal launches worlds Climate-Smart Snow Leopard Plan
Nepal launched climate-smart snow leopard landscape management plan for safeguarding endangered
species and its habitat. This is the first climate-smart landscape management plan for snow leopard
conservation in the world. The plan aims to safeguard the endangered species of snow leopard and its
habitat.
26 August 2017
Nungthaang Tampak: North Easts rst 100% Computer Literate Village
Nungthaang Tampak village in Manipur has emerged as the 100% computer literate in the entire North
East. It is also Indias second 100% computer literate village after Keralas Chamravattom village. Both
villages have completed computer course affiliated to All India Society for Electronics and Computer
Technology (AISECT) organized as a part of the Digital India program.
J&K Government set up rst eco-park
The Jammu and Kashmir Government has approved proposal to build states first Eco-Park in Rajouri
district. The park is a first-of-its-kind project in the region. It will have a botanical garden, cactus garden,
cafeteria and other amenities.
RBI appoints committee to link home loan rates to Repo Rate
The Reserve Bank of India (RBI) has appointed committee on Household Finance to link home loan rates of
banks to the RBIs repo rate. In the past three years RBI has reduced the policy rate by 200 basis points(basis
point is 0.01 percentage point), but the weighted average lending rates have fallen by 145 basis points.
Nandan Nilekani appointed as new non-executive chairman of Infosys board
Infosys co-founder and the architect of Aadhaar Nandan Nilekani was appointed as the new non-executive
chairman of the board of directors of Infosys. His appointment comes in an attempt to restore stability after
the resignation of Vishal Sikka as CEO. Earlier, Nilekani had served as Infosyss CEO from 2002 to 2007.
Kerala Government launches E-Waste Disposal scheme
The Kerala government has launched E-Waste Disposal scheme in its state schools with an aim to process
the electronic waste in the IT labs, offices and stores. The programme is a joint initiative of Kerala
Infrastructure and Technology for Education (KITE), formerly IT@School project and Clean Kerala
Company. Through this scheme, government aims to scientifically recycle over 1 crore kilogram of e-waste
from over 10,000 schools and offices and use the funds earned through this e-waste disposal will be use to
buy new computers for the same institutions.
Digambar Behera wins 2016 Biju Patnaik Award for Scientic Excellence
Prof Digambar Behera was conferred prestigious Biju Patnaik Award for Scientific Excellence for the year
2016. Prof Behera is senior professor and Head, Department of Pulmonary Medicine, Postgraduate Institute
of Medical Education and Research (PGIMER) in Chandigarh. Beside, Prof Prasanta Mohapatra, a professor
at the University of California, was also conferred this award for the year 2015. The Biju Patnaik Award was

2017 GKToday | All Rights Reserved | www.gktoday.in

27
Current Aairs: August, 2017 [Date-wise Compendium]

instituted by Odisha Bigyan Academy (OBA) and is awarded annually to any Odia scientist working
anywhere in India or abroad for their lifetime research contributions in field of science and technology.
27-28 August 2017
Nitasha Biswas crowned First Miss TransQueen India
Nitasha Biswas, a transwoman from Kolkata (West Bengal) was crowned first Miss TransQueen India 2017
at the first transgender beauty contest Miss TransQueen India 2017 held in Gurugram, Haryana. Manipurs
Loiloi was crowned the first runner-up. Biswas is currently pursuing Masters in Business Management in
Kolkata, West Bengal. She will now participate Miss International Queen to be held in Thailand while Loiloi
will compete at Miss Transsexual Australia.
Long-Range Surface-to-Air Missile handed over to Indian Navy
The Long-Range Surface-to-Air Missile (LRSAM) jointly developed by India and Israel was handed over to
the Indian Navy, at Bharat Dynamics Ltd (BDL) in Hyderabad, Telangana. LRSAM is an advanced combat
suit for missile defence against air targets and missiles, and has full capabilities of air and surface surveillance,
threat alert and fire control. Public sector BDL is the missile production agency for the three armed forces.
Kenya imposes worlds toughest law against plastic bags
Kenya has imposed worlds toughest law against used of plastic bags. The law aims at reducing plastic
pollution came into effect. Under it, selling or even using plastic bags will led to imprisonment of up to four
years or fines of $40,000. With this, Kenya joins more than 40 other countries around the world that have
banned, partly banned or taxed single use plastic bags, including France, Italy, China and Rwanda.
Olympian footballer Ahmed Khan passes away
Indias two-time Olympic footballer and East Bengal legend Ahmed Khan passed away. He was 90. He had
played in the 1948 and 1952 Olympics. He was one of the most famous of the legendary quintet of forwards
called the Five Pandavas of India who played for East Bengal Football Club. He made his international
football debut against France in the London Olympics.
Lewis Hamilton wins 2017 Belgian Grand Prix
Mercedes Formula One driver Lewis Hamilton from United Kingdom won Belgian Grand Prix title. It was
fith Grand Prix title of the season and overall 58th career title. Ferraris Sebastian Vettel came second, while
Daniel Ricciardo was placed third.
IRDAI launches Central Database of Insurance Sales Persons
The Insurance Regulatory and Development Authority of India (IRDAI) launched Central Database of all
Insurance Sales Persons in the country. It has been named as `Envoy. The centralised database will ensure
that all licensed insurance sales persons working for insurers and intermediaries including entities such as
insurance agents, broker specified persons of corporate agents, qualified persons do not work with multiple
insurers and intermediaries in the same business category.
Dharmendra Kumar appointed as Director General of RPF
The Union Government has appointed senior IPS officer Dharmendra Kumar was appointed as Director
General of the Railway Protection Force (RPF). Kumar, is 1984 batch IPS officer of Arunachal Pradesh-Goa-
Mizoram-Union Territories (AGMUT) cadre. Prior to this appointment he was CISF ADG. He will tenure

2017 GKToday | All Rights Reserved | www.gktoday.in

28
Current Aairs: August, 2017 [Date-wise Compendium]

of one year till September 2018. RPF is entrusted with protecting railway passengers, passenger area and
railway property of the Indian Railways. It is the only central armed police force (CAPF) which has the
power to arrest, investigate and prosecute criminals. The force is under the authority of Union Ministry of
Railways.
ICGS Varuna: Coast Guard Training Ship decommissioned
The Indian Coast Guard Ship (ICGS) Varuna was decommissioned after it illustrious service spanning about
30 years. The ship was was part of tFirst Training Squadron of Indian Navy. It was the fourth of in its class
of offshore patrol vessels (OPV) built by Mumbai-based Mazagon Dock. The ship had numerous operational
achievements to its credit, including a record number of apprehension of smugglers with gold worth cores of
rupees during the late 1980s. ICGS Varuna now is replaced by ICGS Sarathi in First Training Squadron of
Indian Navy.
1st edition of Rural Games or Grameen Khel Mahotsav held in New Delhi
The first edition of Rural Games or Grameen Khel Mahotsav will be held in Delhi from 28th August to 3rd
September 2017. The purpose of these games is to popularize the indigenous games like wrestling and
athletics and will also have fun games like Matka Race, Tug of War for senior citizens.
29 August 2017
Kerala CM launches FIFA U-17 World Cup logo for Kochi venue
Kerala Chief Minister Pinarayi Vijayan launched the host city logo for Kochi, one of the venues of FIFA
U-17 World Cup India 2017. The logo launched represents Kochis identity and serve as valuable tool in
linking local community in a global World Cup. The tournament will be hosted in the Jawaharlal Nehru
Stadium of the city.
PM Narendra Modi dedicates to nation 11 completed highway projects in Rajasthan
Prime Minister Narendra Modi dedicated 11 completed highway projects in Rajasthan, covering a total
length of about 873 kilometres to the nation. The projects include six Lane Cable Stayed Bridge across River
Chambal at Kota, four laning of Gomati Chauraha Udaipur section and four laning of Rajsamand
Bhilwara section.
North Korea res ballistic missile over Japan
North Korea fired a ballistic missile over Japans northern Hokkaido Island into the sea. The test appears to
be of recently developed intermediate-range Hwasong-12 missile. The test has been one of the most
provocative ever from the isolated state.
International Solar Alliance to be ratied multilateral agency of UN by December 2017
The International Solar Alliance (ISA) is likely to be a recognised multilateral agency of the United Nations
by December 2017. It was announced by Minister of State (Independent Charge) Coal, Power, New and
Renewable Energy and Mines, Piyush Goyal at the 8th World Renewable Energy Technology Congress. ISA
was launched by Prime Minister Narendra Modi and French President Francoise Hollande in November
2015. It aims to channel $300 billion in 10 years to promote renewable energy projects under a global mega
fund for clean energy. It was instituted to connect nearly 121 solar-resource-rich nations for research, low-
cost financing and rapid deployment of clean energy.

2017 GKToday | All Rights Reserved | www.gktoday.in

29
Current Aairs: August, 2017 [Date-wise Compendium]

30-31 August 2017


CSIR Ranked 9th Public Research Institution in World
Indias largest public research organisation Council of Scientific and Industrial Research (CSIR) was ranked
9th in in the 2017 Scimago Institutions ranking World Report. It was only Indian organisation to be placed
amongst the Top 100 Global Institutions. The ranking (Government Institutions) was topped by Chinese
Academy of Sciences.
CSIR is an autonomous body and Indias premier research and development (R&D) organisation funded by
the Union Ministry of Science and Technology. It was established in 1942. It operates as an autonomous
body registered under the Registration of Societies Act of 1860. Its R&D activities include aerospace
engineering, ocean sciences, structural engineering, metallurgy, life sciences, chemicals, mining, food,
petroleum, leather, and environment.
Reliance Foundation awarded Rashtriya Khel Protsahan Puraskar
The Reliance Foundation was awarded with prestigious Rashtriya Khel Protsahan Award at Rashtrapati
Bhawan for its contribution to the promotion of sports. Chairperson of the foundation, Nita Ambani
collected the award from President Ram Nath Kovind. Kendriya Vidyalaya Sangathan (KVS) was awarded
Rashtriya Khel Protsahana Purushkar for its Identification and Nurturing of Budding and Young Talent
(IDCO) program.
RK Raghavan appointed as Indias new High Commissioner to Cyprus
The Union External Affairs Ministry has appointed former CBI Director RK Raghavan as Indias new High
Commissioner to Cyprus. He was IPS officer of 1963 batch from Tamil Nadu cadre. He was CBI Director
from January 1999 to April 2001. He was head of Supreme Court constituted Anti-Ragging Monitoring
Committee in 2006. His specialization is cybersecurity and established Indias first cybercrime investigation
cell. He is a recipient of Indian Police Medal for Meritorious Service and Presidents Police Medal for
Distinguished Service.
PV Sindhu appointed as rst brand ambassador of Bridgestone India
Ace Indian shuttler and Olympic silver medallist PV Sindhu was appointed the first brand ambassador of tyre
maker Bridgestone India. She has signed a three year deal with Bridgestone India, a subsidiary of Bridgestone
which is one of the sponsors for the Olympics 2020 in Japan. She recently had won Silver at the World
Badminton Championship. She was first Indian woman to win silver medal in the 2016 Rio Olympics.
Unicity launches worlds rst ever Genomeceuticals Range of Products in India
Unicity International has launched worlds first ever Genomeceutical (gene controlling) range of products to
India. The advanced Genomecutical technology used by Unicity International in its products helps to
maintain healthy body function at their most basic levels. Genomeceutical is a ground-breaking technology
whereby certain secondary metabolites found in nature can be used to maintain the gene response whenever
needed by the body and thus help to maintain overall metabolic health.
RBI grants Scheduled Bank Status to Ujjivan SFB
The Reserve Bank of India (RBI) has granted scheduled bank status to Ujjivan Small Finance Bank Ltd, a
wholly-owned subsidiary of Ujjivan Financial Services Ltd. Currently, Ujjivan SFB has 65 full-fledged brick

2017 GKToday | All Rights Reserved | www.gktoday.in

30
Current Aairs: August, 2017 [Date-wise Compendium]

and mortar branches in 8 states and two Union Territories. It had commenced its operations as a Small
Finance Bank (SFB) from February 2017.
Date Wise Current Affairs
August 1, 2017
BSF and NSDC sign MoU on Skill Development
The Border Security Force (BSF) signed memorandum of understanding (MoU) with the National Skill
Development Corporation (NSDC).
The MoU will enable NSDC to provide skill training to retired and retiring BSF personnel, as well as to the
population residing in the border areas.
Key Facts
The collaboration between BSF and NSDC aims to train and provide gainful employment to retired BSF
personnel and their families, school students, youth, differently abled for five years (April 2017 to April
2022). The programme will be appropriately named keeping in view the objectives and will have BSF co-
branding with Skill India and NSDC.
It will contribute to Skill Development Agenda of the country by ensuring the successful roll out of NSQF
(National Skill Qualification Framework) based skill training to the retiring or retired BSF personnel and
local population. The skill training will be based NSQF compliant training programmes at multi-skill
development centres run by BSF. This will help in meeting the aspirations of the stakeholders for training
and employment.
The skill training project will be of a limited to 5 years period in mission mode. Moreover, the curriculum
development and capacity building under the programme will be encouraged to enable the model to be self-
sustaining over time.
About National Skill Development Corporation (NSDC)
The NSDC is a Public Private Partnership (PPP) under the Ministry of Skill Development &
Entrepreneurship. Its overall objective of NSDC is to create training capacity in the country; fund vocational
training initiatives and create a market ecosystem for skill development. Its mandate is to train 150 million
people by 2022.
India-Bhutan New Trade and Transit agreement come into force
The new bilateral Agreement on Trade, Commerce and Transit between India and Bhutan came into force
from July 2017. The new agreement aims to further strengthen the bilateral trade relations between both
countries.
The new agreement signed in November 2016 provides for a free trade regime between the territories of
India and Bhutan. It also provides for duty-free transit of Bhutanese merchandise for trade with third
countries.
Bilateral trade between both countries will continue to be transacted in Bhutanese Ngultrums and Indian
Rupees. Moreover, the new agreement removes requirement of having an annual consultation between
2017 GKToday | All Rights Reserved | www.gktoday.in

31
Current Aairs: August, 2017 [Date-wise Compendium]

India and Bhutan for the transit of goods of both countries along with the goods originating in third country
Background
The bilateral trade relations between India and Bhutan are governed by the Agreement on Trade, Commerce
and Transit. The agreement provides for a free trade regime between the territories of India and Bhutan.
The last agreement was renewed on July 29, 2006 for ten years. The validity of this agreement was extended
(from July 29, 2016) for one year or till new agreement comes into force.
19th RCEP Trade Negotiating Committee Meeting held in Hyderabad
The 19th round of the Regional Comprehensive Economic Partnership (RCEP) Trade Negotiating
Committee (TNC) meetings and other related meetings were held in Hyderabad, Telangana.
Besides, the 19th round of TNC, parallel meetings were also held by the three main Working Groups on
Trade in goods, services and investment. Working Groups in other areas like electronic commerce,
Intellectual Property, Legal and Institutional Issues etc. also had held their meetings.
Key takeaways of 19th round
All RCEP Participating Countries (RPCs) agreed that RCEP agreement has immense potential to deliver on
new economic opportunities including job creation that are much needed in todays uncertain world. India
held that RCEP can offer a forward looking alternative in face of growing protectionism in world.
RPCs also expressed shared commitment to work collectively and in cooperative manner to progress the
negotiations in an accelerated way and achieve a comprehensive, modern, high-quality and mutually
beneficial agreement that balances and addresses sensitivities and aspirations of participating countries.
About Regional Comprehensive Economic Partnership (RCEP)
RCEP is a proposed comprehensive regional economic integration agreement (mega Free Trade Agreement)
amongst the 10-ASEAN countries (Brunei, Indonesia, Cambodia, Laos, Myanmar, Malaysia, Philippines,
Thailand, Singapore and Vietnam) and its six Free Trade Agreements (FTAs) partners, viz. Australia, New
Zealand, India, China, Japan and Korea.
RCEP negotiations were formally launched at 2012 ASEAN Summit in Cambodia. RCEP is viewed as an
alternative to the Trans-Pacific Partnership (TPP), a proposed trade agreement that includes several Asian
and American nations but excludes China and India.
Till 2017, RCEP member states accounted for a population of 3.4 billion people with a total GDP (in terms
of PPP) of $49.5 trillion, approximately 39% of the worlds GDP (combined GDPs of China and India makes
up more than half that amount).
SC bans use of five heavy metals in firecrackers
The Supreme Court has imposed ban on use of five harmful heavy metals like lithium, antimony, mercury,
arsenic and lead in the manufacturing of firecrackers as they cause air pollution.
The SC order came on a 2015 petition filed by three Delhi children who had requested the apex court to
intervene to regulate the use of crackers and fireworks during festivals like Diwali.
SC order
The apex court held that it is the responsibility of the Petroleum and Explosives Safety Organisation (PESO)
2017 GKToday | All Rights Reserved | www.gktoday.in

32
Current Aairs: August, 2017 [Date-wise Compendium]

to ensure compliance of ban order particularly in Sivakasi in Tamil Nadu where there are large number of
firecracker manufacturers. It also asked CPCB (Central Pollution Control Board) and PESO to make
collaborative efforts for setting up of standards with regard to air pollution caused by the bursting of fire-
crackers.
Heavy metals in firecrackers
Lithium is a metal used to impart red colour to fireworks. Antimony is used to create glitter effects. Lead
oxide provides a special crackling effect which, if inhaled, in high concentration can cause damage to the
nervous system.
About Petroleum and Explosives Safety Organisation (PESO)
PESO is the apex department to control and administer manufacture, storage, transport and handling of
explosives, petroleum, compressed gases and other hazardous substances in India. It functions under the
Department of Industrial Policy and Promotion (DIPP), Ministry of Commerce and Industry. It is headed by
Chief Controller of Explosives and is headquartered at Nagpur, Maharashtra.
Dhrupad maestro Ustad Sayeeduddin Dagar passes away
Ustad Hussain Sayeeduddin Dagar, one of the foremost exponents and custodian of the venerable Dhrupad
tradition of Hindustani classical music passed away in a Pune. He was 78.
About Sayeeduddin Dagar
Ustad Sayeeduddin Dagar was born on April 29, 1939 in Alwar, Rajasthan. He was member of the illustrious
Dagar family of musicians and was the youngest of the famous seven Dagar Bandhus (Brothers), all
exponents of the ancient, complex and elaborate Dhrupad tradition.
He had dedicated his life to keeping the Dhrupad tradition alive. He had performed at some of the most
prestigious venues and festivals in India and abroad, including the Tansen Savai Gandharva, Samaroh,
Dhrupad Samaroh, Dagar Saptak, Dhrupad Mela, Dhamar Samaroh etc.
He also travelled abroad extensively and enthralled audiences with his Dhrupad performances. He was also
President of Dhrupad Society Jaipur and Pune. He also had conducted workshops and interactive lecture
demonstrations on Dhrupad in India and twice a year in Holland, Germany, France and Belgium.
About Dhrupad
Dhrupad is a genre in Hindustani classical music. It is one of oldest forms of compositions in classical Indian
music. Dhrupad is a Sanskrit name, derived from words dhruva (permanent) and pad (verse) and in
combination it means pillar.
The roots of Dhrupad are ancient and it is mentioned in Natyashastra (200 BCE 200 CE). It is also
described in other ancient and medieval Sanskrit texts, such as Bhagavata Purana (~8001000 CE).
The nature of Dhrupad music is spiritual and does not seek to entertain, but to induce feelings of peace and
spirituality in the listener. It is primarily a form of worship, in which offerings are made to the divine
through sound or Nada.
Dhrupad was initially sung only in the temples, the singer facing the Lord. From this early chanting, it
evolved into a sophisticated classical form of music. One significant characteristic of Dhrupad is the
2017 GKToday | All Rights Reserved | www.gktoday.in

33
Current Aairs: August, 2017 [Date-wise Compendium]

emphasis on maintaining purity of the Raga.


Arvind Panagariya resigns as Vice-Chairman of NITI Aayog
Arvind Panagariya (64) has resigned as Vice Chairman of NITI (National Institution for Transforming India)
Aayog and announced to return to academics in US.
He resigned after his public service leave for two years from the Columbia university, where he is professor
of Indian Political Economy did not receive extension. He was appointed as the as Vice Chairman of NITI
Aayog in January 2015 and held cabinet minister rank.
About Arvind Panagariya
Before joining Niti Aayog, he was a professor at Columbia University and chief economist at the Asian
Development Bank (ABD). He has also worked for the World Bank, International Monetary Fund (IMF),
World Trade Organisation (WTP), and the UN Conference on Trade and Development (UNCTD) in
various capacities.
He has earned PhD in economics from prestigious Princeton University. He has written around 10 books
and the latest was India: The Emerging Giant published in 2008. He was honoured with Padma Bhushan in
2102.
About NITI Aayog
NITI Aayog was formed on January 1, 2015. NITI Aayog via a resolution of the Union Cabinet. It is the
premier policy Think Tank of the Union Government, providing both directional and policy inputs. It had
replaced the 65-year old Planning Commission established in 1950. Prime Minister is ex-officio Chairperson
of the Aayog.
NITI Aayog plays important role in designing strategic and long term policies and programmes for the
Government. It also provides relevant technical advice to the Centre and States. It also acts as the
quintessential platform of Union Government to bring States to act together in national interest, and
thereby fosters Cooperative Federalism.
Shahid Khaqan Abbasi elected as Prime Minister of Pakistan
Pakistans Petroleum Minister Shahid Khaqan Abbasi was elected the 18th Prime Minister of Pakistan by the
National Assembly. Abbasi, nominee of ruling Pakistan Muslim League (N) secured 221 votes in the House
of 342.
Abbasi succeeds Nawaz Sharif who was disqualified by the Supreme Court over the Panama Papers scandal.
He will be interim PM of Pakistan. He will vacate the position for Nawaz Sharifs brother Shahbaz Sharif,
Punjab Chief Minister gets elected to the National Assembly.
About Shahid Khaqan Abbasi
He was born 27 December 1958 in Karachi, Sindh and comes from a military family. Prior to entering
politics, he was an electrical engineer and businessman owning Pakistans most successful private airline
Airblue. He had started his political career after the death of his father in 1988, and since then he has been an
elected Member of the National Assembly six times from Rawalpindi District.
He is member of the Pakistan Muslim League (N) (PML-N) in the National Assembly. He had served as the
2017 GKToday | All Rights Reserved | www.gktoday.in

34
Current Aairs: August, 2017 [Date-wise Compendium]

Minister of Petroleum and Natural Resources (2013 to 2017) in the third Nawaz Sharif Government. He had
briefly held the cabinet portfolio of Minister for Commerce in the Gillani Government in 2008. He also had
served as Chairman of Pakistan International Airlines (1997 to 1999) during the second Sharif Government.
Chemical from turmeric treats cancer in children: study
Scientists from United States have found that curcumin, the bioactive component of turmeric that is widely
used in Indian cuisine can treat cancer in children.
They have found that nanoparticles loaded with curcumin can target and destroy neuroblastoma tumour cells,
which commonly affects children aged five or less.
Key Facts
In their study, scientist had attached curcumin to cerium oxide nanoparticles and tested the formulation in cell
lines of a high-risk form of neuroblastoma. It was found that curcumin-cerium formulation induced
substantial cell death in neuroblastoma cells while producing no or only minor toxicity in healthy cells.
Significance: The study demonstrates cancer treatment method without the toxicity of agressive therapy. It
also shows that nanoparticles can be an effective delivery vehicle for cancer drugs. The use of cerium oxide
nanoparticles also help to remove curcumins low solubility and poor stability which had earlier restricted its
use in therapeutic applications
Neuroblastoma (NB)
NB is a type of cancer that forms in certain types of nerve tissue. It most frequently starts from one of the
adrenal glands normally near the kidneys. But it can also develop in the neck, abdomen, chest or spine. Its
symptoms may include bone pain, lump in the abdomen, neck or chest, or painless bluish lump under the
skin. It is mostly resistant to anti-cancer drugs, and is known to cause health problems, such as hearing loss
and disabilities, even after successful treatment.
August 2, 2017
Eminent Scientist Pushpa Bhargava passes away
Veteran molecular biologist Pushpa Mittra Bhargava passed away in Hyderabad. He was 89. Bhargava was
the founding director of Hyderabad based Centre for Cellular and Molecular Biology (CCMB)
About Pushpa Mittra Bhargava
Born in Ajmer on February 22, 1928. He had earned his Ph.D. in synthetic organic chemistry from Lucknow
University. He had started his career with Central Laboratories for Scientific and Industrial Research.
His pioneering vision and efforts led to the founding of CCMB in 1977 as an institution for research in basic
biology and for seeking its application for benefit of society. Under his leadership, CCMB had done
pioneering research in cell, DNA and molecular biology with close industry linkages. He was strident critic
of introduction of genetically modified crops in India.
He was actively engaged with issues in science and policy and served vice chairman of National Knowledge
Commission constituted in 2005. He was chairman of the Drafting Committee for the Assisted Reproductive
Technologies (Regulation) Bill 2013. He was awarded Padma Bhushan in 1986.

2017 GKToday | All Rights Reserved | www.gktoday.in

35
Current Aairs: August, 2017 [Date-wise Compendium]

India, Somali sign agreement on transfer of sentenced persons


India and Somalia have signed an agreement for transfer of sentenced prisoners in each others country. The
agreement was signed in New Delhi during the ongoing visit of the Somalias Foreign Minister to India.
During the visit, both countries discussed entire gamut of bilateral relations, including the issue of piracy and
maritime security. They also discussed regional issues pertaining to developments in the Gulf region and
Indias neighbourhood.
The agreement opens a new front for collaboration in anti-piracy operation in Eastern Africa as the
incidents of piracy originating from Somalia have affected Indian interests in the Indian Ocean region near
the Horn of Africa.
Background
Somalia is situated near the Horn of Africa with the Gulf of Aden to its north. It is among the partner
countries of Indias Pan African e-Network project. India-Somalia bilateral trade stood at $391.05 million in
2014-15, a more than 51% increase over the figure of $257.27 million in 2013-14.
Diplomatic contacts between India and Somalia have been infrequent, however, Somalia had participated in
the 2015 India-Africa Forum Summit in New Delhi. Somalia plays important role in containing Chinas
presence in Indian Ocean region. China opened its first overseas military base in Djibouti. Somalia shares
border with Djibouti on the west.
India also offers Indian Technical and Economic Cooperation (ITEC) training scholarships and Indian
Council for Cultural Relations (ICCR) scholarships to Somalia. In recent years, India also has extended
development assistance which included mini buses to the war-battered country.
Parliament passes Right of Children to Free and Compulsory Education
(Amendment) Bill, 2017
Parliament has passed the Right of Children to Free and Compulsory Education (Amendment) Bill, 2017
with Rajya Sabha approving it. Lok Sabha had passed it earlier.
The Bill amends the Right of Children to Free and Compulsory Education Act (RTE), 2009 by extending the
deadline for teachers to acquire the prescribed minimum qualifications for appointment.
Key Facts
In the parent Act, provision to possess minimum qualifications by teachers was relaxed for a period not
exceeding five years (till March 2015) in case state does not have adequate teacher training institutions or
sufficient number of qualified teachers.
The amendment bill gives last chance to inadequately qualified teachers as on March 31, 2015, working after
enactment of the RTE Act, 2009 to acquire minimum qualifications within a period of four years (till March
31, 2019) to hold their jobs as teachers.
Comment
The RTE Act, 2009 envisages free and compulsory elementary education to every child in the age group of
6-14 years. The amendments to RTE Act, 2009 will enable the in-service untrained elementary teachers to

2017 GKToday | All Rights Reserved | www.gktoday.in

36
Current Aairs: August, 2017 [Date-wise Compendium]

complete their training and ensure that all teachers at the elementary level have certain minimum standard
of qualifications in order to maintain the standard of teaching quality.
Government launches e-RaKAM portal
The Union Government has launched e-Rashtriya Kisan Agri Mandi (e-RaKAM) portal to provide a
platform for farmers to sell agricultural produce.
e-RaKAM is a first of its kind initiative that leverages technology to connect farmers from the smallest
villages to the biggest markets of the world through internet.
About e-RaKAM
e-RaKAM is a digital initiative that aims to bring together the farmers, PSUs, civil supplies and buyers on a
single platform to ease the selling and buying process of agricultural products. It has been developed by state-
run-auctioneer MSTC Limited and CRWC (Central Railside Warehouse Company) Limited.
Under this initiative, e-RaKAM centres are being developed throughout the country in a phased manner to
facilitate farmers for online sale of their produce. The farmers will be paid through e-Payment directly into
their bank accounts under the e-RaKAM initiative.
Government constitutes Committee to study issues related to data protection
The Union Ministry of Electronics & Information Technology (MEITY) has constituted an expert
Committee to study and identify key data protection issues and recommend methods for addressing them.
The ten-member committee will be headed by Supreme Court Judge (retired) Justice B N Srikrishna and
comprise of members from government, academia and Industry.
Terms of Reference of Committee
The committee will study and identify key data protection issues and recommend methods for addressing
them. It will also suggest a draft Data Protection Bill.
Background
It is seen that protection of data and keeping personal data of citizens secure and protected is expected to
provide big boost to Digital economy of the country. The governments decision to focus on data protection
comes on the back of a wave of privacy and data breaches from corporates such as McDonalds, Reliance Jio
and Zomato to government agencies that have leaked the personal data and Aadhaar of over 100 million
citizens.
India, China home to 39% of young Internet users: UN report
According to recently released report-ICT Facts and Figures 2017, India and China are home to 39% of the 830
million young people worldwide who use the Internet.
The report was released by International Telecommunication Union (ITU), the United Nations specialised
agency for Information and Communication Technologies (ICTs).
Key Highlights of Report
Great strides are being made in expanding Internet access through the increased availability of broadband
networks. Digital connectivity plays a critical role in bettering lives, as it opens the door to unprecedented

2017 GKToday | All Rights Reserved | www.gktoday.in

37
Current Aairs: August, 2017 [Date-wise Compendium]

knowledge, financial and employment opportunities for billions of people worldwide


There has been significant increase in broadband access and subscriptions, with China leading the way.
Youths (15-24 years old) are at the forefront of Internet adoption.
Least Developed Countries (LDCs): Up to 35% of individuals in LDCs are using the Internet are aged
15-24, compared with 13% in developed countries and 23% globally. LDCs saw the highest growth-rate of
mobile broadband subscriptions between 2012 and 2017. Despite this, the number of mobile subscriptions
per 100 inhabitants in LDCs is the lowest globally at 23%.
Fixed-broadband subscriptions: It has increased by 9% annually in the last five years with up to 330
million subscriptions added. There has been an increase in high-speed fixed broadband subscriptions parallel
to the growth in the number of fibre connections. Most of these subscriptions were in developing countries
accounting for 80 % of all fixed-broadband subscriptions.
Mobile broadband subscriptions: It has grown more than 20% annually in the last five years and is
expected to reach 4.3 billion globally by the end of 2017.
Internet user gender gap: It has narrowed in most regions since 2013. The proportion of men using the
Internet remains slightly higher than the proportion of women using the Internet in two-thirds of countries
worldwide. In 2017, the global Internet penetration rate for men stands at 50.9% compared to 44.9% for
women.
International Internet bandwidth: It grew by 32% between 2015 and 2016, with Africa registering an
increase of 72% during this period, the highest of all regions.
Global telecommunication revenues: It declined by 4% from US $2.0 trillion in 2014 to $1.9 trillion in
2015. Developing countries, which are home to 83% of the global population, generate 39% of the worlds
telecommunication revenues.
About International Telecommunication Union (ITU)
ITU is the UN specialised agency for information and communication technologies (ICTs). It has 193
countries as its members and also 800 private-sector entities and academic institutions. It is headquartered in
Geneva, Switzerland.
ITU is responsible for allocating global radio spectrum and satellite orbits. It also develops the technical
standards that ensure networks and technologies to seamlessly interconnect. It also strives to improve access
to ICTs among the underserved communities worldwide.
Scientists successfully use CRISPR to fix a genetic mutation causing
Hypertrophic Cardiomyopathy disease
Scientists for the first time have successfully repaired a genetic mutation in human embryos by using a gene-
editing tool called CRISPR-Cas9.
It has freed embryos of faulty DNA that causes deadly hereditary heart disease. It potentially opens the door
to preventing 10,000 disorders that are passed down the generations.

2017 GKToday | All Rights Reserved | www.gktoday.in

38
Current Aairs: August, 2017 [Date-wise Compendium]

Key Facts
Using CRISPR-Cas9 gene-editing technology, scientists were able to repair mutation in the MYBPC3 gene
which causes a common heart condition called hypertrophic cardiomyopathy, which is marked by
thickening of the heart muscle.
Hypertrophic cardiomyopathy is an inherited cardiac disease and the presence of even one copy of the
defective gene can cause symptoms, which usually manifest as heart failure. There is currently no cure for
the condition. The correction of the mutation in the embryo using CRISPR-Cas9 has ensured that the child
is born healthy and the defective MYBPC3 gene is not passed on to future generations.
How it was achieved?
The CRISPR-Cas9 gene-editing tool was used to remove the genetic mutation and introduce healthy gene in
sperm of man with hypertrophic cardiomyopathy. This sperm was later fertilised with eggs from 12 healthy
women. It was found that 42 out of the 58 fertilised embryos did not carry the mutation. The remaining 16
embryos had unwanted additions or deletions of DNA. It showed that probability of inheriting the healthy
gene increased from 50 to 72.4%.

CRISPR-Cas9 gene-editing technology


Using CRISPR system, scientists can remove, add or alter specific DNA sequences in the genome of higher
organisms. It has two components, a single-guide RNA (sgRNA) that contains a sequence that can bind to
DNA, and Cas9 enzyme which acts as a molecular scissor that can cut specific DNA sequence. In order to
2017 GKToday | All Rights Reserved | www.gktoday.in

39
Current Aairs: August, 2017 [Date-wise Compendium]

selectively edit a desired sequence in DNA, the sgRNA is designed to find and bind to the target.
Rising temperatures drive up farmer suicides in India: US study
According research report from the University of California, US, climate change may have led to over 59,000
farmer suicides over the last 30 years in India.
The study was carried out using data for all States and Union Territories. It tested the link between climate
change, crop yields and suicide by comparing the number of suicides across India between 1967 and 2013
with crop yield and climate data. Data on suicides were collected from the National Crime Records Bureau.
Key Highlights of Study
The increase in temperature during the cropping season reduces crop yields, resulting in increased suicides.
Even a 1C increase in temperature above 20 C in a single day during the crop growing season results in
about 70 suicides on average. Similar increase in temperature during other seasons did not result in a rise in
suicides.
Crop losses due to heat damage causes additional burden on farming households leading to suicides.
Moreover, increase in rainfall by 1 cm during the growing season decreases about 0.8 deaths per 100,000,
thus lowering the suicide rate by 7% on average.
The effect of climate variation has revealed that past growing season temperature strongly influences suicide
rates in the following years up to five years. For instance, abundant rainfall during one growing season leads
to dip in the suicide rates for the next two or three years. Drought does not have any effect on suicide rates.
South India, which is generally hotter, has higher farmer suicide rates. States where the yields are more
affected by high temperatures are also the States which report higher suicide rates. Maharashtra, Tamil
Nadu, Karnataka and Andhra Pradesh have severe suicide rates in responses to temperature and crop yield is
also more negatively affected by higher temperature.
Limitations of Study
The study has not looked at other factors that could have contributed to suicides. It did not find any adaptive
behaviour to prevent suicides in response to climate change.
India can construct Kishanganga, Ratle Hydro Power Plants: World Bank
World Bank has allowed India to construct Kishanganga, Ratle hydroelectric power facilities on tributaries
of the Jhelum and Chenab rivers with certain restrictions under the 1960 Indus Waters Treaty (IWT).
The World Banks comments came as officials from India and Pakistan concluded the secretary-level talks on
the technical issues of the IWT in Washington, US.
Key Facts
Pakistan had opposed the construction of the Kishanganga (Jhelum River) and Ratle (Chenab River)
hydroelectric power plants built by India in Jammu and Kashmir. Both countries had disagreed over the
technical design features of the two hydroelectric plants, as these two rivers are part of Western Rivers
along with Indus River under. IWT has given Pakistan full control over these three western rivers for
unrestricted use. Besides, it also allows India to construct hydroelectric power facilities on these rivers along
with other uses, subject to constraints specified in annexures to the treaty.
2017 GKToday | All Rights Reserved | www.gktoday.in

40
Current Aairs: August, 2017 [Date-wise Compendium]

Background
Due differences over these projects, Pakistan had asked World Bank to facilitate the setting up of a Court of
Arbitration to look into its concerns. On the other hand, India had asked for the appointment of a neutral
expert to look into the issues, citing that concerns raised by Pakistan were technical ones. After that,
representatives of the World Bank had held talks with India and Pakistan to find a way out separately.
About Indus Waters Treaty (IWT)
IWT is a water-distribution treaty between India and Pakistan which was brokered by the World Bank
(then the International Bank for

Reconstruction and Development). It deals with sharing of water of Indus water


system having six rivers Beas, Ravi, Sutlej, Indus, Chenab and Jhelum between
the two countries.
It was signed by then Prime Minister Jawaharlal Nehru and President of Pakistan Ayub Khan in Karachi on
September 19, 1960. It is most successful water treaty in world. Even, it has survived India-Pakistan wars of
1965, 1971 and the 1999 Kargil standoff besides Kashmir insurgency since 1990.
As per treaty, control over three eastern rivers Ravi, Beas and Sutlej was given to India. While control over
three western rivers Indus, Jhelum and Chenab was given to Pakistan. It allows India to use only 20% of the
water of Indus river, which flows through it first, for irrigation, power generation and transport.
Most disagreements and disputes have been settled via legal procedures, provided for within the framework
of the treaty. Under it, Permanent Indus Commission (PIC) was set up as a bilateral commission to
implement and manage the Treaty. The Commission solves disputes arising over water sharing. Besides,
treaty also provides arbitration mechanism to solve disputes amicably. The World Banks role in relation to

2017 GKToday | All Rights Reserved | www.gktoday.in

41
Current Aairs: August, 2017 [Date-wise Compendium]

disputes and differences with respect of IWT is limited to the designation of people to fulfil certain roles
when requested by either or both of the parties.
RBI cuts repo rate by 25 bps to 6%
The Monetary Policy Committee (MPC) of the Reserve Bank of India (RBI) has reduced short-term lending
rate, or repo rate, by 25 basis points to 6%. It was RBIs third bimonthly policy review for the financial year
2017-18.
The decision of the MPC was consistent with a neutral stance of monetary policy in consonance with the
objective of achieving the medium-term inflation target of 4% within a band of +/- 2%, while supporting
growth.
Policy Rates
Repo rate: It is the rate at which RBI lends to its clients generally against government securities. It was
reduced by 25 basis points to 6%. The rate cut comes after a slump in food prices in consumer inflation to a
record low of 1.54%.
Reverse Repo Rate: It is the rate at which banks lend funds to the RBI. It was reduced by 25 bps to 5.75%.
Marginal Standing Facility (MSF) Rate: It is rate at which the scheduled banks can borrow funds
overnight from RBI against government securities. It is a very short term borrowing scheme for scheduled
banks. It adjusted to 6.25%.
Bank Rate: It is rate charged by the central bank for lending funds to commercial banks. It was set to 6.25%.
It influences lending rates of commercial banks. Higher bank rate will translate to higher lending rates by the
banks.
Cash Reserve Ratio (CRR): It is the amount of funds that the banks have to keep with the RBI. It was
unchanged at 4%. The RBI uses the CRR to drain out excessive money from the system.
Statutory Liquidity Ratio (SLR): It was unchanged 20%. It is amount that banks have to maintain a
stipulated proportion of their net demand and time liabilities (NDTL) in the form of liquid assets like cash,
gold and unencumbered securities, treasury bills, dated securities etc.
About Monetary Policy Committee (MPC)
MPC is a committee of the central bank Reserve Bank of India, headed by its Governor. It was set up by
amending the RBI Act to provide for a statutory and institutionalised framework for MPC.
The 6 member MPC is entrusted with the task of fixing the benchmark policy interest rate (repo rate) to
contain inflation within the target level. The majority voice of the committee will be final in deciding the
interest rates.
Composition of MPC includes Governor of RBI (ex officio Chairperson), Deputy Governor of RBI, in
charge of Monetary Policy (Member), one officer of RBI (Member) and three members appointed by Central
Government as members. Each member has one vote and governor has casting vote in case of tie.
August 3, 2017

2017 GKToday | All Rights Reserved | www.gktoday.in

42
Current Aairs: August, 2017 [Date-wise Compendium]

Indian Army develops Humraaz app for soldiers to track promotions, postings
The Indian Army has developed Humraaz mobile application through which serving soldiers can track
details like postings and promotions. The app will also enable soldiers to view their monthly salary slips,
Form 16 and also download them.
Key Facts
The Humraaz mobile app has been developed in-house by the Army. For security reasons, its installation has
been linked to verification of Aadhar details. On installing the app, serving soldiers will be verified with the
Army database over National Information Centre (NIC) cloud and they will get a one-time-password (OTP)
on their registered mobile number. For accessing the app, soldiers have to link their latest mobile number to
their Aadhar number.
Union Cabinet approves MoU between India and Spain on cooperation in
renewable energy
The Union Cabinet has approved Memorandum of Understanding (MoU) between India and Spain on
India-Spain Cooperation in renewable energy. The MoU was signed in May 2017 at Spain.
The MoU aims at strengthening bilateral cooperation between both countries with the exchange of expertise
and networking of information in field of in renewable energy.
Key Facts
Under the MoU, both sides will establish the basis for a cooperative institutional relationship to encourage
and promote bilateral cooperation on new and renewable energy issues on the basis of mutual benefit
equality and reciprocity. The MoU also envisages establishing a Joint Working Committee (JWG) to review,
monitor and discuss matters relation to areas of cooperation.
Lok Sabha passes two key bills for implementation of GST in J&K
The Lok Sabha has passed the Central Goods and Services Tax (Extension to Jammu and Kashmir) Bill, 2017
and the Integrated Goods and Services Tax (Extension to Jammu and Kashmir) Bill, 2017.
The two bills will replace the ordinances promulgated earlier in this regard to complete rolling out GST
regime the Himalayan State.
The first bill provides for the extension of the Central Goods and Services Tax (CGST) Act, 2017 to J&K and
the other also provides for the extension of the Integrated Goods and Services Tax (IGST) Act, 2017 to J&K.
Background
The Central government earlier had promulgated ordinances to make the Central GST and Integrated GST
(IGST), which deals with inter-state commerce, applicable to all states. However, provisions of these Central
laws were not applicable to J&K unlike the rest of India, as the state is having a special constitutional status.
Under this different constitution procedure, President Pranab Mukherjee had promulgated two ordinances
regarding the introduction of the GST in J&K.
Cabinet approves MoU to set up BRICS Agriculture Research Platform
The Union Cabinet has approved for a Memorandum of Understanding (MoU) signed among India and
2017 GKToday | All Rights Reserved | www.gktoday.in

43
Current Aairs: August, 2017 [Date-wise Compendium]

various BRICs countries for establishment of the BRICS Agriculture Research Platform (BRICS-ARP).
The BRICS-ARP will promote sustainable agricultural development and poverty alleviation through
strategic cooperation in agriculture to provide food security in the BRICS member countries.
BRICS Agriculture Research Platform (BRICS-ARP)
BRICS-ARP will serve as a global platform for science-led agriculture-based sustainable development for
addressing the issues of world hunger, under-nutrition, poverty and inequality (between farmers and non-
farmers income) and enhancing agricultural trade, climate resilient agriculture and bio-security. It will further
intensify cooperation among BRICS countries in agricultural research policy, science and technology,
innovation and capacity building, including technologies for small-holder farming in the BRICS countries
Background
The establishment of BRICS-ARP was proposed by Prime Minister Narendra Modi during the 7th BRICS
Summit held in July 2015 at Ufa, Russia. MoU on establishment of this platform was signed by the foreign
Ministers of BRICS countries in the 8th BRICS Summit held in October, 2016 at Goa, India.
SEBI constitutes TK Viswanathan committee on fair market conduct
Market regulator Securities and Exchange Board of India (SEBI) has set up a committee on fair market
conduct. It will be headed former law secretary T K Viswanathan.
The committee will suggest measures for improving surveillance of the markets and strengthen rules for
algorithm trades, among other norms.
Its members include representatives from law firms, mutual funds, retail and institutional brokers, forensic
auditing firms, foreign portfolio investors, stock exchanges, chambers of commerce, data analytics
companies and the markets regulator.
Need
The securities market environment is dynamic, so there is need for periodic review of regulations and
surveillance mechanisms in order to effectively discharge the objectives of SEBI.
Terms and Reference of Committee
The committee will suggest measures for improvement in PFTUP (Prohibition of Fraudulent and Unfair
Trade Practices) regulations, PIT (Prohibition of Insider Trading) norms and norms mainly related to
trading plans and handling of unpublished price sensitive information during takeovers.
It will suggest short term and medium term measures for improved surveillance of the markets as well as
issues of high frequency trades, harnessing of technology and analytics in surveillance. It will suggest
evidentiary issues in anti-fraud enforcement. It will be also responsible for recommending steps to align
insider trading regulations with Companies Act provisions.
About Securities and Exchange Board of India (SEBI)
SEBI is the statutory regulator for the securities market in India established in 1988. It was given statutory
powers through the SEBI Act, 1992. Its mandate is to protect the interests of investors in securities, promote
the development of securities market and to regulate the securities market.
SEBI is responsive to needs of three groups, which constitute the market, issuers of securities, investors and
2017 GKToday | All Rights Reserved | www.gktoday.in

44
Current Aairs: August, 2017 [Date-wise Compendium]

market intermediaries. It has three functions quasi-legislative (drafts regulations in its legislative capacity),
quasi-judicial (passes rulings and orders in its judicial capacity) and quasi-executive (conducts investigation
and enforcement action in its executive function).
Union Cabinet clears proxy voting for NRIs
The Union Cabinet has cleared a proposal to extend proxy voting to Non-Resident Indians (NRIs) and
overseas Indians. Currently, only service personnel are permitted to vote through proxy.
To make proxy voting for NRIs a reality, government is going to introduce bill to amend the Representation
of People Act.
Key Facts
This decision will enable nearly 16 million NRIs to cast vote their in Indian elections (assembly and Lok
Sabha elections) by appointing proxies residing in their constituencies. Overseas electors will have to
appoint a nominee afresh for each election one person can act as proxy for only one overseas voter.
Government (Election Commission) will frame suitable rules and guidelines to make this move effective
soon.
Need for Proxy Voting
Currently, Indian voters residing abroad can only cast their votes in their respective constituencies where
they are registered. This regulation is seen as restrictive as only a few thousand Indians living overseas have
registered themselves as voters (maximum from Kerala). Of these, barely anyone has travelled to the country
to exercise his or her franchise during the elections.
Expert committee Recommendation
An expert committee in the Election Commission of India (ECI) working on the issue in 2015 had forwarded
the legal framework to the Union Law ministry to amend electoral laws to allow overseas Indians use proxy
voting.
Comment
According to rough estimates, there are about 1 crore Indians settled abroad, of which 60 lakh may be within
eligible voting age. By granting them proxy voting rights, NRIs will be able to exercise franchise during
elections and also need not to spend foreign currency to come to India during elections. Moreover, this
decision will also enable NRIS and overseas Indian to considerably sway in election results, especially in
states such as Punjab, Kerala and Gujarat where a number of expats hail from.
BRICS nations vow to fight protectionism
India and other four other members of the BRICS grouping members (Brazil, Russia, China and South
Africa) have pledged to fight protectionism.
Decision in this regard was taken at the two-day meeting of the trade ministers from BRICS nations held at
Shanghai, China. They released joint statement highlighting various issues and need to address them
collectively.
Highlights of Joint statement
BRICS countries should jointly tackle the challenges of economic globalisation, promote open and equitable
2017 GKToday | All Rights Reserved | www.gktoday.in

45
Current Aairs: August, 2017 [Date-wise Compendium]

world economy. They should uphold the common interests of the emerging markets and developing
countries with a view to promoting strong, sustainable, balanced and inclusive growth.
BRICS agreed to firmly oppose protectionism and pledged to annul any protective measures, calling on other
countries to follow the same suit. They also encouraged more countries to participate in the World Trade
Organisation (WTO), urging the institution to show more commitment to receiving the least developed
countries (LDCs). They also agreed to play a major part in global economic governance.
About BRICS
BRICS is the acronym for an association of five major emerging national economies: Brazil, Russia, India,
China and South Africa. It was established in 2009 as BRIC. In 2011, South Africa joined this informal group
and BRIC became BRICS. The first formal BRICS summit was held in Yekaterinburg, Russia in 2009. So far,
eight such summits have taken place.
The BRICS members are distinguished by their large, fast-growing economies and significant influence on
regional and global affairs. The BRICS countries are home to 42% of the worlds population. Their total
share in the global economy has risen from 12% to 23% in the past decade and collectively contribute they
more than half of global growth. They all developing or newly industrialised countries and all five are G-20
members. They are
Israel launches first environmental research satellite Venus
Israel has launched its first environmental research satellite named as Venus (Vegetation and Environment
Monitoring New Micro-Satellite). It is a joint venture between the Israel Space Agency (ISA) and its French
counterpart CNES.
It was launched on board of Arianespace Vega launcher from the European spaceport in Kourou, French
Guiana.
Key facts
The Venus satellite is an earth-observation micro-satellite. It is considered the smallest satellite of its kind in
the world. It weighs only 265 kilogram. It will be placed in sun-synchronized orbit at an altitude of 720
kilometers.
It will circle the earth 29 times in each 48-hour period and will stay in commission for 4.5 years. The mini-
satellite is equipped with a special camera that can visualize details on Earth that are not visible to the naked
eye.
Venus Satellite has a dual mission-scientific and technological. The scientific mission will monitor Earths
vegetation using a camera capable of recording 12 narrow spectral bands. The technological mission will test
the operation of an innovative electric propulsion system based on the Israeli-designed Hall Effect Thrusters
(HET). HET is a relatively low power device used to propel a spacecraft after entering orbit or farther out
into space.
Applications: Venus satellite will be used to survey and monitor large areas to study soil, vegetation,
forests, agriculture, water and air quality and other aspects of the environment. It will help to obtain high-
resolution photographs of specific sites to track environmental issues such as erosion, desertification,
2017 GKToday | All Rights Reserved | www.gktoday.in

46
Current Aairs: August, 2017 [Date-wise Compendium]

pollution, natural disasters, and other phenomena linked to climate change.


Lok Sabha passes Banking Regulation (Amendment) Bill, 2017
The Lok Sabha has passed the Banking Regulation (Amendment) Bill, 2017 by voice vote. It will replace the
Banking Regulation (Amendment) Ordinance, 2017.
The bill seeks to amend the Banking Regulation Act, 1949 to insert provisions for handling cases related to
stressed assets or non-performing assets (NPAs) of banks. Stressed assets (NPAs) are loans defaulted by
borrower in repayment or the loan which has been restructured by changing the repayment schedule.
Key Features of the Bill
Initiating insolvency proceedings: It will enable the Central government to authorize the Reserve Bank
of India (RBI) to direct banking companies to resolve specific stressed assets by initiating insolvency
resolution process. These proceedings will be under the Insolvency and Bankruptcy Code, 2016.
Issuing directions on stressed assets: It empowers RBI to issue directions to banks for resolution of
stressed assets from time to time.
Committee to advise banks: It enables RBI to specify committees or authorities to advise banks on
resolution of stressed assets. RBI will appoint or approve members on such committees.
Applicability to State Bank of India (SBI): It inserts provision to make above provisions applicable to the
SBI and its subsidiaries and also Regional Rural Banks (RRBs).
Need for Amendment
Non-performing assets (NPAs) or bad loans of banks have risen to over Rs. 9 lakh crore resulting in choking
the banking system. So it had become necessary for the RBI to intervene in order to take urgent measures for
their speedy resolution.
Lok Sabha passes NABARD (Amendment) Bill, 2017
The Lok Sabha has passed the National Bank for Agriculture and Rural Development (Amendment) Bill,
2017 by voice vote. The Bill seeks to amend the NABARD Act, 1981.
NABARD is responsible for providing and regulating facilities like credit for agricultural and industrial
development in the rural areas.
Key Features of the Bill
Increase in capital of NABARD: The Bill allows Union Government to increase capital of NABARD to Rs
30,000 crore from Rs. 5000 crore. Further, it allows Union Government to increase it to more than Rs
30,000 crore in consultation with the RBI, if necessary.
Transfer of the RBIs share to Union government: The Bill provides that the Union Government alone
must hold at least 51% capital share of NABARD. Further, it transfers share capital held by the RBI valued at
Rs 20 crore to the Union Government. Currently RBI holds 0.4% of the paid-up capital of NABARD and the
remaining 99.6% is held by the Union government and this causes conflict in the RBIs role as banking
regulator and shareholder in NABARD.
Adds Micro, small and medium enterprises (MSME) terms: The Bill replaces the terms small-scale

2017 GKToday | All Rights Reserved | www.gktoday.in

47
Current Aairs: August, 2017 [Date-wise Compendium]

industry and industry in the tiny and decentralised sector with the terms micro enterprise, small
enterprise and medium enterprise as defined in MSME Development Act, 2006. Further, it allows
NABARD to provide financial assistance to banks if they provide loans to the MSMEs.
Consistency with the Companies Act, 2013: The Bill substitutes references to provisions of the
Companies Act, 1956 with references to the Companies Act, 2013. It includes provisions dealing with
definition of a government company and qualifications of auditors.
August 4, 2017
Qatar to become first Arab state to offer permanent residency to some non-
citizens
Qatars Cabinet has approved a draft law making permanent residency available to some non-citizens. It
marks a partial shift from Qatars heavy reliance on its longstanding visa-sponsorship system.
With this the energy-rich Qatar becomes first Gulf Arab state to make permanent residency available to
some non-citizens.
Key Facts
Under the new law, residency permits will be granted to children of Qatari women married to foreigners.
Moreover it will be also granted to people who offered valuable services to the country and those with
special skills. The residency permit will give holders similar rights as Qataris in terms of property ownership
and run some businesses without needing a Qatari partner. It will also entitle them free state education and
healthcare services. The interior ministry of Qatar will establish committee to review requests of granting
permanent residency ID in line with the provisions of the law.
Background
Gulf Arab countries have a high number of expatriate workers but do not allow naturalization of foreigners
except in rare cases and under strict conditions. Qatar which is worlds wealthiest country per capita has
population of 2.7 million including some 300,000 citizens. Qatars native population is far surpassed by
foreigners, so the energy-rich Arab state is reluctant to extend residency rights out of concern for the
demographic balance.
Maharashtra Government launches two online portals MahaDBT, MahaVASTU
Maharashtra Government has launched two online portals MahaDBT and MahaVASTU to streamline the
process of direct benefit transfer (DBT) and to bring in more transparency in construction sector.
With this, Maharashtra became first state in the country to launch such a system that will provide direct
benefits with Aadhaar authentication.
Key Facts
MahaDBT portal: It is an Aadhaar-authenticated electronic mechanism that will enable direct transfer of
benefits for over 40 schemes currently implemented by states various departments. It is based on Union
Governments DBT scheme and Aadhaar will be mandatory to claim benefits through this portal. It will help
to speed up transfer of benefits and ensure transparency by minimising human intervention.
2017 GKToday | All Rights Reserved | www.gktoday.in

48
Current Aairs: August, 2017 [Date-wise Compendium]

MahaVASTU portal: It is an online building permission management system, through which construction
approvals will be sanctioned with full transparency. It will bring all building permissions online in all
municipal councils of the state.
Suns core rotates 4 times faster than its surface: Study
A team of researchers have discovered solar seismic waves which revealed that Suns core is rotating four
times faster than its surface. Earlier it was assumed that suns core rotate at same speed as the surface.
The discovery was made using 16 years of observations from GOLF (Global Oscillations at Low Frequency)
instrument on Solar and Heliospheric Observatory (SOHO) spacecraft, a joint project of ESA and NASA.
Key Facts
Researchers had use Helioseismology to probe the solar interior by studying sound waves reverberating
through it. They had studied surface acoustic waves in the Suns atmosphere, some of which penetrate to the
Suns core, where they interact with low frequency gravity waves (g-waves) known as g-modes that have a
sloshing motion.
From those observations, they detected the sloshing motions of the solar core. By carefully measuring the
acoustic waves, the researchers precisely determined the time it takes an acoustic wave to travel from the
surface to the centre of the Sun and back again.
On the basis of the signature of the g-waves, researchers determined that the g-waves are shaking the
structure of the suns core. The signature of the imprinted g-waves suggested that the inner core of the Sun
is rotating once every week, nearly four times faster than the observed surface and intermediate layers.
Significance of Discovery: The rotation of the Suns core may give a clue to how the Sun formed. It may
also help to find linkages between the Suns core rotation and sunspots.
Parliament passes IIIT (Amendment) Bill, 2017
Parliament has passed the Indian Institutes of Information Technology (Amendment) Bill, 2017 after the
Rajya Sabha approved it. The Lok Sabha has already passed it. The Bill amends the IIIT Act, 2014.
The IIIT Act, 2014 declares certain Institutes of Technology as institutions of national importance. It aims to
develop new knowledge in information technology and provide manpower of global standards for the
information technology industry.
Key Features of the Bill
Appointment of Director: The Bill modifies the composition of the search-cum-selection committee
which recommends names to the central government for the appointment of the Director of an IIIT.
Further, it replaces the Director of an IIIT with the Director of an Indian Institute of Technology.
Appointment of Assistant Professor and other above posts: The Bill permits the Board of Governor of
Institutes to appoint Assistant Professors and all the posts above that level as well.
Incorporation of an institute: It declares the Indian Institute of Technology, Design and Manufacturing
(ITDM), Kurnool in Andhra Pradesh as an institution of national importance. Further it adds ITDM,
Kurnool to the Schedule of the parent Act which consists of other institutions of national importance.

2017 GKToday | All Rights Reserved | www.gktoday.in

49
Current Aairs: August, 2017 [Date-wise Compendium]

Government proposes to set up 14 new AIIMS hospitals under PMSSY


The Union Government has proposed to establish an additional 14 new AIIMS (All India Institute of
Medical Science) under various phases of Pradhan Mantri Swasthya Suraksha Yojana (PMSSY) in different
parts of the country.
It was announced by Union Health Minister JP Nadda while replying to supplementaries in Lok Sabha on
AIIMS and related healthcare system in India.
Besides, Lok Sabha was also informed that under the national programme for prevention and control of
cancer, diabetes, cardio-vascular diseases and strokes, Government is going to the establish 20 state cancer
institutes and 50 tertiary care cancer centres in different parts of the country.
About Pradhan Mantri Swasthya Suraksha Yojana
Pradhan Mantri Swasthya Suraksha Yojana (PMSSY) was announced in 2003. Its objectives are correcting
regional imbalances in the availability of affordable/reliable tertiary healthcare services and augment
facilities for quality medical education in the country. It establishes AIIMS in various regions of India apart
from different government colleges. It is funded from different centrally sponsored schemes relating to
creating infrastructure on health.
ISRO, NASA working towards realisation of NISAR Mission by 2021
ISRO and NASA are jointly working on the NASA-ISRO Synthetic Aperture Radar (NISAR) mission to
co-develop and launch a dual frequency synthetic aperture radar (SAR) satellite by 2021.
The satellite will be the worlds most expensive earth-imaging satellite till date, costing around $1.5
billion. It aims to study global environmental change and natural disasters.
Salient Features
NISAR is a dual frequency (L & S Band) Radar Imaging Satellite. It will be the first radar
imaging satellite to use dual frequency.
In this joint mission, NASAs Jet Propulsion Laboratory (JLP) will be responsible for design
and development of L-band SAR, GPS system, 12m unfurlable antenna and data recorder.
ISRO will be responsible for design and development of S-band SAR, Spacecraft Bus,data
transmission system, spacecraft integration & testing, launch using GSLV and on-orbit
operations.
Mission Life: It is expected to be launched in year 2021. It will have mission life of 3 years.
Operational orbit: The satellite is planned to be launched into a Sun-synchronous dawn
to dusk orbit.
Applications: It will provide an unprecedented detailed view of Earth by taking snapshots
every week using advanced radar imaging.
It is designed to observe and take measurements of some of the planets most complex
processes, including ecosystem disturbances, natural hazards such as earthquakes, tsunamis,
2017 GKToday | All Rights Reserved | www.gktoday.in

50
Current Aairs: August, 2017 [Date-wise Compendium]

volcanoes and landslides.


It will be used also for natural resources mapping & monitoring, estimating agricultural
biomass over full duration of crop cycle.
It will be also monitor floods and oil slicks, coastal erosion, coastline changes and variation of
winds in coastal waters, surface deformation studies due to seismic activities etc.
Implementation Process: Implementation Arrangement (IA) of mission, defining roles and
responsibilities of ISRO and NASA was signed between two space agencies in September
2014.
So far, ISRO has completed Baseline Design Reviews of satellite and S-band SAR payload.
NASAs JLP has successfully completed Mission Concept and Key Decision Point reviews.
The first Joint Steering Group (JSG) meeting of NISAR was held in July 2015.
Cost of the project: Costing around $1.5 billion, it will be worlds most expensive earth-
imaging satellite till date.
Space Diplomacy: The mission will establish a general pathway for future joint missions
between ISRO and NASA for Mars exploration. It will also further strengthen the Indo-US
ties.
Indias first private missile production facility unveiled in Hyderabad
Indias first private sector missile sub-systems manufacturing facility Kalyani Rafael Advanced Systems
(KRAS) plant was inaugurated near Hyderabad, capital of Telangana.
It is 51:49 joint venture between Indias Kalyani Group and Israels Rafael Advanced Defence Systems Ltd. It
has been established line with the Make in India initiative and Governments policy to encourage private
sector participation in defence production.
Key Facts
KRAS aims to be a one-stop solution provider to locally re-design, develop, re-engineer and manufacture
various land and airborne products and systems in India. It has invested in designing, developing and
manufacturing of weapon systems including Spike Anti-Tank Guided Missile (ATGM) systems and remote
weapon systems.
Besides, it will develop a wide range of advanced capabilities that include command control and guidance,
remote weapon systems, electro-optics, precision guided munitions and system engineering for system
integration. It will be supplying defence equipment to the Indian Army and also export to South East Asian
countries.
The plant would employ more than 300 engineers and provide indirect employment to 1,000 people. The
localisation content of the plant is 90% and most of the vendors are in and around Hyderabad.
Recommendations: Justice Srikrishna committee on Institutionalization of

2017 GKToday | All Rights Reserved | www.gktoday.in

51
Current Aairs: August, 2017 [Date-wise Compendium]

Arbitration Mechanism
A High Level Committee set up to review the Institutionalization of Arbitration Mechanism in India has
submitted its report to the Union Law Ministry. It was headed by Justice (retired) BN Srikrishna.
The Union Government is committed for speedy resolution of commercial disputes and to make India an
international hub of Arbitration. So government has decided to look into the recommendations and
amended laws according to need.
Key Recommendations
Arbitration Promotion Council of India (APCI): The committee has recommended for setting up APCI,
an autonomous body having representatives from all stakeholders for grading arbitral institutions in India.
The APCI may recognize professional institutes providing for accreditation of arbitrators. It may be also
empowered to hold training workshops and interact with law firms and law schools to train advocates with
interest in arbitration and with a goal to create a specialist arbitration bar comprising of advocates dedicated
to the field.
Specialist Arbitration Bench: It also recommended for creation of a specialist Arbitration Bench to deal
with such Commercial disputes, in the domain of the Courts.
Changes in Arbitration and Conciliation Act: It has suggested in various changes inprovisions of the
2015 Amendments in the Arbitration and Conciliation Act with a view to make arbitration speedier and
more efficacious and incorporate international best practices.
National Litigation Policy (NLP): The Committee also has opinion that the NLP must promote
arbitration in Government Contracts.
August 5, 2017
Government to launch AGRI UDAAN-Food and Agribusiness Accelerator 2.0
programme
The Union Government is going to launch AGRI-UDAAN Food and Agribusiness Accelerator 2.0
programme in an attempt to promote innovation and entrepreneurship in agriculture. It will mentor
startups and help them connect with potential investors.
Key facts
The programme will help convert innovative ideas from Indias rural youth into viable businesses startups
and help them connect with potential investors. It aims to attract the youth from rural India and elsewhere,
and train them so they can add value to the farmers produce.
It is managed by ICAR-NAARMs (Indian Council of Agricultural Research-National Academy of
Agricultural Research Management) technology incubator, a-IDEA along with IIM Ahmedabads Centre for
Innovation.
Under the programme, start-ups will get incubation space to run their businesses and have access to research
laboratories and libraries. It will also help the selected start-ups with regulatory services like company
registration and environmental compliances.
2017 GKToday | All Rights Reserved | www.gktoday.in

52
Current Aairs: August, 2017 [Date-wise Compendium]

In the first round of the programme 40 start-ups will be shortlisted who will pitch their ideas to a panel of
evaluators. Out of these, 8 to 12 start-ups will be selected for the final capacity building workshop. Following
an intensive training lasting six months, the new start-ups will be connected to investors for funding.
India signs loan pact with AIIB for Rural Roads Project
India has signed a loan agreement with China led-Asian Infrastructure Investment Bank (AIIB) for financing
of $329 million for Gujarat Rural Roads Project (GRRP).
The objective of the GRRP is to improve the rural road connectivity and accessibility by providing all
weather road connectivity to 1,060 villages in all the 33 districts of Gujarat, benefiting about 80 lakh people.
It will also benefit the service providers such as public transport operators, hospitals, educational
institutions, local markets and traders.
About Asian Infrastructure Investment Bank (AIIB)
AIIB is a multilateral development bank backed by China, seen as a rival to the World Bank and Asian
Development Bank (ADB). It was established to fund various infrastructure projects including energy,
transportation, urban construction and logistics as well as education and healthcare in Asia-Pacific region.
The AIIB was officially established with 57 founding members and started to function in January 2016. But as
of May 2017, the bank has 52 members. It has an authorised capital of US $100 billion. It is headquartered in
Beijing.
Voting Share: China is the largest shareholder with 26.06% voting shares. India is the second largest
shareholder with 7.5% voting shares followed by Russia (5.93%) and Germany (4.5%).
ISRO and CSIR-NPL sign MoU for time and frequency traceability services
The ISRO Telemetry Tracking and Command Network (ISTRAC) and CSIR-National Physical Laboratory
(NPL) have signed MoU to make indigenous regional positioning system-NavIC independent from the US
clock system.
Until now, the satellites on NavIC (Navigation with Indian Constellation) managed by the ISRO relied on
the US GPS to ensure that the clocks aboard were working at the high-precision required in satellite-based
communication.
Significance of MoU
The MoU will help the NAVIC to get formally synchronized with the Indian Standard Time (IST) which is
being maintained by the Delhi-based NPL the timekeeper of India. It will also help in making NAVIC to
get fully operational in the market for commercial purposes as time synchronisation is essential for all kinds
of services such as financial transactions, digital archiving, stock handling, time stamping, national security
or prevention of cyber-crimes etc.
NavIC
Formally called the Indian Regional Navigation Satellite System (IRNSS) is designed to provide accurate
position information service to users in India as well as the region extending up to 1500 km from its
boundary. NavIC System consists of constellation of seven satellites (namely IRNSS-1A, 1B, 1C, 1D, 1E, 1F

2017 GKToday | All Rights Reserved | www.gktoday.in

53
Current Aairs: August, 2017 [Date-wise Compendium]

and 1G) of which three are geostationary and four are non-geostationary.
National Physical Laboratory (NPL)
The NPL is part of the Council of Scientific & Industrial Research (CISR). It is the measurement standards
laboratory of India. It maintains standards of SI units in India and calibrates the national standards of
weights and measures. It is only agency in India authorised to maintain Indian Standard Time (IST) and
hosts most accurate clocks in the country with accuracy of 20 nanoseconds through the Primary Time Scale
(PTS), an ensemble of five caesium clocks and one hydrogen maser.
Indian Railways signs first EPC contract with L&T to speed up electrification
The Indian Railways has signed its first EPC (engineering, procurement, construction) contract with Larsen
and Toubro (L&T) for electrification of railway lines.
The contract valuing Rs 1,050 crore was awarded by Central Organisation for Railway Electrification
(CORE) and Konkan Railways for electrification of 781 route kilometres (RKM). It will cover Delhi Sarai
Rohilla Rewari and Alwr Bandikui Jaipur Phulera Ajmer (353 rkm) section and Roha Verna (428
rkm) of Konkan Railway.
Besides, eight such agreements were also exchanged between Zonal Railways and Public Sector
Undertakings (PSUs) for 1,735 rkm of electrification projects valuing Rs 1,746 crore.
Background
The EPC contracts were awarded as part of a strategy to speed up the electrification of railway lines under
Mission Electrification plan unveiled November 2016. So far only 42% of the lines are currently electrified.
The Rail ministry is planning to increase its electrified network to 52,400 rkm by 2021. The Indian Railways
is expecting to cut its fuel bill by Rs. 3,300 crore annually by 2020-21, by electrifying 90% of all broad gauge
tracks under Mission Electrification plan. In 2017-18 Budget, it was announced the Railways would tap the
EPC route for expediting electrification projects.
Comment
Indian Railways consumes 2% of the countrys total power generation. The power could be effectively
utilized in other sectors. The Mission Electrification plan will help in reducing the energy bill of railways
considerably. It would also make the travel through Railways faster.
Union Government launches Bharat 22 ETF to sell stakes in 22 firms
The Union Finance Ministry has launched second exchange-traded fund (ETF), named Bharat 22. It will
help to speed up Governments disinvestment programme budgeted to raise a record Rs 72,500 crore in the
FY 2018.
Bharat 22 comprise of 22 stocks including those of central public sector enterprises (CPSEs), public sector
banks (PSBs) and its holdings under the Specified Undertaking of Unit Trust of India (SUUTI).
Exchange-traded funds (ETFs)
Exchange-traded funds (ETFs) are essentially index funds that are listed and traded on exchanges like stocks.
They are basically basket of stocks with assigned weights that reflects the composition of an index. They are

2017 GKToday | All Rights Reserved | www.gktoday.in

54
Current Aairs: August, 2017 [Date-wise Compendium]

similar to mutual funds in a certain manner but are more liquid as they can be sold quickly on stock exchanges
like shares.The ETFs trading value is based on the net asset value of the underlying stocks that it represents
Key Facts
Bharat 22 is a well-diversified ETF spanning six sectors basic materials, energy, finance, industrials, FMCG
and utilities. The sector wise weightage in the Bharat 22 Index is basic materials (4.4%), energy (17.5%),
finance (20.3%), FMCG (15.2%), industrials (22.6%), and utilities (20%).
The banking segment includes stocks from State Bank of India (SBI), Axis Bank, Bank of Baroda (BoB),
Indian Bank, Rural Electrification Corporation and Power Finance Corporation. The energy segment
includes Oil and Natural Gas Corporation (ONGC), Indian Oil Corporation (IOC), Bharat Petroleum (BP),
and Coal India.

2017 GKToday | All Rights Reserved | www.gktoday.in

55
Current Aairs: August, 2017 [Date-wise Compendium]

Background
The first CPSE ETF was launched in March 2014. The first CPSE ETF consisted of stocks of 10 public sector
entities. It is currently managed by Reliance Capital Ltd. Government was able to raise Rs. 8,500 crore by
selling it in three tranches.

2017 GKToday | All Rights Reserved | www.gktoday.in

56
Current Aairs: August, 2017 [Date-wise Compendium]

Venkaiah Naidu elected 13th Vice President of India


Former union minister Muppavarapu Venkaiah Naidu (68) was elected as 13th Vice President of India. He
will succeed Hamid Ansari, who held the position for two consecutive terms of 5 years.
With this victory, BJP has occupied top three constitutional posts of India- President, Vice-President and
Prime Minister.
Background
Over 98% voting was recorded in the 2017 vice-presidential election. 98.21% polling was registered for the
election with 771 out of 785 MPs casting their vote, 11 votes were found to be invalid. 381 votes was the
quota set to win the polls. M. Venkaiah Naidu emerged victorious by winning 516 votes, whereas opposition
nominee Gopalkrishna Gandhi has secured 244 votes.
About M. Venkaiah Naidu
Venkaiah Naidu was born on July 1, 1949 in Nellore district of Andhra Pradesh in farmers family. He holds a
law degree with a specialisation in international law from the Andhra University College of Law in
Vishakhapatnam.
He had first entered in the mainstream politics with 1972 Jai Andhra Movement and later joined BJP in
1980. He won the assembly election twice in 1978 and 1983 from Udayagiri constituency and became the
Rajya Sabha MP thrice in 1998, 2004 and 2010. He was elected as the national President of BJP in 2002.
He served as a cabinet minister both NDA governments under the Atal Bihari Vajpayee (1999-2004) and
Narendra Modi (2014-present). He was Cabinet Minister of Information and Broadcasting, Minister of
Urban Development and also as a Parliamentary Affairs Minister under Modi government.
Vice-President Elections
The Vice-President is selected through a secret ballot by the members of the electoral college consisting of
the members of both Houses of Parliament (Lok Sabha and Rajya Sabha) in accordance with the system of
proportional representation by means of the single transferable vote. The nominated Members of Rajya
Sabha as well as of Lok Sabha are also eligible to be included in the Eeectoral college and, therefore, are
entitled to participate in the election.
Economist Rajiv Kumar appointed as Vice Chairman of NITI Aayog
The Union Government has appointed economist Rajiv Kumar as the new vice-chairman of National
Institution for Transforming India (NITI) Aayog.
Rajiv Kumar will succeed Arvind Panagariya. As Vice Chairman of NITI Aayog he will hold cabinet minister
rank. Prime Minister is ex-officio Chairperson of the Aayog.
Government also appointed Vinod Paul, head of the Pediatrics department at AIIMS as a member of the
NITI Aayog.
About Rajiv Kumar
Rajiv Kumar holds a DPhil in economics from Oxford and PhD from Lucknow University. He is a senior
fellow at public policy think-tank Centre for Policy and Research (CPR). He has authored several books on
the Indian economy and Indias national security.
2017 GKToday | All Rights Reserved | www.gktoday.in

57
Current Aairs: August, 2017 [Date-wise Compendium]

Kumar also has served as Secretary General of the Federation of Indian Chambers of Commerce and
Industry (FICCI). He had also held positions as Director and Chief Executive of Indian Council for Research
on International Economic Relations (ICRIER) and chief economist of the Confederation of Indian
Industries (CII), as well as in positions with the Asian Development Bank (ADB), Indian Ministry of
Industries, and Ministry of Finance.
Moreover, he is also member of boards of several international and national institutions, including the King
Abdullah Petroleum Studies and Research Centre in Riyadh, Economic Research Institute for ASEAN and
Asia (ERIAA) in Jakarta, State Bank of India (SBI), and the Indian Institute of Foreign Trade (IIFT). He was
also member of the Indian Governments National Security Advisory Board (NSAB) from 2006 and 2008.
NITI Aayog
NITI Aayog is the premier policy Think Tank of the Union Government, providing both directional and
policy inputs. It had replaced the 65-year old Planning Commission established in 1950.
August 6, 7, 2017
Aparna appointed as Executive Director of World Bank
The Appointments Committee of the Cabinet (ACC) has appointed senior IAS officer from Gujarat S.
Aparna as Executive Director of the World Bank.
At the World Bank, she will represent the constituency of India, Bangladesh, Bhutan and Sri Lanka for three
years. She will replace Subhash Garg, who was recently appointed Economic Affairs Secretary, Union
Ministry of Finance.
About S. Aparna
She is 1988-batch IAS officer, currently Principal Secretary to Gujarat Chief Minister Vijay Rupani. Besides,
she also has senior positions in Gujarat such as Economic Affairs, Surat Municipal Commissioner and Joint
Secretary, Urban Development at centre.
About World Bank
World Bank is one of five institutions created at the Breton Woods Conference in 1944. It is part of the
United Nations system, but its governance structure is different. Its headquarter is situated at Washington
DC. World Bank provides loans to developing countries for capital programmes.
World Bank comprises only two institutions viz. the International Bank for Reconstruction and
Development (IBRD) and International Development Association (IDA). In contrast, World Bank Group
comprises three more viz. Multilateral Investment Guarantee Agency (MIGA), International Finance
Corporation (IFC) and International Centre for Settlement of Investment Disputes (ICSID).
August 7, 2017
7 August: National Handloom Day
The National Handloom Day is being observed every year on 7 August to honour the handloom weavers in
the country and also carter an impetus to Indias handloom industry. This year it was third edition of the
National Handloom Day after it was instituted in 2015.
2017 GKToday | All Rights Reserved | www.gktoday.in

58
Current Aairs: August, 2017 [Date-wise Compendium]

The celebration of the day seeks to highlight the contribution of handloom to the socioeconomic
development of the country and promote handlooms to increase income of weavers and also enhance their
pride.
Background
The Union Government had declared 7 August as the National Handloom Day in July 2015 with the
objective to generate awareness about importance of Indias handloom industry. The date August 7 was
chosen to commemorate the Swadeshi Movement which was launched on this date in 1905 in the Calcutta
Town hall to protest against partition of Bengal by the British Government. The movement had aimed at
reviving domestic products and production processes. The first National Handloom Day was inaugurated on
7 August 2015 by Prime Minister Narendra Modi at Chennai, Tamil Nadu.
Comment
Handloom sector contributes to nearly 15% of the cloth production in the country and also contributes to the
countrys export earnings. 95% of the worlds hand woven fabric comes from India.
Astronomers found WASP-121b exoplanet with glowing water atmosphere
Astronomers using NASAs Hubble Space Telescope have found that exoplanet WASP-121b which provides
strongest evidence about existence of a stratosphere on it along with glowing water molecules.
The stratosphere is a layer of atmosphere in which temperature increases with higher altitudes.
Key Facts
Scientists had used spectroscopy in order to study the exoplanets stratosphere and to analyse how the
planets brightness changed at different wavelengths of light. The discovery shows common trait of most of
the atmospheres in our solar system, which is a warm stratosphere and can also be found in the atmospheres
of exoplanets. This phenomenon can be compared with the same processes that happen under different sets
of conditions in our own solar system.
About WASP-121b exoplanet
The WASP-121b exoplanet is gas giant commonly referred to as a hot Jupiter. It has 1.2 times greater mass
and 1.9 times greater radius than Jupiter. It is located approximately 900 light years from Earth. It orbits
around its host star every 1.3 days in very close proximity.
IIT Delhi researchers develops new nanotechnology-based drug delivery system
An all-women team of researchers from Indian Institute of Technology (IIT) Delhi has developed a new
drug delivery platform using nanoparticles.
The nanotechnology-based drug delivery system has ability to boost the efficacy of antibiotics at the cellular
level and improve chances of recovery from cancer-related bacterial infections.
Need
In traditional drug delivery system, if the bacterial infection in cancer remains untreated, it can infect the
host even after the cancer cells are killed by chemotherapy. Similarly antibiotics used in most conventional
therapeutics has several issues such as improper biodistribution, lack of target specificity, poor water
solubility and loss of efficacy over time due to the emergence of drug resistance in pathogenic bacteria.
2017 GKToday | All Rights Reserved | www.gktoday.in

59
Current Aairs: August, 2017 [Date-wise Compendium]

Moreover, 50% of these antibodies prescribed to humans are either not needed or not effectively utilised as
prescribed.
Key Facts
In the nanotechnology-based drug delivery system, a peptide called sushi-peptide was conjugated [bound] to
gold nanoparticles. It was found that this peptide bound to gold nanoparticles was able to kill E. coli and
Salmonella typhi bacteria more efficiently at lower dosages at much lower concentration than free peptides.
The nanoconjugate was able to kill 50% of bacteria at much lower concentration (400 nM) while the free
peptides antibacterial activity was not significant at the same concentration.
IISc researchers develop low-cost, sensitive CO sensor
Indian Institute of Science (IISc) researchers from Bengaluru have developed a highly sensitive, low cost
nanometre-scale carbon monoxide (CO) sensor, with potential applications in environmental pollution
monitoring.
The sensor was developed using novel fabrication technique that does not involve costly and time
consuming lithography technology.
Carbon Monoxide (CO)
CO is a colorless, odorless gas. It is harmful when inhaled in large amounts The greatest source of CO is
internal combustions (IC) engines of cars, trucks and other vehicles or machinery that burn fossil fuels.
Breathing high concentration of CO reduces the amount of oxygen that can be transported in the blood
stream to critical organs like the heart and brain.
Key Facts
The nanometer-sized sensor was made using zinc-oxide (ZnO) nanostructure on a silicon wafer substrate.
Tiny polystyrene beads were also used on the wafer. These beads were first added on the on the oxidised
silicon wafer arrange themselves into what is called a hexagonal close-packed structure.
Reasonable level of vacuum is maintained between the wafer and beads. When a high voltage is applied, it
etches away the surfaces of the beads until a gap of desired thickness is formed between adjacent beads. Then
ZnO is deposited on the system. This occupies the spaces between the beads, forming a honeycomb like
nano-mesh that can function as a nanosenor.
Significance
The nanometre-scale CO sensor is able to detect a difference in CO level as low as 500 parts per billion
(ppb). It can selectively respond to CO even in the presence of other gases. It also significantly cuts down the
time and cost involved in making nanostructured gas sensors.
Two new species of Cycas discovered
Research conducted on Cycas pschannae, a lone tree found in the Acharya Jagadish Chandra Bose Indian
Botanic Garden, West Bengal has revealed two new species of Cycas to the world. This discovery takes the
total number of Cycas species found in India to 14.
Cycas
Cycas are one of the most ancient plants whose fossils date to the Jurassic period. They are often referred to
2017 GKToday | All Rights Reserved | www.gktoday.in

60
Current Aairs: August, 2017 [Date-wise Compendium]

as living fossils. They have evolved on the earth as the first seeded plants and they grow very slowly, adding
only a few centimetres every year. Nearly 65% of Cycas are threatened. There are over 100 species of Cycas
found across the globe.
Key Facts
Initial studies on the lone Cycas pschannae tree revealed that it was Cycas, a gymnosperm. Further research
based on its anatomical and morphological characters led to the discovery of new species of Cycas pschannae
and later Cycas dharmrajii in the Andaman and Nicobar Islands.
Cycas dharmrajii is characterised by the abnormal branching habit of its giant trunk and its swollen base. It
has well-defined 10 to 28 hook-like structures in the apex of the mega sporophyll which makes it distinct
from other Cycas found in the country. Sporophylls are spore-bearing leaf-like female sex organ of the plant.
The sporophylls of Cycas pschannae are characterised by the presence of two lateral horn-like structures.
Government launches Tele-Law scheme in Bihar
The Union Ministry of Law & Justice in association with the National Legal Service Authority (NALSA)
launched Tele-Law scheme in Bihar.
The scheme aims at providing legal aid services to marginalised communities and citizens living in rural
areas through digital technology. It is continuation to the Access to Justice Project to Marginalised Persons
implemented by United Nations Development Programme (UNDP) in 2008.
Key Facts
The scheme is initiative of the Union Law Ministry and Ministry of Electronics and Information
Technology (MeITY). Under it, Tele-Law portal has been launched which is available across the Common
Service Centre (CSC) network at panchyat levels.
The portal will connect the citizens from rural areas to have access to legal consultation with the help of
para-legal volunteers (PLVs). It will enable people to seek legal advice from lawyers through video
conferencing. The National Legal Services Authority (NALSA) will also provide a panel of lawyers from
state capitals.
The scheme would help poor people in getting legal aid easily. The services of the Right of Public Service
Act and Public Grievance Redressal Act will be also made available at CSCs. Besides, various other services
like making Aadhaar card, PAN, applying for passports, reservation of train berths and bill payments can be
done from CSCs.
Government launches second phase of Measles-rubella vaccination campaign
The Union Ministry of Health and Family Welfare has launched the second phase of measles-rubella (MR)
vaccination campaign to reduce measles morbidity and mortality,
The second phase of campaign will cover 8 states and union territories namely Andhra Pradesh, Chandigarh,
Dadra and Nagar Haveli, Daman and Diu, Himachal Pradesh, Kerala, Telangana and Uttarakhand.
Key Facts
The Health Ministry has initiated MR vaccination campaign in the age group of 9 months to less than 15

2017 GKToday | All Rights Reserved | www.gktoday.in

61
Current Aairs: August, 2017 [Date-wise Compendium]

years in a phased manner in the country. It was started across the country under the universal immunisation
programme (UIP).
The campaign aims to rapidly build up immunity for both measles and rubella diseases in the community to
knock out the disease. Under the second phase, more than three crore children are expected to be covered in
these 8 states.
Under this campaign, MR vaccine has replaced two doses of measles vaccine provided in the routine
immunisation programme in these states. For those children who have already received such vaccination,
the campaign dose will provide additional boosting to them.
Measles-rubella (MR)
Measles is one of the important causes of death in children. It is highly contagious and spreads through
coughing and sneezing of an infected person. It can also make a child vulnerable to life threatening
complications such as diarrhoea, pneumonia and brain infection. Symptoms of the infection can include
cataracts and deafness. It can also affect the heart and the brain.
Rubella disease is commonly known as German Measles (or three-day measles). It is symptomatically similar
to measles. It is generally a mild infection, but has serious consequences if infection occurs in pregnant
women, causing congenital rubella syndrome (CRS). CRS is characterized by congenital anomalies in the
foetus and newborns affecting the eyes (cataract, glaucoma), brain (microcephaly, mental retardation), ears
(hearing loss) and heart defects.
August 8, 2017
India to MGC nations: Expand areas of cooperation
India at 8th Mekong Ganga Cooperation Ministerial Meeting held in Manila, Philippines has called for
expanding the areas of cooperation among the member countries of the Mekong Ganga Cooperation (MGC)
bloc.
MGC countries have proposed to add newer areas of cooperation to the traditional areas on tourism, culture,
education, transport and communication to further deepen mutually beneficial partnership and cooperation.
Key Facts
India and member states agreed to look forward to sustain this progress in the years to come and in further
deepening our mutually beneficial partnership and cooperation. They also have shared the concept notes on
formation of task forces on maritime connectivity and extension of (India-Myanmar-Thailand) trilateral
highway into Cambodia, Laos and Vietnam
Mekong Ganga Cooperation (MGC) bloc
The MGC is an initiative by India and five ASEAN countries, Cambodia, Laos, Myanmar, Thailand and
Vietnam for cooperation in tourism, culture, education, as well as transport and communications. It was
launched in 2000 at Vientiane, Laos. It is named after Ganga and the Mekong which both are civilisational
rivers. MGC initiative aims to facilitate closer contacts among the people inhabiting these two major river
basins.

2017 GKToday | All Rights Reserved | www.gktoday.in

62
Current Aairs: August, 2017 [Date-wise Compendium]

Government merges NCRB with BPRD


The Union Home Ministry has merged three-decade old National Crime Records Bureau (NCRB) with
Bureau of Police Research and Development (BPRD).
The merger aims at improving administrative efficiency and optimal utilisation of resources for
development works related to policing. It will also boost crime data collection and research efforts.
Key Facts
Now BPR&D will oversee all the data collection related to Crime in India, Suicides and Accidental Deaths,
Prison Statistics and Fingerprints which was earlier done by NCRB. The BPRD Director General (DG) will
be the head of the merged new entity, where the NCRB Director, an Additional DG rank post will report to
the former.
Reasons for merger
BPR&D has been given charge of NCRB so that there is more research based and methodological data
collection in future as data collection by NCRB as of now was that of taken from states and was sometimes
inaccurate. It was also felt that if the researchers of BPR&D there will be more accurate and research based
data collection.
About NCRB and BPRD
The NCRB was established in 1986 as the central police organisation to collect crime data, on various
parameters, from across all the states of the country. The BPRD was established in 1970 as the national
police organisation to study, research and develop on subjects and issues related to policing. Both the
organisations were functioning under the aegis of Union Home Ministry.
Entire Assam declared as disturbed area under AFSPA for another month
The Union Home Ministry has declared entire Assam state as a disturbed area under the Armed Forces
(Special Powers) Act for one more month, citing various violent activities by insurgent groups ULFA,
NDFB, and others.
The Home Ministry also declared 20 km belt along Meghalayas border areas adjoining Assam and three
districts in Arunachal Pradesh (Tirap, Changlang and Longding) as disturbed under the AFSPA for two
more months.
Background
AFSPA has been continuing in Assam since November 1990. In 2016, there were 75 incidents of violence
reported in Assam in which 33 people, including four security personnel, were killed and 14 others were
abducted. The violence was perpetrated in Assam by insurgent groups like ULFA, NDFB and others.
Similarly, continuing violence perpetrated by NSCN(K), NSCN(IM), ULFA, NDFB, and others were key
reasons for continuing with AFSPA in Arunachal Pradesh. Meghalaya too has witnessed violence by ULFA,
NDFB militants in the recent past.
About Armed Forces (Special Powers) Act
The AFSPA was enacted by the parliament in 1958. It grants extra-ordinary powers and immunity to the
armed forces to bring back order in the disturbed areas. Some of these extra-ordinary powers include fire
2017 GKToday | All Rights Reserved | www.gktoday.in

63
Current Aairs: August, 2017 [Date-wise Compendium]

upon anyone after giving warning who is acting against law & order in the disturbed area, arrest anyone
without warrant, stop and search any vehicle or vessel, legal immunity to armed forces personnel for their
actions.
Note: Presently AFSPA is enforced in the 6 states of North East (Arunachal Pradesh, Assam, Manipur,
Meghalaya, Mizoram and Nagaland) and Jammu and Kashmir. It was lifted in Tripura in May 2015.
India, Iran call for early operationalisation of Chabahar Port
India and Iran have expressed commitment for early completion and operationalisation of Chabahar Port
besides strengthening bilateral ties.
Both countries in a series of meetings held during the Union Road Transport, Highways and Shipping
Minister Nitin Gadkari two-day visit to Iran to attend the inauguration of Hassan Rouhani second tenure as
President.
Key Facts
Both countries reviewed and assessed the progress in implementation of the decisions taken during Prime
Minister Narendra Modis visit to Iran in 2016, including the progress in the development of Chabahar Port.
India also has conveyed its readiness to aid Iran in taking up operations in Chabahar Port during the interim
period between the actual activation of contract.
Background
India had requested Iran for an early submission of loan application for Chabahar Port Development to Exim
Bank of India so that the contract agreement can be activated. The Iranian side earlier had requested India to
provide up to $150 million credit and had made it a condition for activation of the Chabahar Port contract.
The application for loan to Exim Bank of India is still awaited.
Chabahar Port
Chabahar port is strategically located in the southeastern Sistan-Balochistan province, on the Gulf of Oman,
near Irans border with Pakistan. In May 2016, India and Iran had signed a bilateral agreement, under which
India agreed to refurbish one of the berths at Shahid Beheshti port, and reconstruct a 600-metre long
container handling facility at the port.
Significance of Chabahar Port
Alternative route to Afghanistan: The Chabahar port, intended to provide an alternative route for trade
between India and Afghanistan,

2017 GKToday | All Rights Reserved | www.gktoday.in

64
Current Aairs: August, 2017 [Date-wise Compendium]

by bypassing Pakistan. It will also ensure in the establishment of a politically


sustainable connectivity between India and Afghanistan which will lead to better
economic ties between them. It will also ease connectivity to Zaranj-Delaram road
constructed by India in 2009 which give access to Afghanistans Garland
Highway. From a diplomatic perspective, Chabahar port can also be used for
humanitarian operations in Afghanistan.
INSTC: It will boost Indias access to Iran, the key gateway to the International North-South Transport
Corridor (INSTC) that has sea, rail and road routes between India, Russia, Iran, Europe and Central Asia. It
can significantly boost import of iron ore, sugar and rice to India from Afghanistan. It can also help to
reduce import cost of oil to India.
Countering Chinese presence: It will be beneficial to India in countering Chinese presence in the Arabian
Sea as China with help of Pakistan is developing the Gwadar port which is less than 400 km from Chabahar
by road and 100 km by sea.
Maharashtra Government to raise $270 m from Green Climate Fund
The 2030 Water Resources Group (2030WRG) has decided to help the Maharashtra Government to raise
$270 million from the Green Climate Fund (GCF).
The amount raised will be invested in state governments integrated watershed programmes such as Jalyukt
Shivar Yojana which has successfully augmented the ground water in drought affected areas of the state.
Moreover, the raised fund will be also used to further connect efficient use of water at the village level,
where the fund will be useful for technology and infrastructure development. It will also be used in crop
diversification and adding market linkages.
Maharashtra Water Resources Multi-Stakeholder Platform (MWRMP)
2030WRG along with the Maharashtra Government have launched the MWRMP to address critical water
resources challenges in the State. The platform will be chaired by the Chief Secretary of Maharashtra.
The platform aims to accelerate water use efficiency programme in irrigated areas, along with agriculture

2017 GKToday | All Rights Reserved | www.gktoday.in

65
Current Aairs: August, 2017 [Date-wise Compendium]

growth and income enhancement. It will also develop public-private-community partnership models for
implementation of the programmes. It will develop viable financing solutions, including blended finance
models for promoting efficient use of water.
2030WRG
The 2030WRG is hosted by the International Finance Corporation (IFC), a member of the World Bank
Group. It is a public-private-civil society collaboration for water resources reform in developing economies.
Its ultimate aim is to close the gap between water demand and supply by the year 2030. It facilitates open,
trust-based dialogue processes to drive action on water resources reform in water stressed countries in
developing economies.
Green Climate Fund (GCF)
The GCF is a fund under the UN. It is guided by the principles and provisions of the UN Framework
Convention on Climate Change. It aims to help developing countries to finance clean energy projects other
mitigation efforts and adaptation to climate change.
Government constitute committee to make drugs more affordable
The Union Ministry of Chemicals and Fertilisers has constituted a committee of joint secretaries for
ensuring enhanced affordability, availability and accessibility of drugs for the citizens.
The committee will suggest ways to make pricing policy in favour of poor patients.
Terms of reference of the committee
The committee will review the scope of Drugs (Price Control) Order 2013 (DPCO 2013) and suggest ways
for strengthening the regulatory provisions of the order. It will also suggest ways to make DPCO 2013 more
comprehensive in light of past experience of implementing the order.
It will also delve into making the existing mechanism for collection of market based data on prices of
medicines more robust. It will also suggest ways for strengthening the existing pharmaceutical database
management system.
It will also look at procedural improvements and process of re-engineering in National Pharmaceutical
Pricing Authority (NPPA) to ensure better and quicker implementation of the government policy and bring
greater transparency to reduce litigations and review petitions.
Justice Dipak Misra appointed as 45th Chief Justice of India
President Ram Nath Kovind has appointed Justice Dipak Misra (63), senior-most judge of the Supreme
Court as the next Chief Justice of India. He will be 45th CJI and succeeds Justice JS Khehar.
He will have tenure of over 13 months till October 2018. He will be the third person from Odisha to be
appointed the CJI, after Justices Ranganath Misra and G B Pattanaik.
About Justice Dipak Misra
Justice Dipak Misra was born on 3 October 1953. He is the nephew of Justice Ranganath Misra who was CJI
from September 1990 November 1991.
He had enrolled as an advocate in 1977 and had practised in constitutional, civil, criminal, revenue, service
and sales tax matters in the Orissa High Court and Tribunals before was elevated to the High Court bench.
2017 GKToday | All Rights Reserved | www.gktoday.in

66
Current Aairs: August, 2017 [Date-wise Compendium]

He was appointed Additional Judge of the Orissa High Court in January 1996 and later was transferred to the
Madhya Pradesh High Court in March, 1997 where he became a permanent judge.
In 2009, Justice Misra was elevated as Chief Justice of the Patna High Court and later as the Chief Justice of
the Delhi High Court in May, 2010. He was elevated as a Judge of the Supreme Court in October 2011. In
Supreme Court he has already presided over several key cases and verdicts.
He had led the SC bench which rejected the 1993 Mumbai serial blasts convict Yakub Memons appeal to
stop his execution. He also had led a three judge SC bench which had upheld the death sentence awarded to
the four convicts of the Nirbhaya rape case in May, 2017.
He also has given significant judgment directing states and Union Territories to upload First Information
Reports (FIRs) on websites within 24 hours of their registration for enabling accused and others to file
appropriate pleas in the courts for redress.
August 9, 2017
75th anniversary of Quit India Movement
The 75th anniversary of the Quit India Movement was observed across the country. This years theme was
Sankalp se Siddhi- the attainment through resolve.
Prime Minister Narendra Modi in his recent Mann Ki Baat programme had called for launching the mega
campaign Sankalp Se Siddhi. The campaign pledges to build new India by 2022. It calls on everyone to pledge
together towards Clean India, Povertyfree India, Corruption-free India, Terrorism-free India,
Communalism-free India, Casteism-free India.
Prime Minister Modi also had termed the month of August as a month of revolution as the non-cooperation
movement was launched on August 1, 1920, Quit-India movement on August 9, 1942 and on August 15,
1947 India became independent.
Quit India Movement
The Quit India Movement (Bharat Chhodo Andolan or the August Movement (August Kranti)) was an
important milestone in the Indian freedom struggle. It was a civil disobedience movement launched at the
Bombay session of the All-India Congress Committee (AICC) by Mahatma Gandhi on 8 August 1942
demanding an end to British Rule of India. It was launched after Mahatma Gandhi had made a call to Do or
Die in his Quit India speech delivered in Bombay at the Gowalia Tank Maidan on 7 August 1942. Gandhi
hoped to bring the British government to the negotiating table as the Cripps Mission had failed. Quit India
Resolution drafted by Jawaharlal Nehru and was moved by him on 8th August 1942 in AICCC session and
Sardar Patel seconded it. Under the leadership of Mahatma Gandhi, people across India came together to
uproot imperialism.
Commerce Ministry to shut down DGS&D
The Union Ministry of Commerce and Industry has started the process of shutting down Directorate
General of Supplies and Disposals (DGS&D), over 100-year-old government procurement arm, in this fiscal.
Earlier, the Union Cabinet in April 2017 had ordered the DGS&D to wind up operations by October 2017.
2017 GKToday | All Rights Reserved | www.gktoday.in

67
Current Aairs: August, 2017 [Date-wise Compendium]

Now the public procurement of goods and services will be managed by the Governments e-market platform
(GeM).
Before closing, the Commerce Ministry will decide upon the issues concerning manpower, land and office
spaces of the directorate, which are present all over the country.
Directorate General of Supplies and Disposals (DGS&D)
DGS&D has its origin in the India Stores Department which was established in London for purchase of
imported stores in 1860. In 1922, India Stores Department was established in India to arrange purchases for
Government Civil Departments and Military Purchases.
In 1951, the set-up of Purchase and Inspection was reorganised and DGS&D was constituted, in its present
form, to handle all government purchases centrally. A minor restructuring was undertaken in 1974 to allow
defence and railways to procure items meant for their exclusive use.
The directorate has two professional service cadres Indian Supply Service (ISS) and Indian Inspection
Service (IIS). Currently, the directorate has four regional offices including Mumbai, Kolkata, Hyderabad and
Chennai. It has 12 Purchase Directorates (Commodity-wise) at its headquater in New Delhi. Besides, there
are 20 offices and the staff strength is over 1,200.
Need for shutting down DGS&D
The Union Ministry of Commerce and Industry had launched GeM (Governments e-market platform) to
bring greater transparency and efficiency in public procurement. At present, the Union and state
governments procure goods and services worth over Rs. 5 lakh crore annually. Currently, over 20,000 sellers
are registered on this GeM portal for selling over 50,000 products. The GeM portal has eliminated human
interface in the procurement process such as order placement, vendor registration and payment processing
to a great extent.
NITI Aayog partners with 6 states to transform Health & Education sectors
The NITI (National Institution for Transforming India) Aayog has announced partnership with three states
each to radically transform their Health and Education sectors under its Sustainable Action for
Transforming Human Capital (SATH) initiative.
The shortlisted states for transforming their health sector are Uttar Pradesh, Assam and Karnataka. Madhya
Pradesh, Jharkhand and Odisha were selected for transforming their education sector.
Key Facts
NITI Aayog had invited all states and Union Territories to participate in its SATH initiative to transform
Health and Education sectors in order to give major push to competitive, cooperative federalism. Under this
initiative, a Program Management Unit will now be available in the six chosen States for a period of 30
months to push for efficiency and efficacy in governance structures and service delivery.
It is expected that these three years of focussed attention and support from NITI Aayog will lead to a marked
transformation and provide a model for other States to replicate and adapt. It will be three-way partnership
between NITI, State Governments and knowledge partner for each of the sectors is part of the SATH
initiative of NITI Aayog.
2017 GKToday | All Rights Reserved | www.gktoday.in

68
Current Aairs: August, 2017 [Date-wise Compendium]

Selection of States
These six states were chosen after a rigorous competitive process based on comprehensive metrics to
determine potential for impact and likelihood of success. States had made presentations for each sector
which was assessed by a committee comprising of senior members of NITI Aayog and Human Resources
Development and Health ministries. The States had highlighted the initiatives undertaken by them and their
willingness to accelerate improvement. On thorough technical evaluation, the chosen States have committed
to time-bound, governance reforms in both sectors.
Sustainable Action for Transforming Human Capital (SATH) initiative
NITI Aayog is working to foster co-operative federalism by ranking states through health, water, education,
and agricultural indices. The SATH initiative was launched to go beyond ranking states and to handhold
them in improving their social sector indicators.
The vision of SATH initiative is to initiate transformation in the education and health sectors by providing
technical support to states from NITI. It aims to identify and build three future role model states for health
and education sectors.
Under it, NITI will work in close collaboration with state machinery to design a robust roadmap of
intervention, develop governance structure, set up monitoring and tracking mechanisms and provide
support on a range of institutional measures to achieve the end objectives.
NITI Aayog
NITI Aayog is the premier policy Think Tank of the Union Government, providing both directional and
policy inputs. It is essentially an advisory body that seeks to provide critical directional and strategic inputs
across spectrum of key elements of policy to the centre.
NSDC, Google India launch mobile app development programme
The National Skill Development Corporation (NSDC) and Google India have launched Android Skill
Development programme to improve mobile developer ecosystem in the country.
The programme aims to impart mobile development training on the android OS and web platform and also
improve the quality of mobile app development ecosystem in India.
Key Facts
Under the initiative, NSDC will introduce a specific course to create applications for the mobile platform
that can be used on smartphones and tablets running on the Android OS. NSDCs partner agencies will
design and deliver 100-hour duration course outside the formal education system.
Under the programme, Google India and NSDC will work together to focus on up-skilling Android trainers.
The programme will accelerate mobile skill training and create opportunities for youth in the country. It will
focus on achieving the target of the Union Governments ambitious Skill India initiative.
National Skill Development Corporation (NSDC)
The NSDC is a Public Private Partnership (PPP) under the Ministry of Skill Development &
Entrepreneurship. Its objective is to create training capacity in the country; fund vocational training
initiatives and create a market ecosystem for skill development. Its mandate is to train 150 million people by
2017 GKToday | All Rights Reserved | www.gktoday.in

69
Current Aairs: August, 2017 [Date-wise Compendium]

2022.
Government to expand Logistics Data Bank project to South India
The Union Ministry of Shipping is planning to expand Logistics Data Bank (LDB) project to the countrys
southern region. So far, it had covered only the western logistics corridor.
The LDB project was launched at the Jawaharlal Nehru Port (JNPT), Mumbai in July 2016, making it first
port in India to provide this service. Later its operations were expanded to the container terminals at Adani
Port Special Economic Zone (APSEZ), Mundra and Adani Hazira Port both in Gujarat.
About Logistics Data Bank (LDB) project
The LDB project was unveiled in July 2016 to make Indias logistics sector more efficient through the use of
Information Technology. Under this project, every container in the port facility is attached to a Radio
Frequency Identification Tag (RFID) tag and then tracked through RFID readers.
It is being implemented through a Special Purpose Vehicle (SPV) called Delhi Mumbai Industrial Corridor
Development Corporation Logistics Data Services Ltd. (DLDSL). DLDSL is jointly (50:50) owned by the
Delhi Mumbai Industrial Corridor (DMIC) Trust and Japanese IT services major NEC Corporation.
The LDB project covers the entire movement of containers through rail or road till the Inland Container
Depot and Container Freight Station. Its service integrates information available with the agencies across the
supply chain to provide detailed, real-time information within a single window.
Significance of project
It has helped to cut the overall lead time of container movement as well as reduced transaction costs that
consignees and shippers incur. It also has aided importers and exporters in tracking their goods in transit. It
is billed as a major ease of doing business initiative aimed at boosting Indias foreign trade and ensuring
greater transparency.
Government launches Swachh Survekshan Gramin 2017
The Union Ministry of Drinking Water and Sanitation has released Swachh Survekshan Gramin 2017, third
party survey report to assess the progress Swachh Bharat Mission in rural India
The survey was undertaken by the Quality Council of India (QCI) between May and June 2017 to take into
account status of rural sanitation in all States and UTs. The survey covered 4626 villages across all States and
Union territories.
Key Highlights of Swachh Survekshan Gramin
QCI claims that 62.45% of the households have access to a toilet and 91.29% of the people who had access to
a toilet also used it. Over 4.54 crore household toilets have been constructed since the launch of the Swachh
Bharat Mission Gramin. 160 districts, 2,20,104 villages and 5 States declared have been declared Open
Defecation Free (ODF). Sanitation Coverage has increased from 39% in October 2016 to 66% in August
2017.
Top performing states: Northeastern States of Sikkim, Nagaland and Manipur with 95% rural households
covered by toilets were top performers. Himalayan States of Himachal Pradesh and Uttarakhand were also

2017 GKToday | All Rights Reserved | www.gktoday.in

70
Current Aairs: August, 2017 [Date-wise Compendium]

top performers with over 90% toilet coverage of the rural houses.
Performance of Big States: Almost all rural households in Kerala and Haryana have access to a toilet. In,
Gujarat 85% of rural households have access to toilets. Tamil Nadu has 79% rural households access to a
toilet.
Worst performing States: Bihar (30% coverage) and Uttar Pradesh (37%), Jharkhand (37%) were among
the worst performers in terms of rural sanitation.
About Quality Council of India (QCI)
QCI was set up in 1997 by Union Government jointly with Indian Industry as an autonomous body. Its
mandate is to establish and operate the National Accreditation Structure (NAS) for conformity assessment
bodies and providing accreditation in the field of health, education and quality promotion.
It also promotes the adoption of quality standards relating to Food Safety Management Systems (ISO 22000
Series), Quality Management Systems (ISO 14001 Series) and Product Certification and Inspection. It has
been assigned task of monitoring and administering the National Quality Campaign and also oversee
function of National Information and Enquiry Services.

India ranks 43rd in 2017 Global Retirement Index


India was ranked last at 43rd position in 2017 Global Retirement Index (GRI) published by French asset
management company Natixis Global.
2017 GKToday | All Rights Reserved | www.gktoday.in

71
Current Aairs: August, 2017 [Date-wise Compendium]

The index ranks 43 countries on the basis of four factors viz. the material means to live comfortably in
retirement (Material Well-being); access to quality health services (Health); access to quality financial
services to help preserve savings value and maximize income (Finances) and a clean and safe environment
(Quality of Life ).
The 43 countries include members of the Organization for Economic Co-operation and Development
(OECD), International Monetary Fund (IMF) advanced economies and the BRIC countries (Brazil, Russia,
India and China).
Key Highlights of 2017 GRI
Top three countries in 2017 GRI are Norway, Switzerland and Iceland. India ranks 43rd and has the same score
compared to 2016 GRI. India

also ranks the lowest among the BRIC economies.


Indias rank in all sub-indices is also in the bottom five. India position in all sub-indices is Material Well-
being (41st), Health (43rd), Finances (39th) and Quality of Life (43rd).
India has the lowest income per capita of all countries in the GRI. Additionally, its score for the income
equality indicator has declined compared to 2016 GRI. It has the lowest scores for all indicators within the
sub-index including in insured health expenditure compared to 2016 GRI.
India ranks first in old-age dependency, second in tax pressure and sixth in interest rates. Interms of
governance indicator India ranks as the fifth worst among all countries in the GRI. It also has the tenth-
lowest score for the bank non-performing loans indicator. India ranked last in the Quality of Life sub-index.
Indias environmental factors indicator has improved due to progress in CO2 emissions per GDP. But it still
ranks in bottom for happiness, water and sanitation, and air quality as well as scores second-worst for

2017 GKToday | All Rights Reserved | www.gktoday.in

72
Current Aairs: August, 2017 [Date-wise Compendium]

biodiversity and habitat among all GRI countries.


India ratifies 2nd commitment period of Kyoto Protocol
India has ratified the second commitment period of the Kyoto Protocol (or Doha Amendment) that commits
countries to contain the emission of greenhouse gases (GHGs). In this regard, India deposited its Instrument
of Acceptance of the Doha Amendment to the Kyoto Protocol.
With this, India became the 80th country to accept the amendment relating to the second commitment
period (2013- 2020) of the Kyoto Protocol.
About Kyoto Protocol
The Kyoto Protocol is an international GHGs emissions reduction treaty linked to the United Nations
Framework Convention on Climate Change (UNFCCC). It commits its Parties by setting internationally
binding GHGs emission reduction targets. It was adopted in Kyoto, Japan, in December 1997 and entered
into force in February 2005.
The protocol is based on principle of equity and Common but differentiated responsibilities and respective
capabilities (CBDR). It places obligations on developed nations to undertake mitigation targets to reduce
emissions and provide financial resources and technology to developing nations. Developing countries like
India have no mandatory mitigation obligations or targets under the Kyoto Protocol.
The first commitment period under the Kyoto Protocol was from 2008-2012. The second commitment
period of the Kyoto Protocol or Doha Amendment for 2013-2020 period was adopted in 2012. The
amendment includes new commitments for parties to the Protocol who agreed to take on commitments in a
second commitment period and a revised list of GHGs to be reported on by Parties.
Comment
The ratification of second protocol reaffirms Indias stand on climate action. It also further underlines Indias
leadership in the comity of nations committed to global cause of environmental protection and climate
justice. It will encourage other developing countries also to undertake this exercise. Under the second
commitment period, implementation of clean development mechanism (CDM) projects will help Indian
attract some investments.
August 10, 2017
Anti-dumping duty on 93 products from China: Government
The Union Government has announced that anti-dumping duty is in force on 93 Chinese products to
protect profitability of the domestic industry from import of cheap Chinese goods.
These products fall in the broad groups of chemicals and petrochemicals, steel and other metals, machinery
items, fibres and yarn, rubber, plastic products, electric and electronic items and consumer goods, among
others.
In addition to these 93 products, 40 cases concerning imports from China have been initiated by Directorate
General of Anti-Dumping and Allied Duties (DGAD).

2017 GKToday | All Rights Reserved | www.gktoday.in

73
Current Aairs: August, 2017 [Date-wise Compendium]

Anti-dumping duty
Anti-dumping duty is a protectionist stance taken by a government to cushion domestic companies from an
increase in cheap price imports. It is imposed by government on imported products which have prices less
than their fair normal values in their domestic market.
Why it is imposed?
When a country exports its products to other countries with a selling price below the cost price of the same
product in other countries then it is called as dumping of products. This harms the profitability of domestic
companies. Anti-dumping steps are taken to ensure fair trade and provide a level-playing field to the
domestic industry.
Mechanism for imposition
Anti-Dumping Duty is imposed under the multilateral World Trade Organisation (WTO) regime and varies
from product to product and from country to country. In India, anti-dumping duty is recommended by the
Union Ministry of Commerce (i.e. by DGAD) and imposed by the Union Finance Ministry.
NCERT launches web portal to supply Textbooks online
The National Council of Educational Research and Training (NCERT) has launched a web portal for supply
of textbooks to schools and individuals.
The web portal of NCERT will enable schools to place orders for their books online. It will also enable
individual buyers to place their orders by logging onto the portal and books will be delivered at their
doorstep.
Key Facts
The portal will ensure better distribution of textbooks across the country and also address the apprehensions
of schools and parents regarding unavailability of NCERT textbooks. It will enable schools to place orders
for the session 2018-19 by enabling them to log on to the portal, by entering their respective Boards
affiliation numbers and other details.
Moreover, schools will also have option of directly procuring textbooks either from their nearest NCERT
vendors or from the Regional Production-cum-Distribution Centres (RPDCs) of the NCERTs located at
Kolkata, Guwahati, Ahmedabad and Bengaluru.
The move will help parents save money spent on sourcing expensive books from private publishers, often
due to non-availability of NCERT books. NCERT is also exploring the option of having ecommerce vendors
such as Flipkart and Amazon host its books and deliver them too to streamline the process further.
About National Council of Educational Research and Training (NCERT)
The NCERT is an autonomous organisation set up in 1961 by the Union Government as a literary, scientific
and charitable Society under the Societies Registration Act. Its mandate is to assist and advise the Union and
State Governments on policies and programmes for qualitative improvement in school education.
Its major objectives are to undertake, promote and coordinate research in areas related to school education,
prepare and publish model textbooks, supplementary material, organise pre-service and in-service training
of teachers etc. It acts as a nodal agency for achieving goals of Universalisation of Elementary Education.
2017 GKToday | All Rights Reserved | www.gktoday.in

74
Current Aairs: August, 2017 [Date-wise Compendium]

In addition, NCERT is an implementation agency for bilateral cultural exchange programmes with other
countries in the field of school education. Textbooks published by NCERT are prescribed by the Central
Board of Secondary Education (CBSE) from classes I to XII.
August 10: International Biodiesel Day
The International Biodiesel Day (IBD) is celebrated every year on August 10 in a bid to create awareness
about non fossil-fuels (Green Fuels). The day also honours the research experiments by Sir Rudolf Diesel
who ran an engine with peanut oil in the year of 1893. His research experiment had predicted that vegetable
oil is going to replace the fossil fuels in the next century to fuel different mechanical engines.
About Biodiesel
The Biodiesel is an alternative fuel which can be used in place of fossil fuels. It is manufactured from
vegetable oils, recycled grease, algae, and animal fat. It is produced through a chemical process called
transesterification, in which glycerine is separated from the vegetable oil or fat. It can be seen as alternative
to conventional fossil fuels.
Benefits of Biodiesel
Biodiesel is a renewable fuel. It can be used in diesel engines with little or no modification. It can be
produced locally. It is biodegradable, sustainable, non-toxic environment friendly fuel. On burning, it emits
60% less carbon dioxide (CO2). The energy produced by biodiesel on combustion is approximately 90% of
that of energy produced by petroleum diesel. It is also used in non-engine applications such as to remove
paint etc. Byproductsmethyl esters and glycerine obtained during production of biodiesel can be used
preparation of soaps and other products
UP Government launches Namami Gange Jagriti Yatra
The Uttar Pradesh Government has launched Namami Gange Jagriti Yatra, an awareness drive along banks
of Ganga River to maintain cleanliness. The campaign was flagged off by Chief Minister Yogi Adityanath
from the state capital in Lucknow.
Key Facts
The drive has been organised by Uttar Pradesh Homeguard volunteers to spread awareness in 25 districts
situated on the banks of river Ganga for maintaining the cleanliness and ever-flowing nature of the river. It
will pass through 108 blocks of 25 districts of UP from Bijnore to Ballia along Ganga covering around 1,025
kms.
Under it, above one lakh Homeguard volunteers will organise public gathering and other awareness
programmes in the district under Namami Gange Project. It will urge people to refrain from practices and
other things that pollute the Ganga River.
ISRO to develop full-fledged Hyperspectral Imaging Earth observation satellite
The Indian Space Research Organisation (ISRO) is planning to launch a full-fledged niche Earth observation
(EO) satellite called the Hyperspectral Imaging Satellite (HySIS).
The HySIS satellite has critical chip called an optical imaging detector array indigenously developed by

2017 GKToday | All Rights Reserved | www.gktoday.in

75
Current Aairs: August, 2017 [Date-wise Compendium]

ISRO. Its launch will allow ISRO to enter the domain of operational hyperspectral imaging from earth orbit.
Hyperspectral Imaging
Hyperspectral imaging or hyspex imaging (imaging spectroscopy) combines the power of digital imaging and
spectroscopy. It collects and processes information from across the electromagnetic spectrum. Hyspex
imaging enables distinct identification of objects, materials or processes on Earth by reading the spectrum
for each pixel of a scene from space. The hyspex technology is still an evolving science. In recent times, it has
become trend that is being experimented globally. It has ability to add a new dimension to plain-vanilla
optical imagers.
Key Facts
HySIS satellite developed by ISRO can see in 55 spectral or colour bands from 630 km above ground. It can
be used for a range of applications from monitoring the environment, crops, looking for oil and minerals,
military surveillance. The architecture of the optical imaging detector array chip on board of satellite has
been designed by the payloads development centre, Space Applications Centre, Ahmedabad. It was
manufactured at ISROs electronics arm, the Semi-Conductor Laboratory, Chandigarh. It can read upto 1000
x 66 pixels.
Background
ISRO for first time had tried out hyspex imaging technology in an 83-kg IMS-1 experimental satellite in May
2008. The same year, it also had mounted hyperspectral camera on Chandrayaan-1 and used to map lunar
mineral resources. Globally so far, very few space agencies have such a satellite.

2017 GKToday | All Rights Reserved | www.gktoday.in

76
Current Aairs: August, 2017 [Date-wise Compendium]

National Cyber Coordination Centre made operational: Government


The Union Ministry of Electronics and Information Technology (MEITY) has announced that the first
phase of National Cyber Coordination Centre (NCCC) has become operational.
NCCC has been set up to scan the countrys web traffic to detect cyber security threats. It scans the
2017 GKToday | All Rights Reserved | www.gktoday.in

77
Current Aairs: August, 2017 [Date-wise Compendium]

cyberspace in the country for cyber security threats at metadata level to generate situational awareness.
Key Facts
NCCC is a multi-stakeholder cyber-security and e-surveillance agency implemented by Indian Computer
Emergency Response Team (CERT-In), Union Ministry of Electronics and Information Technology. It
derives its powers as per provisions of section 69B of the Information Technology Act, 2000 and the
Rules notified thereunder.
It will be Indias first layer for cyber threat monitoring and all communication with government and private
service providers will be monitored by it. Its mandate is to scan internet traffic and communication metadata
coming into the country to detect real-time cyber threat and alert various organisations as well as internet
service providers (ISPs) for timely action.
NCCC also will coordinate between intelligence agencies, specifically during network intrusions and cyber-
attacks. It will have virtual contact with the control room of all ISPs to scan traffic within the country,
flowing at the point of entry and exit, including international gateway. Apart from monitoring the Internet,
the NCCC will also look into various threats posed by cyber-attacks. It will have top experts from the cyber
security field and will run like similar organisation in other countries such as the US, the UK, France,
Germany, etc.
Government to launch Aajeevika Grameen Express Yojana under DAY-NRLM
The Union Ministry of Rural Development is going to launch Aajeevika Grameen Express Yojana (AGEY), a
sub-scheme under Deendayal Antyodaya Yojana National Rural Livelihoods Mission (DAY-NRLM).
The scheme aims at providing an alternative source of livelihood to members of Self Help Groups (SHGs) by
facilitating them to operate public transport services in backward rural areas.
Key Facts
AGEY will help to provide safe, affordable and community monitored rural transport services to connect
remote villages with key services and amenities such as access to markets, education and health for the
overall economic development of backward rural areas.
The transport services will be provided by e-rickshaws, 3 and 4 wheeler motorised transport vehicles to
connect remote villages with the help of Self Help Groups (SHGs) under DAY-NRLM. This will provide an
additional avenue of livelihood for SHGs.
The scheme will be implemented initially in 250 blocks in the country on a pilot basis for a period of 3 years
(2017-20) with each block provided upto six vehicles to operate transport services. The Community
Investment Fund (CIF) provided to Community Based Organization (CBOs) under DAY-NRLM will be
utilized to provide interest free loan to SHG member for purchase of transport vehicles.
About Deendayal Antyodaya Yojana National Rural Livelihoods Mission (DAY-NRLM)
DAY-NRLM is a poverty alleviation scheme implemented by Union Ministry of Rural Development to
promote self-employment and organization of rural poor. It was launched as NRLM in 2011 but was
succeeded by Deen Dayal Antyodaya Yojana in 2016. It is supported by World Bank.
The scheme aims to organize the poor into SHGs and make them capable for self-employment. It has a
2017 GKToday | All Rights Reserved | www.gktoday.in

78
Current Aairs: August, 2017 [Date-wise Compendium]

special focus on women empowerment including a dedicated component for promoting farm and non-farm
based livelihoods for women farmers in rural areas.
The scheme will help rural poor to set up self-employment ventures by ensuring easy access to credit. It is
implemented across the country in all States and Union Territories (except Chandigarh and Delhi ). It is one
of the worlds largest initiatives to improve the livelihood of poor.
NGT bans use of non-biodegradable plastic bags in Delhi
The National Green Tribunal (NGT) imposed an interim ban on use of non-biodegradable plastic bags
which are less than 50 microns in the entire national capital Delhi.
It also slapped a fine of Rs 5,000 on anyone found in possession of non-biodegradable plastic bags less than
50 microns, which is the thickness of a human hair.
NGT order
NGT ordered Delhi government to seize entire stock of the banned plastic bags within a week. Besides, it
asked Delhi Pollution Control Committee (DPCC) and Delhi Government to file an affidavit on steps taken
to implement the ban. It also imposed an environment compensation of Rs. 10,000 on vegetable vendors and
slaughter houses for throwing garbage in public places.
Plastic waste
Non-biodegradable plastic waste has been identified as one of the major source of environmental pollution
as they dont decay naturally. These bags have zero reusable value. Besides choking drains, water-bodies and
adding to the load of the already-exhausted landfill sites, there are also cases of cattle eating and dying after
choking on plastic bags.
According to plastic bag manufacturers, the market share of thin plastic bags (less than 50 microns) is not
more than 25%. Mostly the banned bags are made by unregistered units which find many takers because of
their cheaper products.
Most of the garbage bags used at homes are above 50 microns. Their use and manufacturing is already
banned under the Plastic Management Handling Rules, 2016. The NGT in 2017 had banned the use of
disposable plastic in Delhi and NCR with effect from January 1, 2017 and directed the city government to
take steps to reduce dumped waste.
Lok Sabha passes State Banks (Repeal and Amendment) Bill, 2017
The Lok Sabha has passed the State Banks (Repeal and Amendment) Bill, 2017. The bill seeks to repeal the
two Acts namely State Bank of India (Subsidiary Banks) Act, 1959, and State Bank of Hyderabad Act, 1956.
It also seeks to amend the State Bank of India (SBI) Act, 1955 to remove references to subsidiary banks and
powers of SBI to act as an agent of the RBI for subsidiary banks.
Key Facts
Repealing of SBI (Subsidiary Banks) Act and State Bank of Hyderabad Act: These two acts have
established the State Bank of Bikaner, State Bank of Patiala, State Bank of Mysore, State Bank of Hyderabad
and State Bank of Travancore. These banks were subsidiaries of the SB). This repealing of two acts is
consequent to the decision of the Union Cabinet February 2017 to approve merger of five subsidiary banks
2017 GKToday | All Rights Reserved | www.gktoday.in

79
Current Aairs: August, 2017 [Date-wise Compendium]

of SBI were merged with it.


Amendments to the SBI Act: The Bill removes references related to subsidiary banks from the parent Act.
These references include the definition of a subsidiary bank and powers of SBI to act as an agent of the RBI
for subsidiary banks.
Background
The Union Cabinet had approved the merger of five associate banks along with Bharatiya Mahila Bank with
SBI. The five banks were State Bank of Bikaner and Jaipur, State Bank of Hyderabad, State Bank of
Travancore, State Bank of Mysore and State Bank of Patiala. After the acquisition by SBI, the subsidiaries
banks have ceased to exist and, therefore, it was necessary to repeal the two Acts. The merger had made
State Bank of India one of the 50 biggest banks of the world. The main motive behind the merger was to
increase the capital base of the bank to enable it to disburse more loans.
August 11, 2017
India and US to co-host Global Entrepreneurship Summit
India and US are going to co-host the Global Entrepreneurship Summit at Hyderabad, Telangana from 28th
to 30th November 2017. American delegation will be led by US President Donald Trumps daughter Ivanka
Trump.
The Summit will serve as a unique opportunity for bringing together entrepreneurs and start-ups with
global leaders. Earlier it was hosted by the US State Department.
Key Facts
The three-day Global Entrepreneurship Summit will be organised by the NITI (National Institution for
Transforming India) Aayog in coordination with Ministry of External Affairs.
The summit aims at connecting American entrepreneurs and investors to their international counterparts to
form lasting relationships. It seeks to create new opportunities for investment, partnership and collaboration
around the world.
The summit includes a wide range of panels, ignite talks, workshops, pitch competitions, mentoring and
networking sessions. These events at the summit mainly aim to give participants tailored opportunities to
gain skill and relationships that will help them to grow. It also create opportunities for participants to
connect with industry experts.
Parliament passes Banking Regulation (Amendment) Bill, 2017
Parliament has passed the Banking Regulation (Amendment) Bill, 2017 with the Rajya Sabha approving it.
The Lok Sabha has already passed it. The bill will replace the Banking Regulation (Amendment) Ordinance,
2017 promulgated by President in May 2017.
The bill seeks to amend the Banking Regulation Act, 1949 to insert provisions for handling cases related to
stressed assets or non-performing assets (NPAs) of banks. Stressed assets (NPAs) are loans defaulted by
borrower in repayment or the loan which has been restructured by changing the repayment schedule.

2017 GKToday | All Rights Reserved | www.gktoday.in

80
Current Aairs: August, 2017 [Date-wise Compendium]

Key Features of the Bill


Initiating insolvency proceedings: It will enable the Central government to authorize the RBI to direct
banking companies to resolve specific stressed assets by initiating insolvency resolution proceedings under
the Insolvency and Bankruptcy Code, 2016.
Issuing directions on stressed assets: It empowers RBI to issue directions to banks for resolution of
stressed assets from time to time.
Committee to Advise banks: It enables RBI to specify committees or authorities to advise banks on
resolution of stressed assets. RBI will appoint or approve members on such committees.
Applicability to State Bank of India (SBI): It inserts provision to make above provisions applicable to the
SBI and its subsidiaries and also Regional Rural Banks (RRBs).
Need for Amendment
NPAs in the banks stand at Rs. 6.41 lakh crore in the public sector banks while total stressed assets are at Rs.
8.02 lakh crore. It has resulted in choking the banking system. So it had become necessary for the RBI to
intervene in order to take urgent measures for their speedy resolution. Government is also expanding
infrastructures in Debt Recovery Tribunal, National Company Law Tribunal to deal with stressed assets.
National Mission for Clean Ganga approves 10 projects
The National Mission for Clean Ganga (NMCG) has approved ten projects in Uttar Pradesh, Bihar and West
Bengal to the tune of about Rs. 2,033 crore.
These projects were approved in the 5th meeting of the Executive Committee of NMCG. Eight of the ten
projects pertain to sewage infrastructure and treatment, one each to ghat development and Ganga
Knowledge Centre.
Approved Projects
In Bihar, three major sewage infrastructure projects were approved in Barh, Kankarbagh and Digha. These
projects will create additional sewage treatment capacity in Kankarbagh and Barh.
In West Bengal, three projects were approved. Out of these two pertain to sewage infrastructure while the
third one is for ghat development. In Uttar Pradesh, sewage infrastructure work in Chunar, district
Mirzapur, has also been approved at an estimated cost of around 28 crore rupees.
Project to establish Ganga Monitoring Centres (GMCs) was also approved in five riparian Ganga states. The
objective of GMCs is to establish efficient monitoring of wholesomeness of river including flow levels,
pollution levels, point and non-point sources of pollution, among other parameters. Two pilot
demonstrative projects of treatment of drains using bioremediation method were also approved.
About National Mission for Clean Ganga (NMCG)
NMCG is the implementation wing of National Council for Rejuvenation, Protection and Management of
River Ganga (referred as National Ganga Council). It was established in 2011 as a registered society under
Societies Registration Act, 1860.
It has a two tier management structure and comprises of Governing Council and Executive Committee. Both
of them are headed by Director General (DG), NMCG. Executive Committee is authorized to approve
2017 GKToday | All Rights Reserved | www.gktoday.in

81
Current Aairs: August, 2017 [Date-wise Compendium]

projects under mission up to Rs.1000 crore.


Similar to structure at national level, State Programme Management Groups (SPMGs) acts as implementing
arm of State Ganga Committees. This structure attempts to bring all stakeholders on one platform to take a
holistic approach towards the task of Ganga cleaning and rejuvenation.
Venkaiah Naidu sworn in as 13th Vice President of India
Former Union Minister M Venkaiah Naidu was sworn in as 13th Vice President of India. He was
administered the oath of office by President Ram Nath Kovind at a special function at Rashtrapati Bhavan.
After the swearing in he took charge as the Chairman of Rajya Sabha. He also chaired the proceedings of the
Upper House. He was elected as the Vice President after defeating opposition candidate Gopal Krishna
Gandhi.
About M Venkaiah Naidu
Muppavarapu Venkaiah Naidu was born on 1st July 1949 at Chavatapalem, in Nellore district of Andhra
Pradesh. He acquired his bachelors degree in law with specialisation in international law from Andhra
University, Visakhapatnam.
He was elected as the president of the students union of colleges affiliated to his University and joined RSS
and ABVP during his college days. He came into spotlight for his prominent role in the Jai Andhra
Movement and for his contribution in Jayaprakash Narayan Chhatra Sangharsh Samiti.
He has held several positions and carried his successful political career during his three decade long public
life. He has served as national president of BJP (2002 to 2004). In the Atal Bihari Vajpayee Government he
was the Union Cabinet Minister for Rural Development.
In present Narendra Modi led government he had served as the Minister of Housing and Urban Affairs,
Parliamentary Affairs and Information and Broadcasting. Before getting elected as Vice President, he had
held several positions and carried his successful political career during his three decades in public life.
About office of Vice President
The officer of Vice-President is second highest public office in India. It is second highest constitutional post
of the country after President. Practically, office of Vice President has been created to provide political
continuity of the Indian state. His main function is in the form of ex-officio Chairman of Rajya Sabha. He
works as acting President when President is not available.
15th BIMSTEC Ministerial meeting begins in Kathmandu
The 15th edition of Bay of Bengal Initiative for Multi-Sectoral Technical and Economic Cooperation
(BIMSTEC) ministerial meeting was held in Kathmandu, Nepal.
The two-day meet was inaugurated by Prime Minister of Nepal Sher Bahadur Deuba and will be chaired by
Deputy Prime Minister and Minister for Foreign Affairs Krishna Bahadur Mahara. India was represented by
External Affairs Minister Sushma Swaraj.
Key Facts
The meeting will review the progress and achievements in the identified areas of cooperation and discuss the
ways to accelerate the partnership under the BIMSTEC process. The key areas expected to be focused in the
2017 GKToday | All Rights Reserved | www.gktoday.in

82
Current Aairs: August, 2017 [Date-wise Compendium]

meeting are trade, investment, energy, transport, tourism, poverty alleviation and counter-terrorism.
The main agenda of this edition of the meeting is to finalise a free trade agreement among the BIMSTEC
member countries. It will also include negotiations on market access for professionals and reducing the duty
on trading goods.
About BIMSTEC
The BIMSTEC is a sub-regional grouping comprising of seven countries of South Asia and South East Asia.
It was established on 6 June 1997 through the Bangkok Declaration. It is headquartered at Dhaka,
Bangladesh.
BIMSTEC mainly comprises of seven countries viz. Bangladesh, Bhutan, India, Nepal, Sri Lanka, Myanmar
and Thailand lying in the littoral and adjacent areas of the Bay of Bengal.
Its objectives are technological and economical cooperation among south Asian and south East Asian
countries along the coast of the Bay of Bengal. It also covers cooperation in commerce, technology,
investment, agriculture, tourism, human resource development, fisheries, transport and communication,
textiles, leather etc.
US Scientists develop micropropulsion system based on liquid water
Engineers from Purdue University in US have designed and tested a micropropulsion system called a Film-
Evaporation MEMS Tunable Array (FEMTA) thruster that uses liquid water as the propellant for orbital
maneuvering of tiny satellites called CubeSats.
CubeSats
CubeSats are basically miniature satellites which typically weigh around two kilograms. In the future, they
have potential to carry out tasks like imaging and remote-sensing currently performed by heavier satellites
which are expensive to build and launch. CubeSats at present cannot totally replace their larger counterparts
as they are incapable of changing orbit or performing complex manoeuvres as they donot have a propulsion
system. If CubeSats would be having system would allow such tiny satellites to correct their orbit or
maintain their altitude, thereby prolonging their operating life in space before becoming space debris.
Film-Evaporation MEMS Tunable Array (FEMTA)
The FEMTA system uses an innovative design of small thrusters that deliver bursts of water vapor to
manoeuvre the spacecraft into different orbits. It uses pure water as the propellant since it is safe, green, easy
to use and free from the risk of contaminating sensitive instruments by the backflow from plumes as in the
case of thrusters using chemical propellants.
It uses capillaries thinner than human hair through which the propellant water can flow. Small heaters
located near the ends of the capillaries turn the water into vapor, which, on escaping provides the thrust.
The minuscule capillaries act like valves that can be turned on and off by activating the heaters. It is compact
and not power hungry. The technology is said to be similar to the inkjet printer, which uses heaters that fire
dots of ink of ink at the paper.

2017 GKToday | All Rights Reserved | www.gktoday.in

83
Current Aairs: August, 2017 [Date-wise Compendium]

August 12, 2017


12 August: International Youth Day
The International Youth Day is observed across the world on 12 August to draw attention to a given set of
cultural and legal issues surrounding youth.
The theme for this year is Youth Building Peace. It aims at celebrating young peoples contributions in
preventing conflicts and bringing about transformations along with social justice, inclusion and sustainable
peace. It also reaffirms the important role youth can play in deterring and resolving conflicts, and are key
constituents in ensuring the success of both peacekeeping and peacebuilding efforts
Background
The International Youth Day (IYD) was instituted by the United Nations General Assembly (UNGA) by
passing resolution 54/120 in December 1999. It was first observed on 12 August, 2000.
The day was instituted by UNHA on the recommendation made by the World Conference of Ministers
Responsible for Youth held in Lisbon (Portugal) from 8 to 12 August 1998.
MCX gets SEBI approval to launch Indias first gold options contract
Commodity derivatives bourse Multi Commodity Exchange Ltd (MCX) has received markets regulator
Securities and Exchange Board of Indias (SEBI) approval to launch Indias first gold options contract.
The gold futures contract will have bi-monthly duration. The option will also have the existing gold kilo
futures contract as its underlier.
Key Facts
The launch of gold options is one of the major reforms SEBI has taken for the commodity derivatives
market. Earlier in June 2017, SEBI had allowed options trading in commodities to deepen the market but
permitted each exchange to launch options on futures of only one commodity initially.
SEBI is going to put strict eligibility criteria and options could be launched on futures contract of only those
commodities that are among the top five in terms of total trading turnover value of previous 12 months. It
also has stipulated necessary guidelines with regard to the product design and risk management framework
to be adopted for trading in options on commodity futures.
About Multi Commodity Exchange Ltd (MCX)
The MCX is the countrys largest metals and energy commodity bourse. It is countrys first listed commodity
futures exchange that facilitates online trading, and clearing and settlement of commodity futures
transactions, thereby providing a platform for risk management.
It began functioning in November 2003 and operates within the regulatory framework of the Forward
Contracts Regulation Act, 1952 (FCRA, 1952). It offers futures trading in bullion, ferrous and non-ferrous
metals, energy, and a number of agricultural commodities (mentha oil, cardamom, potatoes, palm oil and
others). Globally, MCX ranks no. 1 in silver, no. 2 in natural gas, no. 3 in crude oil and gold in futures
trading.

2017 GKToday | All Rights Reserved | www.gktoday.in

84
Current Aairs: August, 2017 [Date-wise Compendium]

Odisha government and Facebook launches SheMeansBusiness programme


Odisha government and social networking giant Facebook has launched SheMeansBusiness programme to
train women entrepreneurs in the state.
It was launched in partnership between Odishas MSME (micro, small and medium enterprises) department
and Project Mission Shakti.
Key Facts
Under this initiative, 25, 000 women entrepreneurs and self-help group (SHG) members from the state will
be given training on digital marketing skills within by the end of 2018. Facebook will monitor the growth of
the MSMEs in Odisha after imparting training to them in next one year.
It will also prepare a data base of the MSMEs through registrations in which they will take the email ids and
mobile numbers of entrepreneurs in all the workshops.
SheMeansBusiness programme
The programme has been launched in 16 countries around the world including India. It is designed to reach
out to aspiring and established women entrepreneurs to help them build and grow their business online.
Prasoon Joshi appointed as new chairman of CBFC
The Union Information and Broadcasting Ministry has appointed Prasoon Joshi as the new Chairman of
Central Board of Film Certification (CBFC). He will replace Pahlaj Nihalani and shall have tenure of three
years.
Besides, the ministry by exercising its powers conferred by sub-section (1) of section 3 of the Cinematograph
Act, 1952 and the Cinematograph (Certification) Rules, 1983 also reconstituted the existing CBFC Board for
a period of three years or until further orders, whichever is earlier. The new board will be headed by
Prasoon Joshi.
Prasoon Joshi
Prasoon Joshi is renowned songwriter-poet known for his contribution to various films and for designing
successful ad campaigns. He has been honoured with Padma Shri. He has penned the theme song for Prime
Minister Narendra Modis Swachh Bharat Abhiyan and other campaigns.
About Central Board of Film Certification (CBFC)
The CBFC also known as Censor Board is a statutory body under Union Ministry of Information and
Broadcasting. It grants certificate to regulate the public exhibition of films in India under the provisions of
the Cinematograph Act 1952.
The mandate of CBFC is to promote recreation, education, good and healthy entertainment to public
according to the provisions of the Cinematograph Act, 1952, and Cinematograph (Certification) Rules, 1983.
Films can be publicly exhibited in India only after they are certified by the CBFC.
The CBFC board is headed by central government appointed chairperson and 12-45 non-official members.
The members are eminent persons from social science, education, law, arts or films background appointed.
Its headquarters are in Mumbai, Maharashtra.

2017 GKToday | All Rights Reserved | www.gktoday.in

85
Current Aairs: August, 2017 [Date-wise Compendium]

IIP output contracts 0.1% in June 2017


Indias factory output, measured by the Index of Industrial Production (IIP) has registered negative 0.1%
growth in June 2017. It was mainly due to a fall in output of the manufacturing and capital goods sectors.
According to data released by the Central Statistics Office (CSO) it is the first negative fall since June 2013.
In June 2016, it had grown 8%.
Key Facts
Manufacturing sector: It contracted by 0.4% in June 2017.
Mining output: It rose by 0.4%.
Electricity generation: It increased by 2.1%.
Capital goods output: is a barometer of investment. It shrank by 6.8% in June 2017.
Consumer durables output: It contracted by 2.1%.
Consumer non-durables production: It rose by 4.9%.

About Index of Industrial Production (IIP)


The IIP is a composite indicator that measures the short-term changes in the volume of production of a
basket of industrial products during a given period with respect to chosen base period. It is compiled and
published monthly by the Central Statistical Organization (CSO), Ministry of Statistics and Programme
Implementation.
Base year: The CSO had revised the base year of the IIP from 2004-05 to 2011-12 in May 2017 to capture
structural changes in the economy and improves the quality and representativeness of the indices. The
revised IIP (2011-12) reflects the changes in the industrial sector and also aligns it with the base year of other
macroeconomic indicators like the Wholesale Price Index (WPI) and Gross Domestic Product (GDP).

2017 GKToday | All Rights Reserved | www.gktoday.in

86
Current Aairs: August, 2017 [Date-wise Compendium]

Sector wise items and weightages: It covers 407 item groups. Sector wise, the items included falls into 3
categories viz. Manufacturing (405 items), Mining (1 items) & Electricity (1 item). The weights of the three
sectors are 77.63%, 14.37%, 7.9% respectively. The revised eight core Industries have a combined weightage
of 40.27% in the IIP. Decreasing order of weightage of core industries is Electricity> Steel> Refinery
Products> Crude> Coal> Cement> Natural Gas> Fertilizers.
Economic Survey 2016-17 Volume II: Agriculture and Food Management
Reforms Measures
The Economic Survey 2016-17 Volume II released recently has taken into consideration various challenges
faced by the Agriculture sector in India. It has suggested multi-dimensional Agricultural and Food
Management Reforms Measures.
Challenges
Small Operational Land Holdings: The average farm size in India is small and declining since 1970-71. It
is a major limitation to reap the benefits of economies of scale in agriculture operations.
Credit: The predominance of informal sources of credit for farmers is a concern. Moreover, there is regional
disparity in the distribution of agricultural credit.
Post- harvest losses: The horticulture sector in India faces problems like post-harvest losses, availability of
quality planting material and lack of market access for horticultural produce of small farmers.
Reforms suggested
Price risks in agriculture and allied sectors: Strengthening and building marketing infrastructure along
the entire value chain.
Production risks: The share of irrigated area should be expanded by increasing the coverage of water
saving irrigation systems like micro irrigation systems.
Increase productivity of crops: Standards should be set and enforced for better quality, pest and disease
resistant seeds.
Trade and domestic policy changes: It should be announced well before sowing and should stay till
arrivals and procurement is over.
Enhance womens involvement in the dairy projects: Funds should be earmarked through appropriate
mechanisms.
Formal and institutional credit: It should be provided in timely and affordable manner to the small and
marginal farmers is the key to inclusive growth. Regime based on timely interventions needs to be adopted.
Economic Survey proposes Transparency of Rules Act
The Second volume of Economic Survey 2016-17 has proposed Transparency of Rules Act (TORA), a
progressive legislation to end any asymmetry of information regarding rules and regulations faced by an
average citizen.
The objective of TORA is to help citizens overcome an opaque mesh of complicated rules that often leads to
corruption and endless litigation.
2017 GKToday | All Rights Reserved | www.gktoday.in

87
Current Aairs: August, 2017 [Date-wise Compendium]

Need
At present due to opaque mesh of regulations prevalent in India make life of ordinary citizens (as well as
businesses) difficult as it is not easy for ordinary citizens for them to navigate the multitude of rules,
regulations, forms, taxes and procedures imposed by various tiers of government. Moreover, these rules
frequently change and sometimes contradict each other. Even government officials struggle to keep up with
the latest version of complicated rules. They also act as a magnet for corruption and endless litigation.
Key Features of TORA
TORA will require all government departments to mandatorily place all citizen-friendly rules on their
website. Government Officials will not be able to impose any rule not mentioned beforehand. It will make
mandatory for updating all existing laws by the department.
Government websites will also have to notify the date and time of each change made. TORA will normally
be applicable after a specified time after the rule has been posted. TORA compliant departments will ensure
that citizens get authentic and updated information.
Way Forward
India will benefit enormously if the average citizen could easily access the latest rules and regulations in a
comprehensible format. Transparency of Rules Act (TORA) will be a possible solution for this.
Government launches nationwide campaign Gaj Yatra to protect elephants
The Union Ministry of Environment and Forest has launched a nationwide campaign Gaj Yatra to protect
elephants. It was launched on the occasion of World Elephant Day observed on August 12.
The ministry also released the All India Census 2017 Report on elephants and Agreed Points of Action on
Trans-Boundary Conservation of Elephants by India and Bangladesh.
Gaj Yatra Campaign
The campaign is planned to cover 12 elephant range states across the country. It is 15 months campaign the
will be led by the Wildlife Trust of India (WTI). The Gaju mascot, which was released by the Ministry in
2012, will be helm of the campaign.
The campaign aims create awareness about elephant corridors to encourage free movement in their habitat.
During the period of the campaign, artists and craftsmen will create life-size works on the theme of
elephants in places along the route of the roadshow using local art and craft. Specially fabricated vehicles will
be also deployed to display these on pre-determined routes with campaigners.
World Elephant Day
The World Elephant Day is observed every year on August 12 to support various stakeholders involved in
supporting various conservation policies to protect elephants. It also seeks to support stakeholders involved
in improving enforcement policies to prevent illegal poaching and trade in ivory, conserving elephant
habitats, providing better treatment for captive elephants and reintroducing captive elephants into
sanctuaries.
The World Elephant Day was conceived in 2011 by Canadian filmmakers Patricia Sims and Michael Clark of
Canazwest Pictures and Sivaporn Dardarananda, Secretary-General of Elephant Reintroduction Foundation
2017 GKToday | All Rights Reserved | www.gktoday.in

88
Current Aairs: August, 2017 [Date-wise Compendium]

of Thailand. Now it is supported by over 65 wildlife organisations and many individuals in several countries
across the globe.
Elephants
As per the available population estimates, there are about 400,000 African elephants and 40,000 Asian
elephants across the globe. Asian elephants are listed as endangered and African elephants are listed as
vulnerable in the IUCN Red List of threatened species.
Indian Coast Guard Ship Shaurya commissioned in Goa
Indian Coast Guard Ship (ICGS) Shaurya was commission in Vasco, Goa. ICGS Shaurya is an offshore patrol
vessel designed and built indigenously by Goa Shipyard Ltd (GSL).
It will be based at Chennai under administrative and operational control of Commander Coast Guard
Region (East).
About ICGS Shaurya
ICGS Shaurya is fifth of a series of six OPVs designed in-house and built by GSL. It is 105-metre long and
capable of cruising at a speed of 23 knots. It has a range of 6000 nautical miles.
It is powered by twin diesel engines and carries four boats for boarding party operations. It is fitted with
state-of-the-art advanced navigation and communication equipment. Its weaponry package has one 30 mm
gun and two 12.7 mm guns associated with fire control system.
The ship has capacity to carry twin engine light helicopter and five high speed boats including quick
response interceptor boats for quick boarding operations, search and rescue, law enforcement and maritime
patrol. It is also capable of carrying pollution response equipment to combat oil spill contamination.
CSIR-IGIB and NCDC Researchers develop DNA sensor for quick pathogen
detection
Researchers form CSIR-Institute of Genomics and Integrative Biology (CSIR-IGIB) and National Centre for
Disease Control (NCDC) have developed an ultrasensitive DNA chip based sensor for quick pathogen
detection.
The senor can detect S. pyogenes, a bacterium which causes a wide range of diseases in about 30 minutes.
The conventional method takes 18-24 hours identification of the S. pyogenes.
Key Facts
The DNA chip based sensor consists of a carbon electrode embedded with gold nanoparticles. By means of a
bioinformatics study, researchers were able to design probes which are specific for S. pyogenes. The working
electrode surface of the sensor is attached with several small-sized, single-stranded DNA probe specific to
the pathogen.
When patients DNA, isolated from throat swabs are placed on the surface of the sensor, they bind to the
complementary single-stranded DNA on the device leading to electrochemical change. This is measured
using a differential pulse voltammetry.
Significance: The sensor is highly sensitive and could detect even 60-65 bacteria in a 6 microlitre sample. It

2017 GKToday | All Rights Reserved | www.gktoday.in

89
Current Aairs: August, 2017 [Date-wise Compendium]

could identify the pathogen even at very low concentrations of DNA.The sensor was found to be stable for
12 months with only 10% loss in initial current peak on storage at 4 degree C.
S. pyogenes infections
S. pyogenes cause diseases ranging from mild skin and throat infections to life-threatening toxic shock
syndrome. If not treated during early stages of the infection, it can even lead to rheumatic heart disease
(heart valves damage). It affects 700 million people every year.
August 13, 14, 2017
India-ASEAN Youth Summit begins in Bhopal
The India-ASEAN Youth Summit began in Bhopal, Madhya Pradesh to commemorate 25th anniversary of
Association of Southeast Asian Nations (ASEAN)India dialogue partnership in the ongoing year.
The first of its kind event was organised by Union Foreign Ministry, Union Sports Ministry, Madhya
Pradesh government and India Foundation.
Key Facts
The five day Summit will commemorative this years theme of Shared Values, Common Destiny. Madhya
Pradesh was chosen for this event as the Sanchi stupa located here strengthens the states cultural affiliation
with the ASEAN countries.
Around 100 delegates from ASEAN countries Indonesia, Singapore, Philippines, Malaysia, Brunei, Thailand,
Cambodia, Laos, Myanmar and Vietnam are going to participate in the summit. It will also have
participation of 75 youth leaders from India.
The participants will represent some of the largest institutions in their countries, including leaders from
ruling as well as opposition parties, think tanks, media and universities to renew and revitalise engagement
with the region reflecting close cultural and civilisational links.
About Association of South East Asian Nations (ASEAN)
ASEAN regional organisation comprising 10 South east Asian countries that aims to promote
intergovernmental cooperation and facilitates economic integration amongst its members. It came into
existence on August 8, 1967 after ASEAN declaration (also known as Bangkok declaration).
Principal aims of the ASEAN are to accelerate economic growth, intergovernmental cooperation, social
progress, and sociocultural evolution among its members, protect of regional stability and facilitate political,
military, educational and cultural integration amongst its members and Asian states.
Its 10 members are Brunei, Cambodia, Indonesia, Laos, Malaysia, Myanmar, Philippines, Singapore,
Thailand and Vietnam. Its motto is One Vision, One Identity, One Community.
Indias first online course on Infusion Therapy launched by INS
The Infusion Nurses Society (INS)-India in its 6th National conference launched countrys first online course
on Infusion Therapy for nurses. INS is global authority in infusion therapy.
Under the online course, the INS aims to train over 3000 nurses in the vital medical procedure. The course
will enable members to access the various infusion therapy modules and presentations through its website.

2017 GKToday | All Rights Reserved | www.gktoday.in

90
Current Aairs: August, 2017 [Date-wise Compendium]

About Infusion therapy


Infusion therapy involves the administration of medication drug intravenously i.e. through intramuscular
injections and epidural routes. It is prescribed when a patients condition is severe that it cannot be treated
effectively by oral medications.
Nine of ten patients admitted in hospitals receive infusion therapy for therapeutic or diagnostic purposes.
Some of the diseases requiring infusion therapy include infections that are unresponsive to oral antibiotics,
cancer and cancer-related pain, gastrointestinal diseases, dehydration and more. Improper infusion practices
may lead to complications, causing an increase in morbidity, mortality, duration of hospital stay and
healthcare costs.
Indra: India, Russia for first time conduct tri-services exercise
India and Russia for first time are going to conduct Indra tri-services exercise, involving their armies,
navies and the air forces in October 2017 to further ramp up military ties.
It will be for the first time India will participate in a tri-services exercise with a foreign country with such
large scale participation by Army, Navy and Air Force.
Key Facts
The 2017 Indra exercise will be held in Russia including in mountainous Vladivostok region. It will
primarily focus on achieving coordination between forces of the two countries in a tri- services integrated
theatre command scenario.
A total of 350 Indian Army personnel will participate at the mega exercise. The contingents by the Indian
Navy and AirForce (IAF) will also be sizeable in numbers. Russian army, navy and air force also for time
holding tri-services exercise with other country.
Comment
The exercise will take place in the midst of evolving unpredictable security situation in the region. It will be
conducted at a time when Indias ties with China have nosedived over the border dispute issue and also at
time when its ties with Pakistan have deteriorated over cross-border terrorism and number of other issues.
Background
India is significantly ramping up its defence capability and has lined up billions of dollars of procurement
proposals as part of military modernisation. Russia has been a long-standing defence partner of India and
both the countries are now eyeing to further deepen the ties. During Prime Minister Narendra Modis visit
to Russia in June 2017, both countries had decided to upgrade and intensify defence cooperation through
joint manufacture, co-production and co-development of key military hardware and equipment. A vision
document, issued then also had mentioned that both the countries also decided to work towards a
qualitatively higher level of military-to-military cooperation.
Scientists develop worlds first plant-based Zika vaccine
Scientists from US have developed the worlds first plant-based Zika vaccine that may be more effective,
safer and cheaper than other vaccines against the mosquito-borne Zika virus.
The vaccine was developed using proteins derived from Tobacco plant. Currently, there are no licensed
2017 GKToday | All Rights Reserved | www.gktoday.in

91
Current Aairs: August, 2017 [Date-wise Compendium]

vaccines or therapeutics available to combat Zika.


Key Facts
The plant-based Zika vaccine works against DIII, a part of a Zika viral protein that plays a key role for the
virus to infect people. All flaviviruses have the envelope protein on the outside part of the virus. It has three
domains. The domain III has a unique stretch of DNA for the Zika virus.
Researchers exploited domain III to generate a robust and protective immune response that is unique for
Zika. First they had grown the envelope protein in bacteria then prepared the DIII protein domain in
tobacco plants. The multiple immunisation experiments of the vaccine performed on mice shows 100%
protection against multiple Zika virus strains in mice.
Significance: The plant protein-based vaccine uses smallest and most unique part of the Zika virus that can
still elicit a potent and robust immune response. It produces a potent protective immune response against
Zika Virus and also not produce antibodies that may be cross reactive for West Nile fever, Dengue, Yellow
fever or others.
About Zika virus
Zika virus is vector borne disease transmitted primarily by Aedes aegypti mosquitoes, the same mosquito
that transmits dengue, West Nile fever, Yellow fever. The virus has been named after Zika forest in Uganda
where it was first isolated in 1947.
The virus is capable of causing serious birth defects i.e. neurological disorders and foetal deformation known
as Microcephaly in which infants are born with abnormally smaller heads. Besides a possible link between the
virus and Guillain-Barr syndrome (a condition in which the bodys immune system attacks part of the
nervous system) is also suspected. It is also found that, sexual transmission of Zika virus disease is possible.
The worldwide Zika threat first emerged in 2015, infecting millions in South America. The World Health
Organization (WHO) had declared the disease as epidemic and global health emergency but later revoked it
after the outbreak was brought under control.
IMMSAREX: IONSs maiden maritime search and rescue exercise will be held in
November 2017
Indian Ocean Naval Symposiums (IONS) maiden International Maritime Search and Rescue Exercise
(IMMSAREX) will be held in November 2017 in the Bay of Bengal under the chairmanship of Bangladesh.
The ships and aircraft of the members and observers of the IONS including China, India and Pakistan will
participate in the exercise. In addition, Bangladesh is also scheduling to host extraordinary conclave of
Chiefs, a meeting of chiefs of Navy of IONS members before it hands over the Chair to Iran.
Comment
The side-by-side participation of the Indian and Chinese Navies in IMMSAREX exercise will come amid
growing Indias skepticism of Chinese naval activities in the Indian Ocean region (IOR) and establishment of
Chinas first overseas base at Djibouti. India has considerably expanded its engagement with countries to
further its own interests as well as to check the rapid expansion of Chinese naval forays in the IOR. Other

2017 GKToday | All Rights Reserved | www.gktoday.in

92
Current Aairs: August, 2017 [Date-wise Compendium]

countries in the region are also engaged in rapid expansion of their military capabilities.
About Indian Ocean Naval Symposium (IONS)
The IONS is a regional forum of Indian Ocean littoral states launched by India in February 2008. It is a
voluntary initiative that seeks to increase maritime co-operation among navies of the littoral states of the
Indian Ocean Region (IOR) by providing an open and inclusive forum for discussion of regionally relevant
maritime issues.
It endeavors to generate a flow of information between naval professionals that would lead to common
understanding and possibly cooperative solutions on the way ahead.These states are represented by the
heads of their navies.
Under the IONS charter of business adopted in 2014, the forum has working groups on Humanitarian
Assistance and Disaster Relief (HADR), Information Security and Interoperability (IS&I) and maritime
security (anti-piracy).
Members to IONS
There are 35 members navies of the IONS which have been geographically grouped into the following four
sub-regions:
South Asian Littorals: India, Bangladesh, Maldives, Pakistan, Seychelles and Sri Lanka
West Asian Littorals: Bahrain, Iran, Iraq, Kuwait, Oman, Qatar, Saudi Arabia, UAE and
Yemen
East African Littorals: Comoros, Djibouti, Egypt, Eriteria, France, Kenya, Madagascar,
Mauritius, Mozambique, Somalia, South Africa, Sudan and Tanzania
South East Asian and Australian Littorals: Australia, Indonesia, Malaysia, Myanmar,
Singapore, Thailand and Timor Leste.
Observer Countries: China, Japan etc.
El Nino of 2014-16 aided in massive carbon dioxide release: Study
According to recent study conducted by scientists, the monster El Nino of 2014-16 caused over 3 billion
tonnes of carbon to get released into the atmosphere, pushing carbon dioxide (CO2) concentration to record
levels.
The study was based on analysis of data collected by NASAs Orbiting Carbon Observatory-2 (OCO-2)
satellite, which measures level of CO2 in the atmosphere. These are the first measurements for satellite
tracking CO2 levels.
El Nino
El Nino is a complex periodic climate event that causes waters to warm up in east-central Pacific Ocean. The
warming of ocean causes huge changes in wind directions which bring less rain to south-east Asia and
Indian subcontinent, while increasing rain in other parts of the world.
Key Highlights of study
The El Nino led to excessive carbon dioxide releases in three ways. They are (i) Hot weather and drought
2017 GKToday | All Rights Reserved | www.gktoday.in

93
Current Aairs: August, 2017 [Date-wise Compendium]

caused extensive wildfires in south-east Asia, (ii) Drought in the Amazon rainforest stunted plant growth,
reducing the amount of carbon they absorb while growing (iii) Warmer weather and near normal rainfall in
Africa caused forests to exhale more CO2.
The rate of growth of CO2 in the atmosphere had hit an all-time high of 2.94 parts per million per year in
2015 and slightly below that at 2.89 ppm per year in 2016. In other words, CO2 was being added to the
atmosphere at a much higher rate than ever before even though carbon emissions were flat.
In 2014 and 2015, CO2 emissions from burning of fossil fuels had flattened out to about 36.2 billion tonnes.
Projections for 2016 too indicated that emissions were still flat.
The industrialised countries do not appear to be on course to meet the targets that they pledged at the Paris
Climate Accord of 2015. Emissions from European Union countries have actually increased in 2015, the rate
at which emissions from US and Japan are declining does not comply with what they had pledged at Paris.
Earths Largest volcanic region found in Antarctica
Researchers from the University of Edinburgh in Britain have discovered the largest volcanic region on
Earth, two km below the surface of the vast ice sheet in west Antarctica.
They have found 91 previously unknown active volcanoes in the region known as the West Antarctic rift
system, adding to the exiting 47 volcanoes that were discovered earlier. This makes it largest volcanic region
on the Earth.
Key Facts
These active peaks are concentrated in the west Antarctic rift system region which stretches 3,500 km
from Antarcticas Ross ice shelf

to the Antarctic peninsula. The height of these newly discovered active volcanoes range from 100 to 3,850
2017 GKToday | All Rights Reserved | www.gktoday.in

94
Current Aairs: August, 2017 [Date-wise Compendium]

metres, with the highest being almost as tall as Switzerlands Eiger mountain (3,970 metre). All of these
volcanoes are covered in thick layers of ice.
This region is larger than east Africas volcanic ridge which is currently rated as the densest concentration of
volcanoes in the world. Any volcanic activity of Antarctic rift system may have crucial implications for the
rest of the planet.
If one of the volcanoes in Antarctic rift system erupts, it could further destabilise west Antarcticas ice sheets.
If it causes the melting of ice on eruption may speed up the flow of ice into the sea. It will enhance sea level
rises that are already affecting our oceans due to climate change.
August 16, 2017
GeM and CII sign MoU to establish GeMSamvad
The Public procurement portal Government eMarketplace (GeM) and industry body Confederation of
Indian Industry (CII) have signed a MoU to establish GeMSamvad, a GeM -Industry forum.
GeMSamvad aims to create awareness about online purchase of goods and services by government agencies
and departments,
Key Facts
GeMSamvad will be a participative and collaborative approach with the industry for promoting Indian
industry and entrepreneurship. The MoU will facilitate GeM and CII to work together to create GeM
related awareness across the country.
The MoU facilitates establishment of GeM-Industry Forum for collaboration with the Indian Industry for
seeking inputs on the technical specifications of products and service-level agreement of services to be
procured by government agencies. It also calls for improving the quality of products procured from the
MSMEs and organize annual Public Procurement Convention of all stakeholders. It also calls for setting up
of GeM Resource Centres at CII Regional Offices.
About Government eMarketplace (GeM)
GeM is an online market platform to facilitate procurement of goods and services by various Ministries and
agencies of the Government. It was established by Directorate General of Supplies and Disposal (DGS&D),
Ministry of Commerce and Industries.
GeM aims to enhance transparency, efficiency and speed in public procurement. It provides the tools of e-
bidding, reverse e-auction and demand aggregation to facilitate the government procurement. It is a
completely paperless, cashless and system driven e-market place that enables procurement of common use
goods and services with minimal human interface.
Scientists develop injectable tissue bandage to repair hearts
Scientists from the University of Toronto, Canada have developed an injectable tissue bandage dubbed as
AngioChip, smaller than a postage stamp that can repair damaged hearts.
The AngioChip is a tiny patch of heart tissue with its own blood vessels and heart cells beating with a regular
rhythm. It is made out of the biocompatible, biodegradable polymer.

2017 GKToday | All Rights Reserved | www.gktoday.in

95
Current Aairs: August, 2017 [Date-wise Compendium]

Key Facts
Repairing heart tissue destroyed by a heart attack or medical condition with regenerative cells usually
requires invasive open-heart surgery which usually poses more risks than potential benefits. The newly
developed technique lets researchers to inject a repair patch (AngioChip) using a small needle, without the
need to open up the chest cavity. Researchers by injecting the patch into rat hearts have shown that it can
improve cardiac function after a heart attack.
Black Carbon released by aeroplanes may be affecting ozone, monsoon: Study
According to a recent study by climate researchers, aeroplanes may be ejecting significant amounts of black
carbon (BC) which in turn is affecting monsoon, depleting the ozone layer and quickening glacier melt.
The study was conducted by climate researchers from multiple institutions in India including from the
Indian Institute of Science and ISROs Vikram Sarabhai Space Centre.
Key Facts
Earlier it was believed that airborne BC is unlikely to travel upward of 4 km and dissipate and settle down in
few months under the influence of wind and rain. However, this study shows that such particles exist up to
18 km into the stratosphere, a stable region of the atmosphere.
Given the shape and location of these BC particles, researchers believe they could only derive from emissions
from burning of aviation fuel in aeroplanes. As BC particles absorb heat, they warm the surrounding air,
become lighter and rise to greater heights by a process called self-lift and persist for longer time in the air.
The airborne BC particles released by aeroplanes possess a problem because they can linger long time,
enough to provide a fertile ground for other chemical reactions that can deplete the ozone layer. As, BC
particles strongly absorb solar and terrestrial radiation and heats up the atmosphere it can also upset the
monsoon system. If deposited on snow, it could accelerate the heating of snow and quicken the melting of
glaciers.
Significance of Study: This is the first time that any group of climate researchers in the world has shown
that black carbon from aircraft can go to the stratosphere and affect the ozone layer.
India to host steering committee meeting of TAPI gas pipeline
India is going to host the next steering committee meeting of the proposed 1,814 kilometre-long
Turkmenistan-Afghanistan-Pakistan-India (TAPI) gas pipeline.
Decision in this regard was taken at the sixth joint India-Turkmenistan Inter-Governmental Committee
(IGC) meeting on trade, economic, scientific and technological cooperation.
The IGC discussions between both countries comprehensively reviewed the entire gamut of bilateral
relations and took stock of the progress achieved in the areas of energy, security cooperation, civil aviation,
information technology, science and technology, trade and economic cooperation, transport and
connectivity, cultural and educational matters.
About Turkmenistan-Afghanistan-Pakistan-India (TAPI) gas pipeline
TAPI gas pipeline project or Trans-Afghanistan Pipeline is a proposed trans-country natural gas pipeline
developed by the Asian Development Bank (ADB). The pipeline first proposed in 1995 will transport natural
2017 GKToday | All Rights Reserved | www.gktoday.in

96
Current Aairs: August, 2017 [Date-wise Compendium]

gas from energy rich Caspian Sea (Galkynysh gasfields: fourth largest in the world) in Turkmenistan to India
through Afghanistan into Pakistan. Construction on the project started in Turkmenistan on December 2015.
It is expected to be operational by 2019.
The project will supply both India and Pakistan about 38 mmscmd of natural gas whereas Afghanistan will
get 14 mmscmd supply. The life of supply is expected to last for period of 30 years and shall be operational
from 2019. Member countries of the project have recommended establishing an inter-government joint
security task force (JSTF) to serve as the nucleus of the safety of the pipeline.

Comment
TAPI gas pipeline project is considered as an important initiative of these four countries as it connects
energy rich Central Asia with energy starved South Asia. It will enhance economic engagement through
regional connectivity by economically integrating region stretching from the Bay of Bengal to the Caspian
Sea. It will not just be a commercial project, but also help in providing peace and security in the region.
From Indias perspective, TAPI Project will provide an alternative supply source of gas with dependable
reserves leading to enhanced energy security. It will further diversify the fuel basket to the benefit of Indian
economy as it would be used mainly in power, fertilizer and city gas sectors.
Nepal, China sign three pacts to boost energy, economic ties
China and Nepal have signed three pacts to further boost their ties in power, energy and economic sectors.
The agreements signed cover oil and gas resources exploration, economic and technological cooperation and
promotion of investment.
These agreements were signed after visiting Chinese Vice-Premier Wang Yang held discussions with

2017 GKToday | All Rights Reserved | www.gktoday.in

97
Current Aairs: August, 2017 [Date-wise Compendium]

Nepals Deputy Prime Ministers Bijaya Kumar Gachhadar and Krishna Bahadur Mahara.
Key Facts
The agreements cover three different domains viz. (i) economic and technological cooperation, (ii)
framework agreement on promotion of investment and economic cooperation and (iii) China-aid oil and gas
resources survey project.
These pacts also include a feasibility study for excavating natural gas and petroleum in mountain areas and
plains of Terai region of the Himalayan country. They will help to make significant contributions in Nepals
socio- economic transformation.
Both countries also agreed to establish hydropower projects and transmission lines and take steps for
technological development in Nepal, which suffers from acute shortage of electricity. China also agreed to
upgrade 114-kilometre Araniko highway- an old route connecting two countries, which was damaged and
subsequently closed after the massive 2015 Nepal earthquake.
CCEA approves procedure and mechanism for Strategic Disinvestment
The Cabinet Committee on Economic Affairs (CCEA) chaired by the Prime Minister Narendra Modi has
approved revision in the procedure and mechanism for strategic disinvestment.
In this regard, the proposal was forwarded by the Department of Investment and public Asset Management
(DIPAM). The approval will help in speedy completion of strategic disinvestment transactions.
Revised mechanism for strategic disinvestment
Setting up an Alternative Mechanism (AM): It will decide on the matters relating to terms and
conditions of the sale from the stage of inviting of Express of Interests (Eols) till inviting of financial bid. It
will consist of the Finance Minister, Minister for Road Transport & Highways and Minister of
Administrative Department.
Empowering the Core Group of Secretaries (CGD): It will enable CGD to take policy decisions with
regard to procedural issues and consider deviations as necessary from time to time for effective
implementation of decisions of CCEA.
Strategic Disinvestment
In Strategic disinvestment, significant proportion of a Public Sector Units (PSU) share and the management
control goes to a private sector which is considered as strategic partner. It is different from the ordinary
disinvestment in which management of PSU is retained with Government.
Department of Investment and public Asset Management (DIPAM)
DIPAM is the nodal agency of Union Finance Ministry mandated to advise the Union Government in the
matters of financial restructuring of PSUs and also for attracting investment through capital markets. It will
also deal with all matters relating to sale of Union Governments equity in PSUs through private placement
or offer for sale or any other mode in the erstwhile Central PSUs.
Union Cabinet approves new Metro Policy
The Union Cabinet has approved a new policy for expanding and regulating metro rail services in cities
across India. This is the first such policy document prepared by the Union Urban Development Ministry
2017 GKToday | All Rights Reserved | www.gktoday.in

98
Current Aairs: August, 2017 [Date-wise Compendium]

since metro rail operations began in Delhi in 2002.


The policy gives a big boost to private players by making private participation mandatory for all the three
funding options. They are (i) Public-private partnership (PPP) model with central assistance under the
Viability Gap Funding (VGF) scheme of the Union Finance Ministry, (ii) Grant from the Centre under
which 10% of the project cost would be given as a lump sum, (iii) 50:50 equity sharing model between the
Central and State governments.
Salient Features of new policy
Private investments: The policy opens a big window for private investments across a range of metro
operations by making PPP component mandatory for availing central assistance for new metro projects. It
will help to meet the huge resource demand for capital intensive high capacity metro projects.
Last mile connectivity: The new policy seeks to ensure focusing on a catchment area of five kms for last
mile connectivity on either side of metro stations. It makes mandatory for States to commit last mile
connectivity in project reports by providing necessary feeder services, Non-Motorised Transport
infrastructure like cycling and walking pathways and para-transport facilities. States, proposing new metro
projects will be required to indicate such proposals and investments to be made for such services in the
project reports.
Alternatives: It mandates Alternate Analysis, requiring evaluation of other modes of mass transit like Light
Rail Transit, Tramways, BRTS (Bus Rapid Transit System), Metro Rail and Regional Rail in terms of
demand, capacity, cost and ease of implementation to ensure that least cost mass transit mode is selected for
public transport
Urban Metropolitan Transport Authority (UMTA): It makes mandatory for setting up UMTA to
prepare Comprehensive Mobility Plans for cities for ensuring complete multi-modal integration for optimal
utilization of capacities.
Third party assessment: It calls for rigorous assessment of new metro proposals and also independent
third party assessment by agencies to be identified by the Government like the Institute of Urban Transport
and other such Centres of Excellence.
Transit Oriented Development (TOD): It mandates TOD to promote compact and dense urban
development along metro corridors since TOD reduces travel distances besides enabling efficient land use in
urban areas. It will ensure that urban mass transit projects will not merely be seen as urban transport
projects but more as urban transformation projects.
Role of States: States need to adopt innovative mechanisms like Value Capture Financing (VCF) tools to
mobilize resources for financing metro projects. It must be done by capturing a share of increase in the asset
values through Betterment Levy. States will be also required raise low cost debt capital through issuance of
corporate bonds for metro projects. It also empowers States to set up permanent Fare Fixation Authority
(FFA) for timely revision of fares and make rules and regulations in this regard.
Financial viability: It requires the States to clearly indicate in the project report the measures to be taken

2017 GKToday | All Rights Reserved | www.gktoday.in

99
Current Aairs: August, 2017 [Date-wise Compendium]

for commercial/property development at stations and on other urban land to ensure financial viability of
metro projects. States are also require to raise revenues through other means of maximum non-fare revenue
generation through lease of space, advertisements etc., backed by statutory support.
Proposed ways for private sector participation in O & M of metro services
Cost plus fee contract: Private operator will be paid a monthly/annual payment for O&M of metro system.
It can have a fixed and variable component depending on quality of service. Operational and revenue risk
will be also borne by the owner.
Gross Cost Contract: Private operator will be paid a fixed sum for the duration of the contract. Operator
will bear the O&M risk while the owner will bear the revenue risk.
Net Cost Contract: Operator collects the complete revenue generated for the services provided. If revenue
generation is below the O&M cost, then owner may agree to compensate.
NPPA caps prices of orthopaedic knee implants
The drug price regulator National Pharmaceuticals Pricing Authority (NPPA) has capped the prices of
orthopaedic knee implants upto 70% lower than current market rates.
It will be valid for all orthopaedic knee implants sold within India whether domestic or imported. Earlier
in February 2017, the NPPA had capped the prices of stents used in cardiac surgery.
Background
According to World Health Organization (WHO) estimation, osteoarthritis is going to be the fourth largest
cause of immobility in globally by 2020. NPPA in its preliminary analysis had found that importers,
distributors and hospitals are earning hefty trade margins ranging from 67% to as high as 449% orthopaedic
knee implants.
Capped Prices
Now, the most widely used Cobalt chromium knee implant will cost around Rs. 55000 instead of current
average price of over Rs. 1.58 lakh. Titanium Oxidized Zirconium surgery implant has been capped at
Rs.76,600 against the current average prices of Rs.2.5 to 4.50 lakh. The High flexibility implant costing from
Rs. 1.80 to 4.5 lakh has been capped at Rs. 56,490. The revision implants for second surgery has been capped
at over Rs. 1.13 lakh. Besides, the Specialized implants for cancer and tumor will be charged at over Rs. 1.13
lakh.
Comment
This decision has been taken keeping peoples interest and health security of the nation into consideration. It
is considered as a major relief to patients suffering from knee problems. It will help check unethical
profiteering in the implant surgery. It will benefit around 1.5 to 2 crore knee patients and result in savings of
Rs 1,500 crore annually to patients requiring surgery.
National Pharmaceutical Pricing Authority (NPPA)
NPPA is an independent body under Department of Pharmaceuticals under the Union Ministry of Chemicals
and Fertilizers. Its functions are to fix/revise the controlled bulk drugs prices and formulations, enforce
prices and availability of the medicines under the Drugs (Prices Control) Order, 1995/2013, recover amounts
2017 GKToday | All Rights Reserved | www.gktoday.in

100
Current Aairs: August, 2017 [Date-wise Compendium]

overcharged by manufacturers for the controlled drugs from the consumers and monitor the prices of
decontrolled drugs in order to keep them at reasonable levels.
Cabinet approves creation non-lapsable Madhyamik and Uchchtar Shiksha Kosh
The Union Cabinet has approved creation of a single corpus fund non-lapsable known as Madhyamik and
Uchchtar Shiksha Kosh (MUSK).
Under it, all proceeds of secondary and higher education cess will be credited and utilised for the
governments schemes in education sector.
Madhyamik and Uchchtar Shiksha Kosh (MUSK)
The funds arising from MUSK will be utilised for schemes in education sector, which will be available for
benefit of students of secondary and higher education all over the country. It will be administered and
maintained by the Union Ministry of Human Resource Development.
The MUSK will be maintained as a Reserve Fund in the non-interest bearing section of the Public Accounts
of India. It will be operationalised as per the present arrangements under Prarambhik Shiksha Kosh (PSK)
wherein the proceeds of cess are used for Sarv Shiksha Abhiyan (SSA) and Mid-Day Meal (MDM) Schemes.
The expenditure on ongoing schemes of the HRD Ministry will be initially incurred from the gross
budgetary support (GBS) in any financial year and the expenditure will be financed from MUSK only after
the GBS is exhausted.
Background
During the 10th Plan, an education cess of 2% was levied on all central taxes from 2004 to make available
additional resources for basic education/elementary education to augment the existing budgetary resources.
Later in 2007, additional cess of 1% on central taxes (called Secondary and Higher Education Cess) was levied
through Finance Act, 2007 to give fillip for universalizing access to secondary education and expanding the
reach of the higher education sector. In February 2016, HRD Ministry had sought the approval of the
Department of Economic Affairs for creation of non-lapsable fund in the Public Account called MUSK.
August 17, 2017
Malpelo plate: New tectonic microplate discovered off the coast of Ecuador
Researchers from US-based Rice University have found new tectonic mircoplate off Ecuadors coast in the
eastern Pacific Ocean. It has been named Malpelo plate, after a Colombian island and an oceanic ridge it
contains.
It is overall 57th tectonic mircoplate to be discovered so far and the first in nearly a decade. Microplates are
tectonic plates with an area less than 1 million km2.
Key Facts
Malpelo microplate is located west of the Galapagos Islands off the coast of Ecuador. It is wedged in-between
the Nazca, Cocos, and Caribbean minor plates. It is linked to a nearby oceanic ridge along the Ring of Fire.
Earlier it was that assumed most of the region east of the known Panama transform fault was part of the
Nazca plate. But recent study showed that it is different tectonic plate moving independently in a different

2017 GKToday | All Rights Reserved | www.gktoday.in

101
Current Aairs: August, 2017 [Date-wise Compendium]

direction.
Evidence for the Malpelo plate came with the researchers identification of a diffuse plate boundary that runs
from the Panama Transform Fault eastward to where it intersects a deep oceanic trench just offshore of
Ecuador and Colombia.

Tectonic Plate
A tectonic plate is essentially a part-crust, part-mantle that is pushed around by the convecting currents of
superheated rock (magma) beneath them. There are total eight major tectonic plates (with an area greater
than 20 million km2), ten minor tectonic plates (area between 1 and 20 million km2). Interactions at the
edges of moving plates account for most earthquakes and volcanic eruptions on the Earth.
India has 27,312 elephants: 2017 Census Estimation
According to the census report, released by the Union Ministry of Environment, Forests and Climate
Change (MoEFCC) the population estimation of Asian elephant in India is around 27,312.
The census was conducted between March and May 2017 by the Asian Nature Conservation Foundation
(ANCF) at the Indian Institute of Science (Bengaluru), several NGOs and independent conservationists aided
the Project Elephant Directorate and forest departments of 23 states. It was first-ever synchronised all-India
Elephant Population census that covered 1.10 lakh square kilometres and spanned four regions of the country:
the northeast, south, east-central and north.
Key Highlights of the survey
The population estimation of Asian elephants in 2017 census is lower than from the last census estimate in
2017 GKToday | All Rights Reserved | www.gktoday.in

102
Current Aairs: August, 2017 [Date-wise Compendium]

2012 (between 29,391 and 30,711). Karnataka has the highest number of elephants (6,049), followed by
Assam (5,719) and Kerala (3,054).
The estimate was based on preliminary results from the first-ever synchronised all-India Elephant
Population Estimation derived from the sighting-based direct count method alone. The final count will be
confirmed with estimates from the indirect dung-count method in three months.
According to experts parallels cannot be drawn between 2012 and 2017 census, because in the 2012 census,
various states had used different methodologies and it was not synchronised effort across the country. It
might have caused errors and duplication that led to overestimation.
Comment
The 2017 census indices indicate that the elephant population is increasing including birth rate and even
their geographical range has increased. However, it shows marginal increase in elephant poulation ever since
the 1990s. Due to habitat fragmentation, elephants are moving out to agricultural landscapes leading to an
increase in man-elephant conflict resulting in both crop damage and loss of lives of elephants.
Melbourne tops in 2017 EIU Liveability index
According to the 2017 Global Liveability Report compiled by the Economist Intelligence Unit (EIU),
Australian city Melbourne is the most liveable city in the world.
The EIU Liveability index scores 140 major cities of the world on the scale ranging from 0 (least liveable
city) to 100 (most liveable city) based on healthcare, education, stability, culture, environment and
infrastructure parameters. No Indian city was ranked in the top ten or bottom ten.
Key Facts
10 most liveable cities list includes Vienna (2nd), Vancouver (3rd), Toronto (4th), Calgary (5th), Adelaide
(6th), Perth (7th), Auckland

2017 GKToday | All Rights Reserved | www.gktoday.in

103
Current Aairs: August, 2017 [Date-wise Compendium]

(8th), Helsinki (9th) and Hamburg (10th). The rankings of top five cities remain
unchanged as compare to previous year.
For the seventh consecutive year, Melbourne has topped the list. Three Canadian cities made it into the top
five: Vancouver, Toronto and Calgary.
The survey lists Damascus (140th) as the least liveable city, followed by Lagos (139th), Tripoli (138th),
Dhaka (137th), Port Moresby (136th), Algiers (135th), Karachi (134th), Harare (133rd), Douala (132nd) and
Kiev (131st). Iraq, Libya, Syria and Turkey remain the subject of high-profile civil unrest and armed
conflicts.
US designates Hizbul Mujahideen as Foreign Terrorist Organisation
The United States has named Pakistan based Kashmiri terrorist outfit Hizbul Mujahideen as a Foreign
Terrorist Organisation (FTO). The designation slaps a series of US sanctions on the outfit.
With this, Hizb Mujahideen joins the other terrorist outfits Jaish-e-Mohammad (JeM) and Lashkar-e-Taiba
(LeT) on the US list of FTO designated in 2001. Moreover, two other anti-India groups, al-Qaeda in the
Indian Subcontinent (2016) and Indian Mujahideen (2011) are also in the list.
What does it means?
Such designations expose and isolate terrorist groups and individuals, and deny them access to the US
financial system. It also assists the law enforcement activities of US agencies and other governments. It
blocks all of Hizbul Mujahideens property and interests in property subject to US jurisdiction. It also
prohibits US citizens from engaging in any transactions with the group. It purpose is to deny Hizbul
Mujahideen the resources it needs to carry out terrorist attacks.

2017 GKToday | All Rights Reserved | www.gktoday.in

104
Current Aairs: August, 2017 [Date-wise Compendium]

Background
The designation comes against the backdrop of upsurge in the terror activities of the militant group in
Kashmir in recent months. Earlier in June 2017, US had adding the Hizbul Mujahideens Pakistan-based
chief Syed Salahuddin to its list of designated global terrorists.
Comment
The US decision to designate Hizbul Mujahideen as FTO marks a severe blow to Pakistan which has been
projecting the militant group as a voice of Kashmiri people. It also validates Indias position on cross-border
terrorism as the reason behind the Kashmir issue and the support terrorist organisations getting from
Pakistan. It will also help India to put the squeeze on Pakistan, whose top political and military leaders are
portraying Hizb cadres as freedom fighters at a time of heightened terrorist activity in Kashmir.
About Hizbul Mujahideen
Hizbul Mujahideen (meaning Party of Holy Warriors) is one of the largest and oldest terrorist groups
operating in Kashmir. It was started in 1989 by Muhammad Ahsan Dar as Kashmiri separatist group and
holds pro-Pakistan ideology. Its current commander is a Sayeed Salahudeen who is based in Rawalpindi and
Pakistan-occupied Kashmir (PoK). It has been already designated a terrorist organisation by India and
European Union.
Cabinet approves MoU between India and Sweden on IPRs
The Union Cabinet has approved Memorandum of Understanding (MoU) between India and Sweden on
cooperation in the field of Intellectual Property (IPRs).
The MoU establishes wide ranging and flexible mechanism, allowing both countries to work together and
exchange best practices on training programs and technical exchanges to raise awareness and protect IPRs.
Features of MoU
It facilitates establishment of a Joint Coordination Committee (JCC) with members from both countries. It
aims to enhance cooperation between both countries for exchange of best practices, experiences and
knowledge on IP awareness among the public, businesses and educational institutions.
It will also facilitate exchange and dissemination of best practices, experiences and knowledge on IP with
industry, universities, R&D organisations and SMEs by organising programs and events. It also seeks to
enhance collaboration in training programmes, exchange of experts, technical exchanges and outreach
activities.
It will also facilitate exchange of information and best practices for disposal of applications for patents,
trademarks, copyrights, industrial designs and Geographical Indications as also the protection, enforcement
and use of IP rights.
It will enhance cooperation for understanding protection of traditional knowledge and the exchange of best
practices, including traditional knowledge related databases. It will also facilitate exchange of information
and best practices regarding IP law infringements in digital environment, especially regarding Copyright
issues.

2017 GKToday | All Rights Reserved | www.gktoday.in

105
Current Aairs: August, 2017 [Date-wise Compendium]

Significance of the MoU


It will enable India to exchange experiences in the innovation and IP ecosystems that will substantially
benefit entrepreneurs, businesses and investors on both sides. By facilitating exchange of best practices
between both countries, it will lead to improved protection and awareness about Indias range of Intellectual
creations. It will be will further the objectives of National IPR Policy, 2016 and aid in Indias journey towards
becoming a major player in global Innovation.
Union Government inks Global Environment Facility Grant Agreement with World
Bank
The Union Government has inked US $24.64 million Grant Agreement from the Global Environment
Facility (GEF) of the World Bank for Ecosystem Service Improvement Project.
The Project will be entirely financed by the World Bank out of its GEF Trust Fund. The projects duration
is of five years.
Ecosystem Service Improvement Project
The project will be implemented by the Union Ministry of Environment, Forest and Climate Change
(MoEF&CC) in Chhattisgarh and Madhya Pradesh through Indian Council of Forestry Research &
Education (ICFRE) under the National Green India Mission (GIM).
The objective of the Project is to strengthen the institutional capacity of the Community Organisations and
Departments of Forestry to enhance forest ecosystem services and improve the livelihoods of forest
dependent communities in Central Indian Highlands.
Global Environment Facility (GEF)
GEF is a multilateral financial mechanism that provides grants to developing countries for projects that
benefit the global environment and promote sustainable livelihoods in local communities. Projects under it
address six designated focal areas: biodiversity, international waters, climate change, ozone depletion, land
degradation and Persistent Organic Pollutants. It was established on the eve of the 1992 Rio Earth Summit.
National Mission for Green India (GIM)
GIM is one of the eight key Missions outlined under National Action Plan on Climate Change (NAPCC). It
aims at protecting, enhancing and restoring Indias decreasing forest cover and responding to climate change
by a combination of mitigation and adaptation measures.
The mission acknowledges the influence forests on environmental amelioration through climate change
mitigation, water security, food security, biodiversity conservation and livelihood security of forest-
dependent communities. It hinges on decentralized participatory approach by involving grass root level
communities and organizations in decision making, planning, implementation and monitoring.
Cabinet clears stalled North Koel Reservoir Project in Bihar
The Union Cabinet approved proposal to complete the balance works of North Koel Reservoir Project in Bihar,
stalled since 1993. It also revived the project to address environmental concerns.
Under the revived project, storage of water in Mandal dam to be constructed under it will be will be

2017 GKToday | All Rights Reserved | www.gktoday.in

106
Current Aairs: August, 2017 [Date-wise Compendium]

restricted at lower level than envisaged earlier to reduce the submergence and to protect Betla National Park
and Palamau Tiger Reserve. Moreover, a 24 MW power project as part of the original design also has been
scrapped.
Background
The project is situated on the North Koel river, tributary on Sone River. It aims to provide irrigation to
111,521 hectares of land annually in the most backward and drought prone areas in Jharkhand and Bihar.
The construction of the projected originally had started in 1972 and continued till 1993 when it was stopped
by the Bihar forest department over environmental concerns. The Mandal dam as part of the project had
threatened to submerge portions of the Palamau Tiger Reserve and Betla National Park. The National
Wildlife Board (NWB) had cleared the project in June 2017 with certain conditions.
North Koel River
North Koel River rises on Ranchi plateau in Jharkhand. It joins the Sone River (only right bank tributary of
Ganga River) a few miles north-west of Haidarnagar. Its principal tributaries are the Auranga and the
Amanat. It meanders through the northern part of Betla National Park.
Cabinet approves raising Rs. 9020 crore for Long Term Irrigation Fund
The Union Cabinet gave its approval for raising Extra Budgetary Resources of up to Rs, 9,020 crore for
Long Term Irrigation Fund (LTIF) during the financial year 2017-18.
The funds will be raised by the National Bank for Agriculture and Rural Development (NABARD) through
the issuance of Bonds at 6% per annum as per requirement.
Utilization of Funds
The LTIF will be for the implementation of Accelerated Irrigation Benefits Programme (AIBP) works of 99
ongoing prioritised irrigation projects along with their command area development (CAD) works under the
Pradhan Mantri Krishi Sinchayee Yojana (PMKSY).
Background
Large number of major and medium irrigation projects taken up under the AIBP were languishing mainly
due to inadequate provision of funds. To cater to the large fund requirement and ensure completion of these
projects, the Union Finance Minister in his Budget speech 2016-17, had announced creation of dedicated
LTIF in NABARD with an initial corpus of Rs. 20,000 crore for funding identified ongoing projects under
PMKSY (AIBP and CAD). The corpus of LTIF was to be raised through budgetary resources and market
borrowings to fast track implementation of incomplete major & medium irrigation projects.
Pradhan Mantri Krishi Sinchayee Yojana (PMKSY)
PMKSY has been formulated amalgamating ongoing schemes viz. AIBP, Integrated Watershed Management
Programme (IWMP) and On Farm Water Management (OFWM) component of National Mission on
Sustainable Agriculture (NMSA). It is implemented by Ministries of Agriculture, Water Resources and Rural
Development.
Objectives of PMKSY: (i) Achieve convergence of investments in irrigation at the field level, (ii) Expand
cultivable area under assured irrigation (har khet ko pani), (iii) Enhance the adoption of precision-irrigation
2017 GKToday | All Rights Reserved | www.gktoday.in

107
Current Aairs: August, 2017 [Date-wise Compendium]

and other water saving technologies (More crop per drop), (iv) Improve on-farm water use efficiency to
reduce wastage of water, (v) Enhance recharge of aquifers and (vi) Introduce sustainable water conservation
practices.
August 18, 2017
Veteran photographer S Paul passes away
Veteran photographer S Paul passed away in New Delhi. He was 88.
Paul was a self-taught photographer who discovered his passion for the art as a teenager. He joined The
Indian Express in the 1960s and retired in 1989.
About S Paul
Paul was born on 19 August, 1929 in Jhang, Pakistan. He had moved to India after Partition after his family
shifted to Shimla. His younger brother, Raghu Rai is also a renowned photographer.
He received recognition and fame as a professional photographer after his pictures were selected for
publication in three renowned British publications Miniature Camera, Amateur Photographer and
Miniature Camera World.
Paul was the first Indian to be profiled by The British Journal of Photography in 1967. He was the first
Indian to win the Nikon International Photo Contest in 1971. Famous B&W Magazine of US had described
him as The Henry Cartier-Bresson of India.
Veteran photographer S Paul passes away
Veteran photographer S Paul passed away in New Delhi. He was 88.
Paul was a self-taught photographer who discovered his passion for the art as a teenager. He joined The
Indian Express in the 1960s and retired in 1989.
About S Paul
Paul was born on 19 August, 1929 in Jhang, Pakistan. He had moved to India after Partition after his family
shifted to Shimla. His younger brother, Raghu Rai is also a renowned photographer.
He received recognition and fame as a professional photographer after his pictures were selected for
publication in three renowned British publications Miniature Camera, Amateur Photographer and
Miniature Camera World.
Paul was the first Indian to be profiled by The British Journal of Photography in 1967. He was the first
Indian to win the Nikon International Photo Contest in 1971. Famous B&W Magazine of US had described
him as The Henry Cartier-Bresson of India.
NHAI launches MyFASTag and FASTag Partner mobile apps
The National Highways Authority of India (NHAI) launched two mobile Apps MyFASTag and FASTag
Partner in to facilitate the availability of FASTags for Electronic Toll Collection (ETC) project.
The mobile apps will ease the process, making it possible to buy or recharge FASTags easier using mobile
button.

2017 GKToday | All Rights Reserved | www.gktoday.in

108
Current Aairs: August, 2017 [Date-wise Compendium]

Key Facts
MyFASTag: It is a consumer application that will facilitate consumer to purchase or recharge FASTags on
it. It also helps to keep track of transactions and provides for online grievance redressal.
FASTag Partner: It is a merchant application. It will allow agencies like Common Services Centre (CSC),
banking partners and vehicle dealers to sell and enroll FASTag . In addition, it can be also used to activate
dormant RFID tags built in 74 lakh cars in the country following the 2013 Gazette Notification in this
regard. This application will aid to convert these RFID tags into FASTag (ETC Tag).
Background
NHAI has announced complete rollout of FASTags for ETC on all 371 NHAI toll plazas from 1 September
2017. One lane in every toll plaza will be a dedicated FASTag lane where no other form of payment will be
accepted. In order to enable the availability of FASTags for ETC, NHAI has taken two steps in consultation
with Reserve Bank of India (RBI) and National Payments Corporation of India (NPCI). They are online and
offline sale of sale of FASTags through Common Services Centre (CSC) near toll plazas. Even FASTags can
now be purchased online from Issuer Banks websites, NHAI website and will be delivered by courier at the
door step of the purchaser.
Electronic Toll Collection (ETC)
It is electronic payment system on highway tolls that does not have human involvement. ETC systems use
vehicle-to-roadside communication technologies to perform an electronic monetary transaction between
vehicle and toll collection agency. It aims to make travel more convenient, faster and environment friendly
for people.
FASTags
FASTag is a radio frequency identification device that enables cashless transaction at toll plazas without
stopping the vehicle at the toll plazas. It employs RFID technology for making toll payments directly from
the pre-paid account linked to it. This system aims to remove logistic inefficiencies at toll plazas and has the
potential of saving Rs 60,000 crore in terms of time and fuel bills.
Government launches Harit Diwali, Swasth Diwali Campaign
The Union Ministry of Environment, Forest and Climate Change (MoEFCC) has launched the Harit
Diwali, Swasth Diwali campaign.
The campaign was launched with an aim to reduce adverse environmental conditions especially pollution in
the country after post Diwali celebrations due to excessive bursting of crackers which contributes
significantly to air and noise pollution.
Harit Diwali, Swasth Diwali Campaign
The campaign aims at enlightening the children about harmful fire crackers. It will motivate children to not
to buy fire crackers, instead buy a gift, food items, or sweets for poor and underprivileged children living in
their locality. Under this campaign, the MoEFCC will undertake various activities for creating awareness
among various stakeholders and encourage people to participate in combating air pollution.
Some of the activities will include promoting Green Diwali among school children include stickers/logo
2017 GKToday | All Rights Reserved | www.gktoday.in

109
Current Aairs: August, 2017 [Date-wise Compendium]

distribution, advertisement on public ransport systems, poster competition, public appeal using Radio. Social
media campaign will be also undertaken uder it. Moreover an online video competition will be conducted,
where any individual or organisation can make a video on the theme Pollution Free Diwali.
Background
The weather after the post 2016 Diwali celebrations in Delhi and NCR saw extreme pollution due to
excessive bursting of crackers. The adverse air pollution in Delhi and NCR had reached beyond permissable
levels that led Government to declare emergency situation in the national capital, which led to closing down
of schools, construction sites and power stations.
Defence Ministry clears proposal to procure 6 Apache attack helicopters for
Army
The Union Defence Ministry has cleared a proposal to procure six AH-64-E Apache attack helicopters
along with weapons systems from American aerospace giant Boeing for the Indian Army.
Decision in this regard was taken at the Defence Acquisition Council (DAC) meeting chaired by Defence
Minister Arun Jaitley in New Delhi. DAC is Defence Ministrys highest decision-making body on
procurement.
Decision taken by DAC
AH-64-E Apache attack helicopters: The six will come along with associated equipment, spares and
weapons system. It will be a follow on order to the deal India had signed in September 2015 with Boeing and
the US government for 22 Apache helicopters and 15 Chinook heavy-lift choppers.
The AH-64-E Apache choppers are one of the most advanced multi-role combat helicopters which have
advanced weapons systems and night-fighting features. Till now, Indian Army was not having its own fleet
attack helicopter. Army was using the fleet of Russian Mi-25 and Mi-35 attack helicopters with the help
Indian Air Force.
Purchasing gas turbines: DAC also cleared another proposal for buying gas turbines from Ukraine for two
Admiral Grigorovich class ships being made in Russia for India.
Navika Sagar Parikrama: Indian Navys all-woman team to circumnavigate globe
A six-member all-woman team of the Indian Navy will set sail to circumnavigate the globe on the sailing
vessel INSV Tarini. This is the first-ever Indian circumnavigation of the globe by an all-woman crew.
The expedition has been aptly titled Navika Sagar Parikrama with an aim to promote women
empowerment in the country and ocean sailing by the Indian Navy.
Key Facts
The voyage of Navika Sagar Parikrama will begin from Goa in the first week of September 2017 and
culminate on their return in March 2018. The entire distance will be covered in five legs, with stop overs at
four ports Fremantle (Australia), Lyttelton (New Zealand), Port Stanley (Falklands) and Cape Town (South
Africa) for replenishment of ration and repairs as necessary. The project is considered essential towards
promoting Ocean Sailing activities in the Indian Navy while depicting Union Governments thrust for Nari

2017 GKToday | All Rights Reserved | www.gktoday.in

110
Current Aairs: August, 2017 [Date-wise Compendium]

Shakti i.e. empowering women.


INSV Tarini
INSV Tarini is a 55 foot sailing vessel has been built indigenously by M/s Aquarius Shipyard Pvt Ltd, Goa. It
was inducted into the Indian Navy in February 2017. INSV Tarini is the sister vessel of INSV Mhadei. Its
mast is 25 metres and has been custom-built by M/s Southern Spars, Cape Town (South Africa), for sailing
in extreme conditions. It carries a suite of six sails, including a main sail, head sails (Genoa and stay sails),
downwind sails and storm sail. It has Raymarine navigation suite and an array of satellite communication
systems on board through which contact can be made from anywhere.
Earlier Expeditions
The first Indian solo circumnavigation was undertaken by Captain (Retd) Dilip Donde from August 2009 to
May 2010 onboard the Indian built vessel, INSV Mhadei. The first Indian non-stop solo circumnavigation
was undertaken by Cdr Abhilash Tomy from November 2012 to March 2013.
Scientists develops artificial womb to save premature babies
Scientists have developed an artificial womb that can be used in future to save the lives of extremely
premature human babies. It was successfully used to incubate healthy baby lambs for a week.
The research showed that preterm lambs were successfully maintained in a healthy, infection-free condition
with significant growth, for a period of one week using ex-vivo uterine environment (EVE) therapy.
Key Facts
The artificial womb is a high-tech amniotic fluid bath combined with an artificial placenta. To mimic a
womb, researchers had created a temperature-controlled bag filled with electrolyte solution as a substitute
for amniotic fluid that lambs swallow and take into their lungs. They had attached the umbilical cord of
preterm lamb to the device that exchanges carbon dioxide in blood with oxygen. In early-stage animal
testing, the lamb appeared to grow normally inside the system.
Significance
The artificial wombs can facilitate development of premature babies in a uterus-like environment. Further
development, EVE therapy can prevent the severe morbidity suffered by extremely premature infants. It can
potentially offer a medical technology that does not currently exist.
August 19, 2017
5 Indian-origin persons in Fortunes 2017 40 Under 40 list
Five Indian-origin persons have featured in Fortunes 2017 40 Under 40 list of young and influential people
in the field of business inspiring others with their work. They are Leo Varadkar (age: 38), Divya Nag (26), Rishi
Shah (31), Shradha Agarwal (32) and Leila Janah (31).
Fortunes annual list of 40 young and influential people is an annual ranking of the most influential young
people who are under 40 of age. It has termed them as disruptors, innovators, rebels and artists inspiring
others.
Key Facts
2017 GKToday | All Rights Reserved | www.gktoday.in

111
Current Aairs: August, 2017 [Date-wise Compendium]

The Fortunes 2017 40 Under 40 list has been topped by French President Emmanuel Macron (39) who is
Frances youngest leader since Napoleon and had won presidential elections in May 2017.
The list also includes Facebook founder Mark Zuckerberg on the second spot, followed by Brian Chesky
(3rd), Nathan Blecharczyk (3rd), Joe Gebbia (4th), Serena Williams (7th) etc.
Leo Varadkar: He was ranked fifth on the list. His father was born in India. He is doctor by profession and
is Irelands youngest leader in centuries as well first-ever gay to hold the office.
Divya Nag: She was ranked 27th. She oversees Apples ambitious ResearchKit and CareKit programmes
that encourage developers to build health- related apps She is Stanford dropout founded a stem-cell research
startup and began a medical investment accelerator at the age of 23.
Rishi Shah and Shradha Agarwal: They were ranked 38th on the list. They are founders of health-tech
firm Outcome Health. Their company raised over US $500 million at a valuation of more than $5 billion.
Leila Janah: She was ranked 40th on the list. She is CEO and founder of non-profit Samasource. She
daughter of Indian immigrants who had moved to the US.
RBI to issue new Rs 50 currency note
The Reserve Bank of India (RBI) will shortly issue new banknotes of Rs.50 denomination in the Mahatma
Gandhi (New) Series. The base colour of the new notes will be fluorescent blue.
RBI also has clarified that all banknotes in Rs.50 denomination issued in the earlier series will continue to be
legal tender.
Key Features new Rs.50 note
Its dimension will be 66 mm x 135 mm. It will have a motif of Hampi with Chariot on the reverse, depicting
the countrys cultural heritage

along with the Swachh Bharat logo, numeral 50 in devnagiri, year of printing and
the language panel.
It will have Mahatma Gandhis portrait at the Centre and a see through register with enominational numeral
50. Micro letters , INDIA, RBIand 50 and Ashoka Pillar emblem on the right will also feature on the
front side.

2017 GKToday | All Rights Reserved | www.gktoday.in

112
Current Aairs: August, 2017 [Date-wise Compendium]

The security thread will have inscriptions and RBI. The number panel will have numerals growing from
small to big on the top left side and bottom right side. The banknote will bear the signature of RBI Governor
Dr. Urjit R. Patel on the front side along with the guarantee and promise clause and the RBI emblem.
19 August: World Humanitarian Day
The World Humanitarian Day (WHD) is observed every year on 19 August to recognize work of
humanitarian personnel and those who have lost their lives. The campaign theme for this year is
#NotATarget.
The observance of the day also provides an opportunity to celebrate the spirit that inspires humanitarian
work around the globe. It pays tribute to aid workers who risk their lives in humanitarian service and also to
rally support for people affected by crisis around the world.
Background
The World Humanitarian Day was instituted by the United Nations General Assembly (UNGA) by passing a
resolution A/63/L.49. The resolution passed by UNGA on the Strengthening of the Coordination of
Emergency Assistance of the United Nations was sponsored by Sweden.
The day coincides with the anniversary of the 2003 bombing of the UN headquarters in Baghdad, Iraq. In
this bombing, then Special Representative of the UN Secretary-General to Iraq, Srgio Vieira de Mello and
his 21 colleagues were killed.
Climate change costs India $10 billion every year: Government
In its recent report, Parliamentary Standing Committee on Agriculture has observed that extreme weather
events are costing India $9-10 billion annually. It also observed that climate change is projected to impact
agricultural productivity with increasing severity from 2020 to the end of the century.
It also mentioned that the extreme weather events are not always linked to climate change but their
frequency and severity is increasing and this is being increasingly read as fallout of climate change.
Impact of Climate change on agriculture
Decrease in Productivity
The productivity of major crops will be marginal in the next few years but it could rise to as much as 10-40%
by 2100 unless farming adapts to climate change-induced changes in weather. Major crops such as wheat,
rice, oilseeds, pulses, fruits and vegetables will see reduced yields over the years.
It will force farmers to either adapt to challenges of climate change or face the risk of getting poorer. It could
turn India into a major importer of oilseeds, pulses and even milk. Adaptation to climate change will need
different cropping patterns and suitable inputs to compensate yield fluctuations.
Food Security
Vulnerability of Indian agriculture due to vagaries associated with climate change and low adaptation
capacity of majority of Indian farmers poses risk to food security of the country. By 2030, India may need 70
million tonnes more of foodgrains than the expected production in 2016-17.
The demand for food is also going to increase due to an increasing population, expanding urbanisation and
rising income. To meet increasing demand, India to depend on import if it does not act on time to increase
2017 GKToday | All Rights Reserved | www.gktoday.in

113
Current Aairs: August, 2017 [Date-wise Compendium]

production and productivity of major food crops, pulses, oilseeds and milk by adapting to climate change.
Projected food demand
The ICAR-National Institute of Agricultural Economics and Policy Research has projected food demand of
345 million tonnes (MT) by 2030- almost 30% higher than in 2011. The projected demands for fruits,
vegetables, milk, animal products (meat, eggs and fish), sugar and edible oil, by 2030 is estimated to be 2-3
times more than that in 2011.
Economic Losses
According to the economic survey estimates, India currently incurs losses of about $9-10 billion annually due
to extreme weather events. Of these, nearly 80% losses remain uninsured. The quantum of losses are going
to increase substantially in future if one takes into account the impact of climate change on farm
productivity.
Improve in Yields
Though there is possibility of decrease in yields of certain crops in traditional sown areas due to climate
change but it may increase elsewhere due to change in weather pattern. Though most crops will see reduced
production, but climate change may also help improve yields of soyabean, chickpea, groundnut, coconut
(western coast) and potato (in Punjab, Haryana and western Uttar Pradesh).
NASA successfully launches Tracking and Data Relay Satellite-M
The National Aeronautics and Space Administration (NSAS) has successfully launched Tracking and Data
Relay Satellite-M (TDRS-M) into orbit. It was launched aboard a United Launch Alliance (ULA) Atlas V
rocket.
TDRS-M is third and final satellite in a series of the Tracking and Data Relay Satellite System (TDRSS), next
generation communications satellites.
TDRS-M
TDRS-M as part of NASAs Space Network will provide navigation and high-data-rate communications to
the International Space Station (ISS), Hubble Space Telescope, rockets and a host of other spacecraft. It will
also expand the capabilities and extend the lifespan of the NASAs Space Network which will allow
researchers to continue receiving and transmitting mission data well into the next decade.
Tracking and Data Relay Satellite System (TDRSS)
The TDRSS is a fleet of geosynchronous communications satellites that form part of NASAs Space
Network. It was introduced in the 1980s to support the Space Shuttle. It is still in service, providing scientific
data, relay for communications, telemetry and commands between operators on the ground and spacecraft in
Earth orbit. TDRS-Ms predecessors, TDRS-K and TDRS-L were also launched onboard of Atlas V rockets
in January 2013 and January 2014, respectively. The TDRS fleet is a critical connection for delivering science
and human spaceflight data to researchers on Earth.
India, US establish Two-By-Two Ministerial Dialogue
India and US have established new two-by-two (2 by 2) ministerial dialogue to enhance strategic
coordination between them and maintaining peace and stability in the Indo-Pacific region.
2017 GKToday | All Rights Reserved | www.gktoday.in

114
Current Aairs: August, 2017 [Date-wise Compendium]

The new dialogue format will replace the earlier India-US Strategic and Commercial Dialogue. It will be
similar to the India-Japan 2+2 dialogue format between foreign and defence secretaries of the two countries.
Key Facts
India and US had elevated their Strategic Dialogue in 2009 which mainly focuses on regional security,
economic cooperation, defence, trade and climate challenges. The purpose of two-by-two ministerial
dialogue is to put strategic, defence and security relationship between the two countries at the forefront and
centrestage. The new format would include External Affairs Minister and Defence Minister from India and
their American counterparts Secretary of State and Defence Secretary.
Comment
The new ministerial dialogue would enhance strategic coordination between the two nations. It will aslo
elevate strategic consultations between both countries. It will be helpful to coordinate more closely on
Afghanistan, developments in the Asia Pacific, Indian Ocean and also in the Middle East (West Asia). It will
insulate the India-US strategic relationship from feuds over trade and deep divide on economic integration
policies as trade and commercial issues were discussed in the Strategic and Commercial Dialogue earlier.
Two new species of Earthworm discovered in Kerala
Scientists have discovered two new species of earthworm belonging to the primitive family Moniligastridae
in Western Ghats ranges of Kerala.
The new earthworms distinguished by a set of characters have been named as Drawida polydiverticulata and
Drawida thomasi.
Key Facts
In total, there are about 200 species of earthworm known in genus Drawida. To date, there are 73
earthworm species belonging to genus Drawida living in Indian subcontinent. The greatest concentration
(43 species) is found in Western Ghats.
Drawida polydiverticulata: It has multiple lobes called diverticulums. It is an organ located in the front of
its body which is unique amongst the members of the genus. It was found to be widespread in the protected
shola grasslands of the Munnar region, including Eravikulam National Park, Pampadun Shola National Park
and Chinnar Wildlife Sanctuary.
Drawida thomasi: It was discovered at the Kozhippara Waterfalls near Kakkadampoyil, at the border
between Malappuram and Kozhikode. The species has been named as a tribute to Professor A.P. Thomas
who initiated the taxonomical studies on earthworms in Kerala.
August 21, 2017
China launches first cyber court in e-commerce hub of Hangzhou
China launched its first cyber (internet) court specialising in handling internet-related cases in the e-
commerce hub of Hangzhou, amid a spike in the number of online disputes.
The cyber court will handle cases such as online trade disputes and copyright lawsuits. These cases handled
by the court will be tried online.
2017 GKToday | All Rights Reserved | www.gktoday.in

115
Current Aairs: August, 2017 [Date-wise Compendium]

Key Facts
According latest report of China Internet Network Information Center, China had about 751 million
netizens and 724 million mobile Internet users as of the end of June 2017. With high internet penetration,
Chinese people have increasingly turn to online shopping and payment.
The Hangzhou Internet Court will be located in Hangzhou, capital of east Chinas Zhejiang Province. It will
provide a judicial guarantee for maintaining cyber security, resolving online disputes and promoting the
integration of the Internet with society and the economy.
Hangzhou municipal legislature appointed the president, vice presidents and judges of the court. The
province is home to many Internet companies, including e-commerce giant Alibaba.
Defence Ministry approves delegation of powers to BRO
The Union Defence Ministry has approved delegation of administrative and financial powers to the Border
Roads Organisation (BRO).
The purpose of delegation of powers is to improve the functioning of the BRO and speed up works to avoid
delays in projects and to achieve the desired outcomes according to the requirement of the Armed Force
Need
The BRO is engaged in road construction to provide connectivity to difficult and inaccessible regions in the
border areas of the country. On various occasions, Comptroller and Auditor-General (CAG) and the
Parliamentary Standing Committee has highlighted delay in the completion of strategic border roads. In
pursuance of this, Defence Ministry had intended to bring transformational changes in the BRO in order to
improve the pace of execution of works.
Key Facts
Departmental and Contractual works: Defence Ministry has now approved that for both departmental
and contractual mode of execution of projects. Moreover, Chief Engineer of BRO can accord administrative
approval upto Rs. 50 crore and ADGBR upto Rs. 75 crore and DGBR upto Rs. 100 crore.
In the earlier scheme of delegation of powers, Chief Engineer was able to give administrative approval of
works only upto Rs. 10 crore, that too only for departmental works. The ADGBR had powers to accord
administrative approval only upto Rs. 20 crore. In case of contractual works, all administrative approvals
were given by DGBR, who had powers only upto Rs.50 crore.
Speeding up the tendering process: Defence Ministry also has enhanced the powers of Chief Engineer for
acceptance of bids in tendering process with cost of contract upto Rs. 100 crore and upto Rs. 300 crore of
ADGBR. Even the entire tendering process including acceptance of bids will be now completed at the level
of Chief Engineer/ADGBR for a majority of the contracts.
Fast procurement of latest construction machinery and equipment: Defence Ministry has enhanced
the delegation of powers upto Rs. 100 crore to DGBR for procurement of both indigenous and imported
equipment. Even, DGBR power hiring of construction equipment has been enhanced upto three years and
for Chief Engineers (Project) hiring power has been enhanced from 6 months to one year.

2017 GKToday | All Rights Reserved | www.gktoday.in

116
Current Aairs: August, 2017 [Date-wise Compendium]

About Border Roads Organisation (BRO)


BRO develops and maintains road networks in Indias border areas and friendly neighbouring countries. It
functions under the Defence Ministry. It plays pivotal role in construction, maintenance and upgradation of
strategic infrastructure, in remote and far flung border areas
Officers from Border Roads Engineering Service (BRES) and personnel from General Reserve Engineer
Force (GREF) form parent cadre of BRO. It is also staffed by officers and troops drawn from Indian Armys
Corps of Engineers, Army Service Corps, Electrical and Mechanical Engineers and army personnel on extra
regimental employment.
Indias first calf born to surrogate cow in Pune
Indias first calf delivered by a surrogate or recipient cow through In Vitro Fertilisation (IVF) technology
carried out in a mobile laboratory was born in Indapur, Pune (Maharashtra). It has been named as Vijay.
This is for first time IVF procedure was successfully implemented at farm level with the help of mobile
laboratory. Earlier similar experiment was carried out by the National Dairy Research Institute (NDRI) at
the lab-level.
Key Facts
The purpose of producing surrogate calf was to protect indigenous cow breeds in its original form which
have been destroyed due to cross-breeding. The experiment was successfully under taken by the JK Trust, an
organisation which works at improving cattle breed.
In Vijays case, the donor cow belonged to the indigenous Gir breed, whose population across the country is
around one lakh (unevenly spread across Gujarat, Rajasthan and Maharashtra).
In this experiment, the oocytes collected from the mother cow were kept in a special incubator. Using the
IVF technology, these oocytes were fertilised inside the incubator with semen was collected from a Gir bull.
After the oocytes developed into an embryo, they were transferred to the recipient cow.
In Vitro Fertilisation (IVF) technology
IVF or assisted reproductive technology (ART) is a process of fertilisation. In it, an egg is combined with
sperm outside the body, in vitro (in glass or laboratory dish) to produce embryo. The embryo is then
transferred to the uterus with the intention of establishing a successful pregnancy.
Scientists discover new state of matter
Scientists Los Alamos National Laboratory, US have discovered a potential new state of matter that may help
explain phenomena like superconductivity.
It was discovered in the high-magnetic-field state of the heavy fermion superconductor CeRhIn5. Heavy
fermions are intermetallic compounds, containing rare earth or actinide elements.
Key Facts
In the new state, materials electrons are aligned in such a way, that they apparently reduce the symmetry of
the original crystal. This appearance of electronic alignment in a prototypical heavy-fermion superconductor
is called nematic behaviour. It is phenomenon of electronic symmetry breaking, common among the
superconducting materials in high magnetic fields.
2017 GKToday | All Rights Reserved | www.gktoday.in

117
Current Aairs: August, 2017 [Date-wise Compendium]

It highlights the interrelation of nematicity and unconventional superconductivity, suggesting that


nematicity is common among correlated superconducting materials. It appears to be universal among
unconventional superconductors. Unconventional superconductivity develops near a phase boundary
separating magnetically ordered and magnetically disordered phases of a material.
Superconductivity
Superconductivity is the ability of certain materials to conduct electric current with practically zero
resistance. For a material to behave as a superconductor, low temperatures are required i.e. they act as
superconductors when they cooled below a characteristic critical temperature. Superconductivity was first
observed in 1911 by H. K. Onnes, a Dutch physicist. Superconductivity is extensively used in magnetic
resonance imaging (MRI), particle accelerators, magnetic fusion devices, microwave filters, high-speed
magnetic-levitation trains, ultra-high-speed computer chips and high-capacity digital memory chips etc.
Versius: worlds smallest surgical robot developed by UK scientists
Scientists in the United Kingdom (UK) have developed the worlds smallest surgical robot called Versius. It
was developed using low-cost technology used in mobile phones and space industries.
The robot can mimic human arm and can be used to carry out a wide range of surgical procedures. It can be
controlled by a surgeon using a console guided by a 3D screen in the operating theatre.
Key Facts
Versius can make series of small incisions that will circumvent the need for traditional open surgery. These
include colorectal operations, hernia repairs, as well as prostate, ear, nose and throat surgery. It is much
easier to use than existing systems, and requires about a third of the space of current machines.
It works like a human arm and contains technology that detects resistance to make sure the right amount of
force is used when the instruments are inside the patient. It can help to reduce complications and pain after
surgery and speed up recovery time for patients.
Government imposes anti-dumping duty on tempered glass from China
The Union Finance Ministry (Revenue Department) has imposed anti-dumping duty on certain textured
toughened (tempered) glass imported from China.
The duty aims at protecting the domestic industry from cheaper imports. It will be effective for five years.
Textured toughened (tempered) glass
It is also known as solar glass, low iron solar glass or high transmission photovoltaic glass. It is used as a
component in solar photovoltaic (PV) panels and solar thermal applications.
Background
The dumping duty was imposed by revenue department based on the recommendations of Directorate
General of Anti-Dumping and Allied Duties (DGAD). Investigations of DGAD had found that the tempered
glass has been exported to India from China below its associated normal market value. It was also found that
domestic industry had suffered material injury due to cheap imports from China.
Anti-dumping duty
Anti-dumping duty is an import duty imposed by government on imported products which have prices less
2017 GKToday | All Rights Reserved | www.gktoday.in

118
Current Aairs: August, 2017 [Date-wise Compendium]

than their normal values or domestic price. It is protectionist and counter import measure used by a country
under the multilateral World Trade Organisation (WTO) regime to protect its domestic producers and
market from below-cost/cheap imports. It varies from product to product and from country to country. In
India, it is recommended by the Union Ministry of Commerce (i.e. by DGAD), while the Union Finance
Ministry imposes it.
Anti-dumping duty on Chinese Products
So far, Government has imposed anti-dumping duty on 93 products imported from China. These products
belong to a broad group of chemicals and petrochemicals, fibres and yarn, products of steel and other metals,
machinery items, electric and electronic items, rubber or plastic products and consumer goods, among
others.
Indian Navys second LCU Mark IV L52 ship commissioned at Port Blair
Indian Navys second CU (landing craft utility) Mark IV L52 ship was commissioned at Port Blair, capital of
Andaman and Nicobar Islands.
LCU L52 is the second LCU Mk-IV class ship to be inducted into the Indian Navy. It has been indigenously
designed and built by Garden Reach Shipbuilders and Engineers (GRSE), Kolkata.
Key Facts
LCU L52 ship is an amphibious ship with the primary role to transport and deploy Main Battle Tanks
(MTBs), Armoured Vehicles, troops and equipment from ship to shore. It is 62.8 meters long and 11 metres
wide. It has 90% indigenous content.
The ship has endurance of around 1,500 nautical miles at 12 knots. It is fitted with state-of-the-art
equipment and advanced systems like Integrated Platform Management System (IPMS) and Integrated
Bridge System (IBS).
LCU MK-IV will be based in the Andaman and Nicobar Command. It can be deployed for multirole
activities like beaching operations, search and rescue, disaster relief operations, supply and replenishment
and evacuation from distant islands. It will contribute to the nations maritime security needs.
Government launches digital police portal under CCTNS
The Union Ministry of Home Affairs launched digital police portal under Crime and Criminal Tracking
Network and Systems Project (CCTNS).
The portal is a smart policing initiative of the Government to provide services to citizens and aid efficient
police investigation. It will help in realising Governments dream of Minimum Government Maximum
Governance.
Digital police portal
The police portal is a platform for citizens to file online crime related complaints and also request for
antecedent verification. It will also provide 11 searches and 46 reports from the national database for state
police and central investigation agencies. Central investigating and research agencies also have been
provided logins to the portal to access crime statistics.

2017 GKToday | All Rights Reserved | www.gktoday.in

119
Current Aairs: August, 2017 [Date-wise Compendium]

About Crime and Criminal Tracking Network and Systems Project (CCTNS)
The CCTNS project was initiated in 2009 as part of police modernisation programme under the national e-
governance project. It was conceived to bring all 16,000 police stations in India under a single network to
counter terror challenges.
The core objective of CCTNS is to connect all the police stations so that any of the police stations can
contact any other police station; thereby facilitate collection, storage, retrieval, analysis and transfer the
information among police stations, state headquarters and central police organizations.
August 22, 2017
Karnataka Government launches clouding seeding Project Varshadhari
Karnataka Government has launched clouding seeding Project Varshadhari in Bengaluru under which an
aircraft will spray chemicals to induce rainfall.
The cloud seeding project is an experimental project that will cover Bengaluru, Gadag and Yadgir districts of
the state.
Project Varshadhari
The experimental project aims to condense small water particles by clouds (size less than 10 microns) to
droplets of 50 microns, which can constitute rains. The flight for cloud seeding BQ-100 Beechcraft will
spray Silver Iodide, Sodium Chloride and Potassium Chloride to condense the water particles. A committee,
comprising meteorologists, experts in cloud physics and hydrologists, will monitor the process.
Cloud seeding
Cloud seeding (also known as weather modification technique) is an artificial way to induce moisture in the
clouds so as to cause a rainfall. In this process, either silver iodide, potassium iodide or dry ice (solid carbon
dioxide) are spread onto the clouds causing rainfall. These iodide particles rise into the clouds, cause moisture
to freeze. These particles act as a base for water vapours to settle on that later crystallises and falls as rain or
snow.
This method can be used to bring significant amount of rainfall over a specified area especially in location
where rain is badly needed. Several experiments conducted earlier across India were not very effective and
gave mixed results. It is not fool proof as seeded clouds may actually travel to another location and do not
cause precipitation on the intended location. Moreover there are environmental consequences of spreading
of harmful chemicals in atmosphere. It is also very expensive technique. If it is not controlled properly, it
may result in undesirable weather conditions like flooding, hail, storms, etc.
Scientists discover new therapy to prevent sepsis in new born babies
A team of scientists from US have discovered an inexpensive prevention technique (therapy) against sepsis
in new born babies. The team was led by US-based Indian doctor Dr Pinaki Panigrahi.
The therapy has been found to reduce the risk of sepsis infection by 40% in trials and it can be inexpensive
less than one dollar for a course.
Need
2017 GKToday | All Rights Reserved | www.gktoday.in

120
Current Aairs: August, 2017 [Date-wise Compendium]

Each year globally, more than 600,000 infants die of sepsis most of them in developing countries. In India,
more than a quarter of neonatal deaths are due to sepsis. In a significant number of these cases, the infection
probably begins in the gut.
New prevention technique
In the new oral prevention technique, infants are fed with good bacteria (probiotic bacteria) that populates
in his/her gut and block (kill) harmful bacteria. The probiotic bacterial strain is also aided with a non-
absorbable sugar to enhance its ability to effectively colonize the gut.
Researchers call it as probiotic-sugar combination, a symbiotic. The oral therapy starts during day 2-4 of life,
as a preventive measure, much like a vaccine. The technique is also found beneficial in reducing lower
respiratory tract and other infections and also improving immunity status.
Sepsis
Sepsis is a life-threatening illness primarily caused by bacterial infections that spreads fast through the blood
leading to organ failure. It is caused by an immune response triggered by an infection i.e. bodys response to
infection causes injury to its own tissues and organs. Sepsis infections are also more common in elderly
people and those with a weak immune system.
Common signs and symptoms include fever, increased heart rate, increased breathing rate, and confusion.
There also may be symptoms related to a specific infection, such as pneumonia, kidney infection blood
infection and abdominal infection. Currently, no efficient means of prevention is available.
Swasth Bachche, Swasth Bharat Programme launched
The Union Ministry of Human Resource Development (HRD) launched Swasth Bachche, Swasth Bharat
Programme. It was launched by Union HRD Minister Prakash Javdekar in Kochi, Kerala
The programme is an initiative of Kendriya Vidyalaya Sangathan (KVS) to prepare a physical Health and
Fitness Profile Card for more than 12 lakhs of Kendriya Vidyalaya students.
About Swasth Bachche, Swasth Bharat programme
The programme aims to provide a comprehensive and inclusive report card for children covering all age
groups and children of different abilities. Its main objective is to make students, teachers and parents aware
about the importance of good health and fitness and encouraging 60 minutes of play each day.
The programme also intends to imbibe values of Olympics and Paralympics amongst students. The
objectives are bringing back the childhood amongst children, make recreational games and physical activity
an integral part of learning process, motivating potentially outstanding performers in various games and
sports of excellence, using technology for analytics and data capture and giving access to schools, parents and
teachers.
ICMR, Health Ministry ink MoU with IVI for vaccine R&D
The Indian Council of Medical Research (ICMR) and Ministry of Health inked Memorandum of
Understanding (MoU) with the International Vaccine Institute (IVI) to collaborate on vaccine research and
development.
The signing of the MoU is a continuation of Indias partnership with IVI to provide safe, effective and
2017 GKToday | All Rights Reserved | www.gktoday.in

121
Current Aairs: August, 2017 [Date-wise Compendium]

affordable vaccines for people around the world. With this partnership, India is committing an annual
contribution of US $5,00,000 to IVI.
Background
India is a vaccine industry powerhouse that supplies 60% of the worlds vaccines. India has with a long
history of scientific collaboration with IVI. IVI has been partnering with Indian vaccine manufacturers,
research institutes, government, and public health agencies on vaccine R&D. One of the most successful
collaboration of IVI was with Indias Shantha Biotech on development of Shanchol, worlds first low-cost
oral cholera vaccine. The vaccine was licensed in India in 2009 and WHO-prequalified in 2011.
International Vaccine Institute (IVI)
IVI is Seoul (South Korea) based international nonprofit organization devoted to developing and
introducing new and improved vaccines to protect the people, especially children, against deadly infectious
diseases. It was established in 1997 on the initiatives of the United Nations Development Programme
(UNDP).
Its work is exclusively on vaccine development and introduction specifically for people in developing
countries, with a focus on neglected diseases affecting these regions. Currently, IVI has 40 countries and the
World Health Organization (WHO) as signatories to its Establishment Agreement. India officially became a
signatory to the IVI in 2012.
Indian Council of Medical Research (ICMR)
The ICMR is Indias apex scientific body for the formulation, coordination and promotion of biomedical
research. It was established in 1911 as Indian Research Fund Association (IRFA) making it one of the oldest
and largest medical research bodies in the world. The ICMR functions under the Department of Health
Research, Union Ministry of Health and Family Welfare.
ISA extends Indias exclusive rights to explore Polymetallic Nodules
The International Seabed Authority (ISA) in its 23rd session recently held at Kingston, Jamaica has extended
Indias exclusive rights to explore polymetallic nodules from seabed in Central Indian Ocean Basin (CIOB).
These rights extend over 75000 sq. km of area in international waters allocated by ISA for developmental
activities for polymetallic nodules. The estimated polymetallic nodule resource potential of this area is
around 380 million tonnes, containing nickel (4.7 million tonnes), copper (4.29 million tonnes), cobalt (0.55
million tonnes) and manganese (92.59 million tonnes).
Background
India was the first country to receive the status of a pioneer investor for exploration and utilization of
polymetallic nodules. It was allocated an exclusive area in Central Indian Ocean Basin by United Nations
(UN) in 1987. India is one among the top 8-countries to implement a longterm programme on exploration
and utilization of Polymetallic Nodules. Through Ministry of Earth Sciences it carries survey and
exploration, environmental studies, technology development in mining and extractive metallurgy.
Polymetallic nodules
Polymetallic nodules (also called as manganese nodules) are small potato-sized (from millimetres to tens of
2017 GKToday | All Rights Reserved | www.gktoday.in

122
Current Aairs: August, 2017 [Date-wise Compendium]

centimetres in diameter) lumps of minerals found in deep sea. They contain nickel, copper, cobalt, lead,
cadmium, vanadium, molybdenum, titanium in various proportions of which nickel, cobalt and copper are
considered to be of economic and strategic importance. They are found in abundance carpeting the sea floor
of world oceans in deep sea.
International Seabed Authority (ISA)
ISA is a UN body set up to regulate the exploration and exploitation of marine non-living resources of
oceans in international waters. It was established under the 1982 United Nations Convention on the Law of
the Sea (UNCLOS). Its headquarters are in Kingston, Jamaica.
Its mandate is to organize, regulate and control all mineral-related activities in the international seabed area
beyond limits of national jurisdiction (exclusive economic zone), an area underlying most of worlds oceans.
India actively contributes to the work of ISA. It was re-elected as a member of Council of ISA in 2016.
17th meeting of FSDC held in New Delhi
The seventeenth Meeting of the Financial Stability and Development Council (FSDC) was held in New
Delhi under the Chairmanship of the Union Minister of Finance Arun Jaitley.
The meeting was attended by RBI Governor Dr. Urjit R. Patel, SEBI chairman, IRDAI Chairman along with
heads and other senior officers of the Government and financial sector regulators.
Key Highlights of Meeting
Macro-economic stability: FSDC held that India has macro-economic stability on the back of
improvements in its macro-economic fundamentals, structural reforms, action taken to address the Twin
Balance Sheet (TBS) challenge, extraordinary financial market confidence and long-term positive
consequences of demonetization.
Challenges facing the Indian economy: It also discussed the issues and challenges facing the Indian
economy and members agreed on the need to keep constant vigil and be prepared of managing any external
and internal vulnerabilities.
Progress of Financial Sector Assessment Program for India: It is jointly conducted by the International
Monetary Fund (IMF) and the World Bank. It directed that the assessment report should be finalized by end
of this calendar year.
CERT-Fin and FDMC: It took note of the developments and progress made in setting up of Computer
Emergency Response Team in the Financial Sector (CERT-Fin) and Financial Data Management Centre. It
also discussed measures for time bound implementation of the institution building initiative.
Central KYC Registry (CKYCR) system: It also discussed on the CKYCR system and took note of the
initiatives taken in this regard by the members and discussed the issues in respect of its operationalization.
Regulation of CRAs: It also deliberated on strengthening the regulation of Credit Rating Agencies (CRAs).
About Financial Stability and Development Council (FSDC)
FSDC is super regulatory body for regulating financial sector which is a vital for bringing healthy and
efficient financial system in the economy. The idea to create it was first mooted by the Raghuram Rajan
Committee on Financial Sector Reforms in 2008.
2017 GKToday | All Rights Reserved | www.gktoday.in

123
Current Aairs: August, 2017 [Date-wise Compendium]

The FSDC envisages to strengthen and institutionalise mechanism of (i) maintaining financial stability, (ii)
Financial sector development, (iii) inter-regulatory coordination along with monitoring macro-prudential
regulation of economy.
Composition of FSDC: The Union Finance Minister is its chairman. Besides, heads of the financial sector
regulatory authorities (i.e, RBI, SEBI, IRDA, PFRDA), Finance Secretary and/or Secretary, Department of
Economic Affairs (Union Finance Ministry), Secretary, Department of Financial Services, and Chief
Economic Adviser are its members
Two Core functions: (i) Act as an apex level forum to strengthen and institutionalize the mechanism for
maintaining financial stability. (ii) Enhance inter-regulatory coordination and promoting financial sector
development in the country.
Other functions: (i) Focus on financial literacy and financial inclusion. (ii) Monitor macro-prudential
supervision of the economy. (iii) Assess the functioning of the large financial conglomerates.
Government announces National Sports Awards 2017
The Union Government bestows National Sports Awards every year to recognize and reward excellence in
sports in India.
These awards include Rajiv Gandhi Khel Ratna Award, Dronacharya Award, Arjuna Award, Dhyan Chand
Award, Maulana Abul Kalam Azad (MAKA) Trophy and Rashtriya Khel Protsahan Puruskar .
2017 Rajiv Gandhi Khel Ratna Award
It is Indias highest sporting award given for the spectacular and most outstanding performance in the field
of sports by a sportsperson over a period of four years.
Recipients are: Devendra (Para Athletics), Sardar Singh (Hockey).
2017 Dronacharya Award
It bestowed upon coaches for producing medal winners at prestigious international sports events.
Recipients are: Dr. R. Gandhi (Athletics), Heera Nand Kataria (Kabaddi), G.S.S.V. Prasad (Badminton), Brij
Bhushan Mohanty (Boxing), P.A. Raphel (Hockey), Sanjoy Chakraverthy (Shooting) and Roshan Lal
(Wrestling).
2017 Arjuna Award
It is Indias second highest sporting award bestowed upon sportsperson for consistently outstanding
performance for four years.
Recipients are: V.J. Surekha (Archery), Khushbir Kaur (Athletics), Arokia Rajiv (Athletics), Prasanthi
Singh (Basketball), LaishramDebendro Singh (Boxing), Cheteshwar Pujara (Cricket), Harmanpreet Kaur
(Cricket), OinamBembem Devi (Football), S.S.P. Chawrasia (Golf), S.V. Sunil (Hockey), Jasvir Singh
(Kabaddi), P. N. Prakash (Shooting), A. Amalraj (Table Tennis), Saketh Myneni (Tennis), SatyawartKadian
(Wrestling), Mariyappan (Para-Athletics) and Varun Singh Bhati (Para-Athletics).
2017 Dhyan Chand Award
It is bestowed upon sportspersons for life time contribution to sports development.
Recipients are: Bhupender Singh (Athletics), Syed Shahid Hakim (Football), Sumarai Tete (Hockey).
2017 GKToday | All Rights Reserved | www.gktoday.in

124
Current Aairs: August, 2017 [Date-wise Compendium]

TRAI ink LoI with Malaysian Communications and Multimedia Commission


The Telecom Regulatory Authority of India (TRAI) has signed a letter of intent (LoI) with Malaysian
Communications and Multimedia Commission (MCMC) deepen bilateral cooperation in telecom and
broadcasting regulation.
The LoI was signed on the sidelines of ongoing ITU-TRAI Asia-Pacific Regulators Roundtable jointly organised
by the International Telecom Union (ITU) and TRAI in New Delhi.
Key Facts
The LoI will help both regulators to jointly conduct capacity building exercises in the fields of broadcasting
and telecommunications regulation. It will help them to work closely in enhancing mutual cooperation and
bilateral relations.
It will also facilitate exchange of information and regulatory best practices in the fields of broadcasting and
telecommunications, including new and emerging areas. It will also help in exchange, deployment and
attachment of experts from both the countries.
Telecom Regulatory Authority of India (TRAI)
The TRAI is an independent regulator of the telecommunications business in India. It came into existence by
the Act of the Parliament in 1997. It was established in wake of entry of private sector in telecom industries
after Government had launched the National Telecom Policy (NTP) to attract domestic and FDI investment
in the telecommunication sector. Its mandate is to deliver a fair and transparent environment for fair
competition in telecom market. TRAI also fixes or revises the tariffs for telecom services in India.
August 23, 2017
Former Manipur CM Rishang Keishing passes away
Former Manipur Chief Minister and Veteran Congress leader Rishang Keishing passed away in Imphal
following a brief illness. He was 96.
He was freedom fighter and member of Indias first parliament (1952-57). He had served as the Chief
Minister of Manipur for four terms from 1980 to 1988 and 1994 to 1997.
About Rishang Keishing
Rishang Keishing was born 25 October 1920 and hailed from Bungpa village in Ukhrul district of Manipur.
He was the last surviving member of Indias first post-1947 Parliament. He had started his career as a teacher
before entering into active politics.
He was elected to the Lok Sabha in the first parliamentary elections held in 1952. He had won Lok Sabha
elections as a Socialist Party candidate from Manipur Outer constituency in 1957 and 1962. Later in 1964, he
joined Congress party at the invitation of Prime Minister Jawaharlal Nehru. He retired from the Rajya Sabha
as its oldest member in 2014 after serving two terms since being elected in 2002.
SC strikes down instant Triple Talaq
The Supreme Court of Indian has declared Islamic practice of instant divorce through triple talaq as
unconstitutional, illegal and void and not protected by Article 25 (freedom of religion) of the Constitution.
2017 GKToday | All Rights Reserved | www.gktoday.in

125
Current Aairs: August, 2017 [Date-wise Compendium]

The verdict was given by the 5 member multi-religion constitutional bench of the SC. Justice Kurian Joseph
(Christian), Justice U.U Lalit (Hindu) and Justice Rohington F, Nariman (Parsi) gave majority judgment.
Chief Justice J.S. Khehar (Sikh) and Justice S. Abdul Nazeer (Muslim) dissented against majority judgment.
SC Judgement
In a 3:2 majority judgment, the SC held that triple talaq (referred as Talaq-e-Biddat) is against the basic
tenets of Quran and hence, is unacceptable.
The three majority judges held that triple talaq is manifestly arbitrary and violative of the constitution and
fundamental right under Article 14 (equality before law) so it must be struck down. Two dissenting judges
were in favour of putting the practice on hold for six months and asking the government to come out with a
law.

Background
Triple Talaq is a personal practice followed by Muslim communities in the country as a form of divorce
wherein a man can unilaterally and arbitrarily end his marriage by merely pronouncing the word Talaq
thrice. The man pronouncing the divorce may deliver it through a letter, text message, email or even
through a video recording. This practice also does need valid reason to be provided either.
The controversy surrounding the issue arises from the fact that the divorced woman is not provided any
legal safeguards under the rule of law against such a divorce or any injustice arising from it. It was alleged
that this practise adversely impact rights of women to a life of dignity.

2017 GKToday | All Rights Reserved | www.gktoday.in

126
Current Aairs: August, 2017 [Date-wise Compendium]

It is also held that this practice against constitutional principles such as gender equality, secularism,
international laws etc. It also undermines gender equality is a sacred principle of our constitution and
modern society. Event 21 Islamic theocratic countries including Pakistan, Afghanistan have abolished it.
Nasikabatrachus bhupathi: New frog species with pig face discovered
Indian scientists have discovered Nasikabatrachus bhupathi, a new species of frog that has a snout-shaped
nose, just like a pig in West Ghats. It has been named after the Indian herpetologist S. Bhupathy.
Nasikabatrachus bhupathi species show comparisons with the Purple frog (Nasikabatrachus sahyadrensis)
which was discovered in 2003 in Seychelles.
Nasikabatrachus bhupathi
Nasikabatrachus bhupathi is soiled-dwelling species of purple frog. It inhabits the eastern slopes of the
Western Ghats, near the Srivilliputhur Grizzled Giant Squirrel Wildlife Sanctuary in Tamil Nadu.
It differs from the Purple frog morphologically and acoustically. It is dark brown in colour and each of its
calls consists of four distinct pulses while the Purple frog pauses once between its three-pulse-call.
Significance of Discovery
The discovery is significant as it constitutes additional evidence in favour of the continental drift theory. The
Purple frog inhabitant of Seychelles, and discovery of Nasikabatrachus bhupathi in India suggests that Indian
subcontinent was part of ancient landmass of Gondwana before splitting from Seychelles 65 million years
ago.
NITI Aayog launches Mentor India Campaign
The National Institution for Transforming India (NITI) Aayog has launched Mentor India Campaign, a
strategic nation building initiative to engage leaders for guiding and mentoring students at Atal Tinkering
Labs.
The Mentor India Campaign aims at maximising the impact of Atal Tinkering Labs by engaging leaders who
will nurture and guide students in the Atal Tinkering Labs.
Under this initiative, NITI Aayog is looking for leaders who can spend one to two hours every week in one
or more such labs to enable students learn, experience and practice future skills such as design and
computational thinking.
Atal Tinkering Labs
More than 900 Atal Tinkering Labs have been established across the country as a part of the Atal Innovation
Mission. These labs are meant to be non-prescriptive by nature, and mentors are expected to be enablers
rather than instructors. They are dedicated works spaces where students from Class 6th to Class 12th learn
innovation skills and develop ideas that will go on to transform India.
Atal Innovation Mission
NITI Aayogs Atal Innovation Mission is among one of the flagship programs of the Government to
promote innovation and entrepreneurship in the country and to set up the Atal Tinkering Labs across the
country. It aims to give substantial boost to the innovation ecosystem and to catalyze the entrepreneurial

2017 GKToday | All Rights Reserved | www.gktoday.in

127
Current Aairs: August, 2017 [Date-wise Compendium]

spirit in the country.


It prime objective is to provide an innovation promotion platform involving academics, researchers and
entrepreneurs, drawing upon national and international experiences to foster a culture of innovation in
India. It will also promote a network of world-class innovation hubs, Start-up businesses and other
selfemployment activities, particularly in technology driven areas.
NITI Aayog
NITI Aayog is the premier policy Think Tank of the Union Government, providing both directional and
policy inputs. It is essentially an advisory body that seeks to provide critical directional and strategic inputs
across spectrum of key elements of policy to the centre.
Cabinet approves commission on sub-categorisation of OBCs
The Union Cabinet approved a proposal for setting up of Commission under article 340 of the Constitution
to examine the issue of sub-categorization of the Other Backward Classes (OBCs).
The Commission shall be known as the Commission to examine the sub-categorization of Other Backward Classes
(OBCs). It will submit its report within 12 weeks from date of appointment of the Chairperson.
Terms of references of Commission
The Commission will examine the extent of inequitable distribution of benefits of reservation among the
castes included in the broad category of OBCs, especially with reference to the OBCs included in the Central
list. It will work out the mechanism, norms, criteria and parameters, in a scientific approach, for sub-
categorization within such OBCs. It will also take up the exercise of identifying the respective castes/sub-
castes/communities synonyms in the Central List of OBCs and classifying them into their respective sub-
categories.
Background
The Supreme Court in Indra Sawhney and others vs. Union of India case (1992) had observed that there is
no Constitutional or legal bar on states for categorizing backward classes as backward or more backward. It
had also observed that it is not impermissible in law if a State chooses to do sub-categorization. So far, 9
states/UTs viz., Andhra Pradesh, Karnataka, Haryana, Jharkhand, Telangana, Puducherry, Wst Bengal,
Bihar, Maharashtra and Tamil Nadu have already carried out sub-categorization of OBCs. There is no sub
categorisation in the central list.
Cabinet approves India-Nepal MoU on Prevention of Illicit Trafficking in Narcotic
Drugs
The Union Cabinet approved signing of Memorandum of Understanding (MoU) between India and Nepal
on Drug Demand Reduction and Prevention of Illicit Trafficking in Narcotic Drugs and Psychotropic
Substances.
The MoU lists out the areas of cooperation on narcotic drugs and psychotropic substances, their precursor
chemicals and related matters between both neighbouring countries. It also indicates mechanism of
information exchange and competent authorities between both countries responsible for the

2017 GKToday | All Rights Reserved | www.gktoday.in

128
Current Aairs: August, 2017 [Date-wise Compendium]

implementation of the MoU.


Features of MoU
Develop mutual cooperation: to effectively resolving the issue of illicit traffic in narcotic drugs,
psychotropic substances and their precursors, cooperate in drug demand reduction through prevention,
education, awareness and community based programmes, treatment and rehabilitation.
Exchange of information: It will be in operational, technical and general nature in drug matters. Morever,
there will be exchange literature on their existing laws, rules, best practices and methods of curbing illicit
trafficking in narcotic drugs.
Background
India has always supported global efforts to counter drug trafficking. It is party to several multilateral and
bilateral initiatives in this regard as also United Nations (UN) led initiatives. India has entered into mutiple
bilateral agreements/MoUs with neighbouring countries and countries having direct bearing on countrys
prevailing drug situation in accordance with the spirit of UN Conventions on Narcotic Drugs. Such bilateral
agreements have already been executed with various countries.
CCEA approves closure of Bharat Wagon and Engineering Company
The Cabinet Committee on Economic Affairs (CCEA) has approved closure of Bharat Wagon and
Engineering Company Limited (BWEL). BWEL is a Central Public Sector Enterprise (CPSE) under
Ministry of Railways (MoR).
Proposal in this regard was forwarded to CCEA by the MoR. The CCEA decision will stop flow of financial
support from public funds for carrying on the operations of BWEL resulting in savings for the Government.
Background
CCEA decision comes in view of the continues poor physical and financial performance of the BWEL for
more than 10 years, in spite of financial assistance and other support provided by MoR. The decision to
wound up BWEL comes after its low probability of revival in future.
Bharat Wagon and Engineering Company Limited (BWEL)
BWEL was incorporated as a CPSE in 1978 after taking over two sick private sector companies Arthur
Butler & Company, Muzaffarpur and Britannia Engineering Company, Mokama. It was engaged in
manufacture and repair of wagons. It was having two manufacturing units at Muzaffarpur and Mokama in
Bihar.
The administrative control of BWEL was transferred to the MoR from Department of Heavy Industry
(DHI) in August, 2008, as approved by the CCEA. The Company was referred to BIFR (Board for Industrial
and Financial Reconstruction) in December, 2000 and was declared sick in the year 2002. Since then it
remained sick (loss making) company.
Cabinet approves alternative mechanism Framework for consolidation of Public
Sector Banks
The Union Cabinet has approved Framework for Consolidation of Public Sector Banks (PSBs) through an

2017 GKToday | All Rights Reserved | www.gktoday.in

129
Current Aairs: August, 2017 [Date-wise Compendium]

Alternative Mechanism (AM).


The decision would facilitate consolidation among the nationalised banks to create strong and competitive
banks and also improve their competitiveness and efficiency.
Features of approved Framework for Consolidation of PSBs
Under this framework, the decision regarding creating strong and competitive banks will be solely based on
commercial considerations based on approval of the Boards of Banks (PSBs). The proposals of Banks to
formulate schemes of amalgamation must be approved by the Alternative Mechanism (AM).
There will be a ministerial committee to oversee the mechanism. The members of the panel will be decided
by the Prime Minister. After in-principle approval from AM, the Banks can take steps in accordance with
law and Security Exchange Boad of Indias (SEBIs) requirements. The final amalgamation or consolidation
scheme will be notified by the Union Government in consultation with the Reserve Bank of India (RBI).
Benefits
The decision is expected to facilitate the creation of strong and competitive banks in public sector space to
meet the credit needs of a growing economy, absorb shocks and have the capacity to raise resources without
depending unduly on the state exchequer. It will further add commercial strength and prevent multiplicity
of resources being spent in same area.
Background
The banking scenario has changed since 1970/80s when banks were nationalised, with an increased banking
presence from PSBs, non-banking Financial Companies (NBFCs), Regional Rural Banks (RRBs), Payment
Banks and Small Finance Banks (SFBs). There are now 20 PSBs other than SBI.
In 1991 after announcement of liberalisation of economy, it was suggested that there should fewer but
stronger PSBs in the country. However, the effective action for consolidating PSBs began in May 2016 after
announcement of amalgamation of six subsidiary banks into the State bank of India (SBI). The merger was
completed in record time, unlike earlier mergers of PSBs.
August 24, 2017
Union Cabinet approves new Bridge over Mechi River at India-Nepal border
The Union Cabinet has approved a Memorandum of Understanding (MoU) between India and Nepal for
starting construction of a new Bridge over Mechi River at India-Nepal border.
The new bridge will be constructed at an estimated cost of Rs. 158.65 crore, which will be funded by India
through Asian Development Bank (ADB) loan.
Key facts
The new bridge over Mechi River, covering a length of 1500 meters will be constructed as part of up-
gradation of the Kakarvitta (Nepal) to Panitanki Bypass (India) on NH 327B. National Highway and
Infrastructure Development Corporation (NHIDCL) under Ministry of Road Transport & Highways will be
the implementing agency for this project.
Mechi Bridge is the ending point of Asian Highway 02 (AH02) in India leading to Nepal and provides critical
connectivity to Nepal. The construction of bridge will improve regional connectivity and has potential to
2017 GKToday | All Rights Reserved | www.gktoday.in

130
Current Aairs: August, 2017 [Date-wise Compendium]

strengthen cross border trade between both countries and potential to strengthen industrial, social and
cultural exchanges.
Mechi River: It is a trans-boundary river flowing through Nepal and India. It originates in the Mahabharat
Range in Nepal. It is tributary of Mahananda River.
Government to establish first National Sports Museum in New Delhi
The Union Ministry of Sports and Youth Affairs is going to establish National Sports Museum at Jawaharlal
Nehru Stadium, New Delhi. It will be first of its kind sports museum in India.
The aim of the sports museum is to popularise the sports as a way of life within the country. It will showcase
the Indias achievements in sports and also focus on the traditional sports within the country.
Key Facts
The proposed Museum will display achievements of the Indian greats in sports and also act as an educational
hub and offer opportunity for the young to delve into some sporting activity. It will be interactive venue of
interest for all age groups, going beyond the classical approach of a collection of memorabilia
The museum will also have audio-visual display of rules and legacy events pertaining to various sports as
part of education for the budding sportspersons. It will also have a well-equipped library of sports and
physical fitness, painting gallery and amphitheater for screening of legacy matches, games.
All past and present sports persons and members of public can contribute any sports memorabilia held by
them for permanent display in the museum. The activities planned in museum in various segments will
include sporting activities, sports related computer games and kiosks selling sports memorabilia and sports
equipment.
Government notifies new Rs.200 notes
The Union Finance Ministry on the recommendations of the Central Board of Directors of the Reserve
Bank of India (RBI) has notified currency notes of Rs.200. The new notes are likely to be in circulation
shortly.
The new Rs.200 currency note was issued through an official gazette notification by the Finance Ministrys
Department of Economic Affairs in exercise of powers conferred by sub-section (1) of section 24 of RBI Act,
1934.
Key Facts
The Rs.200 denomination banknotes in the Mahatma Gandhi (New) Series bear signature of Governor Dr.
Urjit R. Patel. It has Motif of Sanchi Stupa on the reverse, depicting the countrys cultural heritage. The base
colour of the note is Bright Yellow.
The new Rs.200 currency note will carry advanced security features to prevent counterfeiting. It has gone
through different levels of security and quality checks at the governments press unit at Hoshangabad
(Madhya Pradesh).
It has also reported that RBI may not allow ATMs to dispense new Rs 200 notes. It may be circulated only
through banks and not ATMs like Rs.50, Rs.20 and Rs.10 notes are circulated.

2017 GKToday | All Rights Reserved | www.gktoday.in

131
Current Aairs: August, 2017 [Date-wise Compendium]

Comment
This is the fourth new note (after Rs.2000, Rs, 500 and Rs.50) to be announced since November 2016
aftermath of demonetisation when Rs. 1000 and Rs.500 old currency notes were demonetised. The
introduction of the new Rs. 200 note is expected to ease the problems faced by people due to high-value
Rs.2,000 notes and ease pressure on lower-denomination currency bills.
Printing of currency notes
The printing of currency notes in India is done either in printing units run by the government-run Security
Printing and Minting Corporation of India Ltd (SPMCIL) or at printing presses in Salboni (West Bengal)
and Mysore (Karnataka), managed by the RBI-owned Bharatiya Reserve Bank Note Mudran Private Ltd
(BRBNMP).
Odisha Government launches rooftop solar project
Odisha Government has launched a grid-connected rooftop solar programme through net-metering system
at the State Secretariat in capital city of Bhubaneswar.
The programme aims at harnessing renewable solar capacity through peoples participation to further
strengthen the grid and reduce power losses and benefit the public. It will also help state to targets in
addressing the climate change action plan.
Key Facts
The programme is being implemented by Odisha Renewable Energy Development Agency (OREDA). Under
it, consumers from state can install subsided small solar plants on their rooftops and consume the generated
solar power during daytime. They can also export excess power to grid at 220V/440V level through their
service cables.
Under this programme, state government will provide subsidy of 30% to consumers belonging to residential
and institutional segments as well as registered societies and trusts. The exported power will be metered on
Net-Metre, a special type of metre provided by the discoms (power distribution companies).
The Net-Metre will help to bill consumer by taking into consideration positive difference of import from the
grid and the export to the grid. It will enable consumer to reduce his power consumption by using his own
solar power and simultaneously a reduce bill on account of export of solar power to the grid.
Karnataka ties up with Microsoft to develop Indias first Farm Price Forecasting
Model
Karnataka Government has signed a Memorandum of Understanding (MoU) with software giant Microsoft
India to develop a unique farm price forecasting model.
The proposed initiative is claimed to be the first-of-its-kind in the country. It will aid farmers from states to
predict farm prices in advance and deal effectively with the unpredictable market.
Key Facts
The farm price forecasting model will use latest IT tools in a bid to help farmers, administrators and other
stakeholders to understand market behaviour in advance. It will be a multi-variate platform that will take
into consideration various factors beyond conventional supply-demand equation while forecasting prices of
2017 GKToday | All Rights Reserved | www.gktoday.in

132
Current Aairs: August, 2017 [Date-wise Compendium]

crops.
The model will examine a slew of related factors including weather, rainfall and external factors that impact
market behaviour. It will help to provide real-time market data, besides helping analyse its implications on
farm sector. The model is expected to be in place for major crops by 2018 kharif season.
Significance
The purpose of the mode is to achieve is predictability as well as stability of farm incomes. Knowing the
market situation in advance, it will help farmers, policymakers and industry make informed decisions.
Moreover, the forecast will aslo help the government prepare in advance for market intervention if there
was an indication of price crash.
CCEA approves renaming SAMPADA Scheme as PM Kisan Sampada Yojana
The Cabinet Committee on Economic Affairs (CCEA) has approved renaming of SAMPADA (Scheme for
Agro-Marine Processing and Development of Agro-Processing Clusters) as Pradhan Mantri Kisan Sampada
Yojana (PMKSY).
The SAMPADA scheme was approved by the CCEA in May 2017 for the period of 2016-20 coterminous
with the 14th Finance Commission cycle. The objective of the scheme is to supplement agriculture,
modernize processing and decrease agricultural-waste.
Pradhan Mantri Kisan Sampada Yojana (PMKSY)
PMKSY is an umbrella scheme which incorporates all ongoing schemes of the Union Ministry of Food
Processing Industries (MoFPI). It includes MoFPIs schemes such as Mega Food Parks, Food Safety and
Quality Assurance Infrastructure, Integrated Cold Chain and Value Addition Infrastructure, etc. It also
includes new schemes like Creation of Backward and Forward Linkages, Infrastructure for Agro-processing
Clusters, Creation/Expansion of Food Processing & Preservation Capacities.
Financial Allocation
PMKSY will have budgetary allocation of Rs. 6000 and is expected to leverage investment of Rs. 31,400
crore, handling of 334 lakh MT agro-produce valuing Rs. 1 lakh 4 thousand 125 crore. It will benefit 2
million farmers and generate over 5 lakh 30 thousand direct or indirect employment in the country by the
year 2019-20.
Significance
The implementation of PMKSY will result in creation of modern infrastructure with efficient supply chain
management from farm gate to retail outlet. It will provide a big boost to the growth of food processing
sector in the country which is important segment of the Indian economy in terms of its contribution to
GDP, employment and investment.
It will help in providing better prices to farmers and is a big step towards doubling of farmers income. It will
create huge employment opportunities especially in the rural areas and also help in reducing wastage of
agricultural produce, increasing availability of safe and convenient processed foods at affordable price to
consumers and enhancing the export of the processed foods.

2017 GKToday | All Rights Reserved | www.gktoday.in

133
Current Aairs: August, 2017 [Date-wise Compendium]

Supreme Court declares right to privacy as Fundamental right under constitution


The nine-judge Constitution Bench of the Supreme Court headed by Chief Justice of India JS Khehar in a
landmark unanimous decision has declared right to privacy a fundamental right under the constitution.
With this, the apex court overruled its own previous eight-judge Bench and six-judge Bench judgments of
M.P. Sharma and Kharak Singh delivered in 1954 and 1961, respectively, both of which had pronounced that
the right to privacy is not protected under the constitution.
Supreme Court Judgment
The apex court ruled that right to privacy is an intrinsic part of Right to Life and Personal Liberty under Article 21
and entire Part III of the Constitution. It overruled the apex courts earlier two judgements that right to privacy
is not protected under the Constitution.
The apex court also had voiced concern over the possible misuse of personal information in the public
domain. The question whether Aadhaar violates right to privacy will be dealt with by the five-judge bench
which has been hearing the petitions since 2015.
Comment
The Supreme Court of Indias judgment gains international significance as privacy enjoys a robust legal
framework internationally, though India had earlier remained circumspect. The judgment finally reconcile
Indian laws with the spirit of Article 12 of the Universal Declaration of Human Rights (1948) and Article 17
of the International Covenant on Civil and Political Rights (ICCPR), 1966, which legally protects persons
against the arbitrary interference with ones privacy, honour and reputation, family, home and
correspondence.
India, Nepal sign 8 MoUs
India and Nepal have signed eight Memorandum of Understanding (MoUs) in various fields, including
Housing grant, prevention of drugs, post-earthquake reconstruction packages in health and education
sectors.
The agreements were inked after delegation level talks between Prime Minister Narendra Modi and his
visiting Nepalese counterpart Sher Bahadur Deuba in New Delhi. The two leaders also jointly inaugurated
the Kataiya-Kusaha and Raxaul-Parwanipur cross border Transmission lines.
Signed MoUs are
Agreement on Cooperation in the field of standardization and conformity assessment.
MoU between Institute of Chartered Accountants of India (ICAI) and the Institute of
Chartered Accountants of Nepal (ICAN).
MoU on Modalities for Utilization of Indias Housing Grant Component to support
Reconstruction of 50,000 Houses.
Three MoUs on Implementation of the Grant Component of Indias Post-Earthquake
Reconstruction Package in the Education Sector, Cultural Heritage Sector and Health Sector
in Nepal.
2017 GKToday | All Rights Reserved | www.gktoday.in

134
Current Aairs: August, 2017 [Date-wise Compendium]

MoU for Implementation Arrangement for Construction of Mechi Bridge under ADBs
SASEC Road Connectivity Programme (Tranche 2), Funded by India.
MoU on Drug Demand Reduction and Prevention of Illicit Trafficking in Narcotic Drugs,
Psychotropic Substances and Precursor Chemical and Related Matters
August 25, 2017
Government appoints Ashwani Lohani as New Chairman of Railway Board
The Union Government has appointed Ashwani Lohani as the new Chairman of the Railway Board
(Ministry of Railways). Prior to this appointment, he was serving as Chairman and Managing Director
(CMD) of Air India.
His appointment comes after A.K. Mital resigned from post, owing moral responsibility of Puri-Haridwar
Utkal Express derailment in Uttar Pradesh due to several lapses in track maintenance work.
About Ashwani Lohani
Mr. Lohani is an Indian Railway Service of Mechanical Engineering (IRSME) cadre. Earlier he had served as
DRM, Delhi Division; Chief Administrative Officer, Rail Alternate Fuel and Director, National Rail
Museum. He holds the Guinness Record for successfully running the Fairy Queen Express, the worlds
oldest working steam locomotive.
He holds four engineering degrees, has authored two books (one on steam engines and the other on
management). He is credited with the turnaround of the Madhya Pradesh Tourism Development
Corporation (MPTDC) and the India Tourism Development Corporation.
Railway Board
It is the apex body of the Indian Railways. It reports to the Parliament through the Ministry of Railways
headed by the Union Railway Minister. The Chairman of the Board with the status of a Secretary to
Government in the Ministry of Railways is responsible for decisions on technical and policy matters of
Indian Railways.
Microsoft launches Project Brainwave for real-time Artificial Intelligence
Software giant Microsoft has launched Project Brainwave, a deep learning acceleration platform for real-
time artificial intelligence (AI).
Its launch is important as the real-time AI has increasingly became important as cloud infrastructures
process live data streams, whether they be videos, sensor streams, search queries or interactions with users.
Key facts
The Project Brainwave uses the massive field-programmable gate array (FPGA) infrastructure which has
been deployed by Microsoft over the past few years. It can processes requests as fast as it receives them with
the help of ultra-low latency.
The Project Brainwave system architecture reduces latency, since its Central Processing Unit (CPU) does not
need to process incoming requests. It als allows very high throughput, with the FPGA processing requests as
fast as the network can stream them.
2017 GKToday | All Rights Reserved | www.gktoday.in

135
Current Aairs: August, 2017 [Date-wise Compendium]

The system has been architected to yield high actual performance across a wide range of complex models,
with batch-free execution. It can handle complex, memory-intensive models such as Long Short Term
Memories (LSTM), without using batching to juice throughput.
Artificial Intelligence (AI)
Artificial intelligence is branch of computer science concerned with making computers behave like humans.
In contrast to normal hardware and software, AI enables a machine to perceive and respond to its changing
environment.
Moss serves as a cheap pollution monitor: Scientists
According to Japanese scientists, delicate mosses found on rocks and trees in cities around the world can be
used as low-cost bioindicator to monitor urban pollution and to measure the impact of atmospheric change.
As a bioindicator, mosses respond to pollution or drought-stress by changing its shape, density or
disappearing. This characteristic will allow scientists to calculate atmospheric alterations and air pollution.
Key Facts
Mosses are a common flowerless plant found in all cities especially in damp (humid) or shady locations. It
generally absorb water and nutrients from their immediate environments, so it can reflect changes to
ecosystems. So it can be used as potential bioindicators.
In the study, Japanese researchers studied the effect of nitrogen pollution, air quality and drought-stress on
moss. They found that drought-stress tends to occur in mosses found in areas with high levels of nitrogen
pollution, which has negative impact on health and biodiversity. It can be cost effective and important for
getting information about atmospheric conditions especially effect of nitrogen pollution and air quality in
urban area.
Government eases norms under UDAN Scheme
The Union Civil Aviation Ministry has eased several norms of Ude Desh Ka Aam Nagrik (UDAN), a flagship
regional flying scheme to attract more airlines and helicopter operators to participate in regional
connectivity scheme (RCS).
The new norms were announced when the process for second round of bidding for the RCS were
announced. . The winners of the second round of bidding will be announced by November 2017.
Key Facts
The relaxations of the norms include dilution of the exclusivity clause mandating that only one airline may
fly on one route in the initial years. It will allow selected airline operator of a particular route may issue no-
objection certificate (NOC) to other airlines who want to operate on the selected RCS route.
The norms restricting two airports in close proximity from participating in the bidding also has been
relaxed. It will allow routes with a stage length less than 150 km for operations through fixed wing aircraft.
About Ude Desh Ka Aam Nagrik (UDAN) Scheme
The UDAN Scheme aims at providing connectivity to un-served and under-served airports of the country
through revival of existing airstrips and airports. It aims to develop the regional aviation market and make
flying affordable. It is applicable on flights which cover between 200 km and 800 km. The distance limit is
2017 GKToday | All Rights Reserved | www.gktoday.in

136
Current Aairs: August, 2017 [Date-wise Compendium]

lower limit for hilly, remote, island and security sensitive regions. It reserves minimum number of UDAN
seats i.e. seats at subsidized rates and also cap the fare for short distance flights. It has a unique market-based
model to develop regional connectivity.
August 26, 2017
GM crops only after biosafety, socio-economic evaluation: Parliamentary
Committee
The Parliamentary Standing Committee has recommended that no genetically modified (GM) crop should
be introduced unless the biosafety and socio-economic desirability is evaluated in a transparent process. It
also called for placing an accountable regime in this regard.
The recommendation was made by department-related parliamentary standing committee on science and
technology and environment and forest in its 301st report on GM crop and its impact on environment.
Report Highlights
The Committee has also recommended that the Environment Ministry (MoEFCC) should examine the
impact of GM crops on the environment thoroughly, in consultation with all stakeholders, so that all its
probable effects are very clear.
It also noted that the GM crop regulator GEAC (Genetic Engineering Appraisal Committee) gave its
approval for commercialisation of GM mustard inspite of matter pending for decision in Supreme Court. It
held that GM mustard being herbicide-tolerant GM organism (GMO), there are evidences on adverse
impacts of such GMOs elsewhere in world.
Background
The Centre for Genetic Manipulation of Crop Plants (CGMCP), Delhi University, had submitted an
application to the GEAC for the environmental release of GM mustard hybrid DMH-11 and the use of
parental events (EH2 mod bs 2.99 and varuna bn 3.6) for the development of a new generation of hybrids.
The MoEFCC had received over 700 comments from various stakeholders, including farmers and
researchers, on the Assessment of Food and Environmental Safety (AFES) report on GM Mustard.
Government constitutes task force on artificial intelligence
The Union Ministry of Commerce and Industry has constituted a Task Force on Artificial Intelligence (AI)
for Indias Economic Transformation.
The 18-member panel will be headed by Dr. V. Kamakoti of IIT Madras. It will comprise of experts,
academics, researchers and industry leaders.
Key Facts
The Task force will explore the possibilities to leverage AI for development across various fields. It will
submit concrete and implementable recommendations for government, industry and research institutions.
In addition to regular members, official participation from the following organizations such as NITI Aayog,
Ministry of Electronics and Information Technology, Department of Science & Technology, UIDAI and
DRDO will be also requested.
2017 GKToday | All Rights Reserved | www.gktoday.in

137
Current Aairs: August, 2017 [Date-wise Compendium]

Background
With rapid development in the fields of information technology and hardware, the world is about to witness
a fourth industrial revolution. It will be driven by the power of big data, high computing capacity, artificial
intelligence and analytics. It is going to digitise the manufacturing sector.
Government bans import of gold, silver items from South Korea
The Directorate General of Foreign Trade (DGFT) in the Union Commerce Ministry has banned duty free
imports of gold, silver items from South Korea.
These restrictions were imposed against the backdrop of sudden surge in imports of precious metals from
South Korea, with which India has a free trade agreement (FTA) since January 2010. Now, importers will
now have to obtain a license from the DGFT for importing gold and silver from South Korea.
Sudden surge
Under the India-South Korea FTA, India has allowed duty free import of gold and silver items. However,
12.5% countervailing duty was imposed to offset equal level of excise duty on gold and silver jewellery items
produced domestically.
After goods and services tax (GST) regime was implemented from 1 July 2017, countervailing duty was
abolished as it subsumed excise duty and only 3% GST was imposed on gold. This created a situation where
importing gold via South Korea became profitable due to its duty free status with India even as government
continued to impose 10% basic customs duty on gold imports from other countries. Between 1 July and 3
August 2017, gold imports from South Korea surged to $339 million against import of only $70.5 million in
2016-17.
Background
India is the worlds second largest gold consumer after China, with consumption of 674 tonnes in 2016.
According to the World Gold Council (WGC), Indias gold demand is projected to rise to between 850
tonnes and 950 tonnes by 2020 from an estimated 650-750 tonnes in 2017.
Directorate General of Foreign Trade (DGFT)
The DGFT is the nodal agency responsible for execution of the import and export Policies of India. It is
entrusted with responsibilities for formulating and implementing the foreign trade policy with the main
objective of promoting Indias exports.
NPCI approves Spice Digitals operation under Bharat Bill Payment System
The National Payments Corporation of India (NPCI) has given a nod to Spice Digital Ltd. for processing bill
payments under the Bharat Bill Payment System (BBPS) as a Bharat Bill Payment Operating Unit (BBPOU).
Spice Money Ltd, a fintech company had received an in-principle approval to set-up as a BBPOU under the
Payment and Settlement Systems Act, 2007 in May 2016. In August 2016, NPCI had launched a pilot project
for BBPS with eight BBPS operating unit, Spice Digital was one of them
Need
Bill payments is a major component of the retail payment transactions in India. It is characterized by the
presence of large number of billers and aggregators who provide a variety of payment options to customers.
2017 GKToday | All Rights Reserved | www.gktoday.in

138
Current Aairs: August, 2017 [Date-wise Compendium]

According to the Digital Payments 2020 report by Google and Boston Consulting Group, cashless payments
contribution in the consumer payments segment will double to 40%.
Bharat Bill Payment System (BBPS)
BBPS is an integrated bill payment system offering interoperable and accessible bill payment service to
customers through a network of agents, enabling multiple payment modes and providing instant
confirmation of payment. The BBPS initiative aims to provide a major push to digital payments as it is a big
step forward in formalizing the bill payment system in the country.
Under the BBPS framework, a customer will be able to pay several bills such as electricity, telephone, water,
gas, and DTH television at a single locationphysical or electronicand receive instant confirmation once
the payment is made.
National Payments Corporation of India (NPCI)
NPCI is the umbrella organisation for all retail payments system in India. It is being promoted the Reserve
Bank of India. It was founded in 2008 as a not-for-profit organisation registered under section 25 of the
Companies Act, 2013.
CSIR-NIIST develop new technique to produce bioethanol from cotton-stalks
Scientists from CSIRs National Institute for Interdisciplinary Science and Technology (CSIR-NIIST),
Thiruvananthapuram have developed new technique to produce bioethanol from discarded cotton-stalks.
The technique uses combination of chemical and biological techniques.
The new technique will help to produce bioethanol, a clean fuel from cotton stalk wastes. Converting the
agro-residues such as cotton stalks and wheat and rice husks etc to ethanol reduces the food vs fuel
competition.
India has about 9.4 million hectares under cotton cultivation and every hectare generates 2 million tonnes of
cotton stalk wastes.
New Technique
In new technique, cotton stalks first are treated with acid, alkali and different enzymes to breakdown its
complex organic polymers. The acid treatment remove hemicellulose, a polymer of cell wall and alkali makes
easier to extract lignin, a binding matrix in cell wall, made of complex phenolics.
These treatments exposes cellulose of cotton stalks made of glucose to the action of enzymes. It converts
glucose into final product bioethanol by the process of fermentation using a novel yeast strain.
The novel yeast strain used was Saccharomycescerevisiae-RRP-03N, obtained from a rotting wild fruit we found
in the Silent Valley National Park, Kerala.
Significance
The Saccharomycescerevisiae-RRP-03N yeast performs better than distillers yeast strains commonly used in
fermenting the cotton stalk hydrolysate. It has glucose conversion efficiency of 76% and it utilised entire
glucose in just 24 hours and converted into alcohol. The final alcohol obtained after distillation can be made
to fuel grade bioethanol (>99% purity).

2017 GKToday | All Rights Reserved | www.gktoday.in

139
Current Aairs: August, 2017 [Date-wise Compendium]

IIT Researchers develop biosensor to detect kidney disorders


Researchers from Indian Institutes of Technology (IIT), Bombay and Indore, have jointly developed a
biosensor that makes it possible to detect kidney disorders in less than eight minutes.
The biosensor can accurately measure both the pH and urea concentration with a single drop of urine. It will
help make a point-of-care test to determine whether the kidneys are functioning normally.
Drawbacks of conventional tests
For conventional kidney function test, doctors estimate pH and urea contents in urine as most kidney
disorders result in reduced pH and higher concentration of urea. In current available methods to detect urea,
patients need to undergo two tests for accuracy. In addition, these methods have accuracy problem due to
contaminations components in urine such as calcium, chloride, ascorbic acid, sodium and potassium.
Biosensor Tests
The newly developed biosensor can detect both pH and urea contents in urine. It is made by encapsulating
an enzyme urease and molecule FITC-dextran in alginate microspheres. The combination glows in fluorescence
colour in response to chemical reaction with urea and changes in pH when urine is added. The fluorescence
reduces when the pH is acidic and increases when it is alkaline. The change in intensity of fluorescence helps
to calculate the values of pH and urea.
Significance
The biosensor made using alginate is safe and non-toxic to handle. It can work in the ideal pH range of 4-8.
It is able detect even low concentrations of urea up to 50 millimolar. It has showed accuracy of more than
97%. Moreover, it is stable for up to a month in a refrigerator and gives results unaffected by other
components in urine samples. It can help to make rapid and accurate point-of-care diagnostic test for kidney
disorders.
August 27, 28, 2017
PV Sindhu wins silver medal at 2017 World Badminton Championship
Indian ace shuttler and 2017 Rio Olympic silver medallist PV Sindhu won the silver medal in 2017 World
Badminton Championship held at Glasgow, Scotland (United Kingdom).
In the final match, Sindhu lost to Japans Nozomi Okuhara by 19-21, 22-20, 20-22 score. Earlier, in Saina
Nehwal had won bronze medal. This is for first time, Indian shuttlers have won two medals in the
Championship.
It was overall Sindhus third medal at the World Championships as she earlier had won bronze medals in
2013 and 2014 editions.
PV Sindhu
PV Sindhu received international attention after getting listed on the Top 20 list of Badminton World
Federation rankings in 2012. In 2013, she created history by becoming first Indian womens singles
badminton player to win medal at World Championships. Sindhu had won Silver medal in 2017 Rio
Olympics. She was the second Indian female shuttler to win an Olympic medal after Saina Nehwal who had

2017 GKToday | All Rights Reserved | www.gktoday.in

140
Current Aairs: August, 2017 [Date-wise Compendium]

won bronze medal in 2012 London Olympics. Sindhu has been awarded Padma Shri in 2015, Indias fourth-
highest civilian honour.
Indias achievements at World Badminton Championship
Prakash Padukone was the first Indian to win a medal at Championship after he had won bronze medal in
the mens singles in 1983. Womens doubles pair Jwala Gutta and Ashwini Ponnappa had won bronze medal
in 2011.
Government imposes anti-dumping duty on sodium nitrate imports from China
The Union Finance Ministrys Revenue Department has imposed a five year anti-dumping duty on Sodium
Nitrite imports from China. It was imposed based on recommendations of Directorate General of
Antidumping and Allied Duties (DGAD).
In its sunset review, the DGAD concluded that there is continued dumping of the sodium nitrite from
China and it is causing injury to the domestic industry.
Sodium Nitrite
Sodium Nitrite is a white crystalline powder. It is an oxidising and also a reducing agent. It is mostly used in
the pharmaceutical and dye industries, lubricants, construction chemicals, rubber blowing agent, meat
processing, and textiles.
Anti-dumping Duty
Countries impose anti-dumping duties to safeguard domestic industry from surge in below-cost imports. It
is not a measure to restrict import or cause an unjustified increase in cost of products. Anti-dumping duty is
imposed inorder to ensure fair trade and provide a level-playing field to the domestic industry.
It is protectionist and counter import measure used by a country under the multilateral World Trade
Organisation (WTO) regime. The DGAD, under the Union Commerce Ministry, recommends the duty,
while the Union Finance Ministry imposes the same. India so far has also imposed similar anti-dumping
duties on import of several other products including steel, fabrics and chemicals from different countries
including China.
Government notifies Banking Regulation (Amendment) Act, 2017
The Union Government has notified the Banking Regulation (Amendment) Act, 2017. The Parliament had
approved the Banking Regulation (Amendment) Bill, 2017 which replaced an ordinance in this regard.
It amended the Banking Regulation Act, 1949 by adding provisions for handling cases related to stressed
assets or non-performing assets (NPAs) of banks.
Key Facts
The Act empowers the Central government to authorize the Reserve Bank of India (RBI) to direct banking
companies to resolve specific stressed assets by initiating insolvency resolution process under the Insolvency
and Bankruptcy Code, 2016. The RBI can specify authorities or committees to advise banks on resolution of
stressed assets. The members on the committees will be appointed or approved by the RBI. The Act also
make these provisions applicable to the SBI and its subsidiaries and also Regional Rural Banks (RRBs).

2017 GKToday | All Rights Reserved | www.gktoday.in

141
Current Aairs: August, 2017 [Date-wise Compendium]

Background
The banking sector in India is saddled with non-performing assets (NPAs) of over Rs.8 lakh crore, of which,
Rs. 6 lakh crore are with public sector banks (PSBs). The Union Government in May 2017 had promulgated
an ordinance authorising the RBI to issue directions to banks to initiate insolvency resolution process under
the Insolvency and Bankruptcy Code, 2016. The RBI had identified 12 accounts each having more than Rs.
5000 crore of outstanding loans and accounting for 25% of total NPAs of banks for immediate referral for
resolution under the bankruptcy law. The bulk of the NPAs are in various sectors including power, steel,
road infrastructure and textiles.
DRDO, IAI to produce MRSAM missile system by 2020 for Indian Army
The DRDO has signed Rs.17,000 crore deal with Israel Aerospace Industries (IAI) for producing advanced
mediumrange surface to air missile (MRSAM) system for Indian Army by 2020. The deal envisages
develping 40 firing units and around 200 missiles.
Key Facts
The MR-SAM, a land-based version of long range surface to air missile (LRSAM) of Indian Navy, has strike
range of up 70 km. It is capable of shooting down enemy ballistic missiles, aircraft, helicopters, drones,
surveillance aircraft and AWACS (Airborne Warning and Control Systems) aircraft.
The MRSAM for Indian Armys Air Defence is an advanced all weather, 360 degree mobile land based
theatre air defence system capable of providing air defence to critical areas against wide variety of threats in
combat zone. The current version of MRSAM is operational with the Indian Air Force and the Navy.
Background
The Indian Army has been pressing the government to enhance its aerial attack capability considering the
evolving security challenges. Indian Army is the first land force in the world to deploy Brahmos missile in
2007. It has raised several regiments of this formidable weapon. In May 2015, the Army had inducted the
indigenously developed supersonic surface to air Akash missile which is capable of targeting enemy
helicopters, aircraft and UAVs from a range of 25 km. The Army thinks procurement of MRSAM will mark
paradigm shift in its strike capability.
Government inaugurates Indias first Videsh Bhavan in Mumbai
The Union Ministry of External Affairs (MEA) launched countrys first Videsh Bhavan by assimilating all
Regional Passport Offices and allied departments under a single roof at the Bandra Kurla Complex (BKC) in
Mumbai, Maharashtra.
Key Facts
Currently, 90-plus RPOs and PoEs across the country function from rented offices. Merging and integrating
them into similar Videsh Bhavans in different state capitals is expected to cut costs besides ensuring
improving efficiency and speeding up the delivery of various services.
Under this first pilot project of Videsh Bhavan in Mumbai, four offices of MEA, Regional Passport Office
(RPO), Protector of Emigrants (PoE) office, Branch Secretariat and Regional Office of ICCR were integrated
and brought to function under one roof at the state-of-art office in BKC.
2017 GKToday | All Rights Reserved | www.gktoday.in

142
Current Aairs: August, 2017 [Date-wise Compendium]

The pilot project is part of Union Governments policy to bring together different offices of the MEA under
a single roof and work closely with states as more and more Indians are going abroad for education,
employment, business and tourism.
Justice Dipak Misra sworn in as 45th Chief Justice of India
Justice Dipak Misra (64) sworn in as the 45th Chief Justice of India (CJI). He was administered the oath of
office by President Ram Nath Kovind at the Rashtrapati Bhawan.
He succeeds Chief Justice J S Khehar. He will have tenure of 14 months and will demit office in October
2018. He is third person from Odisha to be appointed the CJI, after Justices Ranganath Misra and G B
Pattanaik.
About Justice Dipak Misra
Justice Dipak Misra was born on 3 October 1953 in Odhisha. He is the nephew of Justice Ranganath Misra
who was CJI from September 1990 November 1991. He began his career as an advocate in 1977 and has
practised in constitutional, civil, criminal, revenue, service and sales tax matters in the Orissa High Court
and Tribunals before was elevated to the High Court bench.
He was appointed Additional Judge of the Orissa High Court in January 1996 and later was transferred to the
Madhya Pradesh High Court in March, 1997 where he became a permanent judge. In 2009, Justice Misra was
elevated as Chief Justice of the Patna High Court and later as the Chief Justice of the Delhi High Court in
May, 2010.
He was elevated as a Judge of the Supreme Court in October 2011. In the apex court, he has already presided
over several key cases and verdicts. He had led the SC bench which rejected the 1993 Mumbai serial blasts
convict Yakub Memons appeal to stop his execution. He also had led a three judge SC bench which had
upheld the death sentence awarded to the four convicts of the Nirbhaya rape case in May, 2017.
He also has given significant judgment directing states and UTs to upload First Information Reports (FIRs)
filed with police on websites within 24 hours of their registration for enabling accused and others to file
appropriate pleas in the courts for redress.
ICOSIS unveils Ocean forecasting system for Comoros, Madagascar and
Mozambique
The Indian National Centre for Ocean Information Services (INCOIS) of the Ministry of Earth Sciences
inaugurated the Ocean Forecasting System for Indian Ocean countries Comoros, Madagascar, and
Mozambique.
It was unveiled at the third Ministerial Meeting of Regional Integrated Multi-Hazard Early Warning System
for Asia and Africa (RIMES) held at Port Moresby, Papua New Guinea.
Key Facts
The Ocean Forecast System will offer, high wave alerts, port warnings, forecast, oil spill advisory services
along the ship routes in addition

2017 GKToday | All Rights Reserved | www.gktoday.in

143
Current Aairs: August, 2017 [Date-wise Compendium]

to tsunami and storm surge warnings and help in search and rescue operations.
These ocean services are aimed towards safety at the sea.
The ocean forecast and early warming information on winds, high wave, currents, tides, sub-surface ocean
conditions will cater to users like coastal population, fishermen, tourism sector, coastal defence officials, port
authorities, marine police, research institutions and offshore industries of these countries.
Background
INCOIS has been developing Ocean Forecast System for the Indian Ocean countries and already has
provided these operational services to Maldives, Sri Lanka and Seychelles. Moreover, the real-time data from
territories of these countries also help India to improve the ocean forecast and early warning system.
Indian National Centre for Ocean Information Services (INCOIS)
INCOIS is an autonomous organization of the Union Government, under the Ministry of Earth Sciences is a
unit of the Earth System Science Organization (ESSO). It was established in 1999 and is headquartered in
Pragathi Nagar, Hyderabad.
Its mandated is to provide the best possible ocean information and advisory services to government agencies,
society, industry and the scientific community through sustained ocean observations and constant
improvements through systematic and focussed research.
August 28, 2017
ICEX launches worlds first diamond futures contracts
The Indian Commodity Exchange (ICEX) as launched worlds first diamond futures contracts to provide
exporters with a hedging tool.
India is a global diamond polishing hub where 14 out of every 15 rough diamonds in the world are polished.
The futures contracts will enable companies involving in cut and polished gems business to better hedge
price risks.
ICEX has received approval from market regulator Securities Exchange Board of India (SEBI) to commence
trading with diamond futures contracts.
2017 GKToday | All Rights Reserved | www.gktoday.in

144
Current Aairs: August, 2017 [Date-wise Compendium]

Key Facts
The ICEX has started trading in 1 carat/100 cent contracts and eventually in future will add 50 cent and 30
cent contracts. It has put in place necessary infrastructure to handle deposits, grading and sealing, vaulting
and delivery of diamonds for the diamond futures contracts.
To avail futures contracts sellers will need to get the diamond certified by De Beers-promoted International
Institute of Diamond Grading and Research. The certification will allow getting credit in electronic form
equivalent to the carat deposited.
Indian Commodity Exchange (ICEX)
ICEX is deemed recognized Stock exchange providing a nation-wide on-line trading platform in commodity
derivative. It is a public-private partnership with Reliance Capital, MMTC Ltd, Indiabulls Housing Finance
Ltd, Indian Potash Ltd, KRIBHCO and IDFC Bank as major shareholders. It was established under the
SCRA, 1956 in terms of Section 131(B) of Finance Act, 2015 pursuant to central Government notification
issued in August 2015.
August 29: International Day Against Nuclear Tests
The International Day Against Nuclear Tests was observed across the world on 29 August with an aim to
raise awareness about the effects of nuclear weapon test explosions or any other nuclear explosions.
Observance of the day seeks to promote peace and security world-wide and calls for urgent need to prevent
nuclear catastrophes to avert devastating effects on humankind, environment and the planet. It also
highlights urgent need for cessation of nuclear weapons as one of the means of achieving the goal of a
nuclear-weapon-free world.
To mark this day, various events were organised across the world such as symposia, conferences, exhibits,
competitions, publications, instruction in academic institutions.
International Day Against Nuclear Tests
The International Day against Nuclear Tests was instituted by the United Nations General Assembly
(UNGA) by unanimously adopting resolution 64/35 in December 2009. The resolution was initiated by
Kazakhstan with support of large number of sponsors and cosponsors with a view to commemorate closure
of Semipalatinsk Nuclear Test site on 29 August 1991. The Day was first observed in 2010 and since then
observed annually to galvanize the necessity of banning nuclear weapon tests.
Semipalatinsk Nuclear Test site
The Semipalatinsk Test Site (also known as The Polygon) was the primary testing venue for the Soviet
Unions nuclear weapons. It is located on the steppe in northeast Kazakhstan (then the Kazakh SSR part of
USSR), south of the valley of the Irtysh River. The Soviet Union had conducted 456 nuclear tests on this site
from 1949 until 1989 including 340 underground and 116 atmospheric explosions (roughly the equivalent of
2500 Hiroshima atomic bombs) with little regard for their effect on the local people or environment.
August 29: National Sports Day
The National Sports Day is observed every year on 29th august every year to birth anniversary of the
legendary hockey player Dhyan Chand who was born on 29 August 1905.
2017 GKToday | All Rights Reserved | www.gktoday.in

145
Current Aairs: August, 2017 [Date-wise Compendium]

To celebrate the day, Government had conducted various programs throughout the country. On this
occasion, President Ram Nath Kovind presented National sports Awards to 29 players. The President
conferred Rajiv Gandhi Khel Ratna Award, Arjuna Award and Dronacharya Award.
About Dhyan Chand
The legendary hockey player was born on 29 August 1905 in Allahabad, Uttar Pradesh. He is widely
considered as the greatest field hockey player of all time. He had started playing hockey only after joining the
Army at the age of 16.
He was famously known as The Wizard for his superb ball control and had scored more than 1000 goals
during his international career. He is mostly remembered for his extraordinary goal-scoring feats, in
addition to earning 3 Olympic gold medals in field hockey in 1928 (Amsterdam), 1932 (Los Angeles) and
1936 (Berlin).
Government launches National Sports Talent Search Portal
The Union Ministry of Youth Affairs and Sports has launched National Sports Talent Search Portal to
unearth sporting talent from every nook and corner of the country. The portal will be also available as
smartphone application.
Key Facts
The portal will enable any person from across country to upload their information through a simple three
step process for registration, profile creation and also upload their achievements. Using this portal, a child or
his parents, coaches or teachers can upload their bio data or video on the portal.
The Sports Ministry will select talented players based on received information and impart training to them
at Sports Authority India (SAI) centres. Its purpose is to tap sports talent and train selected children. It will
also provide them with platform to showcase their sporting abilities irrespective of their social or economic
status.
The portal will ensure that the grassroot population in India also gets an opportunity to contribute their
share in sports. It will be available in English as well as in other regional languages, in order to ensure
language doesnt serve as a barrier to highlight their achievements.
India, Germany ink agreement to improve grid integration of renewable energy
India and Germany have signed an agreement on technical cooperation under the Indo-German Energy
Programme Green Energy Corridors (IGEN-GEC).
The agreement was signed between Indias Ministry of New and Renewable Energy (MNRE) and Deutsche
Gesellschaft fr Internationale Zusammenarbeit (GIZ) GmbH India on behalf of Germany.
Indo-German Energy ProgrammeGreen Energy Corridors (IGEN-GEC)
The IGEN-GEC programme was launched in 2013 by the Germanys Federal Ministry for Economic
Cooperation and Development (BMZ) and jointly implemented by Indias MNRE and GIZ. The main
objective of this programme is to improve the sector framework and conditions for grid integration of
renewable energies with conventional power grids.

2017 GKToday | All Rights Reserved | www.gktoday.in

146
Current Aairs: August, 2017 [Date-wise Compendium]

This programme aims to supports the implementation of Indias Renewable Energy Management Centre
(REMCs), Green Energy Corridors scheme which are prerequisite for large scale grid integration of
renewable energy to achieve the 175 GW target for renewable energy generation capacity by 2022.
Under this programme, Germany had agreed to provide concessional loans of up to 1 billion euros through
German Development Bank (KfW) and up to 10 million euros under technical assistance.
Green Energy Corridor Project
The Green Energy Corridor Project aims at synchronising electricity produced from renewable sources, such
as solar and wind, with conventional power stations in the grid. Its objective is to synchronise electricity
produced from renewable sources, such as solar and wind, with conventional power stations in the grid.
Need for synchronisation
The is Voltage Fluctuations is the main problem in integrating electricity generated from renewable energy
to conventional power grids. The conventional grids face difficulty in absorbing renewable electricity
because of its varying voltage and supply. The planned transmission system under Green Energy Corridor
Project is made dynamic to handle the voltage variations leading to an integrated grid across the nation.
Government to launch Deep Ocean Mission
The Union Ministry of Earth Sciences (MoES) is going to launch Deep Ocean Mission by January 2018 to
spur research activities in ocean science and develop technology to harness ocean resources.
The proposed inter-ministerial and interdisciplinary project aims to explore and harness mineral resources
beneath the ocean floor.
Need
India has 7,500 km of coastline and 2.4 million square km of exclusive economic zone (EEZ). The oceans
along Indian peninsula contain enormous opportunities in energy, food, medicine and a host of other natural
resources. China has been working on a deep sea mission for mining mineral deposits in the Indian and
Pacific oceans.
Deep Ocean Mission
The mission is intended to harness ocean resources in responsible way and play important transformative
role in prosperity and security of the nation. It will also seek to create jobs and business opportunities in
ocean science. It will not only involve the MoES, but also departments of other ministries like science and
technology and the biotechnology etc.
West Bengals Gobindobhog rice gets geographical indication status
The Geographical Indications Registry (GIR) has granted geographical indication (GI) status to
Gobindobhog rice, a speciality from Burdwan district of West Bengal. The state government had applied for
GI status for Gobindobhog rice in August 2015.
Gobindobhog rice
Gobindobhog variety of rice is a short grain, white, aromatic, sticky rice having a sweet buttery flavor. It
derives its name from principal ingredient usage in preparation of offerings to Govindajiu, the family deity
of Setts of Kolkata.
2017 GKToday | All Rights Reserved | www.gktoday.in

147
Current Aairs: August, 2017 [Date-wise Compendium]

It is primarily cultivated in east Burdwan district in southern basin of Damodar River in Raina 1, Raina 2
and Khandaghosh blocks. Burdwan region (now divided into districts of east and west Burdwan) is known as
the rice bowl of Bengal. The south Damodar belt is traditional area of Gobindobhog rice cultivation.
The rice variety has several advantages. It is cultivated late and therefore not much affected by rains. Its
productivity per area is high and so farmers get better prices for Gobindobhog rice. It is less prone to pests as
well.
Significance
As a result of getting the GI tag, rice produced from other regions or rice of other varieties cannot be
branded as Gobindobhog. It will also strengthen marketability of this rice in the local, national and
international markets.
About Geographical Indication (GI)
GI tag is an insignia on products having a unique geographical origin and evolution over centuries with
regards to its special quality or reputation attributes. The status to the products marks its authenticity and
ensures that registered authorised users are allowed to use the popular product name.
It is covered as an element of intellectual property rights (IPRs) under Paris Convention for the Protection of
Industrial Property. At international level, GI is governed by WTOs Agreement on Trade-Related Aspects
of Intellectual Property Rights (TRIPS).
In India, GI registration is governed by the Geographical Indications of goods (Registration and Protection)
Act, 1999 which came into force in September 2003. It is granted by Chennai based GIR. Darjeeling tea was
the first product to accorded with GI tag in India.
Benefits of GI Status: (i) Legal protection to the products (ii) Prevents unauthorised use of a GI tag
products by others (iii) Helps consumers to get quality products of desired traits (iv) Promotes economic
prosperity of producers of GI tag goods by enhancing their demand in national and international markets.
IAEA opens worlds first low Enriched Uranium bank in Kazakhstan
The UN global nuclear watchdog, International Atomic Energy Agency (IAEA) has opened worlds first low
Enriched Uranium (LEU) Bank in Oskemen in Kazakhstan.
The bank is owned and managed by IAEA. It will be the first of its kind LEU bank not to be under
control of any individual country. The IAEA also manages 123 tons of uranium in Angarsk, Russia but it
is under the control of Russian government.
Key Facts
The LEU Bank has reserve capacity to store 90 tons of LEU, the essential ingredient needed to make the fuel
for light-water nuclear reactors, which generate electricity. This reserve is enough to fully load a light-water
reactor capable of supplying electricity to a large city for three years.
The IAEA has established a series of strict criteria for member state purchasing uranium from the bank. The
project was funded by donors, including the United States, European Union, Kuwait, United Arab Emirates,
Norway and Nuclear Threat Initiative.

2017 GKToday | All Rights Reserved | www.gktoday.in

148
Current Aairs: August, 2017 [Date-wise Compendium]

Purpose of Bank
Last resort Supply: It will serve as last resort source of LEU after IAEA member countries are unable to
either produce fuel or if it becomes unavailable on the international market for whatever reason. It will
ensure that in the event of an international crisis or similar circumstances, countries dependent on nuclear
power would still have access to uranium.
Non-proliferation efforts: It will discourage countries from developing their own uranium enrichment
capacities as seen supposedly that peaceful use of enriched uranium can be converted into weapons-grade
level.
No disruption to nuclear fuel trade: It will not disrupt usual trade of nuclear fuel purchased in the open
market or by bilateral agreement between countries.
Peaceful use of atomic energy: It will ensure peaceful use of atomic energy without the need to develop a
costly enrichment program. It will assure that suppliers in the international markets do not manipulate
prices or when LEU ceases to be supplied for political reasons.
International Atomic Energy Agency (IAEA)
IAEA is an international organization that seeks to promote the peaceful use of nuclear energy, and to
inhibit its use for any military purpose, including nuclear weapons. It was established as an autonomous
organization in 1957 through its own international treaty, the IAEA Statute. It is independent of the United
Nations but reports to both the United Nations General Assembly (UNGA) and Security Council
(UNSC).The IAEA has its headquarters in Vienna, Austria.
August 30, 2017
Government clears 100% strategic sale of CEL
The Union Government (Department of Investment and Public Asset Management) has approved a 100%
disinvestment through strategic sale along with transfer of management control of Central Electronics Ltd.
(CEL).
CEL was incorporated as CPSE in 1974 under the administrative control of Ministry of Science and
Technology. It is wholly owned by the government and has a net worth of Rs. 50.34 crore as on March 2017.
The government is planning to engage an advisor from a consulting firm, investment banker or financial
institution or merchant banker for providing advisory services and managing the disinvestment process.
Alternative mechanism
The strategic disinvestment of CEL will take place through new alternative mechanism which was approved
by the cabinet committee on economic affairs (CCEA) in August 2017. The alternative mechanism was
approved for pursuing the governments strategic sale disinvestment programme. This mechanism comprise
of finance minister, minister for road transport & highways and the minister for the concerned
administrative ministry.
Strategic disinvestment
In Strategic disinvestment, significant proportion of a Public Sector Units (PSU) share and the management

2017 GKToday | All Rights Reserved | www.gktoday.in

149
Current Aairs: August, 2017 [Date-wise Compendium]

control goes to a private sector which is as strategic partner. It is different from the ordinary disinvestment
in which management of PSU is retained with Government.
Delhi Police launches YUVA a skill development programme
The Delhi Police has launched YUVA a skill development programme under Pradhan Mantri Kaushal
Vikas Yojana (PMKVY). It was inaugurated by Union Home Minister Rajnath Singh.
The YUVA initiative aims to connect with youth by upgrading their skill as per their competencies. It will
help youths to get gainful employment under PMKVY. It will also play important role in building up
confidence and faith of youths in police organization.
Key Facts
For implementing YUVA initiative, Delhi Police has tied up with National Skill Development Corporation
(NSDC) and Confederation of Indian Industry (CII) to provide mass job linked skill training for selected
youth. NSDC role will of providing skill training to youth under PMKVY and CII will provide job linked
training through its Sector Skill Councils which are connected to industry to provide job guarantee.
The youth in age group of 17-25 years were selected by Delhi Police for this training based on detailed
exercise conducted in all 13 districts under its jurisdiction. These youths belong to categories such as school
dropouts, victims of crimes, Juvenile offenders and families in dire state mostly from the underprivileged
colonies. So far, 2,269 candidates were selected to be trained by 36 training partners in 45 skills. 32 police
station buildings of Delhi Police have been identified where Skill Development Centres would be opened at
the earliest.
Pradhan Mantri Kaushal Vikas Yojana (PMKVY)
PMKVY is flagship Skill Certification Scheme of Ministry of Skill Development & Entrepreneurship
(MSDE) launched in July 2015. Its objective is to enable a large number of Indian youth to take up industry-
relevant skill training that will help them in securing a better livelihood.
Under this Scheme, individuals with prior learning experience or skills are also assessed and certified under
Recognition of Prior Learning (RPL). Government will pay complete training and assessment fees. The
training includes soft skills, personal grooming, behavioural change.
It is implemented through the national skill development corporation (NSDC). Skill training is based on the
national skill qualification framework (NSQF) and industry led standards.
Government to release new industrial policy
The Department of Industrial Policy and Promotion (DIPP) under the Commerce and Industry Ministry is
going to release a new industrial policy by October 2017. It is expected to replace National Manufacturing
Policy (NMP) released in 2011
The new policy aims at making India a manufacturing hub by promoting Make in India with focus on
encouraging Indian branded products with higher value addition. It will also review the existing foreign
direct investment (FDI) policy regime to facilitate greater technology transfer.
Features of New policy
The proposed policy will aim to set clear vision for role of industry and industrial growth in growth and
2017 GKToday | All Rights Reserved | www.gktoday.in

150
Current Aairs: August, 2017 [Date-wise Compendium]

development of economy. It will be comprehensive, actionable, outcome-oriented policy that will enable
industry to deliver a larger role in economy and fulfil its role as engine of growth and add more value and
jobs.
DIPP has adopted consultative approach for formulating industrial policy by setting up six thematic focus
groups. These six thematic areas are Manufacturing and MSME, Technology and Innovation, Infrastructure,
Investment, Trade and Fiscal Policy, Ease of Doing Business and Skills and Employability for Future.
The policy will aim to attract $100 billion of FDI in a year, up from $60 billion in 2016-17. It will also aim at
retaining investments and accessing technology. It will also ensure that it facilitates greater technology
transfer, leverages strategic linkages and innovation.
It will incorporate measures to facilitate use of smart technologies such as the internet of things (IoT),
artificial intelligence (AI) and robotics for advanced manufacturing. A task force constituted on artificial
intelligence under the chairmanship of V. Kamakoti will also provide inputs for the policy
The policy will aim to harness existing strengths in sectors like automobiles and auto-components,
electronics, new and renewable energy, banking, software and tourism, and create globally scaled-up and
commercially viable sectors such as waste management, medical devices, renewable energy, green
technologies, financial services to achieve competitiveness.
The policy will also push for reforms to enhance labour market flexibility with an aim for higher job creation
in the formal sector and performance linked tax incentives. It will also take into consideration changing
economic and business cycles of world and Indian economy, geopolitical trends and broad policy directions
in country.
NITI Aayog proposes Judicial Performance Index for lower judiciary
The premier government think tank National Institution for Transforming India (NITI) Aayog has
suggested establishment of a judicial performance index (JPI) to check delay in finalisation of cases in lower
judiciary.
It was suggested as part of Three Year Action Agenda (2017-20) recommendations for far-reaching reforms
to expedite justice delivery system, particularly lower courts where nearly 3 crore cases have been pending
for years.
Judicial performance index (JPI)
Establishing of JPI will help the high courts and its chief justices to keep track ofperformance and process
improvement at district courts and subordinate levels for reducing delay. It can also include certain progress
on process steps that have already been approved by high courts, like removing burden of day-to-day activity
of judges and giving it to administrative officials.
The process of calculating index will require fixing non-mandatory time frames for different types of cases.
Using existing infrastructure and data, indexs indicators can be created to check duration of pendency of
case along with percentage of cases that have been delayed and how many cases were disposed in previous
year compared to the year before.

2017 GKToday | All Rights Reserved | www.gktoday.in

151
Current Aairs: August, 2017 [Date-wise Compendium]

Other recommendations
Separate administrative cadre: Establishing separate administrative cadre in judicial system to reduce
workload on judges. This cadre should report to Chief Justice in each high court to maintain judicial
independence.
Use of automation and ICT: High priority should be given to automation process in courts and use of
information and communication technology for e-court and case management, including e-management of
court schedules and migration of all courts to unified national court application software.
Online real time judicial statistics: Steps should be taken for ensuring availability of online real time
judicial statistics for determining the adequacy of judicial manpower and infrastructure to deal with work
load of cases. It will enable priority appointment of judges at lower judiciary levels keeping in mind a
scientific approach to assessing number of judges needed to tackle pendency problem.
Internationally developed measures: Government must look into and adopt internationally developed
measures such as global measures of court performance, created jointly by Australasian Institute of Judicial
Administration, Federal Judicial Center (US), National Center for State Courts (US) and Subordinate Courts
of Singapore.
Cabinet approves MoU on India-Israel Industrial R&D and Technological
Innovation Fund
The Union Cabinet approved Memorandum of Understanding (MoU) to set up US $40 million India-Israel
Industrial R&D and Technological Innovation Fund (I4F). The MoU was signed in July, 2017 during
Prime Minister Narendra Modis visit to Israel.
India and Israel will make contribution of US $4 million (Rs.25 crore) each for Fund, both equivalent
amount, annually for 5 years. The fund will be governed by joint Board which will consist of 4 members
from each country.
Key Facts
The MoU envisages promotion of bilateral Industrial R&D and Innovation cooperation between both
countries in fields of science and technology by extending support to joint projects for innovative or
technology-driven new or improved products, processes or services.
Such collaborative projects will lead to affordable technological innovations in focus areas of mutual interest
such as agriculture, water, energy and digital technologies. These projects will also provide institutional
support in building up consortia including private industry, enterprises and R&D institutions from India and
Israel.
The activities supported by the Joint I4F Fund will increase the techno-economic collaboration between
both countries by investing in jointly developed technology projects and collaborations based on
technological innovation.
It will encourage Israel-Indian joint projects that capitalize on both the national and global marketplace. It
will provide a comprehensive set of support tools to encourage joint projects. It will foster and strengthen

2017 GKToday | All Rights Reserved | www.gktoday.in

152
Current Aairs: August, 2017 [Date-wise Compendium]

eco-system of innovation and techno-entrepreneurship in India and will contribute directly to the Start-up
India programme.
Cabinet approves promulgation of GST (Compensation to States) Ordinance,
2017
The Union Cabinet has given its approval to the proposal of Union Finance Ministry to promulgate an
ordinance to suitably amend the Goods and Services Tax (Compensation to States) Act, 2017.
The approval will allow GST Council to hike the maximum rate of compensation cess levied from the
current cap of 15% to 20%. The ordinance will amend Schedule of Section 8 of the GST (Compensation to a
State) Act, 2017.
Key Facts
It will be only an enabling Ordinance and the decision to increase the compensation cess will be taken by
the GST Council, the apex tax rate setting body under the GST regime. The hike in rate of compensation
cess will be levied on SUVs, mid-sized, large and luxury cars. It should be noted that compensation cess is
applicable not just to cars but also tobacco and coal. It is used to form corpus for compensating the states
which experience tax revenue loss post-GST.
Background
In the new GST regime, cars attract the top tax rate of 28%. On top of this, compensation cess of 1-15% is
levied for the creation of the state compensation corpus. The post introduction of GST, the total incidence
tax on motor vehicles [GST+ Compensation Cess] has come down vis-a-vis pre-GST total tax incidence,
making these SUVs, mid-sized, large and luxury cars cheaper post GST rollout. To rectify the anomaly, the
GST Council, comprising of representatives of all states, had recommended that the Central government
move legislative amendments required for increasing the cess.
Cabinet approves MoU between India-Brazil for cooperation in Zebu Cattle
Genomics and ARTs
The Union Cabinet has apprised Memorandum of Understanding (MoU) signed between India and Brazil in
October 2016 for cooperation in fields of Zebu Cattle Genomics and Assisted Reproductive Technologies
(ARTs).
The MoU aims to strengthen existing friendly relations between both countries and promote development
of Genomics and ARTs in Cattle through joint activities to be implemented through mutually agreed
procedures.
Key Facts
The MoU will promote and facilitate scientific cooperation and setting up of genomic selection programme
in Zebu Cattle through (i) application of genomic in Zebu Cattle and their crosses and buffaloes (ii)
application of ARTs in cattle and buffaloes (iii) capacity building in genomic and ART (iv) Related R&D in
Genomics and ART in accordance with respective laws and regulations of two countries. It is covered under
Rule 7 (d) (i) of 2nd Schedule of Government of India (Transaction of Business) Rules, 1961.

2017 GKToday | All Rights Reserved | www.gktoday.in

153
Current Aairs: August, 2017 [Date-wise Compendium]

An implementation committee will be constituted with an equal number of representatives from both
countries for purpose of regularly determining activities and developing work plans and subsequently their
evaluation. It will be done through joint projects in fields of Productivity Improvement of buffaloes and
cattle, for purpose of broadening existing knowledge base on sustainable dairy development and institutional
strengthening.
Zebu Cattle
Zebu Cattle (also known as indicine cattle or humped cattle), is a species or subspecies of domestic cattle
originating in Indian subcontinent. They are characterised by fatty hump on their shoulders, a large dewlap,
and sometimes drooping ears. They are well adapted to withstanding high temperatures, and are farmed
throughout tropical countries. They are used as dairy cattle, draught oxen and beef cattle.
IIT Researchers develops bioartificial implantable pancreas
Researchers from Indian Institute of Technology (IIT) Guwahati have successfully created an implantable
bioartificial pancreas model grown within 3D silk scaffold.
The bioartificial pancreas encapsulates insulin-producing beta cells, capable of naturally producing insulin in
sustained manner. If successful in animal and human trials, it can be used for treating people with Type 1
diabetes. Type 1 diabetes arises when the bodys immune system kills the insulin-producing beta cells.
Key Facts
The 3D silk scaffold was found to be biocompatible (not toxic to living tissue) as it did not trigger any
immune reaction or cause any adverse reaction after implanted. It was made porous by using salt grains of
specific size to dissolve the silk proteins. These pores were 400-500 micrometre in size which allowed
glucose and oxygen to enter scaffold and insulin released by beta cells with greater survival rate to enter
bloodstream.
The scaffold containing beta cells was coated with a semi-permeable membrane barrier. The membrane
allowed insulin produced to be released into blood stream and does not allow immune cells to cross
membrane and kill the islet cells.
To ensure that the implant is not rejected by the bodys immune system, drugs that suppress the immune
system were embedded in the scaffold. Studies carried in lab showed that beta cells in scaffold were able to
produce adequate amount of insulin in response to different glucose levels within a few seconds.
August 31, 2017
India-Myanmar to sign MoU for conservation of quake-damaged Pagodas at
Bagan
The Union Cabinet has approved the Memorandum of Understanding (MoU) between India and Myanmar
for conservation of earthquake-damaged pagodas at Bagan, Myanmar.
The MoU will be signed during official state visit of Prime Minister Narendra Modi to Myanmar during 6-7
September 2017.
Pagodas are Buddhist temple, typically in the form of a many-tiered tower, built in traditions originating as
2017 GKToday | All Rights Reserved | www.gktoday.in

154
Current Aairs: August, 2017 [Date-wise Compendium]

stupa in historic South Asia. Pagodas at Bagan are one of the premier tourist attractions of Myanmar.
Significance of Project
Implementation of conservation project will strengthen Indias enduring ties in cultural and religious sphere
with Myanmar. It will also generate immense goodwill for India among people of Myanmar due to religious
significance as well as touristic significance of project. It will also contribute to Indias development
partnership with Myanmar. It will provided opportunity to showcase Indias expertise in conservation and
restoration of monuments.
Konkani writer Mahabaleshwar Sail receives 2016 Saraswati Samman
Eminent Konkani writer Mahabaleshwar Sail (74) was bestowed with prestigious Saraswati Samman 2016.
He was awarded for his novel Hawthan () that captures cultural lives of traditional potter community
in Goa.
He was presented the award, comprising plaque and a cash purse of Rs.15 lakh by Union minister for
commerce and industry Nirmala Sitharaman at event held in New Delhi.
Mahabaleshwar Sail
Mahabaleshwar Sail is bilingual writer based in Goa. He has written four Marathi dramas and seven Konkani
novels. He has also written five short stories and novel in Marathi. He is best known for his work Paltadcho
Munis, which had inspired film by same name. Most of his short stories and novels are translated into
English and Hindi. His work Hawtham was published in 2009. He has received Sahitya Akadami Award in
Konkani language for his short story collection Taranga in 1993.
Mahabaleshwar Sail was born on 4 August 1943 in Shejebag in Uttara Kannada district of Karnataka. He had
served in Indian Army and had participated in 1965 Indo-Pakistani War. He also had served as UN
peacekeeper in 1964-1965 at Israel and Egypt border. He also had worked as supervisor in Forest
Department, Goa, Daman and Diu Police and India Post till his retirement.
Saraswati Samman
Saraswati Samman is an annual award bestowed upon Indian citizen for his outstanding literary work
written in any 22 Indian language mentioned in Schedule VIII of Constitution and published during last 10
years. The award has been named after Indian goddess of learning Saraswati.
It is considered one of the highest literary awards in India. It was established in 1991 by the KK Birla
Foundation. It carries cash prize of 15 lakh rupees, a citation and a plaque of goddess Saraswati. Harivansh
Rai Bachchan was first recipient of this award for his autobiography published in four volumes.
CCS approves first phase of Army reforms
The Union Cabinet Committee on Security (CCS) chaired by Prime Minister Narendra Modi has approved
implementation of first phase of reforms pertaining to the Indian Army based on recommendations of Lt
Gen Shekatkar Committee.
The purpose of reforms is to enhance combat capability and also optimize and rebalance defence expenditure
of Indian Army in phased manner by December 2019. It is first ever reform exercise to be undertaken in
Indian Army since independence.
2017 GKToday | All Rights Reserved | www.gktoday.in

155
Current Aairs: August, 2017 [Date-wise Compendium]

The first phase of reforms involves


Redeployment and restructuring of approximately 57,000 posts of officers/Junior
Commissioned Officers (JCOs)/ Other Ranks (ORs) and civilians.
Optimisation of signals establishments: Its purpose is to include Radio Monitoring
Companies, Air Formation Signal Regiments, Corps Air Support Signal Regiments,
Composite Signal Regiments and merger of Corps Operating and Engineering Signal
Regiments.
Restructuring of repair echelons in Army: Its purpose is to include Base Workshops,
Advance Base Workshops and Station Workshops in the field Army.
Redeployment of Ordnance echelons: Its purpose is to include Vehicle Depots, Central
Ordnance Depots and Ordnance Depots apart from streamlining inventory control
mechanisms.
Other Reforms: Better utilization of Supply and Transport echelons and Animal Transport
units. Closure of army postal establishments and military farms in peace locations.
Enhancement of standards for recruitment of clerical staff and drivers in Army. Improving
efficiency of National Cadet Corps (NCC).
Background
The Union Ministry of Defence had constituted a Committee of Experts under Chairmanship of Lt Gen
(Retd) DB Shekatkar for recommending measures to enhance combat capability & rebalancing defence
expenditure of Armed Forces with an aim to increase teeth to tail ratio.
The committee had submitted around 99 recommendations. So far, Defence Ministry has approved 65 of
these recommendations pertaining to Indian Army for implementation. Remaining 34 recommendations
pertaining to Indian Navy, Air Force and Integrated Defence Staff (IDS) are likely to be taken up soon.
These reforms will be completed in all respects by December 2019. The committee had suggested that, if
these recommendations are implemented over the next five years, it can result in savings of up to Rs. 25,000
crore in defence expenditure.
US approves first gene therapy for cancer
The US Food and Drug Administration (FDA) approved first gene therapy, a treatment that uses patients
own immune cells to fight acute lymphoblastic leukaemia.
The approval will usher new approach to treatment of cancer and other serious and life-threatening diseases.
Studies have shown that 83% of patients responded to this treatment, achieving remission within three
months.
Key Facts
The gene therapy treatment was originally developed by researchers at University of Pennsylvania and
licensed to pharma giant Novartis. It was previously identified as CAR-T cell therapy (CTL019) or

2017 GKToday | All Rights Reserved | www.gktoday.in

156
Current Aairs: August, 2017 [Date-wise Compendium]

tisagenlecleucel and now is called Kymriah.


The treatment is completely different compared to present popular immunotherapy drugs called checkpoint
inhibitors used to harness immune system that treat variety of cancers by helping the bodys natural T cells
better spot tumors.
CAR-T treatment
The CAR-T treatment is not a pill or form of chemotherapy. It uses gene therapy techniques not to fix
disease-causing genes (cells) but to turbocharge T cells, immune system cells that cancer can evade. These
cells removed from patients blood along with white blood cells are encoded with viral vector,
reprogrammed in lab. They are reprogrammed to harbor chimeric antigen receptor (CAR) that specifically
targets cancer cells. The revived and reprogrammed cells after returned to the patient continue multiplying
to fight disease for months or years.
EU, India agrees to enhance counter-terrorism cooperation
India and European Union (EU) have agreed to strengthen cooperation between their security and
intelligence agencies to combat threat of terrorism and check terror financing.
Both sides deliberated extensively on combating terror financing and challenges posed by violent extremism
and radicalisation during 10th India-European Union Counter Terrorism Dialogue held in New Delhi.
Key Highlights 10th Dialogue
Both sides strongly condemned terrorism in all its forms and manifestations and underscored need for
deepening cooperation to combat terrorism in long term, sustained and comprehensive manner. They also
exchanged views and assessments on prevailing terrorist threats in their respective regions including state-
sponsored, cross-border terrorism in Afghanistan-Pakistan region.
The also held discussions regarding current challenges posed by various terrorist entities, radicalisation and
violent extremism, foreign terrorist fighters and combating financing of terrorism. They agreed to
strengthen institutional linkages by establishing point of contact between Indian agencies and their EU
counterparts including Europol for closer interaction and cooperation in field of counter-terrorism.
Both sides also agreed to identify relevant training programmes for capacity building of individuals on both
sides working in sphere of countering terrorism and violent extremism. They also emphasised prospects for
deepening counter-terrorism cooperation under the UN and Financial Action Task Force (FATF). The next
India-European Union Counter Terrorism Dialogue will take place in Brussels in 2018.
Financial Action Task Force (FATF)
FATF is an inter-governmental body which monitors the progress of member countries in implementing
necessary measures to check money laundering and terrorist financing.
India, Switzerland ink two MoUs in the field of Railways
India and Switzerland have signed two Memorandum of Understandings (MoUs) for exchanging technical
cooperation in the field of Railways.
They were signed after delegation level talks between Prime Minister Narendra Modi and visiting President
of Swiss Confederation Doris Leuthard in New Delhi. They also issued Joint press statement.
2017 GKToday | All Rights Reserved | www.gktoday.in

157
Current Aairs: August, 2017 [Date-wise Compendium]

Signed Pacts
MoU between Konkan Railway Corporation Limited (KRCL) and Swiss Federal Institute of
Technology, Zurich.
MoU between Ministry of Railways and Federal Department of the Environment, Transport,
Energy and Communications of the Swiss Federation on Technical Cooperation in Rail
Sector.
Key Takeaways of Joint press statement
FDI is a big part of the Indo-Swiss cooperation. Both countries agreed to enhance the business to business
cooperation. Both countries are working on new bilateral investment treaty. India thanked Switzerland for
supporting for its bid for MTCR and Nuclear Suppliers Group membership.
Switzerland agreed that reforms taken by India will serve good base for expanding cooperation in several
areas. Switzerland has accepted automatic exchange of information for transparency in financial transactions
with India to effectively combat black money and tax evasion and its parliament is going to pass legislation
supporting it by year end. India committed to clean energy and green future and termed climate change as a
global challenge.
India-Switzerland
India and Switzerland have long standing relationship and enjoy robust economic ties. Switzerland is being
seventh largest trading partner and eleventh largest foreign investor for India. Both countries share common
views on many major global issues.
Launch of navigation satellite IRNSS-1H unsuccessful: ISRO
The launch of IRSOs eight navigation satellite IRNSS-1H onbaord of PSLV-C39 from Sriharikota, Andhra
Pradesh was unsuccessful. The heat shield did not separate as a result of which satellite did not complete the
4th stage.
The IRNSS-1H was being sent to space to back up and replace the functions of Indias first navigation
satellite IRNSS-1A (launched in July 2013) as its three rubidium atomic clocks on board had stopped
functioning.
The rubidium atomic clocks on board of IRNSS-1A that stopped working were imported from European
aerospace manufacturer Astrium. The atomic clocks are used to find satellites position with an accuracy of
0.5 metre.
IRNSS-1H
IRNSS-1H was Indias first satellite actively built by consortium of private firms led by Bengaluru-based
Alpha Design Technologies. The consortium had completed 25% development work of IRNSS-1H, under
the guidance of ISRO scientists. Prior to development of this satellite, private sectors role was only limited
to supplying components to ISRO.
IRNSS
The Indian Regional Navigation Satellite System (IRNSS) is an independent regional system developed by
India on par with the US-based GPS, Russias Glonass and Galileo developed by Europe. The navigation
2017 GKToday | All Rights Reserved | www.gktoday.in

158
Current Aairs: August, 2017 [Date-wise Compendium]

system offers wide services like terrestrial and marine navigation, disaster management, vehicle tracking and
fleet management visual and voice navigation for drivers. It was named Navic (Navigation with Indian
Constellation) by Prime Minister Narendra Modi.
Compendium of Multiple Choice Questions
Below 260 Questions were published in GKTodays Current Aairs Quiz Section in August,
2017.

1. What is the Indias rank in the 2017 Digital Evolution Index (DEI)?
[A] 53rd
[B] 64th
[C] 29th
[D] 35th
Correct Answer: A [53rd]
Explanation: India has been ranked 53rd among 60 countries in the 2017 Digital
Evolution Index (DEI). The list is topped by Norway, followed by Sweden,
Switzerland, Denmark and Finland. The Fletcher School at Tufts University, in
partnership with Mastercard, presented the Digital Evolution Index (DEI) 2017.
The Index is a comprehensive research that tracks the progress of the digital
economy across 60 countries, taking into account more than 100 dierent
indicators across four key drivers: supply, consumer demand, institutional
environment, and innovation. It reects the state and rate of digital evolution on
the one hand, while also identifying avenues for investment, innovation and policy
priorities.
2. Which state government has launched Apni Gaddi Apna Rojgar scheme for
jobless youth?
[A] Maharashtra
[B] Punjab
[C] Assam
[D] Haryana
Correct Answer: B [Punjab]
Explanation: The Punjab government has recently launched its agship Apni
Gaddi Apna Rozgar scheme for jobless youth by agging o over 100 uberMOTO
bikes. Under the scheme, the state government will provide commercial two
wheelers and four wheelers to unemployed youth at subsidised rates. The scheme
will be a game-changer for the youth of Punjab and will play a catalysing role in
encouraging micro-entrepreneurship opportunities throughout the state. The
target of Uber is to create 10,000 entrepreneurial opportunities in Punjab in one

2017 GKToday | All Rights Reserved | www.gktoday.in

159
Current Aairs: August, 2017 [Date-wise Compendium]

year and a total of 45,000 in next ve years.


3. Who has won the 2017 Formula One Hungarian Grand Prix tournament?
[A] Valtteri Bottas
[B] Lewis Hamilton
[C] Sebastian Vettel
[D] Kimi Raikkonen
Correct Answer: C [Sebastian Vettel]
Explanation: Sebastian Vettel, a German racing driver currently driving in
Formula One for Scuderia Ferrari, has won the 2017 Formula One Hungarian
Grand Prix tournament at the Hungaroring in Mogyorod, Hungary.
4. Which IIT institute has invented a low-cost dirt detector that alerts authorities
when a place needs cleaning?
[A] IIT Bombay
[B] IIT Madras
[C] IIT Indore
[D] IIT Kharagpur
Correct Answer: D [IIT Kharagpur]
Explanation: The researchers at the IIT Kharagpur have developed a low-cost
dirt detector that alerts authorities when a place needs cleaning. The Smart
Hygiene Monitor (SHM) records levels of ammonia, sulphur dioxide, carbon
monoxide, carbon dioxide, volatile organic compounds and particulate matter
through sensors and sends online alerts to stakeholders when a certain threshold
is crossed. It can be installed at any place where cleanliness needs to be
monitored at regular intervals. At present, public facilities such as toilets,
eateries, hospitals, parks, airports and railway stations are generally cleaned at
xed intervals, irrespective of how dirty they are.
5. Sam Shepard, the Pulitzer-winning playwright passed away. He hailed from
which country?
[A] United States
[B] France
[C] Germany
[D] Italy
Correct Answer: A [United States]
Explanation: Sam Shepard (73), the Pulitzer Prize-winning playwright, has
passed away in Kentucky, United States on July 27, 2017. He was Oscar-
nominated actor and celebrated author whose plays chronicled the explosive fault
2017 GKToday | All Rights Reserved | www.gktoday.in

160
Current Aairs: August, 2017 [Date-wise Compendium]

lines of family and masculinity in the American West. He wrote nearly 50 plays
and won the Pulitzer for drama in 1979 for his play Buried Child. His plays are
chiey known for their bleak, poetic, often surrealist elements, black humor, and
rootless characters living on the outskirts of American society.
6. Hussain Sayeeduddin Dagar, the noted Dhrupad Maestro passed away. He
hailed from which state?
[A] Assam
[B] Rajasthan
[C] Madhya Pradesh
[D] Uttar Pradesh
Correct Answer: B [Rajasthan]
Explanation: Ustad Hussain Sayeeduddin Dagar (78), maestro and custodian of
the venerable Dhrupad tradition of Hindustani classical music, has passed away in
Pune, Maharashtra on July 30, 2017. He was known aectionately to his legions of
disciples and admirers as Saeed Bhai. He was born in Alwar in Rajasthan in 1939
and was a part of the Dagar family of musicians. He represented the 19th
generation of Dagar Tradition. Dhrupad a Sanskrit portmanteau of Dhruva
(immovable) and Pad (verse) has its roots since ancient times, mentioned as early
as the 3rd Century B.C. in the Natyashastra.
7. Which city to host the 2024 Summer Olympics?
[A] Los Angeles
[B] London
[C] Paris
[D] New York
Correct Answer: C [Paris]
Explanation: The International Olympic Committee (IOC) has recently announced
that the 2024 Summer Olympics will be hosted by Paris in France, which is
scheduled to be held from August 8-18. Beside this, the IOC has also announced
that the 2028 Olympic Games and Paralympic Games to be hosted by Los Angeles
(United States). Both the decisions will be ratied at the 131st IOC Conference in
Lima, Peru on September 13, 2017. The IOC decision means that Los Angeles will
become a three-time Olympic city, after hosting the 1932 and 1984 Games and
Paris, which held the Games in 1900 and 1924, will host a centenary celebration
in 2024.
8. Who is the newly elected Prime Minister of Pakistan?
[A] Shahbaz Sharif
[B] Naveed Qamar
2017 GKToday | All Rights Reserved | www.gktoday.in

161
Current Aairs: August, 2017 [Date-wise Compendium]

[C] Khaqan Abbasi


[D] Shahid Khaqan Abbasi
Correct Answer: D [Shahid Khaqan Abbasi]
Explanation: Shahid Khaqan Abbasi, the member of the Pakistan Muslim League
(N)(PML-N), is the newly elected interim Prime Minister of Pakistan after defeating
his rival Naveed Qamar of PPP by 221 votes on August 1, 2017. Abbasi was
chosen by Nawaz Sharif as an interim Prime Minister for 45 days which would
allow signicant time for Shehbaz Sharif to contest elections from Nawazs
vacated constituency, be elected to the National Assembly and become eligible to
hold the oce of Prime Ministership. Previously, Abbasi served as the Minister of
Petroleum and Natural Resources in the third Sharif ministry from 2013 to 2017.
Nawaz Sharif was disqualied as the Prime Minister by the Supreme Court in July
2017 in connection with Panama Papers scam.
9. Jeanne Moreau, the legendary film actress passed away. She hailed from which
country?
[A] Italy
[B] Germany
[C] United States
[D] France
Correct Answer: D [France]
Explanation: Jeanne Moreau (89), the legendary French lm actress, has passed
away at Paris in France on July 31, 2017. She was best known for her role in
Francois Truauts New Wave classic Jules et Jim.
10. The Gangotri National Park (GNP) is located in which state?
[A] Chhattisgarh
[B] Bihar
[C] Uttrakhand
[D] West Bengal
Correct Answer: C [Uttrakhand]
Explanation: The Gangotri National Park (GNP) is located in the upper catchment
of Bhagirathi river in the Uttarkashi District of Uttrakhand and covers an area of
about 2,390 square km. The park is home to the snow leopard, ibex, tahr,
himalayan barbet, serow, pheasants, partridges, doves, pigeons, etc. It provides
majestic beauty of coniferous forests and grandeur of glacial world combined with
lush green meadows.
11. Which committee has been constituted by Union Government to identify key
2017 GKToday | All Rights Reserved | www.gktoday.in

162
Current Aairs: August, 2017 [Date-wise Compendium]

data protection issues?


[A] B N Srikrishna committee
[B] A K Mathur committee
[C] H S Khan committee
[D] KK Lahoti committee
Correct Answer: A [B N Srikrishna committee]
Explanation: The Union Ministry of Electronics and Information Technology
(MeitY) has recently constituted a committee of experts under the chairmanship
of a former Judge of Supreme Court Justice B N Srikrishna to study and identify
key data protection issues and recommend methods for addressing them. The
purpose to form the committee is to recognize the importance of data protection
and keeping personal data of citizens secure and protected. The committee will
also suggest a draft Data Protection Bill.
12. The Union government has launched e-RaKAM portal to provide a platform
to sell agricultural produce. What does RaKAM stands for?
[A] Rashtriya Kisan Agri Mandi
[B] Rashtriya Kisan Agriculture Mandi
[C] Rashtriya Kisan Agricultural Mandi
[D] Rashtriya Kisan Agri-farming Mandi
Correct Answer: A [Rashtriya Kisan Agri Mandi]
Explanation: The Union government has recently launched a portal e-Rashtriya
Kisan Agri Mandi (e-RaKAM) which will provide a platform to sell agricultural
produce. The e-RaKAM is a rst of its kind initiative that leverages technology to
connect farmers of the smallest villages to the biggest markets of the world
through internet. Food and Consumer Aairs Minister Ram Vilas Paswan and
Union Minister of Steel Chaudhary Birender Singh jointly launched the portal in
New Delhi on August 1, 2017. The portal is a digital initiative bringing together
the farmers, PSUs, civil supplies and buyers on a single platform to ease the
selling and buying process of agricultural products. Under this initiative, e-RaKAM
centres are being developed in a phased manner throughout the country to
facilitate farmers for online sale of their produce. The farmers will be paid through
e-Payment directly into their bank accounts.
13. Santosh Mohan Dev, the veteran political leader passed away. He hailed from
which state?
[A] Odisha
[B] Assam
[C] Uttar Pradesh
2017 GKToday | All Rights Reserved | www.gktoday.in

163
Current Aairs: August, 2017 [Date-wise Compendium]

[D] Rajasthan
Correct Answer: B [Assam]
Explanation: Santosh Mohan Dev (83), the veteran congress leader and former
union minister, has passed away today in Silchar, Assam. He was rst elected to
the Parliament in 1980, the rst of his seven terms in oce as the Member of
Parliament. Out of these seven, he has represented Silchar, Assam ve times and
he was elected from Tripura twice. He has the rare distinction of being elected
from two dierent states. Dev was the eldest son of the renowned Bengali North
Eastern Freedom Fighter Satindra Mohan Dev.
14. What is the current repo rate, as per recently released RBIs 3rd bi-monthly
monetary policy statement for FY 2017-18?
[A] 6.25%
[B] 6.0%
[C] 5.75%
[D] 6.50%
Correct Answer: B [6.0%]
Explanation: According to the Reserve Bank of India (RBI)s 3rd bi-monthly
monetary policy statement for year 2016-17, the policy repo rate under the
Liquidity Adjustment Facility (LAF) has reduced by 25 basis points from 6.25% to
6.0% with immediate eect. This move is expected to lower EMIs for home, auto
and personal loans. This is the rst rate cut since October 2016 and the interest
rate is now at 6-year low. In line with record low retail ination, the reverse repo
rate under the LAF stands adjusted to 5.75%, and the Marginal Standing Facility
(MSF) rate and the Bank Rate to 6.25%. The RBI Governor headed, six member
Monetary Policy Committee also decided to keep the policy stance neutral in
consonance with the objective of achieving the medium-term target for consumer
price index ination of 4%.
15. Which country hosted the 19th RCEP Trade Negotiating Committee (TNC)
meeting?
[A] Philippines
[B] Singapore
[C] India
[D] South Korea
Correct Answer: C [India]
Explanation: The 19th round of the Regional Comprehensive Economic
Partnership (RCEP) Trade Negotiating Committee (TNC) meetings and other
related meetings were held from 17 to 28 July 2017 in Hyderabad, India. RCEP is a
2017 GKToday | All Rights Reserved | www.gktoday.in

164
Current Aairs: August, 2017 [Date-wise Compendium]

proposed comprehensive regional economic integration agreement amongst the


10-ASEAN countries (Brunei Darussalam, Cambodia, Indonesia, Laos, Malaysia,
Myanmar, Philippines, Singapore, Thailand, and Vietnam) and its six Free Trade
Agreements (FTAs) partners, viz. Australia, New Zealand, Japan, China, Korea and
India. During the 19th round, parallel meetings were held by the three main
Working Groups on Trade Trade in goods, services, and investment. The RCEP
meeting also highlighted the need to have balanced discussions to progress
negotiation across all areas, and to continue to deliver outcomes.
16. The worlds longest pedestrian suspension bridge has opened in which
country?
[A] Japan
[B] Indonesia
[C] United States
[D] Switzerland
Correct Answer: D [Switzerland]
Explanation: The worlds longest pedestrian suspension bridge Europaweg (or
Europe Bridge) has recently opened in Randa, Switzerland. It measures 1,620
feet long and rises as high as 278 feet above the Grabengufer ravine. The bridge
connects a hiking trail through the Alps between Zermatt and Graechen.
17. Pushpa Bhargava, the renowned scientist passed away. He was related to
which field?
[A] Physics
[B] Biology
[C] Chemistry
[D] Maths
Correct Answer: B [Biology]
Explanation: Dr. Pushpa Mittra Bhargava (89), the veteran molecular biologist
and a vehement critic of genetically modied crops, has passed away in
Hyderabad on August 1, 2017. He headed the Centre for Cellular and Molecular
Biology in Hyderabad, India when the government rst set it up. He was a well-
known critic of Indian governmental policies and has attained the post of Vice-
Chairman in the National Knowledge Commission
18. Which Indian shuttler has won the mens singles Lagos International 2017?
[A] Karan Rajan Rajarajan
[B] Rahul Yadav Chittaboina
[C] Manu Attri

2017 GKToday | All Rights Reserved | www.gktoday.in

165
Current Aairs: August, 2017 [Date-wise Compendium]

[D] B Sumeeth Reddy


Correct Answer: B [Rahul Yadav Chittaboina]
Explanation: Indian shuttler Rahul Yadav Chittaboina has won the mens singles
title by defeating his compatriot Karan Rajan Rajarajan by 21-15 21-13 in the
summit clash at the 2017 Lagos International Badminton Classics in Lagos,
Nigeria. Beside him, Manu Attri and B Sumeeth Reddy from India have won the
mens doubles title in the tournament.
19. Veer Dev Gulia, who has won bronze at the Junior World Wrestling
Championships, is related to which freestyle category?
[A] 45kg category
[B] 105kg category
[C] 74kg category
[D] 60kg category
Correct Answer: C [74kg category]
Explanation: Veer Dev Gulia from India has won bronze in the mens 74 kg
freestyle category at the Junior World Wrestling Championships at Tampere in
Finland on August 2, 2017. Gulia nished third after beating Yamasaki Yajuro of
Japan, 8-5 in the bronze medal play-o bout. Beside him, Ravinder from India has
also won bronze medal in 60 kg freestyle section in the tournament.
20. The Sahyadri Tiger Reserve (STR) is located in which state?
[A] Uttrakhand
[B] Kerala
[C] Karnataka
[D] Maharashtra
Correct Answer: D [Maharashtra]
Explanation: The Sahyadri Tiger Reserve (STR) is located in the Sahyadri ranges
of the Western Ghats of Maharashtra. It is in news because the Maharashtra
government is planning to relocate tigers from Vidarbha region to the sanctuaries
elsewhere where their population is smaller, including the Sahyadri Tiger Reserve
(STR). At present, STR has only seven tigers. For this, the state government has
also set up a committee to examine this issue, and a decision will be taken based
on its recommendations. It will also consult experts from the Centre and submit
its report in 2-3 months.
21. The 2017 Aadi Perukku festival is celebrated in which Indian state?
[A] Karnataka
[B] Kerala
2017 GKToday | All Rights Reserved | www.gktoday.in

166
Current Aairs: August, 2017 [Date-wise Compendium]

[C] Andhra Pradesh


[D] Tamil Nadu
Correct Answer: D [Tamil Nadu]
Explanation: In Tamil Nadu, the Aadi Perukku festival is celebrated with
traditional gaiety to pay tribute to waters life-sustaining properties. The festival,
also known as Aadi 18, is celebrated on the 18th of Aadi month in Tamil calendar
year to express gratitude to water bodies especially Cauvery river. The 18th day
of Aadi is usually August 2, is observed as Aadi pirappu, a day of oerings and
prayers to these rivers. The day is an occasion for rejoicing particularly for those
living on the banks of the all the main rivers, its branches and tributaries.
22. Who has been named as the new secretary general of the Federation of Indian
Chambers of Commerce and Industry (FICCI)?
[A] Pankaj Patel
[B] Sanjaya Baru
[C] A Didar Singh
[D] N N Patil
Correct Answer: B [Sanjaya Baru ]
Explanation: Dr. Sanjaya Baru has been named as the new secretary general of
the Federation of Indian Chambers of Commerce and Industry (FICCI). He will
assume the charge from September 1, 2017 and will succeed A Didar Singh.
23. Indian Army has developed which mobile app for soldiers to track promotions
and postings?
[A] Soldier App
[B] Humraaz app
[C] ArmyCloud app
[D] TrackPost app
Correct Answer: B [Humraaz app]
Explanation: The Indian Army has developed a mobile application Humraaz
through which serving soldiers can track details like postings and promotions.
Through the Humraaz app, soldiers can also view their monthly salary slips and
the Form 16 and also download them. For security reasons, the installation of the
application has been linked to verication of Aadhaar details. The Aadhaar details
will be veried with the Army database over National Information Centre (NIC)
cloud and he will get a one-time-password on his registered mobile number. Thus,
to enable usage of this mobile application, the latest mobile number of individuals
should be linked to his Aadhaar number. The mobile app has been developed in-
house by the Army and will be launched in the second week of August for prompt
2017 GKToday | All Rights Reserved | www.gktoday.in

167
Current Aairs: August, 2017 [Date-wise Compendium]

communication of information to junior commissioner ocers and other


personnels.
24. Who has been sworn-in as the new President of Iran?
[A] Mohammad Khatami
[B] Ayatollah Ali Khamanei
[C] Sadeq Larijani
[D] Hassan Rouhani
Correct Answer: D [Hassan Rouhani]
Explanation: Hassan Rouhani has been ocially sworn-in as the 7th President of
Iran for his second term on August 3, 2017. Irans supreme leader Ayatollah Ali
Khamanei gave his ocial approval for the president-elect by giving him a decree
to assign him his duties. Rouhani won re-election in May 2017, after securing 57%
of the votes and defeating his main contender Ebrahim Raeisi. Under Irans
election law, a president-elect is required to gain the Leaders ocial approval
before being sworn in before the Parliament. Rouhani will take the oath of oce in
Irans Parliament on August 5, 2017. The oath of oce will be administered by
Chief Justice, Sadeq Larijani.
25. Which Indian sportsperson has been honored with the East Bengal football
clubs highest honour Bharat Gaurav?
[A] Syed Naeemuddin
[B] Boddupalli Amit
[C] Dhanraj Pillay
[D] Subhash Bhowmick
Correct Answer: C [Dhanraj Pillay]
Explanation: Former Indian hockey captain Dhanraj Pillay has been honored with
the East Bengal football clubs highest honour Bharat Gaurav on its foundation
day on August 1, 2017. Pillay has played a huge role in reviving the Indian team
and performed brilliantly during his 15-year career, representing the country in
four Olympics, World Cups, Champions Trophy and Asian Games. He played total
339 matches for India, scoring 170 goals. He led India to Asian Games glory in
1998 and also won the Asia Cup in 2003. Beside him, Gurwinder Singh and
Boddupalli Amit have won the Footballer of the Year award and the Cricketer of
the Year, respectively. Former India International Subhash Bhowmick, a member
of the 1970 Asian Games bronze medal-winning national football team and former
national team coach Syed Naeemuddin were honored with lifetime achievement
awards.
26. What is the theme of 2017 World Breastfeeding Week (WBW)?
2017 GKToday | All Rights Reserved | www.gktoday.in

168
Current Aairs: August, 2017 [Date-wise Compendium]

[A] Sustaining Breastfeeding Together


[B] Breastfeeding: A key to Sustainable Development
[C] Breastfeeding: SDG 2022 goals
[D] Mothers Absolute Affection
Correct Answer: A [Sustaining Breastfeeding Together]
Explanation: The World Breastfeeding Week (WBW) is observed every year from
August 1st to 7th to focus attention on the important aspect of promotion and
support of breastfeeding. The 2017 theme is Sustaining Breastfeeding Together.
To intensify the eorts for promotion of breastfeeding in India, the Union Health
Ministry has initiated a nationwide programme called MAA-Mothers Absolute
Aection to bring undiluted focus on promotion of breastfeeding and provision of
services towards supporting breastfeeding, along with ongoing eorts of routine
health systems. In addition, National Guidelines on Lactation Management
Centres in Public Health Facilities have been recently released to facilitate
establishment of lactation management centres for ensuring that the sick and
pre-term babies are fed with safe human breast milk.
27. Which Indian has become the Asias second richest man, according to latest
Bloomberg Billionaires Index (BBI)?
[A] Lakshmi Mittal
[B] Mukesh Ambani
[C] Pallonji Mistry
[D] Kalanithi Maran
Correct Answer: B [Mukesh Ambani]
Explanation: Mukesh Ambani has become the Asias second richest man
according to the 2017 Bloomberg Billionaires Index (BBI). According to the 2017
index, Ambanis net worth has risen to $35.8 billion (around Rs 2.2 lakh crore),
making him the 19th richest person in the world, up from 29 at the end of 2016.
The increase in the net worth of the chief of Reliance Industries comes partially
from the rise in stock price of the energy major, which has made major bets on
the retail and telecom industries. Currently, Alibaba Group founder Jack Ma is the
richest person in Asia with a net worth of $43.2 billion.
28. BB Shivappa, the noted Indian political leader passed away. He hailed from
which state?
[A] Tamil Nadu
[B] Kerala
[C] Karnataka
[D] Andhra Pradesh
2017 GKToday | All Rights Reserved | www.gktoday.in

169
Current Aairs: August, 2017 [Date-wise Compendium]

Correct Answer: C [Karnataka]


Explanation: B B Shivappa (89), the veteran Karnataka BJP leader, has passed
away in Bengaluru, Karnataka. He is credited with building the party brick by brick
in its early years in the State and served as its State president from 1983 to 88.
Shivappa played an important role in the formation of the rst ever non-Congress
government in Karnataka, led by Ramakrishna Hegde in 1983.
29. Which international organisation has recently released 2017 ICT Facts and
Figures report?
[A] International Telecommunication Union (ITU)
[B] International Labour Organization (ILO)
[C] International Atomic Energy Agency (IAEA)
[D] International Organization for Migration (IOM)
Correct Answer: A [International Telecommunication Union (ITU)]
Explanation: According to recently released report-ICT Facts and Figures 2017,
India and China are home to 39% of the 830 million young people worldwide who
use the Internet. The report, released by the International Telecommunication
Union (ITU), shows that of the 830 million young people online worldwide, 320
million ( or 39%), are in China and India. The ICT data shows that youths (15-24
years old) are at the forefront of Internet adoption. In Least Developed Countries
(LDCs), up to 35% of individuals using the Internet are aged 15-24, compared with
13% in developed countries and 23% globally. The ITU is the United Nations (UN)
specialized agency for Information and Communication Technologies (ICTs). It has
193 countries as its members and also 800 private-sectors.
30. The Pushpagiri Wildlife Sanctuary (PWS) is located in which state?
[A] Rajasthan
[B] Karnataka
[C] Kerala
[D] Assam
Correct Answer: B [Karnataka ]
Explanation: The Pushpagiri Wildlife Sanctuary (PWS) is located in Somwarpet
taluk of the Kodagu district in Karnataka and spread across an area of 102 kms. It
is home to rare and endangered birdlife such as Nilgiri Flycatcher, Nilgiri wood
pigeon, Malabar Grey Hornbill, Grey Headed Bulbul, blue winged parakeet, small
sunbird, white-bellied blue ycatcher, etc. One can spot varieties of animal
species that includes Indian Hare, Spotted Deer, Giant Flying Squirrel, Otter
Species, Indian Wild Dog, Travancore Flying, Brown Palm Civet, Wild Pig, Indian
Giant Squirrel, etc. It is a must-visit sanctuary to hear the heartwarming sounds of
2017 GKToday | All Rights Reserved | www.gktoday.in

170
Current Aairs: August, 2017 [Date-wise Compendium]

the birds.
31. Which committee has been constituted by the SEBI on fair market conduct?
[A] K M Krishna committee
[B] G S Prasad committee
[C] T.K. Viswanathan committee
[D] P S Chauhan committee
Correct Answer: C [T.K. Viswanathan committee]
Explanation: The Securities and Exchange Board of India (SEBI) has recently
constituted a committee on fair market conduct, which will be headed by former
Lok Sabha secretary and law secretary T K Viswanathan. The panel will have
representations from SEBI, mutual funds, brokers, audit rms, stock exchanges,
data analytics rms and legal rms. The committee has been tasked with
suggesting improvements to the existing SEBI norms, including on insider trading
and Fraudulent and Unfair Trade practices (FUTP). It has been specically asked
to look at trading plans, handling of price-sensitive information during takeovers
and aligning of insider-trading rules with provisions of the Companies Act. The
committee has also been tasked with suggesting short-term and medium-term
measures for improved surveillance of the markets, as well as issues of high-
frequency trades and harnessing of technology and analytics in surveillance. The
committee will submit its report within 4 months.
32. Who has been appointed as the new chairman of the Bombay Stock Exchange
(BSE)?
[A] Pankaj Patel
[B] S Ramadorai
[C] Sudhakar Rao
[D] Dhirendra Swarup
Correct Answer: D [Dhirendra Swarup]
Explanation: Dhirendra Swarup, who was a public interest director at the BSE,
has been appointed as the new chairman of the Bombay Stock Exchange (BSE).
He succeeded Sudhakar Rao. Swarup is a former Chairman of Pension Funds
Regulatory and Development Authority (PFRDA) and has over four decades of
experience across nance, budgeting, public debt management, public policy and
pension reforms. He was also chairman of the Cash and Debt Management
Committee of the Reserve Bank of India (RBI) from 2000 to 2003.
33. Who is the newly elected Prime Minister of Papua New Guinea?
[A] Salamo Injia
[B] Don Polye
2017 GKToday | All Rights Reserved | www.gktoday.in

171
Current Aairs: August, 2017 [Date-wise Compendium]

[C] Peter ONeill


[D] Nick Kuman
Correct Answer: C [Peter ONeill]
Explanation: Peter ONeill has been re-elected as the new Prime Minister of
Papua New Guinea (PNG) for a second ve-year term with a healthy majority, 60
to 46 by a newly elected parliament. He is the leader of the Peoples National
Congress and represents the constituency of Ialibu-Pangia.
34. The 2nd Agri UDAAN, food & agribusiness accelerator program has launched
from which city?
[A] New Delhi
[B] Lucknow
[C] Bhopal
[D] Jaipur
Correct Answer: A [New Delhi]
Explanation: The Union Government has launched the 2nd AGRI-UDAAN
programme in New Delhi on August 4, 2017 to promote innovation and
entrepreneurship in agriculture. It will mentor startups and help them connect
with potential investors. The program focuses on catalyzing scale-up stage food
and agribusiness startups through rigorous mentoring, industry networking and
investor pitching. The 6 month program is a unique platform for scale up stage
innovators, entrepreneurs and startups in the Food and Agribusiness sectors. The
applications will be available at www.aidea.naarm.org.in. Top 40 startups will be
shortlisted and allowed to pitch in front of panel of evaluators during cohort
nalization programme. Out of these about 8 to 12 startups will be selected for
nal cohort for capacity building workshop. The AGRI UDAAN has organised by
NAARM, a-IDEA and IIM-A, CIIE in partnership with Caspian Inpact Investment and
supported by DST.
35. Manju Kumari, who has won bronze in the Junior World Wrestling
Championship, is related to which category?
[A] 67kg category
[B] 51kg category
[C] 44kg category
[D] 59kg category
Correct Answer: D [59kg category]
Explanation: Manju Kumari from India has won the bronze medal in 59kg
category with a 2-0 victory over Ilona Prokopevniuk of Ukraine in the Junior World

2017 GKToday | All Rights Reserved | www.gktoday.in

172
Current Aairs: August, 2017 [Date-wise Compendium]

Wrestling Championship in Tampere, Finland on August 3, 2017.


36. The Maharashtra government has constituted committee on safety measures
for women in IT sector?
[A] Nirmala Patil committee
[B] Rashmi Shukla committee
[C] Sadhana Jadhav committee
[D] R M Savant committee
Correct Answer: B [Rashmi Shukla committee]
Explanation: The Maharashtra government has constituted a committee under
the chairmanship of senior IPS ocer Rashmi Shukla to suggest measures to
ensure safety of women working in the Information and Technology sector. The
committee will suggest measures that can be undertaken to ensure safety of
women working in the IT sector. This information was given by the state
government to Bombay High Court. The government also sought four weeks time
to le a detailed adavit on long-term steps it was contemplating to enhance
womens safety. The court has taken up the PIL suo motu (on its own) on the
issue of womens safety in the state. In January 2017, a woman software engineer
working with Infosys was allegedly killed by a security guard on the companys
premises in Pune.
37. Which state government has launched Aadhaar-authenticated Direct Benefit
Transfer portal MahaDBT?
[A] Kerala
[B] Maharashtra
[C] Madhya Pradesh
[D] Uttar Pradesh
Correct Answer: B [Maharashtra ]
Explanation: The Maharashtra Government has launched two online portals
MahaDBT and MahaVASTU to streamline the process of Direct Benet Transfer
(DBT) and to bring in more transparency in construction sector. With this,
Maharashtra has become the rst state in India to launch such a system that will
provide direct benets with Aadhaar authentication. The MahaDBT portal is an
Aadhaar-authenticated electronic mechanism that will enable direct transfer of
benets for over 40 schemes currently implemented. MahaDBTs design is based
on central governments direct transfer benet scheme and Aadhaar will be
mandatory to claim benets through this portal. On the other hand, MahaVASTU
portal is an online building permission management system, through which
construction approvals will be sanctioned with full transparency.

2017 GKToday | All Rights Reserved | www.gktoday.in

173
Current Aairs: August, 2017 [Date-wise Compendium]

38. Who has been appointed as the new CMD of Small Industries Development
Bank of India (SIDBI)?
[A] Manoj Mittal
[B] Mohammad Mustafa
[C] Ajay Kumar Kapur
[D] Kiran Sharma
Correct Answer: B [Mohammad Mustafa]
Explanation: Mohammad Mustafa, a 1995 batch IAS ocer of Uttar Pradesh
cadre, has been appointed as the new Chairman and Managing Director (CMD) of
Small Industries Development Bank of India (SIDBI) for a period of 3 years. At
present, he is the joint Secretary in Department of Financial Services. SIDBI is an
independent nancial institution aimed to aid the growth and development of
micro, small and medium-scale enterprises (MSME) in India and is headquartered
in Lucknow, Uttar Pradesh.
39. Which city hosted the 2017 meeting of BRICS Trade Ministers?
[A] Shanghai
[B] Braslia
[C] Cape Town
[D] Moscow
Correct Answer: A [Shanghai]
Explanation: The 7th Meeting of BRICS Trade Ministers was held in Shanghai,
China from August, 1-2, 2017. From India, the 6-member delegation led by
Commerce and Industry Minister Nirmala Sitharaman , along with Ambassador J.S.
Deepak, PR to WTO participated in the meeting. The trade ministers of BRICS
nations discussed trade facilitation, economic and technological cooperation,
capacity building and the multilateral trade system in the meeting. And, adopted
the following documents at the conclusion of the meeting: 7th BRICS Trade
Ministers Joint Statement, BRICS trade in services cooperation roadmap, BRICS e-
commerce cooperation initiative, BRICS IPR cooperation guidelines, etc.
40. The Pin Valley National Park (PVNP) is located in which state?
[A] Jammu & Kashmir (J&K)
[B] Arunachal Pradesh
[C] Himachal Pradesh
[D] Sikkim
Correct Answer: C [Himachal Pradesh]
Explanation: The Pin Valley National Park is located within the Lahaul and Spiti

2017 GKToday | All Rights Reserved | www.gktoday.in

174
Current Aairs: August, 2017 [Date-wise Compendium]

district of Himachal Pradesh. The Park forms a natural habitat for a number of
endangered animals such as snow leopard and Siberian ibex with its snow laden
unexplored higher reaches and slopes.
41. India has recently signed how much amount of loan pact with AIIB for Gujarat
road project?
[A] $429 million
[B] $329 million
[C] $229 million
[D] $529 million
Correct Answer: B [$329 million]
Explanation: India has recently signed a $329 million loan agreement with Asian
Infrastructure Investment Bank (AIIB) for Gujarat Rural Roads Project. The
objective of the project is to improve the rural road connectivity and accessibility
to more than one thousand villages in all the 33 districts of the state beneting
about 80 lakh people. The project will also benet the service providers such as
public transport operators, educational institutions, hospitals, local markets and
traders.
42. Indias first private sector missile sub-systems manufacturing facility has set
up in which city?
[A] Guwahati
[B] Pune
[C] Hyderabad
[D] New Delhi
Correct Answer: C [Hyderabad ]
Explanation: Indias rst private sector missile sub-systems manufacturing
facility has inaugurated near Hyderabad on August 4, 2017. It has been
established a line with the Make in India initiative and Governments policy to
encourage private sector participation in defense product. It is a joint venture
between the $2.5 billion Kalyani Group and Israels Rafael Advanced Defence
Systems Ltd. The Kalyani Rafael Advanced Systems (KRAS) plant will make anti-
tank guided missile (ATGM) Spike and the production is expected to start soon
within weeks.
43. Who has will be the new Vice-Chairman of NITI Aayog?
[A] Rajiv Kumar
[B] Vinod Paul
[C] Ritesh Gupta

2017 GKToday | All Rights Reserved | www.gktoday.in

175
Current Aairs: August, 2017 [Date-wise Compendium]

[D] S L Shivappa
Correct Answer: A [Rajiv Kumar]
Explanation: Noted economist Dr. Rajiv Kumar will be the new Vice-Chairman of
NITI Aayog, which is a think-tank that replaced the Planning Commission. He will
succeed Arvind Panagariya, who will leave the government think tank on August
31, 2017 and return to Columbia University. Currently, Rajiv Kumar is a senior
fellow at the Centre for Policy Research, a non-governmental think tank on public
policy based out of New Delhi. He had also been part of a Government appointed
high-level search Panel that went into the appointment of regulators in the Indian
nancial sector. Meanwhile, the Centre has also appointed Vinod Paul, a
Paediatrician at AIIMS, as a member of NITI Aayog.
44. Which country to host the 15th meeting of the Foreign Ministers of BIMSTEC
countries?
[A] India
[B] Sri Lanka
[C] Indonesia
[D] Nepal
Correct Answer: D [Nepal]
Explanation: The 15th meeting of the Foreign Ministers of BIMSTEC countries will
be held in Kathmandu, Nepal from August 10-11, 2017. From India, the Union
External Aairs Minister Sushma Swaraj will represent India in the meeting and
will hold bilateral meetings with the leaders of the members countries. During the
BIMSTEC meeting, progress on several programmes will be discussed.
45. Which committee has recently submitted report on institutionalization of
arbitration mechanism in India?
[A] BN Srikrishna committee
[B] Neil Chatterjee committee
[C] BP Das committee
[D] K L Gupta committee
Correct Answer: A [BN Srikrishna committee ]
Explanation: The high level committee under the Chairmanship of Justice BN
Srikrishna has recently submitted its on review of the Institutionalization of
Arbitration Mechanism in India. The committee has recommended for setting up
an Autonomous Body, Arbitration Promotion Council of India (APCI) having
representatives from all stakeholders for grading arbitral institutions in India. The
APCI may recognize professional institutes providing for accreditation of
arbitrators. The APCI may hold training workshops and interact with law rms and
2017 GKToday | All Rights Reserved | www.gktoday.in

176
Current Aairs: August, 2017 [Date-wise Compendium]

law schools to train advocates with interest in arbitration and with a goal to create
a specialist arbitration bar comprising of advocates dedicated to the eld. It has
also recommended for creation of a specialist Arbitration Bench to deal with such
Commercial disputes, in the domain of the Courts.
46. Who has been appointed as the new Ambassador of India to the Republic of
Kazakhstan?
[A] Rajesh Kumar Sachdeva
[B] Manoj Mittal
[C] Ajay Bisaria
[D] Prabhat Kumar
Correct Answer: D [Prabhat Kumar]
Explanation: Prabhat Kumar, the IFS ocer of 1991 batch, has been appointed
as the next Ambassador of India to the Republic of Kazakhstan. Presently, he is
Ambassador of India to Colombia. He is expected to take up the assignment
shortly.
47. Microsoft has tied up with which Indian state to use its AI-based MINE
platform to screen children for eye problems?
[A] Himachal Pradesh
[B] Telangana
[C] Kerala
[D] Karnataka
Correct Answer: B [Telangana ]
Explanation: The Microsoft India has tied up with the state of Telangana to use
its articial intelligence-based MINE platform to screen children for eye problems
and prevent avoidable blindness. The Microsoft Intelligent Network for Eyecare
(MINE) is an articial intelligence platform to reduce avoidable blindness. With this
tied up, Telangana has become the rst Indian state to deploy articial
intelligence for eyecare screening.
48. Robert Hard, the noted actor has passed away. He hailed from which
country?
[A] France
[B] Germany
[C] England
[D] United States
Correct Answer: C [England]
Explanation: Robert Hard (91), who portrayed the role of Minister of Magic
2017 GKToday | All Rights Reserved | www.gktoday.in

177
Current Aairs: August, 2017 [Date-wise Compendium]

Cornelius Fudge in four of the Harry Potter lms, has passed away in London,
England on August 3, 2017. He was also known for his many portrayals of Winston
Churchill and as the irascible vet Siegfried Farnon in All Creatures Great and
Small. His voice performance as Robin Hood in Tale Spinners For Children, an LP
from the 1960s, is considered one of the best Robin Hood renditions.
49. The Chimmini Wildlife Sanctuary (CWS) is located in which state?
[A] Uttrakhand
[B] Chhattisgarh
[C] Himachal Pradesh
[D] Kerala
Correct Answer: D [Kerala]
Explanation: The Chimmini Wildlife Sanctuary (CWS) is located along the
Western Ghats in Mukundapuram taluk of Thrissur District of Kerala. The
sanctuary is an important bird area with 192 recorded avian species such as grey-
headed bulbul, Indian rufous babbler, white-bellied blue-ycatcher, Ceylon
frogmouth, Indian edible-nest swiftlet, Malabar trogon and Lotens sunbird. The
sanctuary also oers trekking paths for the adventure traveller.
50. Ankur Mittal is associated with which sports?
[A] Sprint
[B] Football
[C] Shooting
[D] Basketball
Correct Answer: C [Shooting]
Explanation: Indian shooter Ankur Mittal has clinched the individual as well as
the team gold medal in mens double trap at the 7th Asian Shotgun Championship
at Astana, Kazakhstan on August 5, 2017. Mittal shot 71 in the six-man nal to
nish at the top of the podium. With scores of Sangram Dahiya and Mohammad
Asab added to Mittals nal total, India managed to bag the top prize in the team
event also. Dahiya had a score of 43 and Asab had shot 34.
51. Who has been appointed as new Executive Director of the World Bank (WB) to
represent the constituency of India, Bangladesh, Bhutan and Sri Lanka?
[A] S. Aparna
[B] Ajay Bhadoo
[C] K C Sampat
[D] Subhash Garg
Correct Answer: A [S. Aparna]

2017 GKToday | All Rights Reserved | www.gktoday.in

178
Current Aairs: August, 2017 [Date-wise Compendium]

Explanation: S. Aparna, a 1988-batch IAS ocer from Gujarat cadre, has been
appointed as new Executive Director of the World Bank (WB) to represent the
constituency of India, Bangladesh, Bhutan and Sri Lanka for three years.
Currently, she is principal secretary to Gujarat Chief Minister Vijay Rupani. At the
World Bank, she will replace Subhash Garg, who was recently appointed Economic
Aairs Secretary, Ministry of Finance.
52. The 2017 National Handloom Day (NHD) is celebrated on which date?
[A] August 6
[B] August 7
[C] August 5
[D] August 8
Correct Answer: B [August 7]
Explanation: The National Handloom Day (NHD) is celebrated every year in India
on August 7 to give an impetus to the sector and encourage the use of handmade
loom. The 7th of August has been chosen to celebrate the day due to its special
signicance in Indias history. It was on this day that the Swadeshi Movement was
launched in 1905. The movement involved revival of domestic products and
production processes. The 3rd edition of National Handloom Day is celebrated
across the country on August 7, 2017 to promote use of Khadi among people.
53. Who has been appointed as the brand ambassador for Swachh Bharat Mission
in Uttar Pradesh?
[A] Sachin Tendulkar
[B] Amitabh Bachchan
[C] Akshay Kumar
[D] Aamir Khan
Correct Answer: C [Akshay Kumar]
Explanation: The Bollywood actor Akshay Kumar has been appointed as the
brand ambassador for Swachh Bharat Mission in Uttar Pradesh. It was recently
announced by Yogi Adityanath, the Chief Minister of Uttar Pradesh. Apart from
this, Yogi Adityanath has also made Kumars upcoming lm Toilet: Ek Prem
Katha tax free. The movie stresses on the importance of building toilets and
ending open defecation in the country.
54. Sajan, who bagged a bronze at the Junior World Wrestling Championships, is
related to which Greco-Roman category?
[A] 65kg class category
[B] 44kg class category
2017 GKToday | All Rights Reserved | www.gktoday.in

179
Current Aairs: August, 2017 [Date-wise Compendium]

[C] 105kg class category


[D] 74kg class category
Correct Answer: D [74kg class category]
Explanation: Sajan from India has bagged a bronze medal in Greco-Roman 74kg
class category after defeating Ali Osman Erbay of Turkey by 6-1 at the Junior
World Wrestling Championships in Tampere, Finland on August 6, 2017. He is the
lone Indian Greco-Roman wrestler to bag a medal at this championship.
55. Indias first helicopter-taxi service will start in which city?
[A] Bengaluru
[B] New Delhi
[C] Chennai
[D] Kolkata
Correct Answer: A [Bengaluru]
Explanation: Indias rst helicopter-taxi (heli-taxi) service will be started in
Bengaluru, Karnataka for those who cannot aord to spend time battling trac to
travel across the city. The Kempegowda International Airport will become the rst
airport in India to have this facility. The service will be operated by Thumby
Aviation Private Limited and is expected to take o within three months. Initially,
the helicopter service will be operated between Bengalurus Kempegowda
International Airport and the Electronics City area. The travel time will be cut
down to 15 minutes instead of the two hours taken by road.
56. Where is the headquarters of the Indira Gandhi National Forest Academy
(IGNFA)?
[A] Bhopal
[B] Dehradun
[C] Kanpur
[D] Udaipur
Correct Answer: B [Dehradun]
Explanation: The Indira Gandhi National Forest Academy (IGNFA) is an
organization under the Ministry of Environment and Forests (India) to impart
knowledge and skills to the professional foresters and help them to develop
competence for managing the country forest and wildlife resources on a
sustainable basis. It is located at the New Forest campus of Forest Research
Institute (FRI) in Dehradun, Uttarakhand. Currently, IGNFA is functioning as a Sta
College for the ocers of the Indian Forest Service (IFS).
57. Who has won the 2017 Rwandan presidential election?
2017 GKToday | All Rights Reserved | www.gktoday.in

180
Current Aairs: August, 2017 [Date-wise Compendium]

[A] Frank Habineza


[B] Juvenal Habyarimana
[C] Philippe Mpayimana
[D] Paul Kagame
Correct Answer: D [Paul Kagame]
Explanation: Paul Kagame, the current president of Rwanda who is ruling the
tiny African nation for 17 years, has won the 2017 Rwandan presidential election
with 99% of votes. He is the leader of the political party Rwandan Patriotic Front
(FPR). With this, he secured his third term in oce as president and will extend
his 17 years in power.
58. Who has been selected for the 2017 Prem Nazir Award?
[A] Suraj Venjaramoodu
[B] Salim Kumar
[C] K.J. Yesudas
[D] Sharada
Correct Answer: D [Sharada]
Explanation: Sharada, the veteran actress and national award winner, has been
selected for the prestigious Prem Nazir Award for the year 2017. The award will be
presented to Sharada by state Devaswom Minister Kadakampally Surendran at a
function at Chirayankeezhu on August 15, 2017 at Thiruvananthapuram, Kerala. It
carries a cash prize of Rs 75,000, a statuette and a citation. The award is
instituted by people of Chirayankeezhu, Prem Nazirs home town in the district, in
memory of Keralas evergreen actor. Beside her, Actor T P Madhavan will be
presented Rs 50,000 cash award announced by Chirayankeezhu panchayat.
59. Which Indian institute has developed a highly sensitive nanometre-scale
carbon monoxide sensor?
[A] Indian Institute of Technology (IIT), Kharagpur
[B] Indian Institute of Technology (IIT), Bombay
[C] Indian Institute of Science (IISc), Bengaluru
[D] Indian Institute of Technology (IIT), Madras
Correct Answer: C [Indian Institute of Science (IISc), Bengaluru]
Explanation: The researchers at Indian Institute of Science (IISc), Bengaluru
have developed a low-cost highly sensitive nanometre-scale carbon monoxide
sensor by employing an innovative fabrication technique. It is known that Carbon
Monoxide (CO) can have adverse eects on the health of people exposed to it.
Thus, it becomes necessary to have good, low-cost carbon-monoxide sensors.

2017 GKToday | All Rights Reserved | www.gktoday.in

181
Current Aairs: August, 2017 [Date-wise Compendium]

Typically, a sensor would be a thin, current carrying plate whose resistance


changes on exposure to carbon monoxide. This in turn changes the value of the
current owing through it. This change when measured indicates the level of
carbon monoxide in the air. The research is published in Sensors and Actuators B:
Chemical.
60. The Pampadum Shola National Park (PSNP) is located in which state?
[A] Uttrakhand
[B] Kerala
[C] Chhattisgarh
[D] Himachal Pradesh
Correct Answer: B [Kerala]
Explanation: The Pampadum Shola National Park (PSNP) is the smallest national
park located in the Idukki district of Kerala. It protects a moderate amount of
montane evergreen shola forest that is associated with the wildlife rich
Eravikulam National Park. Some notable birds found here such as Nilgiri wood-
pigeon, white-bellied shortwing, vernal hanging parrot, blue rock-thrush, blue-
capped rock-thrush and Nilgiri ycatcher, and black-and-orange ycatchers.
61. ISRO and NASA are jointly working on which mission to launch dual
frequency SAR satellite by 2021?
[A] NISAR
[B] INSAR
[C] SARIN
[D] SARNI
Correct Answer: A [NISAR]
Explanation: The ISRO and NASA are jointly working on the NASA-ISRO Synthetic
Aperture Radar (NISAR) mission to co-develop and launch a dual frequency (L & S
Band) synthetic aperture radar satellite. This earth monitoring satellite is
scheduled to be launched in 2021. In this joint mission, NASA will be responsible
for design and development of L-band SAR, 12m un-furlable antenna, GPS system
and data recorder. ISRO will be responsible for design and development of S-band
SAR, Spacecraft Bus, data transmission system, Spacecraft integration and
testing, launch using GSLV and on-orbit operations. It is the rst big collaboration
between NASA and ISRO.
62. Which country to host the 8th edition of Global Entrepreneurship Summit
(GES)?
[A] Turkey
[B] India
2017 GKToday | All Rights Reserved | www.gktoday.in

182
Current Aairs: August, 2017 [Date-wise Compendium]

[C] United States


[D] France
Correct Answer: B [India]
Explanation: The 8th edition of Global Entrepreneurship Summit (GES)-2017 will
be hosted by India and is scheduled to be held on November 28, 2017. The NITI
Aayog in association with the Ministry of External Aairs and the Andhra Pradesh
government have been assigned with organising the event. The GES brings
together investors and entrepreneurs from around the world to create new
opportunities for collaboration. The summit will promote economic growth,
inclusion and opportunities among entrepreneurs from across the world, with a
special focus on emerging nations from Africa and Asia. The summit was started
in 2010 under the leadership of then US President Barack Obama at Washington
DC to bring entrepreneurs from across the globe on one platform.
63. Which union minister has recently launched a Swachh Survekshan (Gramin)
-2017?
[A] Rajnath Singh
[B] Narendra Modi
[C] M Venkaiah Naidu
[D] Narendra Singh Tomar
Correct Answer: D [Narendra Singh Tomar]
Explanation: Narendra Singh Tomar, the Ministry of Drinking Water and
Sanitation, has recently launched a Swachh Survekshan (Gramin)-2017 in New
Delhi to encourage states and districts to improve their sanitation coverage and
Solid Liquid Waste Management (SLWM). Under the Swachh Survekshan Gramin,
all districts of the country will be ranked on the basis of performance,
sustainability and transparency. So far, 4,600 villages and 1.4 lakh households
have been covered under Swachh Sarvekshan (Gramin) Survey. The Union Rural
Development Minister Narendra Singh Tomar has stated that toilet usage have
increased to 91% in the country whereas overall toilet coverage in rural India has
reached to 66%.
64. Which country is hosting the 2017 FIBA Asia Championship?
[A] China
[B] India
[C] Lebanon
[D] Nepal
Correct Answer: C [Lebanon]
Explanation: The FIBA Asia Cup, which is known as the Asias biggest basketball
2017 GKToday | All Rights Reserved | www.gktoday.in

183
Current Aairs: August, 2017 [Date-wise Compendium]

tournament, has started in Beirut, Lebanon from August 8-20, 2017. All 16 teams
who qualify for the tournament also qualify for the rst round of the 2019 FIBA
Basketball World Cup qualication for FIBA Asia and FIBA Oceania. Indian mens
team, led by Amjyot Singh Gill, is participating in the tournament in group A.
65. Where is the headquarters of the Bureau of Police Research and Development
(BPRD)?
[A] Jaipur
[B] New Delhi
[C] Pune
[D] Lucknow
Correct Answer: B [New Delhi]
Explanation: Recently, the Union Government has merged National Crime
Records Bureau (NCRB) with the Bureau of Police Research and Development
(BPRD) to improve administrative eciency and optimal utilisation of resources
related to policing. The BPRD Director General (DG) will be the head of the
merged new entity, where the NCRB Director, an Additional DG rank post will
report to the former. The BPRD is the national police organisation to study,
research and develop on subjects and issues related to policing. The headquarters
of BPRD is located in New Delhi.
66. Which team has won the 2017 All-India MCC-Murugappa Gold Cup hockey
tournament?
[A] Punjab National Bank (PNB)
[B] Bengaluru Hockey Association (BHA)
[C] Oil and Natural Gas Corporation (ONGC)
[D] Indian Oil Corporation (IOC)
Correct Answer: C [Oil and Natural Gas Corporation (ONGC)]
Explanation: The Oil and Natural Gas Corporation (ONGC) has won the 91st
edition of All-India MCC-Murugappa Gold Cup hockey tournament by defeating
Bengaluru Hockey Association (BHA) by 4-2 in the nal at Mayor Radhakrishna
stadium Egmore at Chennai, Tamil Nadu. Man of the match: Rajkumar Pal.
67. The Dawn of cruise tourism in India has launched in which city to promote
cruise tourism in India?
[A] Mumbai
[B] Kochi
[C] Chennai
[D] Kolkata
2017 GKToday | All Rights Reserved | www.gktoday.in

184
Current Aairs: August, 2017 [Date-wise Compendium]

Correct Answer: A [Mumbai]


Explanation: In order to promote cruise tourism in India, a special event The
dawn of cruise tourism in India has inaugurated in Mumbai, Maharashtra on
August 8, 2017. This initiative has a potential revenue gains to all stakeholders of
close to Rs. 35000 crore. The promotion of cruise tourism is expected to create
over 2, 50,000 jobs. With this new initiative, number of cruise ships visiting India
could go up from 158 per year to 955 per year.
68. Amazon India has signed a MoU with which state handloom department to
educate weavers and artisans to directly sell their products to Amazon
customers?
[A] Andhra Pradesh
[B] Madhya Pradesh
[C] Karnataka
[D] Telangana
Correct Answer: D [Telangana ]
Explanation: Amazon India has recently signed a Memorandum of Understanding
(MoU) with the Telangana Department of Handloom and Textiles to educate, train
and enable weavers and artisans to directly sell their products to Amazon
customers across the country. Amazon India will not only provide an online
marketplace for marketing but will also engage with weavers and artisans across
the state to train them on making the products more attractive, appealing and
marketable across the online domain. The partnership also intends to empower
and generate livelihood opportunities and income for the artisans and boost their
economy
69. Who has been appointed as the new Chief Justice of India (CJI)?
[A] Ranganath Misra
[B] K K Mittal
[C] Dipak Misra
[D] G B Pattanaik
Correct Answer: C [Dipak Misra]
Explanation: Justice Dipak Misra, the senior-most judge of the Supreme Court,
has been appointed as the new Chief Justice of India (CJI) and will have a tenure of
nearly 13 months. He will succeed Chief Justice J S Khehar, who will retire on
August 27, 2017. Justice Misra is known for the judgment on Yakub Memon who
made a last minute attempt to escape the gallows. He had rejected his plea for a
stay on the death warrant. Memon was convicted in 1993 Mumbai blasts, in which

2017 GKToday | All Rights Reserved | www.gktoday.in

185
Current Aairs: August, 2017 [Date-wise Compendium]

257 people were killed.


70. The Nal Sarovar Bird Sanctuary (NSBS) is located in which state?
[A] Gujarat
[B] Madhya Pradesh
[C] Chhattisgarh
[D] Uttrakhand
Correct Answer: A [Gujarat]
Explanation: The Nal Sarovar Bird Sanctuary (NSBS) is located to the west of
Ahmedabad near Sanand Village in Gujarat. it is the largest wetland bird
sanctuary in Gujarat and one of the largest in India. It harbors over 250 species of
wetland birds such as rosy pelicans, amingoes, white storks, brahminy ducks and
herons.
71. Kynan Chenai has won bronze in mens trap event at the 7th Asian
Championship Shotgun. He hails from which city?
[A] New Delhi
[B] Hyderabad
[C] Jaipur
[D] Bhopal
Correct Answer: B [Hyderabad ]
Explanation: Indian shooter Kynan Chenai from Hyderabad has won a bronze in
mens trap event at the 7th Asian Championship Shotgun at Astana, Kazakhstan
on August 8, 2017. It was his rst international medal at the senior level. So far,
India has won two golds and a bronze in the competition. Before Chenai, Indian
shooter Ankur Mittal clinched the individual as well as the team gold medal in
mens double trap in the championship. Mittal shot 71 in the six-man nal to nish
at the top of the podium. With scores of Sangram Dahiya (43) and Mohammad
Asab (34) added to Mittals nal total, India managed to bag the top prize in the
team event also.
72. Sanwar Lal Jat, the former Union minister has passed away. He hailed from
which state?
[A] Madhya Pradesh
[B] Haryana
[C] Rajasthan
[D] Punjab
Correct Answer: C [Rajasthan ]
Explanation: Prof Sanwar Lal Jat (62), the BJP MP from Ajmer and former Union
2017 GKToday | All Rights Reserved | www.gktoday.in

186
Current Aairs: August, 2017 [Date-wise Compendium]

minister, has passed away in New Delhi on August 9, 2017. He was member of the
Rajasthan Legislative Assembly for ve terms. He also served as Minister in the
Rajasthan Government thrice. In 2014, he was elected to the Lok Sabha from
Ajmer. He served as Union Minister of State for Water Resources, River
Development and Ganga Rejuvenation till July 2016.
73. Indias first micro forest will be built in which state?
[A] Madhya Pradesh
[B] West Bengal
[C] Uttar Pradesh
[D] Chattisgarh
Correct Answer: D [Chattisgarh]
Explanation: Indias rst micro forest will be built in Raipur, Chattisgarh.
Approximately, 19 acres of land which housed around 70 government oce
buildings will be converted into micro forest in order to create an oxy-zone. The
micro forest will provide fresh air to Raipur which is the 7th most polluted city in
the world. It will reportedly be open to the public in the next 8 months.
74. Which city to host the 2017 India ASEAN Youth summit?
[A] Bhopal
[B] Lucknow
[C] New Delhi
[D] Gandhinagar
Correct Answer: A [Bhopal]
Explanation: The 2017 India ASEAN Youth summit will be held in Bhopal,
Madhya Pradesh from August 14 to 19, 2017 with theme Shared values, common
destiny. This event is being organized in collaboration with the Association of
Southeast Asian Nations (ASEAN) Secretariat and the India Foundation. Over 100
delegates from 10 ASEAN countries Brunei, Cambodia, Lao PDR, Myanmar,
Malaysia, Indonesia, the Philippines, Singapore, Thailand, and Vietnam will
participate in the summit. This forum will enhance people-to-people contacts
between India and ASEAN countries.
75. Shapath Bhardwaj has become the Indias youngest athlete to be named in
the Target Olympic Podium (TOP) scheme. He is associated with which sports?
[A] Boxing
[B] Wrestling
[C] Shooting
[D] Sprint

2017 GKToday | All Rights Reserved | www.gktoday.in

187
Current Aairs: August, 2017 [Date-wise Compendium]

Correct Answer: C [Shooting]


Explanation: India shooter Shapath Bhardwaj (15) from Merrut has become the
Indias youngest athlete to be named in the Union Governments Target Olympic
Podium (TOP) scheme. He is among 45 elite athletes, including Olympic medallists
PV Sindhu, Sakshi Malik and Saina Nehwal, to be selected for the scheme. The
TOP scheme elite athletes identication committee, which is chaired by ace
shooter and Olympic gold medallist Abhinav Bindra, selected the athletes after
evaluating their performance in relation to the international standard. The
purpose of the TOP committee is to identify and support potential medal
prospects for 2020 and 2024 Olympic Games under the TOP scheme.
76. Which fish has been officially declared as one of the rarest species of fish in
the world by the Marine Living Resources Department of Andhra University?
[A] Rohu
[B] Rayichapalu
[C] Catla
[D] Tengra
Correct Answer: B [Rayichapalu]
Explanation: Rayichapalu has been ocially declared as one of the rarest
species of sh in the world by the researchers from the Marine Living Resources
Department of Andhra University. The sh is a regular catch for shermen in
Visakhapatnam, Andhra Pradesh. It is a colourful sh which is around 97mm in
length and lives near the reefs. The researchers stated that this sh has a unique
colour pattern with spectacular diamond shaped bands. Reportedly, it is the only
fth sh in this category of size. Now, the tiny colourful sh will have the scientic
name Pseudathias Vizagensis, after the city where it was rst discovered.
77. Which Indian-origin personality will be honoured with the 2017 Asia Game
Changers Award?
[A] Jasleen Laghari
[B] Anuja Ravindra Dhir
[C] Dev Patel
[D] Ishani Duttagupta
Correct Answer: C [Dev Patel]
Explanation: Indian-origin actor Dev Patel will be honoured with the Asian Game
Changer award for 2017 by the Asia Society for making a transformative and
positive dierence for the future of Asia and the world. He will be felicitated with
the award at a ceremony to be held at the United Nations, New York, in
November, 2017. The Asia Society has stated that Patel has turned his attention
2017 GKToday | All Rights Reserved | www.gktoday.in

188
Current Aairs: August, 2017 [Date-wise Compendium]

and star power to philanthropy. His #lionheart campaign raised over $2,50,000
in support of Indias millions of homeless children. The other honorees include
Afghan rapper and activist Sonita Alizadeh, Cambodian environmental activist
Leng Ouch, nonprot organization Sesame Workshop, Japanese casual wear
retailer UNIQLO founder Tadashi Yanai, Chinese musician and composer Wu Tong
and Jean Liu (President of Chinese company Didi Chuxing).
78. What is the Indias rank in the 2017 Natixis Global Retirement Index (GRI)?
[A] 43rd
[B] 52nd
[C] 35th
[D] 24th
Correct Answer: A [43rd]
Explanation: India has been ranked 43rd among 43 nations in the 2017 Global
Retirement Index (GRI). The list is topped by Norway, followed by Switzerland,
Iceland and Sweden. The multi-dimensional index is compiled by Natixis Global
Asset Management with support from CoreData Research, a U.K.-based nancial
research rm. The index ranks 43 countries on the basis of four factors viz.
material well-being, health, nances and quality of life. The purpose of the index
is to provide a global benchmark for retirees and future retirees so that they can
evaluate and compare the suitability of various nations, and plan their retirement
according to their expectations and needs.
79. The International Day of the Worlds Indigenous Peoples is observed on which
date?
[A] August 7
[B] August 8
[C] August 9
[D] August 10
Correct Answer: C [August 9]
Explanation: The International Day of the Worlds Indigenous Peoples is
observed every year on August 9 to promote and protect the rights of the worlds
indigenous population. This event also recognizes the achievements and
contributions that indigenous people make to improve world issues such as
environmental protection.
80. The Keoladeo National Park (KNP) is located in which state?
[A] Uttrakhand
[B] Chhattisgarh
[C] Himachal Pradesh
2017 GKToday | All Rights Reserved | www.gktoday.in

189
Current Aairs: August, 2017 [Date-wise Compendium]

[D] Rajasthan
Correct Answer: D [Rajasthan]
Explanation: The Keoladeo National Park (KNP) is located in the Bharatpur
district of Rajasthan. It is home to Nilgai, feral cattle, chital deer, wild boar, Indian
porcupine, etc. The park is famous avifauna sanctuary that hosts thousands of
birds, especially during the winter season. The reserve protects Bharatpur from
frequent oods, provides grazing grounds for village cattle, and earlier was
primarily used as a waterfowl hunting ground.
81. The 15th meeting of the Foreign Ministers of BIMSTEC countries has started
in which city?
[A] New Delhi
[B] Colombo
[C] Naypyidaw
[D] Kathmandu
Correct Answer: D [Kathmandu]
Explanation: The 15th meeting of the Foreign Ministers of BIMSTEC countries
has started in Kathmandu, Nepal from August 10, 2017. During the two day
meeting foreign ministers of member countries will review the progress in
identied fourteen areas of cooperation and discuss ways to accelerate
cooperation under the Bay of Bengal Initiative for Multi-Sectoral Technical and
Economic Cooperation (BIMSTEC) process. The Union External Aairs Minister
Sushma Swaraj is representing India in the meeting and will hold bilateral
meetings with the leaders of the members countries.
82. Rinku Hooda, who won silver at the World U-20 Para Athletics
Championships, is associated with which sports?
[A] Javelin throw
[B] Wrestling
[C] Sprint
[D] Judo
Correct Answer: A [Javelin throw]
Explanation: Rinku Hooda (18) from India has won silver in the F46 javelin throw
event at the World U-20 Para Athletics Championships in Nottwil, Switzerland on
August 7, 2017. He got disabled as a three-year-old when his left arm got stuck in
a paddy-sowing machine. He won his rst national level medal in 2013 during a
para nationals meet at the Rajiv Gandhi stadium in Rohtak.
83. The National Skill Development Corporation (NSDC) has launched Android
2017 GKToday | All Rights Reserved | www.gktoday.in

190
Current Aairs: August, 2017 [Date-wise Compendium]

Skill Development programme with which tech giant?


[A] Microsoft
[B] Google
[C] Facebook
[D] Wipro
Correct Answer: B [Google]
Explanation: The National Skill Development Corporation (NSDC) and Google
India has launched Android Skill Development programme to impart mobile
development training on the Android and Web platform and to improve the quality
of mobile app development ecosystem in India. The NSDC is a public-private
partnership enterprise under the Ministry of Skill Development &
Entrepreneurship. Under the initiative, NSDC will introduce a specic course with
the scope to train developers to create applications for the mobile platform that
can be used on smartphone and tablets running on the Android operating system.
The NSDC and Google India will also work together to focus on up-skilling Android
trainers under the programme.
84. Which state government has launched a new awareness drive Namami
Gange Jagriti Yatra?
[A] West Bengal
[B] Bihar
[C] Uttar Pradesh
[D] Uttarakhand
Correct Answer: C [Uttar Pradesh]
Explanation: Yogi Adityanath, the Chief Minister of Uttar Pradesh, has launched
a new awareness drive called Namami Gange Jagriti Yatra from his ocial
residence in state capital Lucknow. The drive will be focusing on cleanliness along
the banks of Ganga river and maintain hygiene. The yatra will be organized by the
Uttar Pradesh Home Guards and will help in spreading awareness in about 25
districts situated on the banks of river Ganga in the state. It will urge people to
maintain the cleanliness and hygiene of the Ganga river. The drive will be
covering a total area of around 1,025 kms and will conclude on September 6,
2017.
85. Which state will become the Indias first state to implement coal utilisation
between power units?
[A] Gujarat
[B] Madhya Pradesh
[C] Rajasthan
2017 GKToday | All Rights Reserved | www.gktoday.in

191
Current Aairs: August, 2017 [Date-wise Compendium]

[D] Assam
Correct Answer: A [Gujarat ]
Explanation: The Gujarat government has recently issued a tender for power
developers to receive cheap coal marked for the state in return for selling coal-
based power at less than Rs 2.82 a unit. Through the auction, Gujarat will transfer
the coal allocated to power generating stations owned by Gujarat State Electricity
Corporation (GSECL) to more fuel-ecient private power plants. With this, Gujarat
will become the Indias rst state to implement coal utilisation between power
units. The move is in line with the initiatives of the central government to ensure
fuel availability to stressed power plants. The aim is to reduce the cost of fuel for
nancially ailing distribution companies and ensure eective distribution of
domestic coal.
86. Which parliamentary panel has recently recommended 100% pension to
widow of deceased soldier?
[A] K P Saxena panel
[B] B C Khanduri panel
[C] P C Chaku panel
[D] K T Prasad panel
Correct Answer: B [B C Khanduri panel ]
Explanation: The Parliamentary Panel headed by Maj. General (Retired) B C
Khanduri has recently tabled its report in Parliament. In the report, the panel has
recommended the 100% pension to widow of deceased soldier so that she can
take care of her family. The report stated that after the death of a soldier, the
pension of the spouse comes down to 60% of his pension. The committee feels
that after the death of a soldier, the spouse usually faces severe problems due to
the loss of the breadwinner. The committee also recommended that liberalized
and special family pensions to be exempted from income tax for the widows of
war like counter insurgency situations or on the line of control as is being given to
disabled soldiers. The panel also supported the demand of Defence Forces
representative should be included in the Central Pay Commission to address the
grievances in respect of pay and allowances and their issues.
87. Which union minister has launched the NCERT web portal for home delivery
of books?
[A] Mukhtar Abbas Naqvi
[B] Dharmendra Pradhan
[C] P P Choudhary
[D] Upendra Kushwaha
2017 GKToday | All Rights Reserved | www.gktoday.in

192
Current Aairs: August, 2017 [Date-wise Compendium]

Correct Answer: D [Upendra Kushwaha]


Explanation: Upendra Kushwaha, the Minister of State for Human Resource
Development, has launched the NCERTs Web portal for supply of textbooks to
schools and individuals at a function in New Delhi on August 9, 2017. The portal
will ensure better distribution of textbooks across the country and address the
concerns of schools and parents, regarding unavailability of textbooks. The Web
Portal can be accessed at www.ncertbooks.ncert.gov.in. The schools will be able
to log on to the portal, by entering their respective Boards aliation numbers
and other details, till 8th of next month for placing their orders for the session
2018-19. Soon, the portal will also be opened to the individual buyers. They can
place their orders by login in onto the portal, and books will be delivered at their
doorsteps with nominal postal charges. The NCERT textbooks can also be
downloaded, free of cost, from its website.
88. Sitaram Panchal, who passed away recently, was associated with which field?
[A] Journalism
[B] Politics
[C] Film Industry
[D] Sports
Correct Answer: C [Film Industry]
Explanation: Bollywood actor Sitaram Panchal (54), who was best known for his
acts in Bandit Queen and Peepli Live, has passed away in Mumbai, Maharashtra
on August 10, 2017. During his lifetime, Panchal worked in lms like Peepli Live,
Pan Singh Tomar, The Legend of Bhagat Singh, Slumdog Millionaire, Jolly LLB 2,
etc.
89. The Mandla Plant Fossils National Park (MPNP) is located in which state?
[A] Uttrakhand
[B] Chhattisgarh
[C] Madhya Pradesh
[D] Himachal Pradesh
Correct Answer: C [Madhya Pradesh]
Explanation: The Mandla Plant Fossils National Park (MPNP) is located in Mandla
district of Madhya Pradesh and covers an area of 274,100 square metres. This
national park has plants in fossil form that existed in India anywhere between 40
million and 150 million years ago spread over seven villages of Mandla District
viz. Ghuguwa, Umaria, Deorakhurd, Barbaspur, Chanti-hills, Chargaon and Deori
Kohani.
90. Which team has won the 2017 UEFA Super Cup football tournament?
2017 GKToday | All Rights Reserved | www.gktoday.in

193
Current Aairs: August, 2017 [Date-wise Compendium]

[A] Real Madrid


[B] Barcelona
[C] Manchester United
[D] Chelsea
Correct Answer: A [Real Madrid]
Explanation: Real Madrid, a professional football club of Spain, has won the 2017
UEFA Super Cup football tournament by defeating Manchester United in the nal
by 2-1 at the Philip II Arena in Skopje, Macedonia, on August 8, 2017. It is their 4th
UEFA Super Cup title.
91. ISRO will launch a full-fledged niche Earth observation (EO) satellite HySIS
to enable distinct identification of objects from space. What does HySIS stands
for?
[A] Hyperspectrum Imaging Satellite
[B] Hyperspectral Imaging Satellite
[C] Hyperspecialist Imaging Satellite
[D] Hyperspectacle Imaging Satellite
Correct Answer: B [Hyperspectral Imaging Satellite]
Explanation: ISRO will soon launch a full-edged niche Earth observation (EO)
satellite called the Hyperspectral Imaging Satellite (HySIS) by using a critical chip
it has developed. The purpose of the HySIS satellite is to enable distinct
identication of objects from about 600 km in space. The Vis-NIR Hyperspectral
payload of HySIS satellite is a hyperspectral imaging sensor operating in the
visible and near infrared regions of the electronmagnetic spectrum. The
Hyperspectral (or hyspex imaging) is said to enable distinct identication of
objects, materials or processes on Earth by reading the spectrum for each pixel of
a scene from space.
92. The Oxford University Press (OUP) has recently launched online dictionaries
in which of the following Indian languages?
[A] Tamil and Gujarati
[B] Assamese and Bangla
[C] Telugu and Marathi
[D] Gujarati and Telugu
Correct Answer: A [Tamil and Gujarati]
Explanation: The Oxford University Press (OUP) has recently launched online
dictionaries in Tamil and Gujarati languages. The launch is a part of Oxford Global
Languages (OGL) initiative with an aim to build dictionaries and lexicographical
2017 GKToday | All Rights Reserved | www.gktoday.in

194
Current Aairs: August, 2017 [Date-wise Compendium]

resources for around 100 of the worlds languages and to make them available
online. Before Tamil and Gujarati versions, the Hindi online dictionary was
launched in 2016. Thus, the OGL is a bold initiative from OUP for a modern
challenge and a huge opportunity.
93. M Venkaiah Naidu has been sworn-in as 13th Vice President of India. He hails
from which state?
[A] Andhra Pradesh
[B] Maharashtra
[C] Tamil Nady
[D] Karnataka
Correct Answer: A [Andhra Pradesh]
Explanation: M Venkaiah Naidu, the former Union Minister, has been sworn-in as
13th Vice President of India on August 11, 2017. He has been administered the
oath of oce by President Ram Nath Kovind at a special function at Rashtrapati
Bhavan in New Delhi. He was born on 1st July 1949 at Chavatapalem, in Nellore
district of Andhra Pradesh. Before being elected to the second highest
constitutional post of the country, Mr Naidu has held several positions and carried
his successful political career during his three decade long public life. He has
served as national president of BJP from 2002 to 2004. In the Atal Bihari Vajpayee
Government, he was the Union Cabinet Minister for Rural Development. In present
Narendra Modi led government, Naidu served as the Minister of Housing and
Urban Aairs, Parliamentary Aairs and Information and Broadcasting.
94. Who has been chosen for the first-ever Kesari Media Award?
[A] Adoor Gopalakrishnan
[B] R S Babu
[C] M G Radhakrishnan
[D] T J S George
Correct Answer: D [T J S George]
Explanation: Veteran journalist T J S George has been chosen for the rst Kesari
Media Award, instituted by the Kesari Memorial Journalist Trust (KMJT). The award
consists of a purse of Rs 50,000, citation and a statuette designed by renowned
artist Bhatathiri. The award would be given away at a function in
Thiruvananthapuram, Kerala, which is scheduled to be held on August 13, 2017.
95. SEBI has recently allowed Multi Commodity Exchange of India (MCX) to
launch gold options. The headquarters of MCX is located in which city?
[A] Lucknow
[B] New Delhi
2017 GKToday | All Rights Reserved | www.gktoday.in

195
Current Aairs: August, 2017 [Date-wise Compendium]

[C] Mumbai
[D] Pune
Correct Answer: C [Mumbai]
Explanation: The Securities and Exchange Board of India (SEBI) , the capital and
commodity market regulator, has recently allowed Multi Commodity Exchange of
India Ltd (MCX) to launch Indias rst gold options contract soon. It will allow
investors and hedgers to minimise their price risk at a fraction of cost compared
to currently available futures trading. The headquarters of MCX is located in
Mumbai, Maharashtra.
96. Who is the chairman of the All India Council for Technical Education (AICTE)?
[A] Alok Prakash Mittal
[B] Manik S Das
[C] Anil Sahasrabudhe
[D] Krishna S Patil
Correct Answer: C [Anil Sahasrabudhe]
Explanation: Recently, the All India Council for Technical Education (AICTE) has
decided to close down those technical colleges which recorded less than 30%
admissions in the last ve years. According to AICTE data, there are around ten
thousand three hundred fty engineering institutions in the country that are
approved by the AICTE with a total intake capacity of over 37 lakh students. Out
of this, 27 lakh seats are lying vacant. Anil D Sahasrabuddhe, Chairman of AICTE,
has stated that council has been actively working on reducing the quantity of
engineering institutions across the country due to poor demand and falling quality
of education. Beside this, the AICTE has also reduced the penalty for closing down
the institution that was a deterrent for many colleges which were willing to shut
amid poor demand.
97. The 2017 International Biodiesel Day (IBD) is celebrated on which date?
[A] August 11
[B] August 12
[C] August 10
[D] August 9
Correct Answer: C [August 10]
Explanation: The International Biodiesel Day (IBD) is celebrated every year on
August 10 with an aim to turn the focus to non-fossil fuels. The day also honours
the research experiments by Sir Rudolf Diesel who ran an engine with peanut oil
in the year of 1893. The Biodiesel is an alternative fuel which can be used in place
of fossil fuels. It is manufactured from vegetable oils, recycled grease, algae, and
2017 GKToday | All Rights Reserved | www.gktoday.in

196
Current Aairs: August, 2017 [Date-wise Compendium]

animal fat.
98. Which city to host the Global Entrepreneurship Summit (GES-2017)?
[A] New Delhi
[B] Mumbai
[C] Chennai
[D] Hyderabad
Correct Answer: D [Hyderabad]
Explanation: India and United States (US) will co-host the Global
Entrepreneurship Summit (GES-2017) at Hyderabad from November 28-30, 2017.
The summit is a unique opportunity for bringing together entrepreneurs and start
ups with global leaders. It will promote economic growth, inclusion and
opportunities among entrepreneurs from across the world, with a special focus on
emerging nations from Africa and Asia.
99. Who has become the first Indian athlete to qualify for javelin throw final at
the IAAF World Athletics Championships (WAC)?
[A] Anil Singh
[B] Davinder Singh Kang
[C] Devendra Jhajharia
[D] Neeraj Chopra
Correct Answer: B [Davinder Singh Kang]
Explanation: Davinder Singh Kang from Punjab has become the rst Indian to
qualify for the nal round of the javelin throw at the IAAF World Athletics
Championships in London. He was under pressure to touch the 83 m in his nal
attempt and he was the last man to take the throw. He did it in style by sending
the spear beyond the qualication mark to the huge relief of the Indian camp.
Kangs best eort of 84.22m put him in the seventh position among the nal
round qualiers.
100. The Vansda National Park (VNP) is located in which state?
[A] Maharashtra
[B] Gujarat
[C] Assam
[D] Rajasthan
Correct Answer: B [Gujarat ]
Explanation: The Vansda National Park (VNP) is a protected area which
represents the thick woodlands of the Dangs and southern Gujarat and is situated
in the Vansda tehsil, Navsari District of Gujarat. It is home to important animals
2017 GKToday | All Rights Reserved | www.gktoday.in

197
Current Aairs: August, 2017 [Date-wise Compendium]

such as Indian leopard, rhesus macaque, common palm civet, Hanuman langur
and small Indian civet. There are 443 species of owering plants like teak, sadad,
khakhro, kadad, humb, timru, kalam, bamboo, dudhkod, mahudo, behda, umaro,
kusum, tanach, asan, shimlo, etc.
101. Who has won the 2017 Kenya Presidential election?
[A] William Ruto
[B] Raifa Odinga
[C] Yoweri Museveni
[D] Uhuru Kenyatta
Correct Answer: D [Uhuru Kenyatta]
Explanation: Uhuru Kenyatta, the current President of Kenya, has ocially
declared the winner of the 2017 Kenya Presidential election amid allegations of
voting fraud by the opposition. According to ocial gures released by the
countrys electoral commission, Kenyatta secured 54.27% of the ballots cast,
while his rival, Raila Odinga, won 44.74%.
102. Maheshwari Chauhan is associated with which sports?
[A] Shooting
[B] Cricket
[C] Boxing
[D] Chess
Correct Answer: A [Shooting]
Explanation: Indian Shooter Maheshwari Chauhan has won a bronze at the 7th
Asian Shotgun Championship to earn Indias rst womens skeet individual medal
in Astana, Kazakhstanon on August 11, 2017. Before that, Maheshwari also led
India to the team silver alongside compatriots Rashmmi Rathore and Saniya
Sheikh in the tournament. Besides Maheshwari , Indian shooter Ankur Mittal
clinched the individual as well as the team gold medal in mens double trap in the
championship. And, Kynan Chenai from Hyderabad won a bronze in mens trap
event as well as also won bronze in mixed team trap event with partner Shreyasi
Singh in the championship. Now, Indias tally stands at two gold, silver and three
bronze medals.
103. What is the theme of 2017 International Youth Day (IYD)?
[A] Change Our World
[B] Youth and Mental Health
[C] Youth Building Peace
[D] Youth Civic Engagement

2017 GKToday | All Rights Reserved | www.gktoday.in

198
Current Aairs: August, 2017 [Date-wise Compendium]

Correct Answer: C [Youth Building Peace]


Explanation: The International Youth Day (IYD) is observed every year on August
12 to draw attention to a given set of cultural and legal issues surrounding youth.
The 2017 theme is Youth Building Peace, which recognizes the contributions of
young people to preventing conict, supporting inclusion, social justice, and
sustainable peace.
104. Which Indian sportsperson has been appointed as the WHO Goodwill
Ambassador for physical activity in the South-East Asia Region (SEAR)?
[A] Mary Kom
[B] Milkha Singh
[C] Pullela Gopichand
[D] Abhinav Bindra
Correct Answer: B [Milkha Singh]
Explanation: Milkha Singh, the legendary athlete, has been appointed as the
World Health Organisations (WHO) Goodwill Ambassador for physical activity in
the South-East Asia Region (SEAR). As goodwill ambassador, he will promote WHO
SEARs Non-Communicable Diseases (NCDs) prevention and control action plan
which seeks to reduce the level of insucient physical activity by 10% and NCDs
by 25% by 2025. Milkha Singh, also known as the Flying Sikh, is a Padma Shri
awardee and had won medals in athletics in Commonwealth and Asian games.
105. Who has been appointment as the new chief of the Central Board of Film
Certification (CBFC)?
[A] Shyam Benegal
[B] Prasoon Joshi
[C] Naresh Chandra Lal
[D] Vivek Agnihotri
Correct Answer: B [Prasoon Joshi]
Explanation: Prasoon Joshi, the famous lyricist, has been appointment as the
new chief of the Central Board of Film Certication (CBFC) for a period of three
years. He succeeded Pahlaj Nihalani, who has been accused of autocratic conduct
and of morally policing lms. Joshi is known for putting forth his opinion on
contemporary issues. He became more prominent after penning songs for lms
like Fanaa, Rang De Basanti, Black, Delhi 6 and Bhaag Milkha Bhaag. He twice
won the National Award for Best Lyricist rst in 2008 for Maa (Taare Zameen
Par) and then in 2013 for Bolo Na in Chittagong. He also wrote the lyrics for the
anthem of Prime Minister Narendra Modis Swachh Bharat Abhiyan.

2017 GKToday | All Rights Reserved | www.gktoday.in

199
Current Aairs: August, 2017 [Date-wise Compendium]

106. Which of the following have won the 2017 The Hindu Playwright Award?
[A] Deepika Arwind and K. Madavane
[B] Manjima Chatterjee and Arshia Sattar
[C] Gerish Khemani and Akshat Nigam
[D] Sunil Shanbag and Keval Arora
Correct Answer: C [Gerish Khemani and Akshat Nigam]
Explanation: Gerish Khemani and Akshat Nigam have won The Hindu Playwright
Award 2017 for their play In Search of Dariya Sagar at the 13th edition of The
Hindu Theatre Fest in Mumbai. In Search of Dariya Sagar is a play that brings
together public events and private lives, political history and personal memory, in
a manner that resonates both the real and the magical. The Rs 2 lakh award,
instituted in 2008 to reward the best new unpublished and unperformed play-
script in English.
107. Which nationwide campaign has been launched by the Union Government to
protect elephants?
[A] Gaj Kesari Yatra
[B] Gajanan Yatra
[C] Gajraj Yatra
[D] Gaj Yatra
Correct Answer: D [Gaj Yatra]
Explanation: Dr. Harsh Vardhan, the Union Environment, Forest and Climate
Change Minister, has launched a nationwide campaign Gaj Yatra in New Delhi to
protect elephants on the occasion of World Elephant Day (WED). The campaign is
planned to cover 12 elephant range states. The World Elephant Day (WED) is
celebrated every year on August 12 to focus the attention of various stakeholders
in supporting various conservation policies to protect elephants, including
improving enforcement policies to prevent illegal poaching and trade in ivory.
108. Which state government has launched mobile therapy vans for elderly
people?
[A] Bihar
[B] Assam
[C] Uttar Pradesh
[D] Kerala
Correct Answer: A [Bihar]
Explanation: Nitish Kumar, the Chief Minister of Bihar, has recently agged o
mobile therapy vans which would provide basic medical assistance to elderly
2017 GKToday | All Rights Reserved | www.gktoday.in

200
Current Aairs: August, 2017 [Date-wise Compendium]

people, widows and other section who requires social security. The therapy vans
will be equipped with basic medical apparatus and accompanied by a technical
team. The van is an outcome of a joint project of the World Bank and the Bihar
government which is run by Social Welfare department.
109. Ruth Pfau, popularly known as Pakistans Mother Teresa has passed away.
She hailed from which country?
[A] Italy
[B] United States
[C] Germany
[D] France
Correct Answer: C [Germany]
Explanation: Dr Ruth Pfau (87), a German doctor and nun who dedicated her life
to eradicating leprosy in Pakistan, has passed in Karachi, Pakistan on August 10,
2017. She was hailed as Pakistans Mother Teresa after her work towards
helping people displaced by the 2010 Pakistan oods. Dr. Pfau trained Pakistani
doctors and attracted foreign donations, founded Pakistans National Leprosy
Control Programme and the Marie Adelaide Leprosy Centre, which has a presence
in every Pakistani province. She received numerous awards in Pakistan and
Germany including the Hilal-e-Imtiaz Pakistans second highest civilian award
in 1979, the Hilal-e-Pakistan in 1989 and the German Staufer Medal in 2015. She
wrote four books in German about her work in Pakistan, including To Light A
Candle, which has been translated into English.
110. The Grizzled Squirrel Wildlife Sanctuary (GSWS) is located in which state?
[A] Uttrakhand
[B] Tamil Nadu
[C] Gujarat
[D] Chhattisgarh
Correct Answer: B [Tamil Nadu]
Explanation: The Grizzled Squirrel Wildlife Sanctuary (GSWS) is located in
Virudhunagar and Madurai districts of Tamil Nadu and covers an area 485.2 sq
Kms. Over 240+ species of birds are seen in this sanctuary including 14 species of
birds endemic to the Western Ghats, such as the critically endangered Oriental
white-backed vulture and the long-billed vulture. In addition to grizzled giant
squirrels, the sanctuary is home to numerous animals such as barking deer,
bonnet macaque, common langur, elephants, ying squirrels, gaur, Indian giant
squirrel, leopard, lion-tailed macaques, mouse deer, Nilgiri langur and Nilgiri
Tahrs. There are over 220 species of butteries including many rare and endemic
species. Some of the rare butteries spotted here include Eversheds ace , Silver
2017 GKToday | All Rights Reserved | www.gktoday.in

201
Current Aairs: August, 2017 [Date-wise Compendium]

royal , Orange awlet , Hampsons hedge blue , etc.


111. The Union Government will launch a new sub-scheme AGEY as part of the
DAY NRLM. What does AGEY stands for?
[A] Aagam Grameen Express Yojana
[B] Aajeevika Grameen Express Yojana
[C] Angad Grameen Express Yojana
[D] Ahilya Grameen Express Yojana
Correct Answer: B [Aajeevika Grameen Express Yojana]
Explanation: The Government of India will soon launch a new sub-scheme
named Aajeevika Grameen Express Yojana (AGEY) as part of the Deendayal
Antyodaya Yojana National Rural Livelihoods Mission (DAY-NRLM). Initially, AGEY
will be implemented in 250 Blocks in the country on pilot basis with each Block
provided upto 6 vehicles to operate the transport services. The Self Help Groups
(SHGs) under DAY-NRLM will operate road transport service in backward areas.
This will help to provide safe, aordable and community monitored rural transport
services to connect remote villages with key services and amenities (such as
access to markets, education and health) for the overall economic development of
backward rural areas. This will also provide an additional avenue of livelihood for
SHGs.
112. Which bank has recently launched two digital products: Bharat QR and
Bharat Bill Payment System (BBPS) for online payments?
[A] Central Bank
[B] Dena Bank
[C] Canara Bank
[D] Punjab National Bank
Correct Answer: C [Canara Bank]
Explanation: The Canara Bank has recently launched two digital products:
Bharat QR, a platform for payment settlement at merchant locations, and Bharat
Bill Payment System (BBPS) is an integrated bill payment system for Net banking
customers. The rst service is a mobile application, which enables a merchant to
accept payments as settlement for sale of goods or services. The second product
(BBPS), an initiative of the RBI oers bill payment services to the banks net
banking customers with network of agents.
113. Vijay Nambisan, who passed away recently, was the renowned personality of
which field?
[A] Poetry
[B] Politics
2017 GKToday | All Rights Reserved | www.gktoday.in

202
Current Aairs: August, 2017 [Date-wise Compendium]

[C] Sports
[D] Law
Correct Answer: A [Poetry]
Explanation: Vijay Nambisan (54), the well-known poet, writer and journalist of
India writing in English, has passed away on August 10, 2017. He was the rst-
ever All India Poetry Champion in 1988 for his poem Madras Central. He co-
authored a book of poems Gemini with Dom Moraes and Jeet Thayil. Nambisan
passionately championed the ethical use of language and made an appeal to
readers, writers and enthusiasts of literature in his monumental book Language
as an Ethic. His poetry served keen observations about politics both internal
and external with gentle wit, often directed at himself.
114. Which state government has launched Facebooks She Means Business
programme to train women entrepreneurs?
[A] Assam
[B] Karnataka
[C] Kerala
[D] Odisha
Correct Answer: D [Odisha]
Explanation: Naveen Patnaik, the Chief Minister of Odisha, has recently launched
Facebooks She Means Business programme in Bhubaneswar to train women
entrepreneurs in the state. For this, Facebook has partnered the Odishas MSME
department and Project Mission Shakti to train 25,000 women entrepreneurs and
self-help group (SHG) members on digital marketing skills by the end of 2018.
After imparting training, Facebook will make a database of entrepreneurs in the
state and will monitor their growth, turnover and prot after one year.
115. Which state is hosting the 2017 India-ASEAN Youth Summit?
[A] Uttar Pradesh
[B] Haryana
[C] Madhya Pradesh
[D] Assam
Correct Answer: C [Madhya Pradesh]
Explanation: The 2017 India-ASEAN Youth Summit has started in Bhopal,
Madhya Pradesh on August 14, 2017 with theme of Shared Values, Common
Destiny. The event is being organised by Foreign Ministry, Madhya Pradesh
government and India Foundation. The ve day Summit has inaugurate by Union
Minister of State for External Aairs General VK Singh, Union Minister of State for
Youth Aairs and Sports Vijay Goel and Madhya Pradesh Chief Minister Shivraj
2017 GKToday | All Rights Reserved | www.gktoday.in

203
Current Aairs: August, 2017 [Date-wise Compendium]

Singh Chouhan. The exercise will mark the 25th anniversary of the ASEAN-India
dialogue partnership in the ongoing year 2017. Madhya Pradesh has been chosen
for this event as the Sanchi stupa here strengthens the states cultural aliation
with the ASEAN countries. Over 175 delegations of youths from Indonesia,
Singapore, Philippines, Malaysia, Brunei, Thailand, Cambodia, Laos, Myanmar,
Vietnam and India are participating in the Summit.
116. Which Indian pair has won gold in the skeet mixed team event at the 7th
Asian Championship Shotgun?
[A] Ankur Mittal and Shreyasi Singh
[B] Mairaj Ahmad Khan and Rashmmi Rathore
[C] Kynan Chenai and Shreyasi Singh
[D] Maheshwari Chauhan and Mairaj Ahmad Khan
Correct Answer: B [Mairaj Ahmad Khan and Rashmmi Rathore]
Explanation: The Indian duo of Mairaj Ahmad Khan and Rashmmi Rathore has
won a gold medal in the skeet mixed team event at the 7th Asian Championship
Shotgun in Astana, Kazakhstan on August 13, 2017. With this, India nished the
championship with a total of 8 medals. Earlier, Ankur Mittal had won a gold in the
mens double trap and had also helped India bag the team gold in the event.
Kynan Chenai won a bronze in the mens trap and also partnered Shreyasi Singh
to claim the trap mixed team bronze. Maheshwari Chauhan clinched a bronze in
the womens skeet, along with team silver in the event before the mens skeet
team picked up another bronze. The results will serve as good encouragement for
the 2017 Shotgun World Championships in Moscow, which is scheduled to start
from August 30.
117. Yisrael Kristal, the worlds oldest man passed away. He hailed from which
country?
[A] Israel
[B] Japan
[C] France
[D] Germany
Correct Answer: A [Israel]
Explanation: Yisrael Kristal (113), the worlds oldest man who lived through both
World Wars, has passed away in Haifa, Israel on August 11, 2017. He was ocially
recognised as the worlds oldest man by the Guinness Book of Records in March
2016. Kristal, a Polish-Israeli supercentenarian, became the worlds oldest
recognized Holocaust survivor in 2014 and the worlds oldest man in 2016.
118. The 2017 World Elephant Day (WED) is observed on which date?
2017 GKToday | All Rights Reserved | www.gktoday.in

204
Current Aairs: August, 2017 [Date-wise Compendium]

[A] August 13
[B] August 14
[C] August 12
[D] August 11
Correct Answer: C [August 12 ]
Explanation: The World Elephant Day (WED) is observed every year on August
12 to focus the attention of various stakeholders in supporting various
conservation policies to protect elephants, including improving enforcement
policies to prevent illegal poaching and trade in ivory. On the occasion of 2017
WED, Dr. Harsh Vardhan, the Union Environment, Forest and Climate Change
Minister, has launched a nationwide campaign Gaj Yatra in New Delhi to protect
elephants in the country. The campaign is planned to cover 12 elephant range
states.
119. Which state government has launched Namo Yuva Rojgar Kendra for
unemployed youth?
[A] Tamil Nadu
[B] Gujarat
[C] Maharashtra
[D] Rajasthan
Correct Answer: C [Maharashtra]
Explanation: Devendra Fadnavis, the Chief Minister of Maharashtra, has
launched Namo Yuva Rojgar Kendra on August 14, 2017 to provide job
opportunities to unemployed youth in Mumbai. Under this initiative, there are
various schemes like farmers weekly market, farmers mobile market and
mothers tin. These products will be made available with the help of mobile vans
all over Mumbai and the information about the location of the vans is made
available through an app.
120. The Orang National Park (ONP) is located in which state?
[A] Manipur
[B] Mizoram
[C] Tripura
[D] Assam
Correct Answer: D [Assam ]
Explanation: The Orang National Park (ONP) is located on the north bank of the
Brahmaputra River in the Darrang and Sonitpur districts of Assam and covers an
area of 78.81 square kilometers. The park has a rich ora and fauna, including

2017 GKToday | All Rights Reserved | www.gktoday.in

205
Current Aairs: August, 2017 [Date-wise Compendium]

great Indian one-horned rhinoceros, pigmy hog, elephants, wild bualo and tigers.
It is the only stronghold of rhinoceros on the north bank of the Brahmaputra river.
121. Which union minister has launched the live twitter wall for promoting
Bharat Ke Veer portal?
[A] Narendra Modi
[B] Narendra Singh Tomar
[C] Rajnath Singh
[D] Arun Jaitley
Correct Answer: C [Rajnath Singh]
Explanation: Union Home Minister Rajnath Singh has launched the live twitter
wall for promoting Bharat Ke Veer portal on the eve of 71st Independence Day.
According to an ocial release, this social media awareness campaign will display
all tweets with hashtag Bharat Ke Veer at Connaught Place, CGO complex and
Khan Market in New Delhi where huge LED display screens have been installed.
This portal facilitates online donation directly to the families of CAPF Martyrs who
have sacriced their lives for the country in line of duty from January 2016. The
Bharat Ke Veer portal was launched by Rajnath Singh and actor Akshay Kumar on
April 2017.
122. Dr Bhakti Yadav, who passed away recently, was the first woman doctor of
which state?
[A] Uttar Pradesh
[B] Rajasthan
[C] Haryana
[D] Madhya Pradesh
Correct Answer: D [Madhya Pradesh]
Explanation: Dr Bhakti Yadav (92), the rst woman doctor of Madhya Pradesh,
has recently passed away in Indore on August 14, 2017. Her life epitomised the
power of compassion and service and she is an inspiration for all. Dr Yadav had
oered free treatment to patients for last 68 years. She was awarded Padma Shri
for her exemplary services in the eld of medicine in April 2017.
123. Indias first-ever railways disaster management centre will come up in which
city?
[A] Kolkata
[B] Bengaluru
[C] Lucknow
[D] Pune
2017 GKToday | All Rights Reserved | www.gktoday.in

206
Current Aairs: August, 2017 [Date-wise Compendium]

Correct Answer: B [Bengaluru]


Explanation: Indias rst-ever railways disaster management centre is expected
to come up near Bengaluru, Karnataka by 2018 and will be used to replicate train
accidents to improve rescue operation eorts. It will also have a virtual reality
centre for analyzing railway accidents. The focus is on imparting state-of-the-art
training on rescue, medical relief and rolling stock restoration techniques in a
classroom and also using various simulated teaching aids. Though, it is the
practical reproduction of accidents on site and the ensuing rescue operations that
are being touted as the big ticket venture.
124. Indias first-ever aviation university will come up in which state?
[A] Uttar Pradesh
[B] Assam
[C] Maharashtra
[D] Tamil Nadu
Correct Answer: A [Uttar Pradesh]
Explanation: The Rajiv Gandhi National Aviation University (RGNAU), which is
Indias rst-ever aviation university, will soon be inaugurated at Fursatganj in Rae
Bareli, Uttar Pradesh. The central university is planned as an autonomous body
under the administrative control of the Ministry of Civil Aviation with an aim to
facilitate and promote aviation studies, teaching, training and research. Air Vice
Marshal (retired) Nalin Tandon has been appointed as the Vice Chancellor.
125. Which country will chair the IONS maiden International Maritime Search
and Rescue Exercise (IMMSAREX)-2017?
[A] India
[B] Bangladesh
[C] China
[D] Nepal
Correct Answer: B [Bangladesh ]
Explanation: The International Maritime Search and Rescue Exercise
(IMMSAREX) will be chaired by Bangladesh at the Indian Ocean Naval Symposium
(IONS) in the Bay of Bengal in November 2017. The ships and aircraft of the
members and observers of the IONS including China, India and Pakistan will
participate in the exercise. The IONS is a regional forum of Indian Ocean littoral
states, represented by their Navy chiefs, launched by India in February 2008. It
presently has 23 members and nine observers.
126. Who has been appointed as the new CMD of Hindustan Copper Limited

2017 GKToday | All Rights Reserved | www.gktoday.in

207
Current Aairs: August, 2017 [Date-wise Compendium]

(HCL)?
[A] Nidhi Sen
[B] Kirti Singh
[C] J P Gupta
[D] Santosh Sharma
Correct Answer: D [Santosh Sharma]
Explanation: Santosh Sharma has been appointed as the new Chairman and
Managing Director (CMD) of Hindustan Coper Limited (HCL). Currently, he is
Director (Operations) in the HCL. According to an order issued by the Department
of Personnel and Training (DoPT), the Appointments Committee of the Cabinet
(ACC) has approved his appointment to the post for a period of ve years.
127. Which country to host 2017 G7 Interior Ministers summit on security issues?
[A] Germany
[B] Canada
[C] Italy
[D] Japan
Correct Answer: C [Italy ]
Explanation: Italy will host a summit of G7 Interior Ministers centered on security
issues, which is scheduled to be held in October 2017. The meeting is being
organised at the request of Italys G-7 partners- Canada, France, Germany, the
United Kingdom, Japan and the United States. Currently, Italy holds the rotating
helm of the G-7 industrialised nations.
128. Who is the newly appointed CEO of Rajya Sabha TV (RSTV)?
[A] Gurdeep Singh Sappal
[B] Surya Prakash
[C] Vinay Sahasrabuddhe
[D] Shashi Shekhar Vempati
Correct Answer: D [Shashi Shekhar Vempati]
Explanation: Shashi Shekhar Vempati, the Chief Executive Ocer (CEO) at
public broadcaster Prasar Bharati, has been given the additional charge of CEO in
Rajya Sabha TV (RSTV). Vempati succeeded Gurdeep Singh Sappal, who has been
serving as the CEO and editor-in-chief of Rajya Sabha TV since 2011. The RSTV is
owned and operated by Rajya Sabha and the Vice-President is its chairman. It
covers the proceedings of the upper house of the Parliament. Prasar Bharati runs
Doordarshan television and All India Radio (AIR).
129. Who has been appointed as the new MD & CEO of India Infrastructure
2017 GKToday | All Rights Reserved | www.gktoday.in

208
Current Aairs: August, 2017 [Date-wise Compendium]

Finance Company Limited (IIFCL)?


[A] Nalin Tandon
[B] Sankara Rao
[C] P S Murthy
[D] Chandan Kumar
Correct Answer: B [Sankara Rao]
Explanation: Sankara Rao has been appointed as the new Managing Director &
Chief Executive Ocer (CEO) of the India Infrastructure Finance Company Limited
(IIFCL). According to an order issued by the Department of Personnel and Training
(DoPT), the Appointments Committee of the Cabinet (ACC) has approved his
appointment to the post for a period of 3 years. Currently, he is the Chief General
Manager at IIFCL. Rao is also currently a director and chief executive ocer at
IIFCL Asset Management Company Ltd.
130. Stephen Wooldridge, the former world cycle champion has passed away. He
hailed from which country?
[A] United States
[B] France
[C] Australia
[D] Germany
Correct Answer: C [Australia]
Explanation: Stephen Wooldridge (39), Australias former Olympic and world
cycle champion, has passed away in Australia on August 14, 2017. The former
star team pursuiter won gold for Australia in the 2004 Athens Olympics, was also
a four-time world champion in the discipline and the 2002 Commonwealth Games
champion. There were no details about when and how Wooldridge died.
131. The CCEA has approved the proposal of DIPAM for strategic disinvestment.
What does DIPAM stands for?
[A] Department of Investment and Proper Asset Management
[B] Department of Investment and Public Asset Management
[C] Department of Investment and Property Asset Management
[D] Department of Investment and Private Asset Management
Correct Answer: B [Department of Investment and Public Asset Management]
Explanation: The Cabinet Committee on Economic Aairs (CCEA) has recently
approved the proposal of Department of Investment and Public Asset
Management (DIPAM) for the strategic disinvestment. The approval will help in
speedy completion of strategic disinvestment transactions. The department will
2017 GKToday | All Rights Reserved | www.gktoday.in

209
Current Aairs: August, 2017 [Date-wise Compendium]

set up an alternative mechanism comprising of the Finance Minister, Minister for


Road Transport and Highways and Minister of Administrative Department, to
decide on the matters relating to terms and conditions of the sale from the stage
of inviting of Express of Interests till inviting of nancial bid. It will also empower
the Core Group of Secretaries to take policy decisions with regard to procedural
issues and to consider deviations as necessary from time to time for eective
implementation of decisions of CCEA.
132. Who has won the 2017 Kalpana Chawla Award for courage and daring
enterprise?
[A] Preethi Srinivasan
[B] K. Satyagopal
[C] S.P. Thyagarajan
[D] V.P. Elayapari
Correct Answer: A [Preethi Srinivasan]
Explanation: Preethi Srinivasan, the former State under-19 cricket captain, has
bestowed with the 2017 Kalpana Chawla Award for courage and daring enterprise
from Chief Minister Edappadi K. Palaniswami during the 71st Independence Day
celebrations at Fort St. George in Chennai, Tamil Nadu. Beside her, the
Government of Tamil Nadu has also awarded S.P. Thyagarajan with the APJ Abdul
Kalam award for discovering a patented drug to treat hepatitis B.
133. Which Indian shuttler has won the mens singles 2017 Bulgaria Open
International Series tournament?
[A] Valiyaveetil Diju
[B] HS Prannoy
[C] Kidambi Srikanth
[D] Lakshya Sen
Correct Answer: D [Lakshya Sen]
Explanation: Young Indian shuttler Lakshya Sen has clinched the 2017 Bulgaria
Open International Series title after defeating Croatias Zvonimir Durkinjak in the
mens singles nal by 18-21 21-12 21-17 in Soa, Bulgaria.
134. The Desert National Park (DNP) is located in which state?
[A] Gujarat
[B] Haryana
[C] Rajasthan
[D] Assam
Correct Answer: C [Rajasthan ]
2017 GKToday | All Rights Reserved | www.gktoday.in

210
Current Aairs: August, 2017 [Date-wise Compendium]

Explanation: The Desert National Park (DNP) is located near Jaisalmer, Rajasthan
and covers an area of 3162 km. The park is an excellent example of the
ecosystem of the Thar Desert. The sand dunes form around 20% of the Park. It is
home to desert fox, Bengal fox, desert cat, wolf, hedgehog, blackbuck and
chinkara.
135. Which country has been declared the worlds most liveable city by The
Economists Global Liveability Report-2017?
[A] Melbourne
[B] Sydney
[C] London
[D] Perth
Correct Answer: A [Melbourne ]
Explanation: Melbourne has been declared the worlds most liveable city for the
7th consecutive year by The Economists annual global liveability survey, despite
soaring housing costs and serious congestion challenges. The annual survey
assesses 140 cities and ranks them according to their stability, healthcare, culture
and environment, education and infrastructure. Others in the 10 most liveable
cities in the world include Vienna (3rd), followed by Vancouver, Toronto, Calgary,
Adelaide, Perth, Auckland, Helsinki and Hamburg. As per the survey, the least
liveable city was Damascus at 140th place, followed by Lagos (139), Tripoli (138),
Dhaka (137), Port Moresby (136), Algiers (135), Karachi (134), Harare (133),
Douala (132) and Kiev (131).
136. Which country to host the steering committee of the proposed TAPI Gas
pipeline?
[A] Turkmenistan
[B] Afghanistan
[C] Pakistan
[D] India
Correct Answer: D [India]
Explanation: India will host the next steering committee meeting of the
proposed 1,814 kilometre-long Turkmenistan-Afghanistan-Pakistan-India (TAPI)
gas pipeline in New Delhi. The decision was taken during the 6th joint Inter-
Governmental Committee (IGC) meeting on trade, economic, scientic and
technological cooperation. The benet of the TAPI project is that it will give an
opportunity to India to secure its interest in Central Asia. The projects success
will also ensure that India, Pakistan and Afghanistan nd ways of cooperating on
other issues as well.
2017 GKToday | All Rights Reserved | www.gktoday.in

211
Current Aairs: August, 2017 [Date-wise Compendium]

137. Indias first museum on the Partition of the Indian Sub-continent will open in
which state?
[A] Uttar Pradesh
[B] Odisha
[C] Punjab
[D] Assam
Correct Answer: C [Punjab]
Explanation: Indias rst museum on the Partition of the Indian Sub-continent
will soon open in Amritsar, Punjab, which focuses on stories and display
memorabilia of survivors. The museum will include newspaper clippings and
donated personal items meant to tell the story of how the regions struggle for
freedom from colonial rule turned into one of its most violent episodes, as
communal clashes left hundreds of thousands of Hindus, Muslims and Sikhs dead
and another 15 million displaced from their ancestral homes. It will also have The
Gallery of Hope, where visitors can scribble messages of love and peace. Tickets
are priced low at Rs10 for Indians and Rs150 for foreigners.
138. NABARD has conferred the best performance award for 2016-17 to which
regional rural bank?
[A] Telangana Grameena Bank
[B] Allahabad UP Gramin Bank
[C] Karnataka Vikas Grameen Bank
[D] Madhyanchal Gramin Bank
Correct Answer: C [Karnataka Vikas Grameen Bank]
Explanation: The National Bank for Agriculture and Rural Development
(NABARD) has conferred the best performance award for 2015-16 and 2016-17
to Karnataka Vikas Grameen Bank (KVGB) for its work in the eld of credit-linking
joint liability groups (JLGs). The KVGB is sponsored by syndicate bank.
139. Which state government has launched Indira Canteen for urban poor?
[A] Karnataka
[B] Himachal Pradesh
[C] Puducherry
[D] Punjab
Correct Answer: A [Karnataka]
Explanation: The Karnataka government and Congress Vice President Rahul
Gandhi have launched Indira Canteen in Bengaluru with an aim to reach out to
the poorer sections of the society. The canteens will oer breakfast at Rs 5 and
2017 GKToday | All Rights Reserved | www.gktoday.in

212
Current Aairs: August, 2017 [Date-wise Compendium]

lunch and dinner at Rs 10 per plate. Of a total of proposed 198 canteens, 101
were launched on August 16, 2017 and the rest would start functioning from
October 2, 2017.
140. Who has authored the book The Adivasi Will Not Dance?
[A] Hansda Sowvendra Shekhar
[B] Abhilash Pudukad
[C] Amit Shankar
[D] Chetan Bhagat
Correct Answer: A [Hansda Sowvendra Shekhar]
Explanation: The book The Adivasi Will Not Dance has been authored by
Hansda Sowvendra Shekhar. It is a mature, passionate, intensely political book of
stories, made up of the very stu of life. Recently, the book is in news because
the Jharkhand government has banned the book on the grounds that it portrayed
the Santhal women in a bad light. The writer himself belongs to the Santhal
adivasi community, which is numerically among the largest indigenous groups.
Shekhar won the Sahitya Akademi Yuva Puruskar in 2015 for his novel The
Mysterious Ailment of Rupi Baskey.
141. S Paul, who passed away recently, was the renowned personality of which
field?
[A] Politics
[B] Sports
[C] Law
[D] Photography
Correct Answer: D [Photography]
Explanation: S Paul (88), the veteran photographer, has passed away in New
Delhi on August 16, 2017. He was a self-taught photographer who discovered his
passion for the art as a teenager. He joined The Indian Express in the 1960s and
retired from the newspaper in 1989. Paul received recognition and fame as a
professional photographer when his pictures were selected for publication in three
renowned British publications Amateur Photographer, Miniature Camera and
Miniature Camera World. B&W Magazine of US had described him as The Henry
Cartier-Bresson of India. Apart from other recognitions, Paul was the rst Indian
to be proled by The British Journal of Photography in 1967. He was the rst
Indian to win the Nikon International Photo Contest in 1971. Paul was born on 19
August, 1929 in Jhang, Pakistan. He moved to India after Partition and his family
shifted to Shimla. His younger brother, Raghu Rai, is also a renowned
photographer

2017 GKToday | All Rights Reserved | www.gktoday.in

213
Current Aairs: August, 2017 [Date-wise Compendium]

142. Which union minister has launched Harit Diwali, Swasth Diwali campaign
to celebrate pollution-free diwali?
[A] Arun Jaitley
[B] Narendra Modi
[C] Rajnath Singh
[D] Harsh Vardhan
Correct Answer: D [Harsh Vardhan]
Explanation: Dr. Harsh Vardhan, Union Minister for Environment, Forest and
Climate Change, has launched the Harit Diwali, Swasth Diwali campaign to
celebrate pollution-free diwali. Diwali is an integral part of our rich tradition and a
festival that embodies joy and happiness. In recent times, the pattern of
celebration has changed somewhat and has got associated with excessive
bursting of crackers, which contributes signicantly to air and noise pollution. As a
result, there has been a signicant impact on the environment and health of the
people. Thus, as a part of the campaign, the Environment Ministry will undertake
various activities to create awareness among various stakeholders and encourage
people to participate in combating air pollution.
143. The National Highways Authority of India (NHAI) has launched which
consumer mobile app for smooth functioning of electronic toll collection?
[A] MyFASTag
[B] MFASTag
[C] MineFASTag
[D] MiFASTag
Correct Answer: A [MyFASTag]
Explanation: The National Highways Authority of India (NHAI) has launched two
mobile Apps namely MyFASTag and FASTag Partner in New Delhi to ensure
smooth functioning of the Electronic Toll Collection system. The mobile apps will
ease the process and will make it possible to buy or recharge FASTags at the click
of a mobile button. Basically, MyFASTag is a consumer App that can be
downloaded from the App Store for both Android and iOS systems. A consumer
can purchase or recharge FASTags on this App. The app also helps to keep track
of transactions and provides for online grievance redressal. While, the FASTag
Partner app is for banks and agencies selling the tag. A total of 10 banks,
including mobile wallets, provide FASTags.
144. Which football club has won the 2017 Spanish Super Cup?
[A] Manchester United
[B] Real Madrid
2017 GKToday | All Rights Reserved | www.gktoday.in

214
Current Aairs: August, 2017 [Date-wise Compendium]

[C] Barcelona
[D] Arsenal
Correct Answer: B [Real Madrid]
Explanation: Real Madrid football club has won the Spanish Super Cup for the
10th time with a 2-0 win over Barcelona at a joyous Bernabeu on August 16, 2017
to complete a 5-1 aggregate victory.
145. Who has taken charge as the new Pakistans high commissioner to India?
[A] Salman Bashir
[B] Sohail Mahmoodm
[C] Mir Basit
[D] Shahid Malik
Correct Answer: B [Sohail Mahmoodm]
Explanation: Sohail Mahmoodm, the senior diplomat, has taken charge as the
new Pakistans high commissioner to India. He succeeded Abdul Basit. The High
Commission of Pakistan in New Delhi is the diplomatic mission of Pakistan in India.
Prior to this, Mahmood served as Pakistans ambassador in Turkey.
146. The 2017 World Photography Day (WPD) is observed on which date?
[A] August 21
[B] August 18
[C] August 19
[D] August 17
Correct Answer: C [August 19]
Explanation: The World Photography Day (WPD) is observed every year on
August 19 to inspire photographers across the planet to share a single photo with
a simple purpose: to share their world with the world. The date of August 19 was
chosen to commemorate the invention of the Daguerreotype, a photographic
process developed by Frenchmen Louis Daguerre and Joseph Nicephore Niepce in
1837. On January 9, 1839, the French Academy of Sciences announced the
Daguerreotype process. On August 19 that same year, the French government
purchased the patent and announced Daguerreotype free to the world as a gift.
This event was an important step in the development of modern-day
photography.
147. BSNL has launched its BSNL Wallet in partnership with which digital
payment company?
[A] Jio Money
[B] PhonePe
2017 GKToday | All Rights Reserved | www.gktoday.in

215
Current Aairs: August, 2017 [Date-wise Compendium]

[C] MobiKwik
[D] Paytm
Correct Answer: C [MobiKwik]
Explanation: The Bharat Sanchar Nigam Ltd. (BSNL) has launched its mobile
wallet BSNL Wallet in partnership with MobiKwik to enable one-tap bill payment
for its over 100 million subscribers. This digital wallet was launched at an event
by Manoj Sinha, Minister for Telecommunications in New Delhi. The mobile wallet
is developed & issued by MobiKwik on behalf of BSNL. It works on both
smartphone and feature phones and enables fast online recharges, bill payments,
shopping, bus & train booking and many more.
148. India has recently signed $24.64 million GEF grant agreement with which
international organization for ecosystems service improvement project?
[A] International Monetary Fund
[B] Economic and Social Council
[C] Food and Agriculture Organization
[D] World Bank
Correct Answer: D [World Bank]
Explanation: The Government of India (GoI) has signed Global Environment
Facility (GEF) Grant Agreement with the World Bank (WB) for $24.64 million for
Ecosystems Service Improvement Project (ESIP). The project is entirely be
nanced by the World Bank out of its GEF Trust Fund. The projects duration is 05
years. The Ministry of Environment, Forest and Climate Change (MoEF&CC) will
implement the Project in the States of Chhattisgarh and Madhya Pradesh through
Indian Council of Forestry Research & Education under the National Mission for
Green India. The purpose of the Project is to strengthen the institutional capacity
of the Departments of Forestry and Community Organisations to enhance forest
ecosystem services and improve the livelihoods of forest dependent communities
in Central Indian Highlands.
149. Where is the headquarters of the Central Board of Film Certification
(CBFC)?
[A] Pune
[B] Mumbai
[C] Kolkata
[D] Chennai
Correct Answer: B [Mumbai]
Explanation: The Central Board of Film Certication (CBFC) is a statutory
censorship and classication body under the Ministry of Information and
2017 GKToday | All Rights Reserved | www.gktoday.in

216
Current Aairs: August, 2017 [Date-wise Compendium]

Broadcasting that regulated the public exhibition of lms under the provisions of
the Cinematograph Act. . The headquarters of CBEC is located at Mumbai,
Maharashtra. The board consists of non-ocial members and a Chairman, all of
whom are appointed by Central Government. Any Film can be publicly exhibited in
the country only after they have been certied by it. Prasoon Joshi, the famous
lyricist, is the newly appointment chairperson of the CBFC. Joshi is known for
putting forth his opinion on contemporary issues. He twice won the National
Award for Best Lyricist rst in 2008 for Maa (Taare Zameen Par) and then in
2013 for Bolo Na in Chittagong.
150. The Parambikulam Tiger Reserve (PTR) is located in which state?
[A] Kerala
[B] Tamil Nadu
[C] Karnataka
[D] Andhra Pradesh
Correct Answer: A [Kerala ]
Explanation: The Parambikulam Tiger Reserve (PTR) is a protected area located
in Chittur taluk in Palakkad district of Kerala and covers an area of 391 square
kilometers. It is alos home to lion-tailed macaques, Nilgiri tahr, elephants, Bengal
tigers, Indian leopards, Nilgiri marten small Travancore ying squirrel, etc.
According to recently conducted tiger census at the regional level in 2016, there
are 33 tigers in Periyar reserve and 25 in Parambikulam reserve respectively,
which is informed by the Forest Minister K Raju in the state assembly on August
18, 2017. It must be noted that as per a tiger census conducted at the national
level in 2010, considered as authoritative gure, there were 38 tigers in
Parambikulam spread over 643 sq feet area and 34 in Periyar that covers an area
of 925 sq feet.
151. Which city hosted the Chief Electoral Officers Conference of all States and
Union Territories?
[A] Pune
[B] Lucknow
[C] Jaipur
[D] New Delhi
Correct Answer: D [New Delhi]
Explanation: The two-day Chief Electoral Ocers Conference of all States and
Union Territories has been inaugurated by Chief Election Commissioner Achal
Kumar Joti and Election Commissioner OP Rawat in New Delhi on August 18, 2017.
The event is being organised by the Election Commission of India (ECI) to review
performance as well as to explore new initiatives and improve the election
2017 GKToday | All Rights Reserved | www.gktoday.in

217
Current Aairs: August, 2017 [Date-wise Compendium]

management and voter experience. The conference stressed on the need of


providing E-services to stakeholders and emphasised the requirement to develop
E-Payment gateways for various election services of Election commission. It also
stressed the need for extensive training programmes on Voter Veriable Paper
Audit Trail (VVPAT) as the ECI has decided to conduct all future elections using it
along with Electronic Voting Machines.
152. The National Conference on Insolvency and Bankruptcy: Changing
Paradigm was held in which city?
[A] New Delhi
[B] Chennai
[C] Mumbai
[D] Patna
Correct Answer: C [Mumbai]
Explanation: Arun Jaitley, the Union Minister of Finance, Defense and Corporate
Aairs, was the Chief Guest at the National Conference on Insolvency and
Bankruptcy: Changing Paradigm, which was held at Mumbai on August 19, 2017.
Besides inaugurating the conference, Jaitley also launched the website for
National federation for corporate governance. The one-day National Conference is
being jointly organized by the Ministry of Corporate Aairs (MCA), National
Foundation for Corporate Governance (NFCG) and Insolvency and Bankruptcy
Board of India (IBBI) to facilitate the better understanding of intricacies of the
Insolvency and Bankruptcy Code (IBC)- 2016, share the experiences gained during
the initial nine months and also discuss the challenges that need to be overcome.
153. Who will be the chairman of the technical committee for the 48th
International Film Festival of India (IFFI)-2017?
[A] Nagesh Kukunoor
[B] Piyush Pandey
[C] Shoojit Sarcar
[D] Prasoon Joshi
Correct Answer: A [Nagesh Kukunoor]
Explanation: The Information and Broadcasting (I&B) Ministry has constituted a
technical committee for the 48th International Film Festival of India (IFFI), which is
proposed to be held from November 20-28, 2017 in Goa. The technical committee
will have 12 members and will be chaired by Nagesh Kukunoor. The I&B ministry
has also constituted a steering committee which will have 13 members, including
lm director Nagesh Kukunoor, lyricist Prasoon Joshi, Jahnu Barua (convener),
Piyush Pandey, creative director of Ogilvy & Mather (India), and lm director and

2017 GKToday | All Rights Reserved | www.gktoday.in

218
Current Aairs: August, 2017 [Date-wise Compendium]

producer Shoojit Sarcar. The director of NFDC is also a member of the committee.
154. The United Nations (UN) has launched which global campaign for 2017
World Humanitarian Day (WHD)?
[A] #ShareHumanity
[B] #NotATarget
[C] #Help
[D] #ProtectCivilians
Correct Answer: B [#NotATarget]
Explanation: The World Humanitarian Day (WHD) is observed every year on
August 19 to pay tribute to aid workers who risk their lives in humanitarian
service, and to rally support for people aected by crises around the world. The
2017 WHD campaign #NotATarget follows the UN Secretary-Generals report on
the protection of civilians in armed conict, which was launched recently. Through
a global online campaign, United Nations (UN) in partnership with Facebook Live
will raise voices to advocate for the most vulnerable people in war zones, and
demand that world leaders do everything in their power to protect civilians in
conict.
155. Who has been appointed as the new Russias new ambassador to India?
[A] Nikolay Kudashev
[B] Alexander Kadakin
[C] Vladimir Lukin
[D] Yuri Ushakov
Correct Answer: A [Nikolay Kudashev]
Explanation: Russian President Vladimir Putin has recently appointed career
diplomat Nikolay Kudashev as Russias new ambassador to India. The
appointment of Kudashev comes nearly seven months after the death of Russias
previous ambassador to India Alexander Kadakin. Kudashev, a specialist in South
East Asia, is currently the Deputy Director General Secretariat of the Ministry of
foreign Aairs of Russia.
156. Indias first world peace university has come up in which state?
[A] Haryana
[B] Uttar Pradesh
[C] Maharashtra
[D] Assam
Correct Answer: C [Maharashtra ]
Explanation: The Dr Vishwanath Karad MIT World Peace University (MITWPU),
2017 GKToday | All Rights Reserved | www.gktoday.in

219
Current Aairs: August, 2017 [Date-wise Compendium]

the rst of its kind in India, has recently inaugurated by chief minister of
Maharashtra Devendra Fadnavis in Pune. The purpose of the university is to
visualize education through a holistic outlook of knowledge building to establish
peace and harmony in the world. Beside this, a two-day Global Summit Science
and Spirituality for World Peace was also organised at Swami Vivekananda
Mandap at MIT World Peace University to discuss issues of national and
international importance and how a dierent outlook towards education could
solve many of them.
157. Which of the followinng has become the first power utility to introduce a QR
code based bill payments system in India?
[A] Tata Power
[B] Adani Power
[C] Reliance Power
[D] Jindal India Thermal Power
Correct Answer: A [Tata Power]
Explanation: Tata Power has become the rst power utility to introduce a QR
code based bill payments system in India. The QR code linked to Unied
Payments Interface (UPI) will be printed on the electricity bills. The customers can
scan the QR code with BHIM app or any other UPI linked bank app and pay their
bills without any hassle. The bill details will be displayed on the app, post which
the customer can authorise the payment within a few seconds and his bill will be
paid instantaneously. Some of the advantages of QR code service are that the
consumer need not visit any Tata Power bill collection/ customer relation centre or
any other payment avenues and can make the payment from the comfort of his
home/oce or on-the-go. The Tata Power Company Limited is an Indian electric
utility company based in Mumbai, Maharashtra and is part of the Tata Group.
158. Who has taken charge as new Executive Director of Insolvency and
Bankruptcy Board of India (IBBI)?
[A] A Unnikrishnan
[B] Ajay Tyagi
[C] G. S. Yadav
[D] Mamta Suri
Correct Answer: D [Mamta Suri]
Explanation: Dr. Mamta Suri has taken charge as Executive Director of
Insolvency and Bankruptcy Board of India (IBBI) in New Delhi on August 16, 2017.
Before joining IBBI, Dr. Suri was serving as Chief General Manager, Insurance
Regulatory Development Authority of India (IRDAI). The IBBI is the regulator for

2017 GKToday | All Rights Reserved | www.gktoday.in

220
Current Aairs: August, 2017 [Date-wise Compendium]

overseeing insolvency proceedings and entities like Insolvency Professional


Agencies (IPA), Insolvency Professionals (IP) and Information Utilities (IU) in India.
Madhusudan Sahoo is the current chairman of the organization.
159. Which country to host the 2017 FIFA Under-17 World Cup 2017
[A] Japan
[B] United Kingdom
[C] India
[D] Russia
Correct Answer: C [India]
Explanation: The 2017 FIFA Under-17 World Cup will be held in India from
October 6 to 28, which is rst-ever FIFA tournament in India. It is in news because
the curtain raiser function for FIFA U-17 World Cup was held in Major Dhyan
Chand National Stadium in New Delhi on August 19, 2017. The trophy for FIFA
World Cup was also displayed at the stadium. The objective of the programme
was to promote football in India, especially among children and youth.
160. The Periyar National Park (PNR) is located in which state?
[A] Odisha
[B] Kerala
[C] Tamil Nadu
[D] Andhra Pradesh
Correct Answer: B [Kerala]
Explanation: The Periyar National Park (PNR) is a protected area near Thekkady
in the districts of Idukki, Kottayam and Pathanamthitta in Kerala and covers an
area of 925 km2. It is notable as both elephant reserve and tiger reserve. The
park is made up of tropical evergreen and moist deciduous forests, grasslands,
stands of eucalyptus, and lake and river ecosystems. It is surrounded by
agricultural regions such as tea, cardamom, and coee. It is also home to gaur,
sambar, wild pig, Indian giant squirrel, Travancore ying squirrel, jungle cat, sloth
bear, Nilgiri tahr, lion-tailed macaque, Nilgiri langur, etc.
161. Which Indian-origin boy has been crowned as the UKs Child Genius in a
popular television quiz competition?
[A] Rahul Doshi
[B] Naveen Jain
[C] Preethi Singh
[D] Kalpana Chauhan
Correct Answer: A [Rahul Doshi ]

2017 GKToday | All Rights Reserved | www.gktoday.in

221
Current Aairs: August, 2017 [Date-wise Compendium]

Explanation: Rahul Doshi (12), an Indian-origin boy, has been crowned as the
UKs Child Genius in a popular television quiz competition, after answering all
questions correctly. He won the Channel 4 show Child Genius by beating his
nine-year-old opponent Ronan by 10-4 in the programmes nale on August 20,
2017. In the nal, Rahul impressed with knowledge of his chosen subject Edward
Jenners medical innovation and methodology in 18th Century England. He
clinched the title by answering a question on 19th Century artists William Holman
Hunt and John Everett Millais. He has an IQ of 162, which is believed to be higher
than the likes of Albert Einstein and Stephen Hawking, and qualies him to be a
member of Mensa club the largest and oldest high IQ society in the world.
162. Which state government has launched an exclusive 247 helpline 181 for
women?
[A] Uttar Pradesh
[B] Telangana
[C] Assam
[D] Jharkhand
Correct Answer: B [Telangana]
Explanation: The Telangana government has recently launched an exclusive
247 helpline 181 for women welfare. The helpline will respond to calls on
dowry harassment cases, stalking, domestic violence, sale of girl child and
immoral tracking. Women with suicidal tendencies or those who are mentally
disturbed can also contact the helpline. Apart from contacting police, the helpline
can also be used to seek medical help, ambulance and other emergency services.
The helpline will work 24X7 on all 365 days. It is funded by Centre & supported by
GVK EMRI.
163. Which city is hosting the 8th World Renewable Energy Technology
Congress?
[A] Pune
[B] Guwahati
[C] New Delhi
[D] Bhopal
Correct Answer: C [New Delhi ]
Explanation: The 8th World Renewable Energy Technology Congress &
Expo-2017 has started in New Delhi from August 21, 2017. The theme of the 3-
day annual conference is Renewable Energy: What Works, which focuses on
new Green Technologies to ensure clean, reliable and aordable Renewable
Energy supplies. It provides an opportunity and open forum to exchange
2017 GKToday | All Rights Reserved | www.gktoday.in

222
Current Aairs: August, 2017 [Date-wise Compendium]

information, share experiences and best practices by bringing together experts,


investors and other multi stake holders of i.e. Public and Private Sector, Advisory
Groups, Governments, NGOs, Non-Prot organizations Environmentalists and
Academia.
164. MK Damodaran, who passed away recently, was the former Advocate
General of which state?
[A] Odisha
[B] Uttar Pradesh
[C] Haryana
[D] Kerala
Correct Answer: D [Kerala]
Explanation: MK Damodaran (77), the veteran lawyer and former Advocate
General of Kerala, has passed away in Kochi, Kerala. He was one of the most
popular gures at the Kerala High Court and was well known for defending the
Naxalites during the late 60s and early 70s. He was expert in both criminal and
civil cases and was jailed during the Emergency period 1975-77.
165. Which state government will launch a new scheme for Compassionate
Family Pension (CFP) in lieu of compassionate appointment?
[A] Assam
[B] Manipur
[C] Arunachal Pradesh
[D] Jammu & Kashmir
Correct Answer: A [Assam]
Explanation: The Assam government will launched a new Scheme called Scheme
for Compassionate Family Pension (CFP) in lieu of Compassionate Appointment.
As per ocials, the existing compassionate appointment policy did not fully serve
the objectives and often the legal heir fails to get an appointment immediately
due to practical diculties such as lack of educational qualications and non-
availability of vacancies within the stipulated 5% of total vacancies. The new
scheme will be applicable to those employees who died in harness on or after
April 1, 2017, but the relevant oce memorandums related to the compassionate
appointments will be applicable to those who died on or before March 31, 2017.
The new scheme will be applicable to all employees of the state government, all
others as eligible under Assam Services (Pension) Rules, 1969, all state
government employees under NPS category and All India Services ocers borne
on Assam cadre.
166. Which state chief minister has been chosen for the National Agriculture
2017 GKToday | All Rights Reserved | www.gktoday.in

223
Current Aairs: August, 2017 [Date-wise Compendium]

Leadership Award-2017?
[A] Madhya Pradesh
[B] Telangana
[C] Maharashtra
[D] Tripura
Correct Answer: B [Telangana ]
Explanation: Telangana Chief Minister K Chandrashekhar Rao has been selected
for the prestigious National Agriculture Leadership Award-2017 by the Indian
Council of Food and Agriculture (ICFA) under the chairmanship of well known
agriculture scientist Prof MS Swaminathan. Rao. The Chief Minister has been
selected for the award for his innovative services rendered to the welfare of
farmers and the farm sector. The award will be presented on September 5, 2017
at Taj Palace Hotel in New Delhi.
167. Jerry Lewis, the famous comedian passed away. He hailed from which
country?
[A] United Kingdom
[B] Germany
[C] United States
[D] France
Correct Answer: C [United States]
Explanation: Jerry Lewis (91), the famous comedian and Filmmaker, has passed
away in Las Vegas, United States. He was one of the most popular comic actors of
the 1950s and 60s and was known for his slapstick humor in lm, television,
stage and radio. He perfected not only the role of the quirky clown but also won
acclaim as a writer, actor and philanthropist in a career spanning six decades. He
became the highest-paid actor in Hollywood, chalking up hits such as The Bell
Boy, Cinderfella and The Nutty Professor.
168. Who is the author of the book I am HIV positive, so what?
[A] Pradip Kumar
[B] Vikram Seth
[C] Jayanta Kalita
[D] Amitav Ghosh
Correct Answer: C [Jayanta Kalita]
Explanation: The book I am HIV positive, so what? has been authored by
journalist Jayanta Kalita and published by Bloomsbury. The book is based on the
life of Manipurs acclaimed bodybuilder Khundrakpam Pradip kumar Singh, who is

2017 GKToday | All Rights Reserved | www.gktoday.in

224
Current Aairs: August, 2017 [Date-wise Compendium]

known for his struggle against HIV-AIDS. The book speaks of the incredible
journey of the Pradip, who mended his failing health, overcame psychological
trauma, and fought stigma and discrimination to pursue his dreams. He won the
Mr. South Asia title and bagged bronze in the Mr World contest in 2012.
169. The Mrugavani National Park (MNP) is located in which state?
[A] Chattisgarh
[B] Karnataka
[C] Tamil Nadu
[D] Telangana
Correct Answer: D [Telangana ]
Explanation: The Mrugavani National Park (MNP) is located at Chilkur in
Hyderabad, Telangana State and covers an area of 3.6 square kilometers. It is
home to Cheetal, Sambar, Wild boar, Jungle Cat, Civet Cat, Mongoose Monitor
Lizard, Python, Russell Viper, King Cobra, etc., and has 600 dierent types of plant
life.
170. The 2017 World Mosquito Day (WMD) is observed on which day?
[A] August 21
[B] August 22
[C] August 23
[D] August 20
Correct Answer: D [August 20]
Explanation: The World Mosquito Day (WMD) is observed every year on August
20 is to raise awareness about the threat of malaria and other diseases
transmitted by one of the worlds deadliest killers the mosquito. The day also
marks the historic discovery by British doctor Sir Ronald Ross in 1897 that female
Anopheles mosquitoes transmit malaria between humans and that laid
foundations for scientists across the world to better understand the deadly role of
mosquitoes in disease transmission and come up with eective innovative
interventions.
171. Who of the following have been selected for the 2017 Rajiv Gandhi Khel
Ratna Award?
[A] Brij Bhushan Mohanty and Syed Shahid Hakim
[B] Varun Singh Bhati and SV Suni
[C] Devendra Jhajharia and Sardar Singh
[D] Harmanpreet Kaur and P N Prakash
Correct Answer: C [Devendra Jhajharia and Sardar Singh]

2017 GKToday | All Rights Reserved | www.gktoday.in

225
Current Aairs: August, 2017 [Date-wise Compendium]

Explanation: Para Athlete Devendra Jhajharia and Hockey player Sardar Singh
have been selected for the 2017 Rajiv Gandhi Khel Ratna Award. Cricketer
Cheteshwar Pujara, Women Cricketer Harmanpreet Kaur and shooter P. N.
Prakash are among the 17 sportsmen who will be given Arjuna Awards for their
outstanding performance. Seven persons have been chosen for Dronacharya
Awards for producing medal winners at prestigious International sports events.
They include late Dr. R. Gandhi in Athletics, Brij Bhushan Mohanty in Boxing and
P.A. Raphel in the eld of hockey. Dhyan Chand Award will be given to Bhupender
Singh in Athletics, Syed Shahid Hakim in Football and Sumarai Tete in Hockey for
their life time contribution to sports development.The awardees will receive their
awards from the President Ram Nath Kovind in a special function at Rashtrapati
Bhawan on August 29, 2017.
172. Where is the headquarters of the Border Roads Organisation (BRO)?
[A] Pune
[B] Guwahati
[C] Srinagar
[D] New Delhi
Correct Answer: D [New Delhi]
Explanation: Recently, the Union Ministry of Defence has granted additional
administrative and nancial powers to the Border Roads Organisation (BRO) to
speed up the construction of roads along the Indo-China border. With this, from
now, a chief engineer in BRO can now accord administrative approval for
contracts of up to Rs50 crore, the additional director general (ADG) can approve
projects worth up to Rs75 crore and the director general (DG) can take decisions
on projects worth up to Rs100 crore. These projects can be implemented through
departmental and contractual modes of execution. The BRO is functioning under
the control of the Ministry of Defence since 2015 and is engaged in road
construction to provide connectivity to dicult and inaccessible regions in the
border areas of the country. Its headquarters is located in New Delhi.
173. Who has been appointed as the new director general of Railway Protection
Force (RPF)?
[A] Brij Mohanty
[B] S H Prakash
[C] Dharmendra Kumar
[D] Nikhil Singh
Correct Answer: C [Dharmendra Kumar]
Explanation: Dharmendra Kumar, IPS (AGMU:84), has been appointed as the

2017 GKToday | All Rights Reserved | www.gktoday.in

226
Current Aairs: August, 2017 [Date-wise Compendium]

new Director General of Railway Protection Force (RPF). Presently, he is working in


CISF as Additional DG. The appointment of Kumar will be eective from the date
of assumption of charge of the post or until further orders, whichever is earlier.
The RPF is a security force of India entrusted with protecting railway passengers,
passenger area and railway property of the Indian Railways .This is the only
central armed police force (CAPF) which has the power to arrest, investigate and
prosecute criminals. The force is under the authority of Ministry of Railways.
174. The Indian Council of Medical Research (ICMR) has signed a pact with
which organization to collaborate on vaccine research and development?
[A] World Food Programme
[B] International Vaccine Institute
[C] World Health Organization
[D] International Committee of the Red Cross
Correct Answer: B [International Vaccine Institute]
Explanation: The Indian Council of Medical Research (ICMR) and the Union
Ministry of Health has inked a pact with the International Vaccine Institute (IVI) to
collaborate on vaccine research and development. With this partnership, India will
commit an annual contribution of $5,00,000 to IVI and joins Sweden and South
Korea as nancially contributing signatories to IVI. In addition, ICMR will be
focussed on research and development of vaccines combatting various infectious
diseases. As per ocial statement, the IVI has 35 signatory countries and India
ocially became a signatory in 2012. The IVI is an international nonprot
organization i.e. devoted to developing and introducing new and improved
vaccine to protect the people, especially children, against deadly infectious
diseases. The headquarters of IVI is located in Seoul, South Korea. The ICMR is the
apex body in the country for the formulation, coordination and promotion of
biomedical research.
175. Velichko Cholakov, the noted weightlifter passed away. He hailed from
which country?
[A] Bularia
[B] France
[C] Italy
[D] Russia
Correct Answer: A [Bularia ]
Explanation: Bulgarian weightlifter Velichko Cholakov (35), an Olympic and
world medalist, has passed away in Smolyan, Bularia on August 20, 2017. He took
+105 kg bronze at the Athens 2004 Olympics and silver in the same division in

2017 GKToday | All Rights Reserved | www.gktoday.in

227
Current Aairs: August, 2017 [Date-wise Compendium]

the 2003 world championships. He was part of the Bulgarian team pulled from the
Beijing 2008 Games after failed dope tests. Cholakov was one of 11 members of
the contingent to test positive for a banned anabolic steroid. In recent years
Cholakov dedicated his time to training young weightlifters in Smolyan.
176. Which union minister has launched Swasth Bachche Swasth Bharat
programme?
[A] Prakash Javadekar
[B] Rajnath Singh
[C] Narendra Modi
[D] Arun Jaitley
Correct Answer: A [Prakash Javadekar]
Explanation: Prakash Javadekar, the Union Ministry of Human Resource
Development (HRD), has recently launched Swasth Bachche, Swasth Bharat
Programme. The programme is an initiative of Kendriya Vidyalaya Sangathan
(KVS) to prepare a physical health and tness Prole Card for over 12 lakh
Kendriya Vidyalaya students. The programme will provide a comprehensive and
inclusive report card for children of all age groups and dierently-abled kids. The
programme aims to make students, teachers and parents aware of the
importance of good health and tness and encourage 60 minutes of play every
day. The programme also intends to imbibe values of Olympics and Paralympics
among students.
177. Who will be the head of a newly constituted preview committee for
International Film Festival of India (IFFI) 2017?
[A] Aniruddha Roy Chowdhury
[B] Khalid Mohamed
[C] Vivek Agnihotri
[D] Narendra Kohli
Correct Answer: C [Vivek Agnihotri]
Explanation: The Information and Broadcasting (I&B) Ministry has recently
constituted a 40-member preview committee with lmmaker Vivek Agnihotri as
its convenor to select movies for the 2017 International Film Festival of India
(IFFI). The other members of the new committee include Pallavi Joshi, Narendra
Kohli, Hrishitaa Bhatt, Aarti Bajaj, Khalid Mohamed, and lm director Aniruddha
Roy Chowdhury. The 2017 IFFI will be held in Panaji, Goa from November 20 to
28. Earlier, the I & B Ministry had constituted the steering committee and the
technical committee for the IFFI.
178. Which state government has launched clouding seeding Project Varshadhari
2017 GKToday | All Rights Reserved | www.gktoday.in

228
Current Aairs: August, 2017 [Date-wise Compendium]

under which an aircraft will spray chemicals to induce rainfall?


[A] Karnataka
[B] Mizoram
[C] Punjab
[D] Kerala
Correct Answer: A [Karnataka]
Explanation: The Karnataka Government has recently launched an ambitious
clouding seeding Project Varshadhari in Bengaluru to deal with poor monsoons in
the state. Under the project, an aircraft will spray chemicals to induce rainfall. The
purpose of the project is to condense small water particles by clouds (size less
than 10 microns) to droplets of 50 microns, which can constitute rains. It is an
experimental project that will cover Bengaluru, Gadag and Yadgir districts of the
state. The Cloud seeding (also known as weather modication technique) is an
articial way to induce moisture in the clouds so as to cause a rainfall.
179. Which union minister has launched digital police portal under Crime and
Criminal Tracking Network and Systems Project (CCTNS)?
[A] Arun Jaitely
[B] Prakash Javadekar
[C] Narendra Modi
[D] Rajnath Singh
Correct Answer: D [Rajnath Singh]
Explanation: Home Minister Rajnath Singh has launched digital police portal
under Crime and Criminal Tracking Network and Systems Project (CCTNS) on
August 21, 2017. The portal is a smart policing initiative to provide services to
citizens and aid ecient police investigation. It is a platform for citizens to le
online crime related complaints and also request for antecedent verication. The
portal will also provide access to authorized persons to use National Database of
crime records for the purpose of investigation. In addition, central investigating
and research agencies have also been provided logins to the portal to access
crime statistics. Till date, the CCTNS national database has around seven crore
records of old and new criminal cases.
180. The Sepahijala Wildlife Sanctuary (SWS) is located in which state?
[A] Odisha
[B] Kerala
[C] Tripura
[D] Manipur
Correct Answer: C [Tripura]
2017 GKToday | All Rights Reserved | www.gktoday.in

229
Current Aairs: August, 2017 [Date-wise Compendium]

Explanation: The Sepahijala Wildlife Sanctuary (SWS) is located in Bishalgarh,


Tripura and covers an area of 18.53 square kilometres. It is famous for its clouded
leopard enclosures. It is woodland with an articial lake and natural botanical and
zoological gardens. It gives shelter to about 150 species of birds and the unique
bespectacled monkey, Phayres langur. The sanctuary has been developed both
as a wildlife sanctuary and as an academic and research centre.
181. The NITI Aayog has launched Mentor India Campaign from which city?
[A] Chennai
[B] Hyderabad
[C] Bengaluru
[D] New Delhi
Correct Answer: D [New Delhi]
Explanation: NITI Aayog CEO Amitabh Kant has launched the Mentor India
Campaign in New Delhi on August 23, 2017. The campaign is a strategic nation
building initiative to engage leaders who can nurture and guide students at more
than 900 Atal Tinkering Labs. The labs are dedicated workspaces where students
from Class 6th to 12th learn innovation skills and develop ideas that will go on to
transform India. The Atal Tinkering Labs have been established across the country
as a part of the Atal Innovation Mission. The aim of campaign is to maximize the
impact of the labs.
182. Under the chairmanship of whom the 17th meeting of the Financial Stability
and Development Council (FSDC) was held?
[A] Rajnath Singh
[B] Arun Jaitley
[C] Narendra Modi
[D] Urjit Patel
Correct Answer: B [Arun Jaitley]
Explanation: Arun Jaitley, the Union Finance Minister, has chaired the 17th
meeting of the Financial Stability and Development Council (FSDC) in New Delhi
on August 22, 2017. The Council discussed the issues and challenges facing the
Indian economy and stressed the need to keep constant vigil to manage any
external and internal vulnerability. The Council noted that India has macro-
economic stability today on the back of improvements in its macro-economic
fundamentals, structural reforms with the launch of the GST and long-term
positive consequences of demonetization. The FSDC also took note of the
developments and progress made in setting up of Computer Emergency Response
Team in the Financial Sector (CERT-Fin) and Financial Data Management Centre.

2017 GKToday | All Rights Reserved | www.gktoday.in

230
Current Aairs: August, 2017 [Date-wise Compendium]

The meeting was attended by RBI Governor Urjit Patel, SEBI Chairman Ajay Tyagi,
IRDAI Chairman TS Vijayan, PFRDA Chairman Hemant Contractor, Finance
Secretary Ashok Lavasa, and other government ocials.
183. Who has been appointed as the new Chairman of Railway Board?
[A] Ashwani Lohani
[B] A K Mittal
[C] P S Thakur
[D] Neelmani Reddy
Correct Answer: A [Ashwani Lohani]
Explanation: Ashwani Lohani, an ocer of the Indian Railway Service of
Mechanical Engineers (IRSME), has been appointed as the new Chairman of
Railway Board after AK Mital tendered his resignation following recent train
accidents. Presently, Lohani is serving as the Chairman and Managing Director of
Air India. Earlier, he served as the Divisional Regional Manager of Delhi and was
also ITDC chairman as well as the director of the Rail Museum in the capital.
184. Rishang Keishing, who passed away recently, was the former Chief Minister
of which state?
[A] Mizoram
[B] Arunachal Pradesh
[C] Manipur
[D] Nagaland
Correct Answer: C [Manipur ]
Explanation: Rishang Keishing (96), the former Manipur Chief Minister and a
member of the rst Lok Sabha, has passed away in Imphal, Manipur on August 23,
2017. The veteran Congress leader and a Naga served as the chief minister of
Manipur from 1980 to 1988 and from 1994 to 1997. The Manipur government has
declared that all government oces and educational institutions will be closed on
August 24 as mark of respect to Keishing.
185. The 58th edition of Subroto Cup International Football Tournament has
started in which city?
[A] New Delhi
[B] Pune
[C] Chandigarh
[D] Patna
Correct Answer: A [New Delhi]
Explanation: The 58th edition of Subroto Cup International Football Tournament
2017 GKToday | All Rights Reserved | www.gktoday.in

231
Current Aairs: August, 2017 [Date-wise Compendium]

has started in New Delhi. A total of 99 teams from India and abroad are
participating across three categories in this edition of famous inter-school football
tournament. Teams from Bangladesh, Indonesia, Nepal, Afghanistan and
Singapore are participating in this edition. The one month long football carnival
will be played in three categories Sub-Junior boys (Under-14), Junior Girls
(Under-17), Junior Boys (Under-17). This season seems to be more promising with
a special emphasis on BETI-BACHAO, BETI-PADHAO, BETI-KHILAO. Some of the
key highlights of this edition will be special awards in dierent categories, an
increase in cash awards, and special initiatives like Delhi Darshan for participants.
186. TRAI has signed a Letter of Intent (LoI) with which countrys telecom
regulator to jointly conduct capacity building exercises in telecom sector?
[A] Indonesia
[B] Malaysia
[C] Vietnam
[D] Maldivies
Correct Answer: B [Malaysia]
Explanation: The Telecom Regulatory Authority of India (TRAI) has signed a
Letter of Intent (LoI) with the Malaysian Communications and Multimedia
Commission (MCMC) in New Delhi to jointly conduct capacity building exercises in
the elds of broadcasting and telecommunications regulation. The agreement
would help both regulators to work closely in enhancing mutual cooperation and
bilateral relations in telecom and broadcasting regulation. The agreement will
facilitate exchange of information and regulatory best practices in the elds of
broadcasting and telecommunications, including new and emerging areas.
187. Brian Aldiss, the renowned science fiction writer passed away. He hailed
from which country?
[A] France
[B] Portugal
[C] England
[D] Italy
Correct Answer: C [England]
Explanation: Brian Aldiss (92), one of the most prolic and inuential science
ction writers of the 20th century, has passed away in Oxford, England. Aldiss
served in India and Burma with the British Army during World War II and later
became a bookseller, publishing his rst stories in a trade magazine. He was the
author of science ction classics including Non-Stop, Hothouse and Greybeard and
Helliconia trilogy. His numerous short stories include Super-Toys Last All Summer

2017 GKToday | All Rights Reserved | www.gktoday.in

232
Current Aairs: August, 2017 [Date-wise Compendium]

Long, formed the basis for Steven Spielbergs 2001 lm AI, while his Horatio
Stubbs saga was based on his time during the war in Burma and the far east.
188. The National Sports Museum will be established in which city?
[A] Indore
[B] Varanasi
[C] Surat
[D] New Delhi
Correct Answer: D [New Delhi ]
Explanation: The National Sports Museum will be established in New Delhi that
will showcase Indias achievements in sports and also focus on the traditional
sports within the country. The purpose of the Museum is to popularise the sports
as a way of life within the country. The union Sports Ministry has stated that the
proposed Museum would go beyond the classical approach of a collection of
memorabilia to become an interactive venue of interest for all age groups. In
addition, it will display achievements of the Indian greats in sports and also act as
an educational hub and oer opportunity for the young to delve into some
sporting activity. The Museum will have audio-visual display of rules and legacy
events pertaining to various sports as part of education for the budding
sportspersons and a well-equipped library of sports and physical tness.
189. Where is the headquarters of the Nehru Memorial Museum and Library
(NMML)?
[A] New Delhi
[B] Thiruvananthapuram
[C] Allahabad
[D] Raipur
Correct Answer: A [New Delhi ]
Explanation: Recently, the Union Home Minister Rajnath Singhhas chaired the
42nd annual general body meeting of the Nehru Memorial Museum and Library
(NMML) on August 22, 2017. The meeting approved the annual report and audited
accounts of the NMML society for the years 2014-15 and 2015-16. During the
meeting various issues including up gradation and renovation of museum,
digitization of documents, acquisition of books in regional languages and setting
up of a museum for former Prime Ministers, were discussed. The NMML is a
museum and library in New Delhi that aims to preserve and reconstruct the
history of the Indian independence movement.
190. The Kasu Brahmananda Reddy National Park (KBRNP) is located in which
state?
2017 GKToday | All Rights Reserved | www.gktoday.in

233
Current Aairs: August, 2017 [Date-wise Compendium]

[A] Kerala
[B] Telangana
[C] Tripura
[D] Tamil Nadu
Correct Answer: B [Telangana ]
Explanation: The Kasu Brahmananda Reddy National Park (KBRNP) is located in
Jubilee Hills in Hyderabad, Telangana and covers an area of 390-acre. The park is
also known as Chiran Fort Palace. The park has over 600 species of plant life, 140
species of birds and 30 dierent varieties of butteries and reptiles. It is home to
pangolin, small Indian civet, peacock, jungle cat, porcupines, etc.
191. The constitution bench of Supreme Court has declared right to privacy as
Fundamental right under constitution. The bench was headed by whom?
[A] D Y Chandrachud
[B] A M Sapre
[C] JS Khehar
[D] R F Nariman
Correct Answer: C [JS Khehar ]
Explanation: In a landmark decision, the Supreme Court has declared right to
privacy a fundamental right under the constitution on August 24, 2017. A nine-
judge constitution bench headed by Chief Justice JS Khehar ruled that right to
privacy is an intrinsic part of Right to Life and Personal Liberty under Article 21
and entire Part III of the Constitution. The ruling on the highly contentious issue
was to deal with a batch of petitions challenging the Centres move to make
Aadhaar mandatory for availing the benets of various social welfare schemes.
Others members of the bench comprising Justices J Chelameswar, S A Bobde, R K
Agrawal, R F Nariman, A M Sapre, D Y Chandrachud, S K Kaul and S Abdul Nazeer
also shared the same view. The nine judges unanimously overruled the two earlier
judgements of the apex court that right to privacy is not protected under the
Constitution.
192. The 2017 International Day for the Remembrance of the Slave Trade and its
Abolition is observed on which date?
[A] August 24
[B] August 25
[C] August 22
[D] August 23
Correct Answer: D [August 23]

2017 GKToday | All Rights Reserved | www.gktoday.in

234
Current Aairs: August, 2017 [Date-wise Compendium]

Explanation: The International Day for the Remembrance of the Slave Trade and
its Abolition is observed every year by UNESCO on August 23 to pay tribute to all
those who fought for freedom, and, in their name, to continue teaching about
their story and the values therein.
193. Which tech giant has launched Project Brainwave for real time Artificial
Intelligence (AI)?
[A] Infosys
[B] Microsoft
[C] Facebook
[D] Google
Correct Answer: B [Microsoft]
Explanation: Microsoft has launched its Project Brainwave deep learning
acceleration platform for real-time articial intelligence (AI). With the help of ultra-
low latency, the system processes requests as fast as it receives them. The
Project Brainwave uses the massive Field-Programmable Gate Array (FPGA)
infrastructure that Microsoft has been deploying over the past few years. The real-
time AI is becoming increasingly important as cloud infrastructures process live
data streams, whether they be search queries, videos, sensor streams, or
interactions with users.
194. Which city will host the first-ever beauty pageant for transgenders Miss
Transqueen India?
[A] Gurugram
[B] Pune
[C] Kochi
[D] Chennai
Correct Answer: A [Gurugram]
Explanation: The rst edition of Miss Transqueen India, a beauty pageant for
transgenders, will take place in Gurugram, Haryana on August 27, 2017. Like
other beauty pageants, the contestants will have to go through swimming
costume, traditional wear, evening gown and question-answer rounds. The winner
will go to Thailand for Miss International Queen and the rst runner-up will
compete at Miss Transsexual Australia.
195. Which state government has launched a grid-connected rooftop solar
programme through net-metering system?
[A] Odisha
[B] Madhya Pradesh
2017 GKToday | All Rights Reserved | www.gktoday.in

235
Current Aairs: August, 2017 [Date-wise Compendium]

[C] Rajasthan
[D] Kerala
Correct Answer: A [Odisha]
Explanation: Naveen Patnaik, the Chief Minister of Odisha, has launched a grid-
connected rooftop solar programme through the net-metering system at the State
Secretariat in Bhubaneshwar in a bid to enable public participation to harness
renewable energy. The State government will provide 30% subsidy to the
consumers belonging to residential and institutional segments as well as to
registered societies and trusts. At present, current investment for installation of a
rooftop project is around Rs 70,000 per KW. The web portal
www.rtsodisha.gov.in was also launched that will provide an opportunity to the
stakeholders to interact with customers for timely delivery of services online.
Under the programme, consumers can instal solar plants on their rooftops and
consume the generated solar power during day time. Excess power can be sold to
the grid at 220 v/440 v levels through service cables. The program is being
implemented by Odisha Renewable Energy Development Agency (OREDA).
196. The Karnataka government has tied up with which tech giant to develop
Indias first Farm Price Forecasting Model?
[A] Google
[B] Infosys
[C] Microsoft
[D] Facebook
Correct Answer: C [Microsoft]
Explanation: The Karnataka Government has signed a Memorandum of
Understanding (MoU) with software giant Microsoft India to develop a unique
farm price forecasting model using latest IT tools in a bid to help farmers,
administrators and other stakeholders to understand the market behaviour in
advance. The proposed initiative is said to be the rst-of-its-kind in the country as
it is a multi-variate one that takes into consideration various factors beyond the
conventional supply-demand equation while forecasting the prices for crops. It will
examine a slew of related factors including weather, rainfall and external factors
that impact market behaviour. The model is expected to be in place for major
crops by next kharif crops.
197. Which country is hosting the International Military Music Festival
(IMMF-2017)?
[A] India
[B] United States

2017 GKToday | All Rights Reserved | www.gktoday.in

236
Current Aairs: August, 2017 [Date-wise Compendium]

[C] China
[D] Russia
Correct Answer: D [Russia]
Explanation: The 2017 International Military Music Festival Spasskaya Tower
has started in Moscow, Russia on August 24 and will continue till September 3,
2017. The Indian Navy Band Tri- Services is participating in the fest. It is the
parade of the best military music bands of Russia and other countries that takes
place every year at Red Square in Moscow. This Music Festival is a mega event,
where military musicians represent the variety of national, artistic and military
traditions of the world. Every year about 1500 musicians, military men and other
artists from around 40 countries perform at the Spasskaya Tower. The Festival
is duly considered one of the largest international events in Russia and attracts
tremendous response of the general public.
198. The Palamau Tiger Reserve (PTR) is located in which state?
[A] Tamil Nadu
[B] Jharkhand
[C] Punjab
[D] Haryana
Correct Answer: B [Jharkhand ]
Explanation: The Palamau Tiger Reserve (PTR) is located in Latehar District of
Jharkhand and covers an area of 1,129.93 square kilometres. Apart from tigers
and elephants, leopards, gaurs, sambars and wild dogs live in the reserve. It
forms part of Betla National Park and Palamau Wildlife Sanctuary.
199. Who has been appointed as the new Chairman and Managing Director
(CMD) of Air India?
[A] Rajiv Bansal
[B] Kirti Das
[C] Nitant Agrawal
[D] Vinay Kumar
Correct Answer: A [Rajiv Bansal]
Explanation: Rajiv Bansal, the 1988 batch IAS ocer of Nagaland cadre, has
been appointed as the new Chairman and Managing Director (CMD) of Air India for
three months. Bansal, who is additional secretary and nancial adviser at the
petroleum ministry, will replace Ashwani Lohani who has been appointed as the
Chairman of Railway Board. It must be noted that Bansal is taking over as Air
India CMD at a time when the government is working on the modalities for the
disinvestment of the loss-making national carrier. He had served at the civil
2017 GKToday | All Rights Reserved | www.gktoday.in

237
Current Aairs: August, 2017 [Date-wise Compendium]

aviation ministry between 2006 and 2008.


200. Who is leading the Indian delegation in the Shanghai Cooperation
Organization (SCO) meeting on disaster prevention?
[A] Rajnath Singh
[B] Sushma Swaraj
[C] Arun Jaitley
[D] Narendra Singh Tomar
Correct Answer: A [Rajnath Singh]
Explanation: The 9th meeting of heads of Governments of Shanghai Cooperation
Organization (SCO) member states on disaster prevention and relief has started at
Choplon Ata in Kyrgyz Republic on August 24, 2017. The 2-day meeting will focus
on prevention and elimination of emergency situations. The Union Home Minister
Rajnath Singh is leading the Indian delegation which includes senior ocers from
Home and External Aairs Ministries and National Disaster Management
Authority. The participation of India as SCO Member will open additional
opportunities for giving new impetus to the development of international
cooperation in the eld of protecting the population and territories from
emergency situations within the framework of the Organisation.
201. Indias first Videsh Bhavan will come up in which of the following states?
[A] Jharkhand
[B] Maharashtra
[C] Madhya Pradesh
[D] Uttar Pradesh
Correct Answer: B [Maharashtra]
Explanation: Sushma Swaraj, the Union External Aairs Minister, will inaugurate
Indias rst Videsh Bhavan on August 27, 2017 at the Bandra Kurla Complex (BKC)
in Mumbai, Maharashtra. As a rst pilot project, four oces of MEA Regional
Passport Oce (RPO), Protector of Emigrants (PoE) oce, Branch Secretariat and
Regional Oce of ICCR have been assimilated and brought under one roof at the
state-of-art oce. The pilot project of the Videsh Bhavan of Mumbai is part of the
Centres policy to bring together dierent oces of the MEA under a single roof
and work closely with states as more and more Indians are going abroad for
employment, education, business and tourism. In other words, the Videsh
Bhavan at Mumbai is the rst integrated oce complex of the Ministry of
External Aairs established in any State of India.
202. Which state power discoms has become the Indias first state-owned discoms
to accept payments through Bharat QR?
2017 GKToday | All Rights Reserved | www.gktoday.in

238
Current Aairs: August, 2017 [Date-wise Compendium]

[A] Andhra Pradesh


[B] Chattisgarh
[C] Rajasthan
[D] Odisha
Correct Answer: A [Andhra Pradesh]
Explanation: Andhra Pradeshs Eastern Power Distribution Company Ltd (EPDCL)
and the Southern Power Distribution Corporation of AP Ltd. (SPDCL) have become
the Indias rst state-owned distribution companies to accept payments through
Bharat QR. The Bharat QR is the worlds rst interoperable quick response code
acceptance solution launched by the Reserve Bank of India (RBI). It is secure,
network-agnostic interface that enables customers to use their smartphones to
make payments without divulging information like card numbers and CVV. To pay
power bills via Bharat QR, customers must log in to their banks app and scan the
QR code on their electricity bill for instant payment. Alternatively, they may enter
their unique customer IDs online and generate a QR code that can be scanned
using their banks app.
203. Who has been appointed as the new non-executive Chairman of Infosys?
[A] Ravi Venkatesan
[B] R. Seshasayee
[C] Nandan Nilekani
[D] U B Pravin Rao
Correct Answer: C [Nandan Nilekani]
Explanation: Nandan Nilekani, the Infosys co-founder and the architect of
Aadhaar, has been appointed as the new non-executive chairman of the board of
directors of Infosys. The sweeping changes have been made to the board in an
attempt to restore stability after the resignation of Vishal Sikka as CEO, and a
bitter rift between Narayana Murthy and the board came to the fore. Nilekanis
appointment will allow Infosys to focus on the strategic changes it needs to make
in order to capitalize on the attractive opportunities in the years ahead. Nilekani
served as Infosyss CEO from 2002 to 2007.
204. Tony de Brum, the well-known climate awareness activist has passed away.
He hailed from which country?
[A] Tonga
[B] Tuvalu
[C] Vanuatu
[D] Marshall Islands
Correct Answer: D [Marshall Islands]
2017 GKToday | All Rights Reserved | www.gktoday.in

239
Current Aairs: August, 2017 [Date-wise Compendium]

Explanation: Tony de Brum (72), the former Marshall Islands foreign, has passed
away in Majuro, Marshall Islands on August 22, 2017. He was the leading
advocate for the landmark Paris Agreement and an internationally recognised
voice in the ght against climate change. de Brum also fought against nuclear
weapons and for his nations independence.
205. The Union government has recently hiked OBC creamy layer ceiling limit to
how much for central government jobs?
[A] Rs 6 lakh
[B] Rs 8 lakh
[C] Rs 7 lakh
[D] Rs 11 lakh
Correct Answer: B [Rs 8 lakh]
Explanation: The Union Cabinet has recently increased the creamy layer ceiling
for the Other Backward Classes (OBC) category to Rs 8 lakh per annum from the
existing Rs 6 lakh for central government jobs. With this, those in the OBC
category earning up to Rs 8 lakh per annum would now get reservation benets.
206. Which city to host Digital Haryana Summit (SHS)?
[A] Rohtak
[B] Gurugram
[C] Panipat
[D] Sonipat
Correct Answer: B [Gurugram]
Explanation: The Haryana government will organize Digital Haryana Summit
(DHS) in Gurugram on September 15, 2017 to discuss issues like digital
governance, smart living in cities and villages, citizen partnership,
entrepreneurship, skilling and geo-governance and sustainability, among others.
The National Association of Software and Services Companies (NASSCOM) will be
partner for this event and the summit would be organised as a part of the states
Swarna Jayanti celebrations. The purpose of the summit is to bring together
dierent stakeholders and develop a singular agenda for the digital
transformation of Haryana, in line with the Digital India initiative.
207. Which former RBI governor has authored the book I Do What I Do?
[A] Duvvuri Subbarao
[B] Rahguram Rajan
[C] Manmohan Singh
[D] Amitav Ghosh
2017 GKToday | All Rights Reserved | www.gktoday.in

240
Current Aairs: August, 2017 [Date-wise Compendium]

Correct Answer: B [Rahguram Rajan]


Explanation: The book I Do What I Do has been authored by Rahguram Rajan,
the former RBI governor. This book conveys what it was like to be at the helm of
RBI in those turbulent but exciting times. Rajan explains economic concepts in a
readily accessible way. Equally, he addresses key issues that are not in any
banking manual but essential to growth: the need for tolerance and respect to
assure Indias economic progress, for instance, or the connection between
political freedom and prosperity. The book is published by HarperCollins India and
will be launched in Chennai on 4th September 2017, exactly a year after Rajan
quit as RBI governor.
208. Sanjivani Jadhav, who won silver in the World University Games, is
associated with which sports?
[A] Boxing
[B] Sprint
[C] Wrestling
[D] Judo
Correct Answer: B [Sprint]
Explanation: Sanjivani Jadhav from India, a marathon runner from Nashik, has
clinched silver medal in the womens 10,000-metre at the 29th World University
Games in Taipei, Taiwan on August 24, 2017. Earlier, she won a bronze in the
5000 metres at the Asian Athletics Championships in Bhubaneswar in July 2017.
209. As per 2017 UNWTO report on least-visited nations, which is the least
visited country in 2016?
[A] Bangladesh
[B] Moldova
[C] Comoros
[D] Tuvalu
Correct Answer: D [Tuvalu]
Explanation: According to 2017 report by the United Nations World Tourism
(UNWTO), the island of Tuvalu was the least visited country in 2016. Only 2,000
people ventured to the South Pacic paradise of Tuvalu in 2016. The latest report
reveals that there are 15 least-visited countries globally. Kiribati is the 2nd least
visited country, followed by Sao Tome and Principe, Montserrat, Comoros,
Djibouti, San Marino, Timor-Leste, Liechtenstein, Sierra Leone, Anguilla, Moldova,
Bangladesh, Bhutan and French Guiana.
210. The Murlen National Park (MNP) is located in which state?
2017 GKToday | All Rights Reserved | www.gktoday.in

241
Current Aairs: August, 2017 [Date-wise Compendium]

[A] Odisha
[B] Karnataka
[C] Mizoram
[D] Punjab
Correct Answer: C [Mizoram]
Explanation: The Murlen National Park (MNP) is located in the Champhai district
of Mizoram and covers an area of 200 square kilometres. The tropical, semi-
evergreen and sub montane Forests of Murlen are home to a rich variety of ora
and fauna. It is home to the tiger, leopard, sambar, barking deer, Malayan giant
squirrel, Humes pheasant, etc. About 15 species of mammals, 150 species of
birds, 35 species of Medicinal plants, 2 species of bamboos, and 4 species of
orchids so far have been recorded in this Park.
211. Which state government has launched a new E-Waste Disposal scheme in
state schools?
[A] Himachal Pradesh
[B] Jharkhand
[C] Uttar Pradesh
[D] Kerala
Correct Answer: D [Kerala ]
Explanation: The Kerala government has recently launched a new E-Waste
Disposal scheme in its state schools with an aim to process the electronic waste
in the IT labs, oces and stores. Through this scheme, over 1 crore kilogram of e-
waste from over 10,000 schools and oces will be scientically recycle, and the
funds earned through this e-waste disposal will be used to buy new computers for
the same institutions. The E-waste includes old monitors, mouses, keyboards and
UPS, among other things that have been lying unused since 2000 in the state. The
programme is a joint initiative of Kerala Infrastructure and Technology for
Education (KITE), formerly IT@School project and Clean Kerala Company.
212. Who has won the Biju Patnaik Award for Scientific Excellence-2016?
[A] Digambar Behera
[B] Sharat Kumar Pradhan
[C] Prasanta Mohapatra
[D] Bhishma Kumar Patel
Correct Answer: A [Digambar Behera]
Explanation: Odisha Chief Minister Naveen Patnaik has recently conferred the
Biju Patnaik Award for Scientic Excellence for the year 2016 to Prof Digambar

2017 GKToday | All Rights Reserved | www.gktoday.in

242
Current Aairs: August, 2017 [Date-wise Compendium]

Behera. Behera is senior professor and Head, Department of Pulmonary Medicine,


Postgraduate Institute of Medical Education and Research (PGIMER) in
Chandigarh. Beside this, Prof Prasanta Mohapatra, a professor at the University of
California, was also conferred this award for 2015. The award was instituted by
Odisha Bigyan Academy (OBA) and is given to any Odia scientist working
anywhere in India or abroad each year for his/her lifetime research contributions
in the eld of science and technology.
213. The first edition of Grameen Khel Mahotsav will kick-start from which city?
[A] Ahmedabad
[B] New Delhi
[C] Pune
[D] Lucknow
Correct Answer: B [New Delhi]
Explanation: The rst edition of Rural Games or Grameen Khel Mahotsav will
kick-start from Delhi and will take place between 28th August to 3rd September
2017. The purpose of the Rural Games is to popularize the indigenous games like
wrestling and athletics and will also have fun games like Matka Race, Tug of War
for senior citizens. In the rst stage, the games will be held in Alipur, Mehrauli,
Nangloi, Najafgarh and Shahadara where ten thousand youngsters will participate.
The Union Ministry of Youth Aairs and Sports will also launch sports talent search
portal. The portal will enable any person from across country to upload his
information and as per that the Ministry will tap their talent and train them further
in their respective disciplines.
214. Which RBI committee has recently recommended that banks should link
home loan rates to repo rate?
[A] Viral Acharya
[B] H R Khan
[C] Tarun Ramadorai
[D] Amitav Ghosh
Correct Answer: C [Tarun Ramadorai]
Explanation: The Reserve Bank of India (RBI) appointed committee on
Household Finance has recently recommended that banks should link their home
loan rates to the RBIs repo rate, the rate at which it lends to banks. The
committee, whose recommendations are not binding, suggests that this will allow
for better and more transparent transmission of interest rates in the economy to
individuals. As per recommendations, banks should quote loans to customers
using the RBI repo rate rather than based on their own MCLR rates. This would
make it easier for customers to compare rates at the time of purchase. The
2017 GKToday | All Rights Reserved | www.gktoday.in

243
Current Aairs: August, 2017 [Date-wise Compendium]

committee was chaired by Dr Tarun Ramadorai, who is Professor of Financial


Economics, Imperial College Business School in London. The committee has also
recommended that all banks use the same reset period of one month for loans.
Under the current system, oating rate loans have a xation period of roughly one
year. The report argues that the current system impedes monetary transmission
mechanism and does not allow borrowers to immediately benet from interest
rate drops.
215. The Institute of Chartered Accountants of India (ICAI) has recently signed
MoU with which country on mutual co-operation?
[A] Nepal
[B] Bhutan
[C] Bangladesh
[D] Sri Lanka
Correct Answer: A [Nepal]
Explanation: The Institute of Chartered Accountants of India (ICAI) has recently
signed the Memorandum of Understanding (MoU) with the Institute of Chartered
Accountants of Nepal (ICAN) to establish mutual co-operation for the
advancement of accounting knowledge, professional and intellectual
development, advancing the interests of their respective members and positively
contributing to the development of the accounting profession in Nepal and India.
ICAI has a membership base of over 5,000 members in Nepal. The ICAI was
established on July 1, 1949 as a statutory body under the Chartered Accountants
Act, 1949 enacted by the to regulate the profession of Chartered Accountancy in
India. Its headquarters is located in New Delhi.
216. Who will officially launch the National Sports Talent Search Portal?
[A] Narendra Modi
[B] Venkaiah Naidu
[C] Vijay Goel
[D] Ram Nath Kovind
Correct Answer: B [Venkaiah Naidu]
Explanation: Venkaiah Naidu, the Vice President of India, will ocially launch the
Sports Ministrys much- awaited National Sports Talent Search Portal at the Indira
Gandhi Indoor Stadium in New Delhi on August 28, 2017. The portal will provide a
three step simple process for registration, prole creation and upload of
achievements. Approx 1000 kids will be selected and they will receive
scholarships of Rs 5 lakh per year for eight years. It will be available in English
and other regional languages so that it is easier for any boy or girl who is talented
in a particular sport to upload his or her achievements from any part of the
2017 GKToday | All Rights Reserved | www.gktoday.in

244
Current Aairs: August, 2017 [Date-wise Compendium]

country. The portal will also be available as App which can be downloaded on
smartphones.
217. Which task force has been constituted by the Union Government on Artificial
Intelligence (AI) for Indias economic transformation?
[A] Gautam Shroff task force
[B] Shantanu Chaudhary task force
[C] V. Kamakoti task force
[D] Vijay Kumar Sankarapu task force
Correct Answer: C [V. Kamakoti task force]
Explanation: Nirmala Sitharaman, the Union Commerce and Industry Minister,
has constituted a task force on Articial Intelligence (AI) for Indias economic
transformation. The 18-member panel, headed by Dr. V. Kamakoti of IIT Madras,
will comprise of experts, academics, researchers and industry leaders. It will
explore possibilities to leverage Articial Intelligence (AI) for development across
various elds. In addition to regular members, it will have ocial participation
from NITI Aayog, Ministry of Electronics and Information Technology, Department
of Science & Technology, UIDAI and DRDO. The task force will submit concrete
and implementable recommendations for government, industry and research
institutions.
218. Jay Thomas, the noted comic actor passed away. He hailed from which
country?
[A] Germany
[B] France
[C] United Kingdom
[D] United States
Correct Answer: D [United States]
Explanation: Jay Thomas (69), the well-known comic and character actor, has
passed away in California, United States. In addition to playing Eddie LeBec in
Cheers, Jay played obnoxious TV talk show host Jerry Gold on Murphy Brown
from 1989 until 1998, and also had his own sit-com, Love and War, from 1992
until 1995. He was also a regular guest on David Lettermans talk show during the
festive season.
219. The Chitwan National Park (CNP) is located in which country?
[A] India
[B] Bhutan
[C] Bangladesh

2017 GKToday | All Rights Reserved | www.gktoday.in

245
Current Aairs: August, 2017 [Date-wise Compendium]

[D] Nepal
Correct Answer: D [Nepal]
Explanation: The Chitwan National Park (CNP) is located in the subtropical Inner
Terai lowlands of south-central Nepal and covers an area of 932 square
kilometres. The park is considered to have the highest population density of sloth
bears with an estimated 200 to 250 individuals. It is also home to golden jackals,
shing cats, jungle cats, leopard cats, large and small Indian civets, Asian palm
civets, crab-eating mongooses and yellow-throated martens.
220. Who has won bronze in the 2017 womens singles World Badminton
championships (WBC)?
[A] Ashwini Ponnappa
[B] Jwala Gutta
[C] P V Sindhu
[D] Saina Nehwal
Correct Answer: D [Saina Nehwal ]
Explanation: Saina Nehwal from India has won bronze in the 2017 womens
singles World Badminton championships (WBC). In the rst womens singles semi-
nal at Glasgow, Nehwal lost to Nozomi Okuhara of Japan in three games 12-21,
21-17, 21-10. It must be noted that Okuhara is the rst shuttler from Japan to
reach the nals of World Championship. Apart from this, Olympic silver medallist P
V Sindhu from India has won 2nd Womens Singles semi-nal by defeating
Chinese Chen Yufei 21-13 21-10 on August 26, 2017 and reached at nals of the
World championship.
221. Which country has won the 2017 South Asian Football Federation (SAFF)
under-15 football Championship?
[A] India
[B] Bhutan
[C] Nepal
[D] Bangladesh
Correct Answer: A [India]
Explanation: Indian football team has won the 2017 South Asian Football
Federation (SAFF) under-15 football Championship by defeating Nepal by 2-1 in
the nal at Kathmandu in Nepal on August 27. Vikram Pratap Singh was declared
best player of the championship. India won all four matches in the championship.
In the league matches, India beat Maldives by 9-0 and host Nepal by 2-1. In the
seminal India defeated Bhutan by 3-0. India scored 16 goals and conceded just

2017 GKToday | All Rights Reserved | www.gktoday.in

246
Current Aairs: August, 2017 [Date-wise Compendium]

two.
222. Who has won the 2017 womens singles World Badminton championships
(WBC)?
[A] Yihan Wang
[B] Nozomi Okuhara
[C] Hsiao Huan Chen
[D] Xin Okuhara
Correct Answer: B [Nozomi Okuhara]
Explanation: Japanese shuttler Nozomi Okuhara has won the 2017 womens
singles World Badminton championships (WBC) by defeating P V Sindhu in the
nal by 12-21, 21-17, 21-10 at the Emirates Arena in Glasgow, Scotland. With this,
Sindhu had to settle for silver in the championship. Earlier, Saina Nehwal ended
her campaign with a bronze. The Badminton Association of India (BAI) has
announced cash award of Rs 10 lakh for Sindhu and Rs ve lakh for Saina
following their medal winning performances at the world Championships. Sindhu,
who had won bronze twice in the 2013 and 2014 editions, became only the
second Indian to win a silver medal after Saina Nehwal had achieved the feat in
the last edition at Jakarta in 2015.
223. Who has won 2017 Formulae 1 Belgian Grand Prix tournament?
[A] Daniel Ricciardo
[B] Sebastian Vettel
[C] Lewis Hamilton
[D] Max Verstappen
Correct Answer: C [Lewis Hamilton]
Explanation: Lewis Hamilton, a British racing driver who races in Formula One
for the Mercedes AMG Petronas team, has won the 2017 Belgian Grand Prix at the
Circuit de Spa-Francorchamps in Stavelot, Belgium on August 27, 2017.
224. The National Sports Talent Search Portal has officially launched in which
city?
[A] Guwahati
[B] Jaipur
[C] Bhopal
[D] New Delhi
Correct Answer: D [New Delhi]
Explanation: Vice-President Venkaiah Naidu has ocially launched the Union
Sports Ministrys much-awaited National Sports Talent Search Portal at a function
2017 GKToday | All Rights Reserved | www.gktoday.in

247
Current Aairs: August, 2017 [Date-wise Compendium]

in the Indira Gandhi Indoor Stadium in New Delhi on August 28, 2017. The portal
will enable any person from across country to upload his information and as per
that the Ministry will tap their talent and train them further in their respective
disciplines. The portal will provide a three step simple process for registration,
prole creation and upload of achievements. Around 1000 children will be
selected and they will receive scholarships of Rs 5 lakh per year for eight years.
The portal will be available in English and other regional languages so that any
boy or girl can upload his or her achievements from any part of the country. The
portal will also be available as App which can be downloaded on smart phones.
225. Who has been sworn-in as the new Chief Justice of India (CJI)?
[A] Navneet Kaur
[B] Dipak Misra
[C] Shekhar Gupta
[D] Nirmal Jain
Correct Answer: B [Dipak Misra]
Explanation: Justice Dipak Misra has been sworn-in as the new Chief Justice of
India (CJI) and will have tenure of nearly 13 months. He succeeded Chief Justice J
S Khehar. Justice Misra was enrolled as an Advocate in 1977 and practiced in
Constitutional, Civil, Criminal, Revenue, Service and Sales Tax matters in the
Orissa High Court and the Service Tribunal. He was appointed as an Additional
Judge of the Orissa High Court in 1996 and became permanent Judge in
December, 1997. He also served as the Chief Justice of Patna and Delhi High
Courts. In 2011, Justice Misra was elevated as a Judge in the Supreme Court.
226. Which country to host the 2017 Indian Ocean Conference (IOC)?
[A] Sri Lanka
[B] India
[C] Bangladesh
[D] Myanmar
Correct Answer: A [Sri Lanka]
Explanation: The 2nd edition of Indian Ocean Conference (IOC) will be held in
Colombo, Sri Lanka on August 31, 2017 with theme Peace, progress, and
prosperity. The 2-day conference is being co-hosted by the India Foundation, a
Delhi-based think tank, in collaboration with RSIS, Singapore, and NIFS, Colombo
both research and study centers. The representatives from around 35 countries
are expected to participate in the event. From India, External aairs minister
Sushma Swaraj will participate in the conference. The conference will also be
attended by ocials from the USA, Australia and Germany among others.

2017 GKToday | All Rights Reserved | www.gktoday.in

248
Current Aairs: August, 2017 [Date-wise Compendium]

227. Which Indian-origin boy has recently got top score in Mensa IQ in United
Kingdom (UK)?
[A] Milan Sharma
[B] Jayeesh Seth
[C] Dhruv Garg
[D] Nitin Das
Correct Answer: C [Dhruv Garg]
Explanation: Dhruv Garg, an Indian-origin boy from Wokingham in south east
England, in the United Kingdom (UK) has got the highest possible score of 162 on
a Mensa IQ test that placed him in the top 1% people in the world who achieved
this feat. Garg, who goes to Reading School, a grammar in Berkshire, has also
been developing an app to combat social isolation by helping lonely people meet
up. The app connects people who live in the same area who want to meet new
people. Mensa is believed to be the largest and oldest high IQ society in the
world. Membership is open to anyone who can demonstrate an IQ in the top two
per cent of the population, measured by a recognised or approved IQ testing
process.
228. The Reserve Bank of India (RBI) has recently approved Spice Digital to
process bill payments under BBPS. Where is the headquarters of the Spice
Digital?
[A] Chandigarh
[B] Noida
[C] Kochi
[D] Chennai
Correct Answer: B [Noida]
Explanation: The Spice Digital Limited (SDL) has received the nal licence from
the Reserve Bank of India (RBI), to process bill payments as a Bharat Bill Payment
Operating Unit (BBPOU) in the Bharat Bill Payment System (BBPS). The BBPS is a
RBI conceptualised system driven by National Payments Corporation of India
(NPCI). It is a one-stop payment platform for all bills providing an interoperable
and accessible Anytime Anywhere bill payment service to all customers across
India with certainty, reliability and safety of transactions. The headquarters of the
Spice Digital is located at Noida in National Capital Territory of Delhi (NCR). The
company provides Telco Solutions, Value Added Services (MVAS), Enterprise
Solutions, Financial technology, GSP and Digital transformation products and
services. It is a subsidiary of listed entity S Mobility, which has diversied interests
in telecom space.

2017 GKToday | All Rights Reserved | www.gktoday.in

249
Current Aairs: August, 2017 [Date-wise Compendium]

229. Which of the following IIT institutes have developed biosensor to detect
kidney disorders?
[A] IIT Bombay and IIT Indore
[B] IIT Kanpur and IIT Bombay
[C] IIT Indore and IIT Delhi
[D] IIT Delhi and IIT Kanpur
Correct Answer: A [IIT Bombay and IIT Indore]
Explanation: The Indian Institutes of Technology, Bombay and Indore, have
jointly developed a biosensor that makes it possible to detect kidney disorders in
less than 8 minutes. The biosensor can accurately measure both the pH and urea
concentration with a single drop of urine. The researchers who developed it say
that it will help make a point-of-care test to determine whether the kidneys are
functioning normally. The biosensor is made by encapsulating an enzyme urease
and a molecule FITC-dextran in alginate microspheres. The combination glows in
response to a chemical reaction with urea and changes in pH when urine is
added. The uorescence reduces when the pH is acidic and increases when it is
alkaline. The changes in uorescence intensity are measured, which helps to
calculate the values of pH and urea. Thus, it will help make a rapid and accurate
point-of-care diagnostic test for kidney disorders.
230. The Abohar Wildlife Sanctuary (AWLS) is located in which state?
[A] Haryana
[B] Himachal Pradesh
[C] Uttar Pradesh
[D] Punjab
Correct Answer: D [Punjab]
Explanation: The Abohar Wildlife Sanctuary (AWLS) is located at a distance of 15
km from the city Abohar in the Fazilka district of Punjab and spread over 186.5 sq
km area. It is a unique open sanctuary which encompasses 13 revenue villages.
The sanctuary is known for highest number of black bucks, besides other
protected species. Its 100-m diameter area is termed as Eco-Sensitive Zone
(ESZ). It is home to several varieties of ora and fauna such as Acacia nilotica,
Azadirachata indica, Blue Bull, Porcupines, Hare and Wild Boar.
231. The 2017 National Sports Day (NSD) is celebrated on which day in India?
[A] August 28
[B] August 29
[C] August 26
[D] August 27
2017 GKToday | All Rights Reserved | www.gktoday.in

250
Current Aairs: August, 2017 [Date-wise Compendium]

Correct Answer: B [August 29]


Explanation: The National Sports Day (NSD) is celebrated every year on 29th
August to mark the birthday of legendary hockey player Dhyan Chand. On this
occasion, in 2017, President Ram Nath Kovind has confered Rajiv Gandhi Khel
Ratna Award, Arjuna Award and Dronacharya Award at Rashtrapati Bhawan in
New Delhi. The Khel Ratna Award is honoured for the players spectacular and
most outstanding performance in the eld of sports over a period of four years at
international level. The Arjuna Awards are given to recognize outstanding
achievement in National sports while the Dronacharya Award is sports coaching
honour.
232. Who will lead Indian delegation at the 2017 BRICS Summit?
[A] Ajit Kumar Doval
[B] Sushma Swaraj
[C] Narendra Modi
[D] Manohar Parrikar
Correct Answer: C [Narendra Modi ]
Explanation: Prime Minister Narendra Modi will visit Xiamen in Chinas Fujian
province to attend the 9th BRICS summit, which will be held from September 3-5,
2017. Subsequently, the Prime Minister will pay a State visit to Myanmar from
September 5th to 7th. This will be Prime Minister Modis rst bilateral State visit to
Myanmar. During the visit, Prime Minister will hold discussions with State
Counsellor Daw Aung San Suu Kyi on matters of mutual interest and also call on
President U Htin Kyaw. Apart from his engagements in the capital city of Nay Pyi
Taw, Mr Modi will visit Yangon and Bagan.
233. Ahmed Khan, who passed away recently, was associated with which sports?
[A] Football
[B] Wrestling
[C] Cricket
[D] Hockey
Correct Answer: A [Football]
Explanation: Ahmed Khan (90), legendary footballer and Olympian, has passed
away in Bengaluru on August 27, 2017. He is best remembered for his role in the
1948 Olympics in London, where India lost its rst-round match in heartbreaking
fashion to France but made a deep impression on the public. He was also part of
the Indian sides that won gold at the Asian Games of 1951 and went to the 1952
Olympics in Helsinki, will be remembered as a gifted inside-left who mesmerised
spectators with his ball control. Khan was also known for being an East Bengal
2017 GKToday | All Rights Reserved | www.gktoday.in

251
Current Aairs: August, 2017 [Date-wise Compendium]

legend and was a part of the famous Pancha Pandavas of red and gold. The
other four were Venkatesh, Saleh, Appa Rao and Dhanraj.
234. Which city hosted the Defence Accounts Department (DAD)s first synergy
conference on Defence Pensions?
[A] Guwahati
[B] Pune
[C] Kanpur
[D] New Delhi
Correct Answer: D [New Delhi]
Explanation: The Defence Accounts Department (DAD) has organized its rst
Synergy Conference on Defence Pensions in New Delhi on August 28, 2017. The
conference was attended by all Defence Pensioners Associations and was open to
all Defence Pensioners. The purpose of the conference was to obtain feedback
from the pensioners that would help the department to serve them better with
constantly improving systems and procedures. Digitization of pension data and
implementation of a pensions app were also discussed in detail.
235. India has signed pact with which country to improve grid integration of
renewable energy?
[A] France
[B] Germany
[C] Italy
[D] Nepal
Correct Answer: B [Germany ]
Explanation: India and Germany has recently signed pact on technical
cooperation under the Indo-German Energy Programme Green Energy Corridors
(IGEN-GEC). The main purpose of this programme component is to improve the
sector framework and conditions for grid integration of renewable energy. As per
the pact, both the countries will work on improving market mechanisms and
regulations for integration of renewable energies, advancing technical and
institutional conditions in specied target states, regions and on a national level,
adding human capacities to handle systemic (strategic, managerial, nancial,
technical) renewable energies integration in an ecient and eective manner.
236. ICEX has launched the worlds first diamond future exchange. What does
ICEX stands for?
[A] Indian Commodity Exchange
[B] International Commodity Exchange

2017 GKToday | All Rights Reserved | www.gktoday.in

252
Current Aairs: August, 2017 [Date-wise Compendium]

[C] Indonesia Commodity Exchange


[D] None of the above
Correct Answer: A [Indian Commodity Exchange]
Explanation: The Indian Commodity Exchange (ICEX) has become the worlds
rst derivatives exchange to launch diamond futures contract. Initially, the
contracts are launched in size of 1 carat with compulsory delivery. This will create
an entirely new market for the diamond players where sellers can deliver their
certied diamonds to dierent buyers.The launch will also lead to ecient price
discovery based on demand and supply and a nationwide market participation.
Till date, over 100 members and around 4,000 clients have registered with the
exchange. The exchange will oer a fair, transparent and nationwide market,
bringing in large market participation into diamond trade. It will result in ecient
price discovery and provide an eective hedging platform for all stakeholders.
The new exchange will be able to make a meaningful contribution towards further
development of commodity markets.
237. Nungthang Tampak, which has become North-East (NE)s first 100%
computer literacy village, is located in which state?
[A] Nagaland
[B] Arunachal Pradesh
[C] Manipur
[D] Mizoram
Correct Answer: C [Manipur ]
Explanation: The Nungthaang Tampak village in Manipur has become North-East
(NE)s rst 100% computer literacy village. The training program was organised
by Mangaal Rural, an NGO working for rural development, as a part of the Digital
India program and is aliated to All India Society for Electronic and Computer
Technology (AISECT). Keralas Chamravattom village is the Indias rst 100%
computer literate village.
238. Who is the newly appointed chief of Bombay Stock Exchange (BSE)?
[A] Narendra Kumar
[B] S S Khan
[C] Dharmendra Singh
[D] Ashish Chauhan
Correct Answer: D [Ashish Chauhan]
Explanation: Ashish kumar Chauhan has been re-appointed as the new
Managing Director and CEO of Bombay Stock Exchange (BSE) for the period of 5
years from November 2, 2017 to November 1, 2022. For this, Securities and
2017 GKToday | All Rights Reserved | www.gktoday.in

253
Current Aairs: August, 2017 [Date-wise Compendium]

Exchange Board of India (SEBI) has recently gave its approval to re-appoint
Chauhan as MD and CEO of the company for next 5 years.
239. Which country has launched the worlds first climate-smart snow-leopard
plan?
[A] Nepal
[B] India
[C] Bangladesh
[D] Bhutan
Correct Answer: A [Nepal]
Explanation: The Government Nepal has launched the worlds rst climate-smart
snow-leopard management plan to safeguard this endangered species of snow
leopard and its habitat. The Nepals conservation plan addressed key current and
emerging threats to snow leopards including climate change and will be used as a
model for other range countries to adopt. It was discussed in the 2017
International Snow Leopard Summit and Ecosystem Forum in Bishkek, Kyrgyzstan,
where world leaders hold critical talks to strengthen previous commitments to
safeguard the future of the snow leopard and its habitat.
240. The Lawachara National Park (LNP) is located in which country?
[A] Sri Lanka
[B] Bangladesh
[C] Nepal
[D] Bhutan
Correct Answer: B [Bangladesh]
Explanation: Lawachara National Park (LNP) is located at Kamalganj Upazila,
Maulvi Bazar District of Bangladesh and covers an area of 12.5 km2. It is a major
national park and nature reserve in Bangladesh. There are 460 species, of which
167 species are plants, 4 amphibian species, 6 reptile species, 246 bird species,
20 mammal species, and 17 insect species. The park is considered of critical
importance because the last viable population of critically endangered Western
Hoolock Gibbons survive here.
241. Who has been crowned as Indias first-ever Miss TransQueen 2017?
[A] Reena Rai
[B] Loiloi Haorongbam
[C] Ragasya
[D] Nitasha Biswas
Correct Answer: D [Nitasha Biswas]

2017 GKToday | All Rights Reserved | www.gktoday.in

254
Current Aairs: August, 2017 [Date-wise Compendium]

Explanation: Kolkata-based Nitasha Biswas has been crowned Indias rst-ever


Miss TransQueen 2017 in Gurugram, Haryana. Currently, Biswas is pursuing
Masters in Business Management in Kolkata, West Bengal. Loiloi Haorongbam
from Manipur became the rst runner-up, while Ragasiya from Tamil Nadu was
the second runner-up. Now, Biswas will represent India at the Miss International
TransQueen in Thailand, one of the most prestigious international pageants for
the transgender community, in March 2018. And, Loiloi will compete at Miss
Transsexual Australia. The event was judged by a panel of eight comprising social
activists, actors, beauty experts and media professionals.
242. The 2017 International Day against Nuclear Tests (IDANT) is observed on
which date?
[A] August 28
[B] August 30
[C] August 29
[D] August 31
Correct Answer: C [August 29]
Explanation: The International Day against Nuclear Tests (IDANT) is observed
every year on August 29 to raise public awareness about the eects of global
nuclear weapon tests or any other nuclear explosions and the need for their
cessation as one of the means of achieving the goal of a nuclear-weapon-free
world. The purpose of the day is to end nuclear testing and to promote peace and
security.
243. Who has become the 2nd Indian women and first Sikh women to be selected
by NASA for the Mars Expedition happening in 2030?
[A] Jasleen Kaur Josan
[B] Simran Bagga
[C] Jaspreet Kaur
[D] Sukhpreet Kaur
Correct Answer: A [Jasleen Kaur Josan]
Explanation: After Kalpana Chawla, Jasleen Kaur Josan has become the 2nd
Indian Women and rst Sikh women to be selected by NASA for the Mars
Expedition happening in 2030. The Orion Mission will be the rst man mission to
Mars, which is on track to take place in 2030. The candidates were selected from
around the world. The 2020 MARS Mission is a one-way mission where the group
inhabiting Mars will not return back to planet Earth. However, Jasleen is part of
the two-way mission which is set to happen in 2030.
244. The worlds first IAEA Low Enriched Uranium (LEU) Bank has opened in
2017 GKToday | All Rights Reserved | www.gktoday.in

255
Current Aairs: August, 2017 [Date-wise Compendium]

which country?
[A] Kazakhstan
[B] Israel
[C] Japan
[D] Kyrgyzstan
Correct Answer: A [Kazakhstan]
Explanation: The International Atomic Energy Agency (IAEA) has launched the
worlds rst IAEA Low Enriched Uranium (LEU) bank at the Ulba Metallurgical Plant
(UMP) in Oskemen, Kazakhstan. The $150 million facility will help ensure a steady
supply of low enriched uranium for countries nuclear power programmes. The
bank is owned and managed by IAEA. It is the rst of its kind LEU bank not to be
under control of any individual country. The IAEA LEU Bank will be a physical
reserve of up to 90 metric tons of low enriched uranium suitable to make fuel for
a typical light water reactor, the most widely used type of nuclear power reactor
worldwide. The LEU Bank will serve as a last-resort mechanism to provide
condence to countries that they will be able to obtain LEU for the manufacture of
fuel for nuclear power plants in the event of an unforeseen, non-commercial
disruption to their supplies. The LEU can be used to make enough nuclear fuel to
power a large city for three years.
245. Who is the current Chairman & Managing Director of National Research
Development Corporation (NRDC)?
[A] B N Sarkar
[B] H Purushotham
[C] Narayan Rao Gali
[D] Satyawati Sharma
Correct Answer: B [H Purushotham]
Explanation: The National Research Development Corporation (NRDC) is the
technology transfer and commercialization arm of the Union Ministry of Science &
Technology. Its primary objective is to help develop and promote technologies
developed at various national R&D institutions. Recently, it is in news because the
organization has won the 2017 ASSOCHAM Services Excellence Award and its
CMD Dr. H Purushotham won an international recognition. The International
Association of Advanced Materials (IAAM) Sweden honored Dr. Purushotham with
the IAAM Medal 2017 for his outstanding R&D contributions in the Advanced
Materials Science & Technology during its award ceremony in Stockholm, Sweden.
246. India and which country will jointly issue postage stamps with Diwali as
theme?
2017 GKToday | All Rights Reserved | www.gktoday.in

256
Current Aairs: August, 2017 [Date-wise Compendium]

[A] Bhutan
[B] Indonesia
[C] Canada
[D] Sri Lanka
Correct Answer: C [Canada]
Explanation: The Union Cabinet chaired by Prime Minister Narendra Modi has
recently informed that India and Canada have mutually agreed to jointly issue a
set of two Commemorative Postage Stamps on the theme Diwali. The joint stamps
will be released on September 21, 2017, as per MoU signed between the postal
departments of the two countries. In this joint issue, the theme Diwali has been
selected as it is a cultural theme for both the countries and also considering the
large presence of Indian Diaspora in Canada.
247. The Uttar Pradesh government has launched Startup Yatra programme from
which city?
[A] Varanasi
[B] Allahabad
[C] Kanpur
[D] Lucknow
Correct Answer: D [Lucknow ]
Explanation: Yogi Adityanath, the Uttar Pradesh chief Minister, has launched the
Startup Yatra programme in Lucknow on August 30, 2017. Beside this, the
government has also launch a mobile application and a call centre for startups. As
per programme, the state government will provide nancial assistance of Rs 15
thousand per month and maximum Rs 10 lakh rupees in a year to those who will
begin startups. For this, the state government will soon sign MoU with Small
Industries Development Bank of India (SIDBI). Under this policy, the government
will appoint one person in each district of the state who with the use of
technology will develop a brand product and connect it with the concerned
district.
248. Which state/UT police has launched a skill development programme
YUVA?
[A] Puducherry Police
[B] Karnataka Police
[C] Delhi Police
[D] Uttar Pradesh Police
Correct Answer: C [Delhi Police]

2017 GKToday | All Rights Reserved | www.gktoday.in

257
Current Aairs: August, 2017 [Date-wise Compendium]

Explanation: Rajnath Singh, the Union Home Minister, has inaugurated the YUVA
a skill development programme and an initiative by Delhi Police under Pradhan
Mantri Kaushal Vikas Yojana (PMKVY) on August 29, 2017. The initiative will
engage street children and youth, providing them with opportunities to hone their
skills and utilise it professionally through skill development training. For YUVA
initiative, Delhi Police has tied up with National Skill Development Corporation
(NSDC) and Confederation of Indian Industry (CII) for providing mass job linked
skill training for the selected youth. The NSDC will provide skill training to the
youth under PMKVY and CII will provide job linked training through its Sector Skill
Councils who are connected to industry and thereby provide job guarantee.
249. The Gobindobhog rice has recently got Geographical Indication (GI) status.
This type of rice is primarily grown in which state?
[A] Karnataka
[B] West Bengal
[C] Manipur
[D] Nagaland
Correct Answer: B [West Bengal]
Explanation: Gobindobhog rice from Burdwan district of West Bengal has
recently got the Geographical Indication (GI) status. As a result of getting the GI
tag, rice from other regions or rice of other varieties cannot be branded as
Gobindobhog. Hence, the marketability of the rice would be strengthened, for
the local, national and international markets. The region of Burdwan (now divided
into the districts of east and west Burdwan) is known as the rice bowl of Bengal.
This variety of rice is primarily cultivated in east Burdwan district in the southern
basin of the Damodar river in the Raina 1, Raina 2 and Khandaghosh blocks. The
south Damodar belt has been the traditional area of Gobindobhog rice cultivation.
The rice has several advantages. It is cultivated late and therefore not much
aected by rains. It is less prone to pests as well. The productivity per area is high
and farmers get better prices for Gobindobhog rice.
250. Sumiteru Taniguchi, who passed away recently, was the prominent nuclear
disarmament campaigner of which country?
[A] Bhutan
[B] Japan
[C] China
[D] South Korea
Correct Answer: B [Japan]
Explanation: Sumiteru Taniguchi (88), the prominent nuclear disarmament
2017 GKToday | All Rights Reserved | www.gktoday.in

258
Current Aairs: August, 2017 [Date-wise Compendium]

campaigner and peace activist, has passed away in Nagasaki, Japan on August 30.
He was the survivor of the 1945 atomic bombing of Nagasaki and a prominent
activist for a treaty prohibiting nuclear weapons, and chairman of the Nagasaki
Council of the A-Bomb Suerers. He was 16 when the atomic attack happened,
and was about 1.8 kilometers away from ground zero when the bomb hit. He
suered severe burns on his back. He appealed for the support of survivors and
elimination of nuclear weapons throughout his life. Ten years after the bombing,
Taniguchi formed a group with other survivors who experienced the ordeal.
251. Who will be the new Comptroller and Auditor General (CAG) of India?
[A] Anita Pattanayak
[B] Rajiv Gauba
[C] Shashi Kant Sharma
[D] Rajiv Mehrishi
Correct Answer: D [Rajiv Mehrishi]
Explanation: Rajiv Mehrishi, the former home secretary, will be the new
Comptroller and Auditor General (CAG) of India. He will replace the present
incumbent Shashi Kant Sharma who completes his term on September 30, 2017.
Meanwhile, three senior bureaucrats were appointed as Deputy Comptroller and
Auditor General. They are Ashwini Attri, Anita Pattanayak and Ranjan Kumar
Ghose.
252. Who is leading the Indian delegation at the Indian Ocean Conference (IOC
2017)?
[A] Narendra Modi
[B] Ajit Doval
[C] Sushma Swaraj
[D] Subrahmanyam Jaishankar
Correct Answer: C [Sushma Swaraj]
Explanation: Sushma Swaraj, the Union External aairs minister, is leading the
Indian delegation at the Indian Ocean Conference 2017 (IOC 2017), which is
started in Colombo, Sri Lanka on August 31, 2017. The theme of the conference is
Peace, progress, and prosperity. The 2-day conference is being co-hosted by the
India Foundation, a Delhi-based think tank, in collaboration with Rajaratnam
School of International Studies (RSIS), Singapore and National Institute of
Fundamental Studies (NIFS), Colombo. The conference will see participation from
around 35 countries and have speakers from 25 countries.
253. Who has taken charge as the new Union Home Secretary?
[A] Nripendra Misra
2017 GKToday | All Rights Reserved | www.gktoday.in

259
Current Aairs: August, 2017 [Date-wise Compendium]

[B] Rajiv Gauba


[C] Ajit Sinha
[D] Rajiv Mehrishi
Correct Answer: B [Rajiv Gauba]
Explanation: Rajiv Gauba, A 1982-batch IAS ocer of Jharkhand cadre, has
recently taken charge as the Union Home Secretary after incumbent Rajiv
Mehrishi retired from service. He will have tenure of two years. As the Union home
secretary, he will have to handle issues like internal security, militancy in Jammu
and Kashmir and the Northeast, Maoist problems in the central and east India
besides other issues. Earlier, Gauba was serving in the Home Ministry as Ocer
on Special Duty. Prior to this, Gauba was Secretary in the Ministry of Urban
Development. He also represented India on the board of International Monetary
Fund (IMF) for four years.
254. Who has been appointed as the new High Commissioner of India to the
Republic of Cyprus?
[A] R.K. Raghavan
[B] PK Sinha
[C] Atul K Chatterjee
[D] B N Mitra
Correct Answer: A [R.K. Raghavan]
Explanation: Dr. R.K. Raghavan, a 1963 cadre IPS ocer, has been appointed as
the new High Commissioner of India to the Republic of Cyprus and is expected to
take up the assignment shortly. Raghavan, who was Central Bureau of
Investigation (CBI) director from January 1999 to April 2001, had investigated high
prole cases like Priyadarshini Mattoo murder case, 2000 South Africa cricket
match xing, 2002 Gujarat riots. Prior to this, he was the director-general of Tamil
Nadus State Vigilance Directorate from 1993 to 1999. He oversaw probe into
corruption cases against AIADMK leader J Jayalalithaa. After his retirement,
Raghavan was also corporate security adviser with Tata Consultancy Services and
served as consulting adviser to O P Jindals Board of Management.
255. ISRO had launched navigation satellite IRNSS-1H into orbit, but the mission
has failed. It was launch by which rocket vehicle?
[A] PSLV C35
[B] PSLV C37
[C] PSLV C38
[D] PSLV C39
Correct Answer: D [PSLV C39 ]
2017 GKToday | All Rights Reserved | www.gktoday.in

260
Current Aairs: August, 2017 [Date-wise Compendium]

Explanation: The Indian Space Research Organisation (ISRO) has launched its
navigation satellite IRNSS-1H into the orbit by the rocket PSLV C39 from the
Satish Dhavan Space Centre at Sriharikotta, Andhra Pradesh on August 31, 2017.
Though, the IRNSS-1H mission was failed as heat shield did not open up for the
deployment of the satellite. The IRNSS-1H was a part of the indigenous navigation
system called NavIC, that was initially planned to have a constellation of seven
satellites. NavIC system is equivalent to the GPS service, having footprint over the
whole of India and about 1500 kilometers from its borders on all sides. It helps
user agencies to plan and monitor their vehicular movement on the road and on
the sea. It can alert shermen while approaching international maritime
boundaries and help public to locate precisely the addresses they require.
256. The Ujjivan Small Finance Bank (SFB) Ltd has received Scheduled Bank
status by the RBI. Where is the headquarters of Ujjivan SFB?
[A] Chennai
[B] Lucknow
[C] Pune
[D] Bengaluru
Correct Answer: D [Bengaluru]
Explanation: The Ujjivan Small Finance Bank (SFB) Ltd, a wholly-owned
subsidiary of Ujjivan Financial Services Ltd., has been included in the Second
Schedule to the Reserve Bank of India (RBI) Act, 1934. With this, Ujjivan SFB has
attained the status of a Scheduled Bank. This status will enhance the market
acceptability of the bank in its eort to garner institutional deposits at a
competitive price and participate more actively in the inter-bank market. It also
opens the door for issuing Certicates of Deposits (CDs), which will be an
important source of funding. Currently, Ujjivan SFB has 65 full-edged brick and
mortar branches in eight states and two Union Territories. It plans to convert 160
existing branches and 29 new Unbanked Rural Centres (URCs) by the end of FY
2017-18 and the remaining branches will be converted over a period of two years
in line with RBI approval. The headquarters of Ujjivan SFB is located in Bengaluru.
257. Which IIT institute has successfully developed an implantable bioartificial
pancreas?
[A] IIT Guwahati
[B] IIT Bombay
[C] IIT Indore
[D] IIT Delhi
Correct Answer: A [IIT Guwahati]

2017 GKToday | All Rights Reserved | www.gktoday.in

261
Current Aairs: August, 2017 [Date-wise Compendium]

Explanation: The researchers at the Indian Institute of Technology (IIT) Guwahati


have successfully created an implantable bioarticial pancreas model grown
within a 3D silk scaold. The bioarticial pancreas, which encapsulates insulin-
producing cells, is capable of naturally producing insulin in a sustained manner. If
successful in animal and human trials, it can be used for treating people with type
1 diabetes. The silk scaold was found to be biocompatible (not toxic to living
tissue) as it did not trigger any immune reaction or cause any adverse reaction
when implanted. In Type 1 diabetes, the pancreas produces little or no insulin.
Insulin is a hormone needed to allow sugar (glucose) to enter cells to produce
energy. The results were published in the journal ACS Biomaterials Science &
Engineering.
258. The Tanintharyi National Park (TNP) is located in which country?
[A] Nepal
[B] Bangladesh
[C] Myanmar
[D] Sri Lanka
Correct Answer: C [Myanmar ]
Explanation: The Tanintharyi National Park (TNP) is located at the Tenasserim
Hills, Myanmar and covers an area of 2072 km2. The main purposes of the park
are habitat conservation. It is home to Sambar Deer, Asian Elephant, Barking
Deer, Serow, Red Goral, Malayan tapir, tiger, Leopard and a number of bird
species.
259. The 10th India-European Union Counter Terrorism Dialogue was held in
which country?
[A] Brazil
[B] India
[C] France
[D] Germany
Correct Answer: B [India ]
Explanation: The 10th India-European Union Counter Terrorism Dialogue was
held at New Delhi on August 30, 2017. The Indian delegation was led by Mahaveer
Singhvi, Joint Secretary for Counter Terrorism at the Ministry of External Aairs of
India, while the European Union side was led by Pawel Herczynski, Director for
Security Policy of European External Action Service (EEAS). Both sides agreed to
strengthen institutional linkages between Indian agencies and their European
Union counterparts including Europol for closer interaction and cooperation in the
eld of counter-terrorism. They also agreed to identify relevant training

2017 GKToday | All Rights Reserved | www.gktoday.in

262
Current Aairs: August, 2017 [Date-wise Compendium]

programmes for capacity building of individuals on both sides working in the


sphere of countering terrorism and violent extremism. Prospects for deepening
counter-terrorism cooperation under the UN and Financial Action Task Force
(FATF) were also emphasised. The 11th India-European Union Counter Terrorism
Dialogue will take place in Brussels in 2018 on a mutually convenient date.
260. Who has been appointed as the new Election Commissioner (EC)?
[A] Gautam Jain
[B] Satinder Nangia
[C] Neelam Joshi
[D] Sunil Arora
Correct Answer: D [Sunil Arora]
Explanation: Sunil Arora, a 1980 batch IAS ocer of the Rajasthan cadre, has
been appointed as the new Election Commissioner (EC). The appointment will
take eect from the date he assumes the charge. Presently, Achal Kumar Joti is
the Chief Election Commissioner (CEC) & Om Prakash Rawat is the other Election
Commissioner. Arora is the former Information and Broadcasting Secretary and
Secretary in the Ministry of Skill Development and Entrepreneurship.

2017 GKToday | All Rights Reserved | www.gktoday.in

263

Das könnte Ihnen auch gefallen